Pyq Fmge
Pyq Fmge
MEDICAL ACADEMY
Building Better Doctors
RISE
FMGE iNSiGHT
2017-DEC
2018-JUNE,DEC
2019-JUNE,DEC
2020-AUG,DEC
2021-JUNE
2020-AUG,DEC
2019-JUNE,DEC
2018-JUNE,DEC Arise Medical Academy
ARISE - Hyderabad
+91 7680929292
040 2351 5252
# 2ⁿ Floor, Above Indian Bank,
Opp. Olive Hospital,
Kakatiya Nagar Colony,
Mehdipatnam, Hyderabad - 500 028
arisemedicalacademy@gmail.com
ARISE - Delhi
+91 9100728811
Goutham Nagar,
arisemedicalacademy@gmail.com
Follow us on
f cy d
Arise Medical Academy
Add us on FB Page : - /arisemedacademy
www.arisemedicalacademy.com
MCI DEC 2017
1. Which of the following is a Selective estrogen receptor modulator -
a. Fulvestrant
b. Flutamide
c. Tamoxifen
d. Clomiphine citrate
Explanation :
a. Fulvestrant – selective estrogen receptor downregulators (or) pure
estrogen antagonists
b. Flutamide - selective antagonist of androgen receptor
c. Tamoxifen – selective estrogen receptor modulator. Used as a major
indication for treatment of breast cancer. Has beneficial effect on bone
and lipid profile but increase the risk of endometrial cancer and
thromboembolism.
d. Clomiphine citrate – binds to both ER- alpha and ER – beta and acts
as a pure estrogen antagonists.
2. which of the following Stone is insensitive to alkaline PH of urine?
a. Xanthine stones
b. Cystine stones
c. Struvite stones
d. Calcium stones
Explanation:
a. Xanthine stones – due to deficiency of xanthine oxidase leading to
xanthine uria. May be either radio opaque or radio lucent .
b. Cystine stones – Due to the defect in renal absorption of amino acid
cysytine leading to cystinuria. Radio opaque due to the presence of
sulphur. They are hexagonal shaped or benzene like crystals.
c. Struvite stones – aka triple phosphate stones. Formed mainly after
infection by proteus, which causes alkaline urine leads to precipitation
of phosphate crystals. Drugs like acetazolamide can also cause
struvite stones.
d. Calcium stones - most common type of stone is calcium oxalate
stones. They are radio opaque.
3. Which of the following is Ocp induced liver malignancy ?
a. Hepatic adenoma
b. Hepatocellular carcinoma
c. Hemangioma
d. Focal nodular hyperplasia
Explanation :
a. Hepatic adenoma: Most common symptomatic benign liver tumour.
Most common associated factors are ocp and steroids. High risk of
rupture and malignant transformation.
4. Which of the following methods of contraception is used within 1 st 6 months of
lactation?
a. Combined Ocps
b. Copper T- A380
c. Progesterone only pills
d. Condoms
Explanation:
a. Combined ocps – are contraindicated during lacation .
b. Copper T-A380 – can also be used during lactation but most common
complication is vaginal bleeding.
c. Progesterone only pills - used mainly within 72 hrs of intercourse to
reduce the risk of pregnancy and is considered as safe during
lactation.
d. Condoms – not much effective because of its high failure rate 2-14
per HWY.
5. Prosopagnosia means ?
a. Inability to write
b. Inability to calculate
c. Inability to identify face
d. Inability to speak
Explanation :
a. Inability to write – agraphia
b. Inability to calculate – acalculia
c. Inability to identify faces – prosopagnosia
d. Inability to speak - aphasia
6. Layers of spinal anaesthesia ?
a. Skin>subcutaneous>supraspinatus ligament>interapinatus
ligament> ligamentum flavum
b. Skin> subcutaneous>supraspinatus ligament >ligamentum
flavum>interspinous ligament
c. Skin>subcutaneous>interspinous ligament> supraspinatus
ligament>ligamentum flavum
d. Skin >supraspinatus ligament >subcutaneous >interspinous ligament
>ligamentum flavum
7. Which of the following is the correct order of endometrial blood supply?
a. Arcuate>radial>basal>spiral
b. Radial>arcuate >spiral > basal
c. Radial >basal > arcuate > spiral
d. Arcuate > radial > spiral > basal
8. SIADH is caused by ?
a. Head trauma
b. Tb meningitis
c. Small cell carcinoma of lung
d. All of the above
Explanation :
SIADH : syndrome of inappropriate ADH.
CAUSES: Tumors like- head and neck cancer
Small cell cancer of lung
Bronchial carcinoma
Carcinoid tumour
Head trauma
Drugs like – carbamazapine
Clofibrate
Chlorpropamide
Vincristine
Infections like – pneumonia
TB
Meningitis
SIADH is characterized by gain of water and hence dilutional hyponatremia is seen.
Rapid correction of hyponatremia produces “central pontine myelinosis”.
Rx: doc is – demiclocycline
9. RDA of calcium in elderly women?
a. 600
b. 1200
c. 1800
d. 2400
Explanation :
Elderly women – 1200mg
Pregnancy – 1300 - 1500mg
10. Calculate the volume of fluid to be replaced in a burn patient of TBS 40% of 50 kg
within 1 st 8 hrs?
a. 4 L
b. 8 L
c. 12 L
d. 16 L
Explanation :
Volume of fluid to be given for burns patient is calculated by parkland formula.
Parkland formula= TBSA% * weight (kg) * 4
= 40% * 50*4
= 8L
In 1st 8 hrs- 50% of fluid should be given ie, 4L
In next 16 hrs – remaining 50% of fluid should be given .
11. Sun burst appearance in osteosarcoma is due to?
a. Abnormal calcium deposits
b. Periosteal reaction
c. Bony destruction
d. Highly vascular
Explanation :
Osteosarcoma:
2nd most common malignant bone tumour, arise from metaphysis of lower end of
femur. Gross appearance shows mutton leg appearance . periosteal reaction is
present.
X ray findings- sun ray appearance
Codmans triangle
Alkalaine phosphatase is highly increased.
Rx: pre- operative chemotherphy ,limb saving surgery , post – operative
chemotherphy .
12. Motor molecule are all of the following except?
a. Kinesin
b. Dyenin
c. Myosin
d. Actin
13. P value in null hypothesis?
a. 0.005 %
b. 0.05%
c. 0.5%
d. 5%
Explanation :
Test of significance:
state the null hypothesis ( ie, state that both groups are same)
state the level of significance (alpha – 0.05)
choose the appropriate statistical test
find “P” value
if “P” value is <0.05 , reject null hypothesis ie, both groups are different .
if “P” value is >0.05 ,accept null hypothesis ie, both groups are same.
22. Ranolazine is not contraindicated in giving with which of the following drugs?
a. Ketoconazole
b. Diltiazem
c. Morphine
d. Clozapine
Explanation:
Dose of ranolazine should be limited to 500 mg twice daily in patients on
MODERATE CYP3A INHIBITORS ,including diltiazem , verapamil , erythromycin ,
fluconazole .
Do not use ranolazine with STRONG CYP3A INHIBITORS, including ketoconazole,
itraconazole, clarithromycin etc.
23. Glycemic index of glucose?
a. 0.5
b. 1
c. 1.5
d. 2
24. Corocoid process of scapula is a example of?
a. Pressure epiphysis
b. Traction epiphysis
c. Atavistic epiphysis
d. Aberrant epiphysis
Explanation :
25. Imbecile?
a. Less than 20
b. 25-34
c. 25-49
d. 50-70
Explanation : less than 20 – profound
0 -24 - idiot
25 – 49 – imbecile
50 – 70 – moron
26. Keratic precipitate is seen in which layer ?
a. Epithelium of cornea
b. Decemets membrane
c. Stomach
d. Endothelium of cornea
Explanation:
Keratic precipitate (KP) is an inflammatory cellular deposit seen on corneal
endothelium. Acute KPs are white and round in shape whereas old KPs are faded
and irregular in shape. Mutton-fat KPs are large in shape and are greasy-white in
color and are formed from macrophages and epithelioid cell. They are indicative of
inflammatory disease. Mutton fat Kps are due to granulomatous iridocyclitis. Another
variant called red KPs may be seen in hemorrhagic uveitis.
27. All of the following shows Increased anion gap are all except?
a. Aspirin poisoning
b. Uremia
c. Diarrhoea
d. Diabetic acidosis
Explanation : diarrhoea is eg of normal anion gap.
28. Respiratory papillomatosis is caused by?
a. HPV
b. EBV
c. H. Influenza
d. Klebsiella
Explanation :
Recurrent respiratory papillomatosis (RRP), which is caused exclusively
by human papilloma virus (HPV), is a rare condition characterized by recurrent
growth of benign papillomata in the respiratory tract. The papillomata can occur
anywhere in the aerodigestive tract but most frequently in the larynx, affecting both
children and adults. The management of this entity remains still challenging since no
specific definitive treatment exists.
29. Positive swartz sign is seen in ?
a. Meneier’s disease.
b. Active otosclerosis
c. Acoustic neuroma
d. Malignant otitis externa
Explanation:
Otosclerosis is an abnormal bone growth in the middle ear that causes hearing
loss. Swartz sign is a pink reflex , seen on the promontory through intact tympanic
membrane ,in the area of oval window. It indicates active focus with increased
vacularity. Otosclerosis usually occurs during pregnancy. Management for active
otosclerosis is surgery and inactive otosclerosis is sodium fluoride theraphy.
30. which of the following complement deficiency leads to Meningococcal meningitis?
a. C3b
b. C1
c. C5-C9 [MAC]
d. C5a
Explanation :
Recurrent meningococcal meningitis is caused by neisseria infections due to
deficiency of MAC complex (c5-9).
C1 esterase deficiency leads to hereditary angioedema.
C3b – is a opsonin ,helps in opsonisation of the pathogens.
C5a – chemokine.
31. In which of the following cattle tract appearance is seen?
a. ARMD (age related macular degeneration )
b. Diabetic retinopathy
c. Retinal detachment
d. CRAO
Explanation :
Atherosclerosis related thrombosis at the level of lamina cribrosa is most common
cause (80%) followed by embolization from heart and carotid artery (20%) producing
a sudden painless loss of vision.
C/F : sudden painless loss of vision ,cherry red spot at fovea, segmentation of blood
in blood vessels producing cattle tract sign .
Rx : ocular massage is most effective method
Decrease IOP by giving manitol
Inhalation of carbogen (95% of O2 and 5% of CO2).
32. Treatment for nasopharyngeal carcinoma?
a. Surgery
b. Chemotheraphy
c. Radiotheraphy
d. Both chemo and radiotheraphy
Explanation :
Mc tumour of head and neck which give rise to secondaries with occult primary . Mc
site is fossa of Rossenmuller . Mc type is non keratinizing undifferentiated carcinoma
followed by keratinazing carcinoma. Mc manifestation is upper neck swelling usually
unilateral due to cervical lymphadenopathy . earliest lymphnode to be involved is
retropharyngeal lymphnode. Mc cranial nerve palsy in nasopharyngeal carcinoma is 5
th followed by 6 th and also 9 th and 10 th CN. Treatment of choice is irradiation
(external beam radiotheraphy) as it is highly radiosensitive. In stage 1 and 2 only
radiotherapy is done and in 3 rd and 4 th stage chemoradiation is the treatement.
33. At 24 weeks of gestation the fundus height is at the level of ?
a. Pubic symphysis
b. Umbilicus
c. Xiphoid sternum
d. Manubrium of sternum
Explanation:
At 12 weeks fundus is above pubic symphysis
At 24 weeks fundus is at the level of umbilicus
At 36 weeks fundus is at the level of xiphi sternum
At 38 weeks fundus is just below the level of xiphi sternum
34. All of the following is true regarding TOF EXCEPT?
a. Ventricular septal defect
b. Normal position of aorta
c. Pulmonary stenosis
d. Right ventricular hypertrophy
Explanation:
Tetralogy of fallot is a cyanotic congenital heart defect.
A hole in the wall between your heart’s main pumping chambers (ventricular septal
defect or VSD).
A valve between your heart and lungs that is too narrow (pulmonary stenosis or PS)
A right heart chamber with walls that are too thick (right ventricular hypertrophy)
A major blood vessel (aorta) that is misplaced or moved (overriding aorta).
a. Allergic rhinitis
b. CSF rhinorrhea
c. atrophic rhinitis
d. Rhinosporidiosis
Explanation:
It is a flow of CSF from nose (due to CSF leakage from sub arachnoid space into
nasal cavity). Usual sites of CSF leak are from cribriform plate >frontal sinus > floor
of the anterior cranial fossa. Mc cause of CSF rhinorrhoea is iatrogenic trauma
during surgery.
C/F: unilateral clear watery discharge dripping on looking down, which increases on
coughing , sneezing or exertion.
37. Optic nerve diameter of adult is?
a. O.5 mm
b. 1 mm
c. 1.5 mm
d. 2 mm
38. Which of the following is the carbonic anhydrase inhibitor is anti glaucogenic?
a. Dorsalamide
b. Lantoprost
c. Brimonidine
d. Dipivefrine
Explanation :
Dorsalamide is a carbonic anhydrase inhibitor which is used in the management of
glaucoma to decrease the IOP by decreasing the production of aqueous humour.
Should not be given to patients who are allergic to sulphur.
39. 1 st trimester miscarriage is most commonly due to?
a. Chromosomal abnormalaities
b. Incompetent cervix
c. Fibroids
d. Infections
Explanation :
The most common cause of recurrent 1 st trimester abortion is chromosomal
abnormalities of which mc is balanced translocation in mother. Mc cause of
recurrent 2 nd trimester abortion is incompetent cervix. Infections can’t cause
recurrent abortions.
40. Quintan fever is caused by?
a. Tularaemia
b. Coxiella burnetti
c. H.ducreyi
d. Bartonella
Explanation:
Trench fever/quintan fever is a clinical syndrome caused by infection with
Bartonella Quintana. The disease is classically a 5-day fever. The onset of
symptoms is sudden with high fever, severe headache, back pain and leg pain and a
fleeting rash. Recovery takes a month or more. Relapses are common.
41. Most common site of carcinoma esophagus?
a. Upper 1/3
b. Middle 1/3
c. Lower 1/3
d. Cricopharyx.
Explanation :
Mc site of esophageal carcinoma in india – middle 1/3 rd
Mc site of esophageal carcinoma in worldwide – lower 1/3 rd
Mc type – squamous cell carcinoma
42. Recurrent laryngeal nerve doesn’t supply which of the following laryngeal
muscles?
a. Cricoarytenoid
b. Thyroarytenoid
c. Cricothyroid
d. Interarytenoids
Explanation :
All the laryngeal muscles are supplied my recurrent laryngeal nerve except
cricothyroid which is supplied by external laryngeal nerve , branch of superior
laryngeal nerve .
43. A boy with fever with chills , Swollen tonsil and it is pushed medially is due to
which of the following?
a. Quinsy
b. Lushcka tonsil
c. Parapharyngeal abcess
d. Retropharyngeal abcess
Explanation :
Quinsy/peritonsillar abcess is the collection of pus between the fibrous capsule of the
tonsil. Mc occurs as a complication of acute tonsillitis and generally unilateral.
Caused by GABHS /mixed flora. Managed by i.v fluids ,antibiotics and analgesics. If
not responding to antibiotics, then interval tonsillectomy is done after 6 weeks of
quinsy.
45. From which of the following parafollicular “c” cells are derived from ?
a. Ultimobrachial body
b. Neuroectoderm
c. splanchnic Mesoderm
d. pharyngeal arches
Explanation :
Parafollicular “c” cellas are derived from neural crest > 4 th ventral endodermal
pouch.
46. Which of the following organ undergoes liquefactive necrosis?
a. Cerebrum
b. Kidney
c. Small intestine
d. Pancreas
Explanation :
Brain – undergoes liqefactive necrosis due to ischemia /abcess formation in brain .
All other organs of the body undergo coagulative necrosis eg.kidney ,small intestine
etc.due to coagulation of proteins.
Pancreas and breast are eg of fat necrosis.
47. Launge hauman classification is for ?
a. Shoulder joint
b. Hip joint
c. Knee joint
d. Ankle joint
Explanation :
The Lauge-Hansen classification system is used for the classification of the ankle
injuries based on injury mechanisms which have predictable patterns and imaging
findings. Along with the Weber classification, these systems are useful tools for
describing and classifying ankle injuries.
Classification
The Lauge-Hansen system uses two-word descriptors:
the first word describes the position of the foot at the time of injury (i.e. supination
or pronation)
the second word describes the deforming force direction (i.e. abduction,
adduction, or external rotation)
a. kanamycin
b. gentimicin,
c. amikacin,
d. streptomycin
Explanation :
Neomycin has the greatest cochleotoxic effect of all antibiotics.
Kanamycin are close to neomycin in cochleotoxic potential and are capable of
causing profound, permanent hearing loss while sparing balance.
Streptomycin tends to cause more damage to the vestibular portion than to the
auditory portion of the inner ear.
Explanation :
Wernicke encephalopathy and koraskoff psychosis is due to deficiency of vit B1
thymine usually seen in alcoholics.
51. Identify the structure shown in arrow.
a. Asterion
b. Pterion
c. Inion
d. Glabella
Explanation :
Explanation :
GCS : max score is 15 and minimum score is 3 . GCS for a dead patient is 3.
53. Best to evaluate adequate fluid resuscitation in a burn patient?
a. Urine output
b. Thirst
c. Fall in Blood pressure
d. Shock
Explanation :
The key for monitoring fluid resuscitation in burns patient is urine output. It
should be between 0.5 and 1 ml/kg body weight per hour. If the urine output is below
this ,the infusion rate should be increased by 50%.
54. 21 teeth present at the age of?
a. 5
b. 6
c. 10
d. 12
Explanation :
The complete set (ie,20) primary teeth have to come in by the time the child is
2 1/2 to 3 yrs old. Rule of thumb is that ~ 4 teeth will erupt for every 6 months of life.
Lower teeth erupt before upper teeth with teeth in both jaws usually erupting in pairs.
Primary teeth are smaller and whiter in color than permanent teeth that will follow.
After age 4 ,the jaw and facial bones of child begin to grow and creating the space
between the primary teeth. Between the age of 6 and 12, a mixture of booth primary
teeth and permanent teeth reside in mouth .
55. Another Term of lesbianism ?
a. Fetischism
b. Eonism
c. Urolangia
d. Tribadism
Explanation :
Lesbianism /tribadism – gratification of sexual desire of a woman by another women.
Urolangia – sexual excitement by sight or odor of urine or faeces
Eonism /transvestism – pleasure of wearing clothes of opposite sex.
Fetischism – sexual gratification by article of opposite sex.
56. Movement of superior rectus muscle?
a. Elevation , intortion, adduction
b. Depression, extortion, adduction
c. Elevation , extortion, abduction
d. Depression , intortion, abduction
Explanation:
Muscles Function
Primary secondary tertiary
Abduction - -
Lateral rectus
Inferior rectus Adduction - -
Inferior rectus Depression Extortion Adduction
Superior rectus Elevation Intortion Adduction
Inferior oblique Extortion Elevation Abduction
Superior oblique Intortion Depression Abduction
57. A Man with bulla on face and neck, flaccid bulla, spreading and nikolsky sign
positive is seen in ?
a. Bullous pemphigoid
b. Pemphigus vulgaris
c. Pemphigus violaceous
d. Dermatitis herpetiformis
Explanation :
Disease Biopsy features DIF
Subepidermal oedema/blister.
Bullous pemphigoid Linear IgG,
Mixed perivascular infiltrate with eosinophils. C3 in BMZ
(lamina
lucida).
Dermatitis Subepidermal vesicles. Granular IgA
herpetiformis Mixed papillary infiltrate with eosinophils. in tips
of papillae.
62. Which of the following is most common cause for oral cancer ?
a. Alcohol
b. Betel nut chewing
c. Sun exposure
d. Infections like EBV,HPV etc.
Explanation :
Most cases of oral cancer (~90%) in south east asia are linked to tobacco
chewing and tobacco smoking. In India mc form of tobacco chewing is betel liquid
which usually consist of betel leaf ,areca nut , lime and tobacco.
a. Intramuscular
b. Intravenous
c. Transdermal
d. Intrathecal
Explanation :
Fentanyl can be given as slow iv or im , it can also be given intrathecal
administration.
64. Site of gluconeogenesis is ?
a. Muscle
b. Liver
c. Kidney
d. Intestine
Explanation:
Gluconeogenesis is the synthesis of glucose from non carbohydrate
compounds. It takes place in liver and kidney . The liver accounts for 90% of
gluconeogenesis in the body and the remaining 10% occurs in kidney and other
tissues of the body. It takes place in both cytoplasm and mitochondria of liver.
65. Hand foot mouth disease is caused by ?
a. HINI
b. Adenovirus
c. Rotavirus
d. Coxsackie virus
Explanation:
Coxsackie virus is single strand + ve sense RNA virus ,which spread
by feco oral route. HFMD, caused by coxsackieviruses, usually causes fever,
malaise, rash, and small blisters that ulcerate. The most frequent locations for
the blisters/ulcers are on the palms of the hand, soles of the feet, and in the mouth.
There is no specific treatment or vaccine available for coxsackievirus infections.
66. Period of viability of the fetus is?
a. 210
b. 220
c. 230
d. 240
Explanation :
Previously ,28 weeks of geststion is considered as a period of viability ie,210 days.
Now it has been changed to 22 weeks to consider fetus as viable. The best option
here is 210 days.
diarrhoea
abdominal pain
fatigue (extreme tiredness)
unintended weight loss
blood and mucus in your faeces (stools).
People with Crohn's disease sometimes go for long periods without symptoms or
with very mild symptoms. This is known as remission. Remission can be followed by
periods where symptoms flare up and become particularly troublesome.
78. Which of the following is not correct regarding reduced ORS ?
a. Sodium 75
b. Trisodium Citrate 10
c. Potassium chloride 20
d. Glucose 90
Explanation :
a. Colorectal carcinoma
b. Diffuse esophageal spasm
c. Intussception
d. Crohn’s disease
Explanation :
The “apple core sign” was first described in barium enema examinations of the
colon. The apple core appearance is the visual manifestation of an annular lesion of
the bowel with irregular overhanging edges and shouldered margins . The apple core
sign is classically seen in cases of colon carcinoma.
82. Which of the following k+ sparing diuretics that doesn’t cause gynecomastia ?
a. Spironolactone
b. Triamterene
c. Amiloride
d. Eplenorone
Explanation :
Amiloride acts by inhibiting the epithelial sodium channel (ENaC). Its mechanism is
different to spironolactone ,which is an aldosterone receptor antagonist. Therefore
amiloride doesn’t cause gynecomastia. Eplenorone has similar action like
spironolactone but it have less adverse effects like gynecomastia and vaginal
bleeding when compared with spironolactone.
83. Lid lag sign of hyperthyroidism is?
a. Moebius sign
b. Vongraefe’s sign
c. Dalyrymptes sign
d. Stellwag’s sign
Explanation :
84. In case of hanging , during autopsy which of the following cavity should be
opened 1st?
a. Cranial cavity
b. Thoracic cavity
c. Abdominal cavity
d. Pelvic cavity
a. Sphincter of oddi
b. Pancreas
c. 3 rd part of duodenum
d. Jejunum
Explanation :
Explanation :
90. Most common tumour to occur in HIV patients is ?
a. Cervical cancer
b. Hodgkin’s lymphoma
c. Kaposi scarcoma
d. Nasopharyngeal carcinoma
Explanation :
Infection with HIV weakens the immune system and reduces the body's ability to
fight viral infections that may lead to cancer. The viruses that are most likely to cause
cancer in people with HIV are:
Hepatitis B virus (HBV) and hepatitis C virus (HCV), which both cause liver
cancer.
Explanation :
Amblyopia is a developmental visual disorder which is characterized by
reduced visual acquity (uncorrectable by lense in the absence of any detectable
defect in eye or visual pathway). Crowding phenomenon is a feature of an amblyopic
eye. It is partial loss of sight , best detected and treated by 3 or 4 yrs of age for best
results. Occlusion of normal eye to encourage the use of amblyopic eye is the most
effective treatment.
92. Man with Lepromatous leprosy who had undergone chemotherapy develops
various nodules is a example of which type of hypersensitivity ?
a. Type 2
b. Type 3
c. Type 4
d. Type 5
Explanation :
Leprosy is a chronic granulomatous infection principally affecting the skin
and peripheral nerves caused by the obligate intracellular organism Mycobacterium
leprae. The disease causes skin lesions and neuropathy. Complications secondary to
the neuropathy can result in deformity and disability. The immunological response
mounted by the host dictates the clinical phenotype that develops. People with
leprosy show a spectrum of clinical types. Tuberculoid disease is the result of high
cell-mediated immunity with a largely Th1 type immune response. Lepromatous
leprosy however is characterized by low cell-mediated immunity with a humoral Th2
response. Type 1 reactions are delayed hypersensitivity(type 4) reactions that occur
in borderline disease. Type 2 or erythema nodosum leprosum (ENL) reactions occur
in borderline lepromatous and lepromatous disease (type -3).
Explanation :
A hordeolum is a common disorder of the eyelid. It is an acute focal infection
(usually staphylococcal) involving either the glands of Zeis (external hordeola, or
styes) or, less frequently, the meibomian glands (internal hordeola). In the early stages
of disease the gland become swollen, hard and painful. Usually the whole edge of the
lid is edematous. Subsequently abscess is formed which points near the base of one
of the cilia. Pain is considerable until pus is evacuated.
94. Intermittent ring causing dysphagia is seen in which of the following disease?
a. Diffuse esophageal spasm
b. Schatzki’s ring
c. Esophageal web
d. Streakhouse syndrome
Explanation :
Schatzki rings are fixed smooth, benign, circumferential, and narrow ring of
tissue in the lower end of the esophagus. Schatzki rings are a common cause of
intermittent dysphagia. Typically, the dysphagia is intermittent or short-lived. Food may
become stuck in the esophagus but can be forced down by drinking liquids or may be
regurgitated. The dysphagia may not recur for months or years. Patients may have
associated symptoms of gastroesophageal reflux. Barium esophagography is the
preferred method of detection of Schatzki rings.
95. Identify the procedure given in the picture?
a. Robotic surgery
b. Laproscopic cholecystitis
c. Endoscopic
d. LAZER ablation
Explanation :
Robotic surgery, or robot assisted surgery, is one of the developments that is at the
forefront of this shift. Robotic surgery involves the use of surgical robots that can be
controlled by doctors through computers. This results in less pain and blood loss for
the patient, and quicker healing time. Even higher accuracy can be achieved when
computer-generated images are used to guide surgeries. One such possibility is the
use of a real time MRI scan to guide surgeries.
96. Most common site of zollinger Ellison syndrome is?
a. Cardiac part of stomach
b. Pylorus part of stomach
c. Duodenum
d. Jejunum
Explanation :
Zollinger-Ellison syndrome (ZES) is a disease of the gastrointestinal system.
People who have ZES develop tumors known as gastrinomas in the pancreas and
duodenum (the first section of the small intestine). The gastrinomas caused by ZES
secrete the hormone gastrin. Because gastrin creates excessive stomach acid, 90
percent of patients with ZES develop stomach and duodenal ulcers.
97. Mummification is seen in which conditions?
a. Moist and humid environment
b. Lead poisoning
c. Desiccation of body in hot and dry weather
d. Electric burns
Explanation :
Mummification is a another modification of putrefaction. Hot and dry climate
favours it. Dehydration and drying causes evaporation of water and shrinkage of
cadaver. The natural appearances and body features are preserved. The mummified
body is odourless, dry and leathery brown. The time required is 3 months to 1 yr. It
begins in the exposed parts of the body like face, hands, feet and then extends to
internal organs. Arsenic and Antimony poisoning favours mummification .
98. In Game keeper’s thumb which of the following is damaged?
a. medial collateral ligament
b. Ulnar collateral ligament
c. anterior ligamnet
d. posterior ligament
Explanation :
Ulnar collateral ligament of thumb is damaged
Explanation :
Carotid sinus massage is performed for three reasons:
• To break junctional tachycardia (IAVNRT and AVRT) by blocking AV nodal
conduction (and cause Bradycardia).
• To reveal on the P wave pattern of an atrial arrhythmia by reducing the frequency of
AV nodal conduction during the tachycardia so that the QRS complexes do not mask
the artrial activity.
• To test for carotid sinus hypersensitivity, which is a severe fall in blood pressure or
heart rate in response to carotid sinus stimulation.
100. Incidence is
a. No. Of new case
b. No. Of existing case
c. 1st case in community
d. 1st case that comes to doctor
Explanation :
Incidence is defined as the no. of new cases occurring in a defined population during
specific period of time. It can be determined from cohort study .
Incidence = no of new case /population at risk *1000
Incidence is a RATE . (expressed per 1000)
101. Invertogram to be done in a new born:
a. Immediately
b. After 2 hrs
c. After 4 hrs
d. After 6 hrs
Explanation :
Invertogram is done for detection of imperforate anus. It is done after six hours
of birth in infant u sufficient air may have collected in large intestine to cast a X-ray
shadow.
102. For hyponatremia select the electrocardiographic finding with which it is most
commonly associated?
a. No known electrocardiographic abnormalities
b. Prolonged QT interval
c. Short QT interval
d. Widened QRS complex
Explanation :
Hypokalemia typically increases automaticity of myocardial fibers, which results
in ectopic beats or arrhythmias. Electrocardiography in hypokalemia reveals
flattening of the T wave and prominent U waves. Hyperkalemia decreases the rate of
spontaneous diastolic depolarization in all pace maker cells. It also results in slowing
of conduction. One of the earliest electrocardiographic signs of hyperkalemia is the
appearance of tall, peaked T waves. More severe elevations of the serum potassium
result in widening of the QRS complex. At serum sodium levels compatible with life,
neither hyponatremia nor hypernatremia results in any characteristic
electrocardiographic abnormalities.
103. Over 75% of the strength of the intact abdominal wall lies in the –
a. Skin
b. Subcutaneous tissue
c. Aponeurosis
d. Peritoneum
Explanation:
Aponeurosis of rectus sheath is very strong and provides maximum strength of intact
abdominal wall.
104. In amblyopia, the treatment of choice is:
a. Surgery
b. Orthoptic exercise
c. Spectacles
d. Conventional occlusion
Explanation :
Amblyopia is a developmental visual disorder which is characterized by
reduced visual acquity (uncorrectable by lense in the absence of any detectable
defect in eye or visual pathway). Crowding phenomenon is a feature of an amblyopic
eye. It is partial loss of sight , best detected and treated by 3 or 4 yrs of age for best
results. Occlusion of normal eye to encourage the use of amblyopic eye is the most
effective treatment. Occlusion therapy must be carried out for as short a period as
may be expected to give improvement in the acuity of the amblyopic eye and yet
avoid the risk of occluded eye itself becoming amblyopic from lack of visual output.
105. Paranoid schizophrenia is seen with which of the following substance abuse?
a. Cocaine
b. Amphetamine
c. Barbiturates
d. Opiods
Explanation :
Schizophrenia like syndrome may be seen with amphetamine use, mescaline
,phencyclidine abuse, LSD abuse , huntington’s chorea, homocystinuria ,
hemochromatosis , wilson’s disease, acute intermittent porphyria. Thought disorder
in paranoid schizophrenia are bizzre delusions ,1st rank symptoms , and formal
thought disorder. Insight is absent.
Explanation:
Cresol emulsions are very powerful disinfectant . Also known as “all
purpose general disinfectant “.
109. Hippus is seen in which poisoning ?
a. Abrus
b. Aconite
c. Datura
d. Alcohol
Explanation :
The condition called hippus is also called pupillary athetosis, or spasmodic,
rhythmic dilation and constriction of the pupils.
Aconite- in late stages Hippus is seen.
Alcohol- pupils are earlier dilated, later constricted,
Barbiturates- pupils are earlier constricted, later dilated.
110. Superior vena caval syndrome is most commonly caused by ?
a. Lung cancer
b. pheochromocytoma
c. cirrhotic liver
d. rib fractures
Explanation :
SVCS is more common for people who have lung cancer, non-Hodgkin lymphoma,
or cancers that spread to the chest. But cancer can cause SVCS in other ways:
A tumor in the chest may press on the superior vena cava.
A tumor may grow into the superior vena cava, causing a blockage.
If cancer spreads to the lymph nodes surrounding the superior vena cava, the
lymph nodes may enlarge and press on or block the vein.
A blood clot may appear in the vein. This is caused by a pacemaker wire or
an intravenous catheter, which is a flexible tube placed in a vein to take out or
put in fluids
Explanation:
Polyuria is defined, as 24-hour urine volume greater than 3 litres.
Hypocalcaemia does not cause polyuria. Hypercalcaemia causes polyuria by solute
diuresis. Solute diuresis is also seen with glucose, mannitol, radiocontrast agent,
urea, resolving ATN.
Lithium causes nephrogenic diabetes insipidus and hence polyuria. ADH deficiency
causes central diabeted insipidus. The various causes are postoperative (removal of
pituitary), trauma supra-or intrasellar tumour, Sheehan’s syndrome, infections etc.
Explanation:
Scrub typhus is caused by Orientia tsutsugamushi, which is maintained in nature
by transovarian transmission in trombiculid mites, mainly of genus Leptotrombidium.
After hatching, the infected larval mites (chigger) inoculate organisms into the skin
while feeding. Clinical features are fever, headache, myalgia, cough, gastrointestinal
symptoms, regional lymphadenopathy etc. Doxycycline or chloramphenicol is used for
treatment.
a. Herpes simplex
b. Impetigo
c. Dermatitis herpetiformis
d. Phemphigus.
Explanation :
Impetigo is the most common bacterial infection in children. primarily caused
by Streptococcus pyogenes or Staphylococcus aureus. Impetigo is classified as either
nonbullous (impetigo contagiosa) (about 70% of cases) or bullous. nonbullous
impetigo commonly have honey-colored crust with a moist erythematous base as
shown in the above image. Bullous impetigo is considered to be less contagious than
the nonbullous form.
122. Skin of the parotid gland is supplied by which of the following nerve?
a. Greater auricular nerve
b. Auriculotemporal nerve
c. Mandibular nerve
d. Vagus nerve
Explanation :
The parotid gland receives sensory and autonomic innervation. The autonomic
innervation controls the rate of saliva production.
Sensory innervation is supplied by the auriculotemporal nerve (gland) and the great
auricular nerve (fascia and skin). The parasympathetic innervation to the parotid
gland has a complex path. It begins with the glossopharyngeal nerve.
The auriculotemporal nerve then carries parasympathetic fibres from the otic
ganglion to the parotid gland. Parasympathetic stimulation causes an increase in
saliva production. Sympathetic innervation originates from the superior cervical
ganglion, part of the paravertebral chain.
123. Which of the following tumour is rathke’s pouch derivative ?
a. Glioblastoma multiforme
b. Craniopharyngioma
c. Vestibular schwanoma
d. Oligodentroglioma
Explanation :
A craniopharyngioma is a benign (noncancerous) tumor arising from small
nests of cells near the pituitary stalk. Adamantinomatous (ordinary)
craniopharyngioma occurs in children and tends to be less solid than papillary
craniopharyngioma. Papillary craniopharyngioma occurs in adults and is a more solid
tumor. Increased pressure within the brain causes many of the symptoms associated
with this tumor. Other symptoms result from pressure on the optic tract and pituitary
gland Surgery to remove the tumor is usually the first step in treatment. If
hydrocephalus (excess water in the brain) is present, a shunt (drainage system) may
be placed during surgery. The shunt will help remove excess cerebrospinal fluid from
the brain and ease the pressure.
124. For diagnosis of schizophrenia symptoms should be present for how long
period?
a. 2 weeks
b. 1 month
c. 6 month
d. 1 year
Explanation :
Two or more of the following ,each present for a significant portion of time of during 1
month
Delusions
Hallucinations
Disorganized speech
Negative symptoms ie, affective flattening, alogia or avolition .
Continous signs of disturbance persist for atleast 6 months.
This 6 month period must include atleast 1 month of symptoms.
125. Leucocyte rolling over endothelial cells of the blood vessel is carried out by
which of the following?
a. Selectins
b. Integrins
c. PECAM 2
d. E- Cadherins
Explanation:
Rolling is a process of transient adhesion of leucocytes with the endothelial
cells. Selectins are the important molecules responsible for it. They interact with
complementary molecules resulting in transient adhesion.
126. Which of the following tumour is most common in post renal transplant
patients?
a. Non Hodgkin’s Lymphoma
b. Hepatocellular carcinoma
c. Non melanoma skin cancer
d. Kaposi’s sarcoma
Explanation :
There is an overall 3–5-fold increase in cancer risk in transplant recipients
compared with the general population, with skin cancers and lymphoma particularly
prevalent. Cancers in transplant recipients are often more aggressive than those in
the general population, with poor prognosis, particularly for gastrointestinal tumours
and lymphomas.
127. In ERG ‘A’ wave corresponds to?
a. Rods and cones
b. Pigment epithelium
c. Nerve bundle layer
d. Artefact
Explanation:
Electroretinography: A test in which the electrical potentials generated by the retina
of the eye are measured when the retina is stimulated by light. Abbreviated as ERG.
In an ERG, an electrode is placed on the cornea at the front of the eye that
measures the electrical response of the rods and cones, the visual cells in the retina
at the back of the eye. An ERG may be useful in the evaluation of hereditary and
acquired disorders of the retina. The instrument used to conduct ERG is an
electroretinograph, and the resultant recording is called an electroretinogram
waves Function
A wave Activity of rods and cones
a. Pithris pubis
b. Scabei
c. Head louse
d. Tick
135. Which of the following is not a metabolites of tyrosine?
a. Melanin
b. Melatonin
c. norepinephrine
d. T3 and t4
Explanation :
Tyrosine is required for the formation of thyroid hormones, melanin and
catecholamines such as epinephrine, norepinephrine, dopamine. Melatonin is a
metabolite of tryptophan.
136. Incubation period of hepatitis – B is
a. 10- 14 days
b. 30- 180 days
c. 15- 60 days
d. 3- 35 days
Explanation :
Type Causative agent Incubation period Common modes of
transmission
Hep -A Enterovirus -72 15 - 45 days Fecal – oral ,
sexual
(picornavirus)
Hep -B Hepadnavirus 30 – 180 days Sexual ,perinatal,
percutaneous
Hep -C Hepacivirus 15 – 160 days percutaneous
(flavivirus)
Hep –D Viriods like 30 – 180 days Sexual , perinatal ,
percutaneous
Hep -E Calcivirus 15 – 60 days Fecal – oral
(alpha virus like)
140. Which of the following is the tumour marker for hepatocellular carcinoma?
a. CEA
b. a – feto protein
c. B- HCG
d. calcitonin
Explanation :
CEA Colon cancer
Pancreatic cancer
Lung and stomach cancer
AFP Yolk sac tumour
Hepatocellular carcinoma
B – HCG Trophoblastic tumors like choriocarcinoma , seminoma etc.
Calcitonin Medullary carcinoma of thyroid
141. H.pylori is a risk factor for causing which of the following cancers?
a. GIST
b. Gastric adenocarcinoma
c. Squamous cell carcinoma of esophagus
d. Hepatic adenoma
Explanation :
H.pylori is a gram –ve flagellated bacteria producing enzymes like phospholipase
and urease. H.pylori is associated with peptic ulcer disease, gastric cancer and
gastric mucosa associated lymphoma.
142. Which of the following class of drugs is not used in treatement of uterine
fibroids?
a. Nafarelin
b. Tranexemic acid
c. Misoprostol
d. Mifepristone
Explanation :
Nafarelin – GnRH agonist is used to decrease the size of fibroids.
Tranexemic acid is a non hormonal drug which can be used to reduce the blood loss
over short periods.
Mifepristone is also used to treat fibroids.
143. Which of the following nerve passes through cavernous sinus ?
a. Occulomotor
b. Abducens
c. Trochlear
d. Trigeminal V1
Explanation :
a. Azygous vein
b. Hemiazygous vein
c. Minor fissure of right lung
d. Major fissure of left lung
Explanation :
In the untreated patient the fourth week ceases the fever, mental state
and abdominal distension slowly improve over a few days, but intestinal
complications may still occur. Convalescence is prolonged, and most relapses occur
at this stage.
Treatement :
The only effective treatment for typhoid is antibiotics. The most commonly used
are ciprofloxacin (for non-pregnant adults) and ceftriaxone. Other than antibiotics, it
is important to rehydrate by drinking adequate water.In more severe cases, where
the bowel has become perforated, surgery may be required.
Explanation :
175. Glutaraldehyde is not used as a disinfectant for which of the following ?
a. Thermometer
b. Endotracheal tube
c. Cystoscope
d. Bronchoscope
Explanation :
Glutaraldehyde is used for cystoscopes, bronchoscopes, rubber tubes, face masks,
and endotracheal tubes. Wiping with alcohol was at least as efficient as immersion in
a phenolic and is recommended for thermometer disinfection in hospitals.
176. Which of the following is associated with rheumatic heart disease?
a. Roth’s spots
b. Pulsus paradoxus
c. Koplik’s spots
d. McCallum plaques
Explanation :
Rheumatic heart disease describes a group of short-term (acute) and long-term
(chronic) heart disorders that occur as a result of acute rheumatic fever. Damage of
the heart valves is a common result of rheumatic fever. It is usually seen in children
who are 5 to 15 years old. Mural endocardial lesions can be seen as MacCallum
plaques in rheumatic heart disease. These plaques appear as map-like areas of
thickened, roughened, and wrinkled part of the endocardium in the left atrium.
177. Burning sensation in extremites is due to ?
a. Pheripheral neuropathy
b. Poliomyelitis
c. Syringomyelia
d. Multiple sclerosis
Explanation :
Peripheral neuropathy is damage to the peripheral nerves and results in a
tingling, painful, or burning sensation in the extremities. It most commonly occurs in
the legs. Other symptoms of peripheral neuropathy can include weakness and
numbness. Poorly controlled diabetes is one of the most common causes of
peripheral neuropathy, but a number of conditions can be responsible for damage to
the peripheral nerves.
178. Burned airways, 1st line management is ?
a. Tracheostomy
b. Elective intubation
c. Fluid replacement
d. Prophylactic corticosteroid
Explanation :
In cases of thermal burns to the airway, many burn surgeons prefer ET intubation
over blind insertion techniques, but supraglottic airways are acceptable if intubation
isn’t available or can’t be established quickly.
a. Ulcus serpens
b. Acanthancochal ulcer
c. Dentritic ulcer
d. Pseudodentritic ulcer
Explanation :
Viral ulcer – caused by herpes simplex
usually attacks the epithelium and
stroma. In epithelium these dentritic
ulcers are seen. These are called true
dentrites and are like branch of tree.
Corneal sensation is reduced and so it is
regarded as most painless ulcer. Steroid
are C/I in dentritic ulcer . If given, it can
lead to geographical ulcer.
182. Delayed onset of puberty in female age is seen at?
a. 12 yrs
b. 13 yrs
c. 14 yrs
d. 15 yrs
Explanation :
Puberty starts when the pituitary gland begins to produce two hormones ,
luteinizing hormone (called LH) and follicle-stimulating hormone (called FSH), which
cause the ovaries to enlarge and begin producing estrogens. The growth spurt starts
shortly after breasts begin to develop, and the first menstrual cycle begins about 2-3
years later. A girl who has not started to have breast development by the age of 13 is
considered to be delayed.
183. Microsporum affects ?
a. Skin , hair and nail
b. Skin and hair
c. Skin and nail
d. Hair and nail
Explanation :
Microsporum spp. mostly infect the hair and skin, except for Microsporum
persicolor which does not infect hair.
184. Who level of Hb conc. Of anemia in pregnancy is ?
a. 10 gm/dl
b. 11 gm/dl
c. 12 gm/dl
d. 13 gm/dl
Explanation :
Anemia is a condition of low circulating haemoglobin (Hb) in which concentration
has fallen below a threshold lying at two standard deviations below the median of a
healthy population of the same age, sex and stage of pregnancy. The WHO
definition for diagnosis of anemia in pregnancy is a Hb concentration of less than 11
g/dl (7.45 mmoL/L) and a hematocrit of less than 33%.
185. Radiological test for staging of endometrial carcinoma invasion is?
a. MRI
b. CECT pelvis
c. TAS
d. TVS
Explanation :
A dedicated MRI protocol is crucial for accurate MRI evaluation of endometrial
carcinomas. In endometrial carcinoma, it is important to accurately assess invasion
depth and preoperative staging. Fusion of T2-weighted magnetic resonance imaging
(T2WI) and diffusion-weighted magnetic resonance imaging (DWI) may contribute to
the improvement of anatomical localization of lesions.
186. For how long immunusuppresive therapy should be given post renal
transplant?
a. Lifelong
b. 3 months
c. 6 months
d. 9 months
First stage: Begins from the onset of true labor and lasts until the cervix is
completely dilated to 10 cm.
Second stage: Continues after the cervix is dilated to 10 cm until the delivery
of your baby.
Third stage: Delivery of your placenta. The third stage is the delivery of the
placenta and is the shortest stage. The time it takes to deliver your placenta
can range from 5 to 30 minutes.
Signs of placental separation :
A gush of blood comes from vagina
Umbilical cord gets longer
Uterus rises over the abdomen
Uterus becomes round in shaped.
192. Most common site for the development of laryngeal web is?
a. Anterior
b. Posterior
c. Subglottic
d. Supraglottic
Explanation :
Laryngeal web is a condition in which child’s larynx (voice box) contains a layer
of web-like tissue. This partially obstructs child’s windpipe, causing frequent
shortness of breath and other symptoms. Laryngeal webs occur in the glottic level
and affect the vocal cords ™More than 90% of laryngeal webs webs are located
anteriorly and extend toward the arytenoids. Other types include the posterior glottic
web, causing interarytenoid vocal cord fixation; subglottic webs, which may occur
with or without cricoid cartilage involvement and subglottic stenosis; and supraglottic
webs.
193. Rise in JVP with inspiration is seen with all except ?
a. Pericardial effusion
b. Pleural effusion
c. Constrictive pericarditis
d. Cardiac tamponade
Explanation :
JVP will decrease during inspiration in the normal state. The JVP will rise during
inspiration (kussmaul’s sign )in the presence of pericardial constriction , right
ventricular infarction , pericardial effusion, or rarely, cardiac tamponade.
Explanation :
204. A 9 yr old school going girl has a difficulty in focusing on one task, day
dreamer, easily distracted, talk nonstop. Possible diagnosis is ?
a. Conduct disorder
b. ADHD
c. AUTISM
d. Pervasive development disorder
Explanation :
ADHD is one of the most common conditions in childhood. It impacts how kids
function in school and in everyday life. symptoms include :
There is no cure for ADHD, currently available treatments may help reduce
symptoms and improve functioning. ADHD is commonly treated with medication,
education or training, therapy, or a combination of treatments.
205. False statement regarding peutz jegher syndrome is?
a. Associated with mucosal, lip and skin melanosis
b. It is autosomal disorder
c. Has multiple polyp in intestine that require surgery to stop bleeding
d. It is a malignant condition
Explanation :
Peutz-Jeghers syndrome (PJS) is an autosomal dominant inherited disorder
characterized by benign intestinal hamartomatous polyps in association with a
distinct pattern of skin and mucosal macular melanin deposition. People with Peutz-
Jeghers syndrome have a high risk of developing cancer during their lifetimes.
Cancers of the gastrointestinal tract, pancreas, cervix, ovary, and breast are among
the most commonly reported
206. B – blocker ointment is used for which of the following conditions?
a. Superficial burns
b. Infantile hemangiomas
c. Contaminated wounds
d. Clean wounds
Explanation :
The treatment of infantile hemangiomas changed from the use of oral
corticosteroids to oral propranolol . Topical beta blockers are now used for lesions
with both deep and superficial components and those that are amblyogenic. When
initiated in the proliferative phase of the lesion, the effectiveness of the treatment can
be seen within days.
a. Chlonorchis sinensis
b. Schistosoma hematobium
c. Fasciola hepatica
d. Paragonimus westermani
Explanation :
testis. Upper and lower motor neuron disorders can cause an absence of
cremasteric reflex.
223. Chance of metastasis to liver in colon cancer is?
a. Less than 25%
b. 25- 50%
c. 75%
d. 90 %
Explanation:
More than 50% of the patients with colorectal cancer (CRC) have or will develop
metastasis, with a quarter having distant metastatic disease at the time of diagnosis,
most frequently in the liver. Liver metastasis is the leading cause of cancer-related
morbidity and mortality in colorectal cancer. Left sided colon cancer has 70%
increased risk of isolated liver metastases. The only potentially curative treatment
for liver metastasis is liver resection, but only 15% to 20% of the patients are suitable
for surgical resection.
224. False statement regarding crohn’s disease is?
a. Skip lesions are seen
b. Presence of non caseating granuloma
c. Intestinal resection is curative
d. Ileum is the most common site to be affected.
Explanation :
225. Which of the following is not a absolute contraindication of OCP?
a. Breast cancer
b. Liver disease
c. Age <30 yrs
d. Congenital hyperlipdemia
Explanation :
229. Which of the following statement is correct regarding fast breathing in a child?
a. > 60 /min in a child > 12 months
b. >50 /min in a child of age between 2 months to 1 yr
c. > 40/min in a child of > 2 months
d. > 70 /min in a child of > 18 months
Explanation :
According to ARI control programme, a case of pneumonia is considered only if
child has fast breathing.
Upto 2 months - >60/min
2- 12 months - > 50/min
>12 months - >40/min
230. Which of the following hormone is increased in ZES?
a. Histamine
b. Pepsin
c. Gastrin
d. Secretin
Explanation :
Zollinger-Ellison syndrome is a rare disorder that occurs when one or more
tumors form in the pancreas and duodenum. The tumors, called gastrinomas,
release large amounts of gastrin that cause the stomach to produce large amounts
of acid. Patients with ZES show gastric acid hypersecretion, which induces refractory
peptic ulcers, severe gastroesophageal reflux disease, diarrhea, and even death.
The plasma concentration of gastrin is measured by RIA or ELISA , and can be used
to diagnose gastrin-secreting tumors, atrophic gastritis, gastric ulcers, and pernicious
anemia. Zollinger-Ellison syndrome treated with medications to reduce gastric acid
secretion and with surgery to remove gastrinomas.
231. Febuxostat is used in the treatment of?
a. Gout
b. Rheumatoid arthritis
c. Polyarteritis nodosa
d. gaint cell arteritis
Explanation :
Febuxostat is used to lower uric acid levels in people with gout. It inhibits xanthine
oxidase . Febuxostat works by reducing the amount of uric acid made by the body.
Increased uric acid levels can cause gout.
232. Dense bodies in smooth muscle is analogues to which part of skeletal
muscle?
a. I band
b. M line
c. Z line
d. A band
Explanation :
Smooth muscle fibers are spindle-shaped and have a single nucleus; they range
from about 30 to 200 μ m and they produce their own connective tissue,
endomysium. Although they do not have striations and sarcomeres, smooth muscle
fibers do have actin and myosin contractile proteins, and thick and thin filaments.
These thin filaments are anchored by dense bodies. A dense body is analogous to
the Z-discs of skeletal and cardiac muscle fibers and is fastened to the sarcolemma.
233. Excessive vomiting is an example of which type of acid base disorder?
a. Metabolic acidosis
b. Repiratory acidosis
c. Metabolic alkalosis
d. Respiratory alkalosis
Explanation:
In a case of vomiting , H+ ions are lost and because of this, bicarbonate ions
remains and the patient develops metabolic alkalosis.
234. Under mental illness act, detaining of mentally ill person is punishable for how
many yrs?
a. 1 yr
b. 2 yr
c. 5 ys
d. 7 yr
Explanation :
The mental heath act (1987), repels the Indian lunacy act (1912).
Defines, “mentally ill person” as a person who is in need of treatment by reason of
any mental disorder other than mental retardation for treatment as in patient a
person can represent himself or represented by family or friends in writing to the
director of a psychiatric hospital. For a preliminary diagnosis , he can be kept in
hospital for 10 days, if more time is required then it has to permit by magistrate who
can give extension of 10 days twice.
Punishment : detains or keeps mentally ill patient otherwise in accordance with the
act is punishable with upto 2 yrs imprisonment or Rs.10000 fine or both.
235. In establishing causal association, most essential criterion is?
a. Consistency of association
b. Temporal association
c. Duration of association
d. Strength of association
Explanation :
The Bradford-Hill criteria are widely used in epidemiology as providing a framework
against which to assess whether an observed association is likely to be causal.
1. Strength of the association.
According to Hill, the stronger the association between a risk factor and outcome, the
more likely the relationship is to be causal.
2. Consistency of findings.
Have the same findings must be observed among different populations, in different
study designs and different times?
3. Specificity of the association.
There must be a one to one relationship between cause and outcome.
4. Temporal sequence of association.
Exposure must precede outcome.
5. Biological gradient.
Change in disease rates should follow from corresponding changes in exposure
(dose-response).
6. Biological plausibility.
Presence of a potential biological mechanism.
7. Coherence.
Does the relationship agree with the current knowledge of the natural history/biology
of the disease?
8. Experiment.
Does the removal of the exposure alter the frequency of the outcome?
Oculoglandular - This form occurs when the bacteria enter through the eye. This
can occur when a person is butchering an infected animal and touches his or her
eyes. Symptoms include irritation and inflammation of the eye and swelling of lymph
glands in front of the ear.
Pneumonic - This is the most serious form of tularemia. Symptoms include cough,
chest pain, and difficulty breathing. This form results from breathing dusts or
aerosols containing the organism. It can also occur when other forms of tularemia
(e.g. ulceroglandular) are left untreated and the bacteria spread through the
bloodstream to the lungs.
The 100-meter sprint requires intense, all-out effort for a very short time. Cells break
down carbohydrates and fats to provide energy, then store that energy in the form of
adenosine triphosphate (ATP). ATP is often termed the cell's energy currency.
Because the breakdown of stored ATP releases energy almost instantaneously, & it
is the primary energy source as a sprinter explodes out of the blocks. However,
muscles only store enough ATP for two to three seconds of maximal power
output. To rapidly replenish ATP, muscle cells break down another high-energy
compound called creatine phosphate (CP). This is called the ATP-CP or phosphagen
energy system, sometimes referred to as the alactic anaerobic system because it
does not require oxygen.
Surgery, if feasible, is the only curative therapy. If the tumor has metastasized
(most commonly, to the liver) and is considered incurable, there are some promising
treatment modalities, such as radiolabeled octreotide or the radiopharmaceutical for
arresting the growth of the tumors and prolonging survival in patients with liver
metastases.
Complete breech is when both of the baby's knees are bent and his feet and bottom
are closest to the birth canal.
Incomplete breech is when one of the baby's knees is bent and his foot and bottom
are closest to the birth canal.
Frank breech is when the baby's legs are folded flat up against his head and his
bottom is closest to the birth canal.
Contraction Relaxation
Endothelins ANP
Angiotensin II Dopamine
Vasopressin PGE2
Nor-epinephrine cAMP Q
Thromboxane A2
PGF2
LT-C4 and D4
Histamine
278. Increased suicidal tendency is associated with alteration in the brain levels of?
a. Dopamine
b. Serotonin
c. GABA
d. Noradrenaline
Explanation :
Decreased serotonin and its metabolite levels in brain is associated with depression
whereas elevated levels of 5 – HT are associated with anxiety.
279. Vaccine given in adolescence is ?
a. Hep - B
b. Td/Tdap
c. MMR
d. Yellow fever
Explanation :
280. Commonest post operative complication of intubation is?
a. Aspiration pneumonitis
b. Sore throat
c. Vocal cord granuloma
d. Tracheal stenosis
Explanation :
Most common post – operative complication of intubation is sorethroat which usually
subsides in 2-3 days without any specific treatment.
281. Hormone responsible for diabetogenic state in pregnancy is?
a. B- HCG
b. AFP
c. HPL
d. Maternal progesterone
Explanation :
Human placental lactogen a/k as human chorionic somatotropin that is secreted by
syncytiotrophoblast. It is mainly responsible for diabetogenic state in pregnancy, as it
antagonises the action of insulin. It leads to maternal lipolysis and increased levels of
circulating free fatty acids thus provides a source of energy for maternal metabolism
and fetal nutrition. It is a potent angiogenic hormone therefore,may play important in
fetal vasculature formation.
282. All of the following are the causes of metabolic encephalopathy except?
a. Carbon monoxide poisoning
b. Electrolytic imbalance
c. Diabetic neuropathy
d. Increased catecholamines levels
Explanation :
Metabolic encephalopathy (toxic metabolic encephalopathy) is a broad category
that describes abnormalities of the water, electrolytes, vitamins, and other chemicals
that adversely affect brain function. Other causes of metabolic encephalopathy
include carbon monoxide or cyanide poisoning, which prevents hemoglobin from
carrying oxygen in the bloodstream and results in tissue anoxia. Toxic metabolic
encephalopathy can also include medication side effects or drug ingestions affecting
the chemical transmitters in the brain. Called neurotransmitters, dopamine,
GABA, serotonin, acetylcholine, and glutamine help nerve endings pass electrical
signals between brain cells.
283. Adult Polycystic disease of kidney may have cysts in all of the following
organs except?
a. Liver
b. Lungs
c. Pancreas
d. Spleen
Explanation :
Hodgkin’s disease
Hodgkin’s disease
a. Candida tropicalis
b. Pityriasis rosea
c. Tuberous sclerosis
d. Roseola infantum
Explanation :
Acute symmetrical self limiting papulosquamatous disorder. Seen specifically on the
trunk in children and young adults common during spring and fall.1st manifestation is
1 2-6 cm annular lesion – herald patch with fine scaling, fine cigarette paper like
collarette of scales usually over the chest. Typical truncal presentation is chirstmas
tree pattern /fir tree pattern. Hnging curtain sign is seen. Associated with HHV 7. As
mentioned previously, it is self limiting and hence no treatment is required. If pruritic
, antihistamines and tropical steroids and UV – B phototheraphy.
289. Chorionic component in early stages of pregnancy is composed of?
a. Cytotrophoblast +syncytotrophoblast + somatic mesoderm + fetal
blood vessels
b. Cytotrophoblast + syncytotrophoblast
c. Syncytotrophoblast + somatic mesoderm + fetal blood vessels
d. None of the above
Explanation :
Placenta
The placenta is a fetomaternal organ. The fetal portion of the placenta is
known as the villous chorion. The maternal portion is known as the decidua
basalis. The two portions are held together by anchoring villi that are
anchored to the decidua basalis by the cytotrophoblastic shell.
Function
The placental membrane separates maternal blood from fetal blood. The fetal part
of the placenta is known as the chorion. The maternal component of the placenta is
known as the decidua basalis.
Although the placental membrane is often referred to as the placental barrier, many
substances, both helpful and harmful, can cross it to affect the developing embryo.
Structure
· Secondary chorionic villi have a core of loose connective tissue, which grows
into the primary villi about the third week of development.
· Tertiary chorionic villi contain embryonic blood vessels that develop from
mesenchymal cells in the loose connective tissue core. These blood vessels
connect up with vessels that develop in the chorion and connecting stalk and begin
to circulate embryonic blood about the third week of development.
290. Earliest markers for down syndrome is?
a. Unconjugated estriol
b. Ultra sound markers ( nuchal translucency)
c. AFP
d. Amniocentesis
Explanation :
ultrasound (also called sonography) is to confirm the gestational age of the fetus. It
can also pick up problems of a serious medical nature, such as blockage of the small
intestine or heart defects. Knowing these defects exist as early as possible will
benefit the treatment of the child after birth.
Studies showed that there was a strong association between the size of a collection
of fluid at the nack of the fetal neck, called nuchal transluceny, and the risk of Down
syndrome. Early attempts to use a measurement of the nuchal area were limited by
a wide variety in measurement techniques. Recently, standardized guidelines on
measuring nuchal translucency along with specific training and certifications have
been instituted, making this ultrasound measurement useful as part of the first
trimester screen.
Explanation :
Impulse: This is a sudden and irresistible force compelling a person to the conscious
performance of some action without motive or fore-thought. A sane person is
capable of controlling an impulse, but an insane person may do things on impulse.
Types:
a. KleptomaniaQ- An irresistible desire to steal articles of little value.
b. Pyromania- an irresistible desire to set things to fire.
c. Mutilomania- An irresistible desire to mutilate animals.
d. Dipsomania- an irresistible desire for alcoholic drinks at periodic intervals.
e. Sexual impulses- including sexual perversions.
f. Suicidal and homicidal impulses.
g. Twilight state is seen in visual hallucinations, Epilepsy, Head injury, Hysteria,
Punch-drunkenness.
The most common cause of shock in the surgical or trauma patient is loss of
circulating volume from hemorrhage. The clinical and physiologic response to
hemorrhage has been classified according to the magnitude of volume loss.
a. Loss of up to 15% of the circulating volume (700 to 750 mL for a 70-kg patient)
may produce little in terms of obvious symptoms, while
b. Loss of up to 30% of the circulating volume (1.5 L) may result in mild tachycardia,
tachypnea, and anxiety.
c. Hypotension, marked tachycardia [i.e., pulse >110 to 120 beats per minute (bpm)],
and confusion may not be evident until more than 30% of the blood volume has been
lost;
d. Loss of 40% of circulating volume (2 L) is immediately life threatening, and
generally requires operative control of bleeding.
CHEMICAL PEEL
1). Chemical peel produces a controlled partial thickness injury to the skin.
2). Following the insult to the skin, a wound healing process ensues that can
regenerate epidermis from surrounding epithelium and adnexal structures, decrease
solar elastosis, and replace and reorient the new dermal connective tissue.
3). The result is an improved clinical appearance of the skin, with fewer rhytids and
decreased pigmentary dyschromia.
Peeling Agents
1). Glycolic acid.
2). Jessner’s solution.
3). Pyruvic acid.
4). Resorcinol.
5). Salicylic acid.
6). Trichloracetic acid.
7). Deep chemical peels.
8). Combinations peels: Salicylic acid +Trichloracetic acid.
300. WHO defines adolescent age between ?
a. 10 – 19 yrs
b. 10 – 14 yrs
c. 10 – 25 yrs
d. 9 – 14 yrs
Explanation :
Adolescence
Adolescence is defined as the period from the onset of puberty to the termination of
physical growth and attainment of final adult hight, i.e. adolescence is considered as
a period of transition from childhood to adulthood. Adolescence is usually the period
10 to 20 yr.
Puberty
Puberty is the biological process in which a child become adult, i.e. biological
process which occurs during transition from childhood to adulthood. Changes include
appearance of secondary sex characteristics and development of reproductive
capacity.
WHO/UN definitions
a) Pressure
b) Traction
c) Aberrant
d) Atavistic
Atavistic epiphysis: A bone that is independent phylogenetically but is now fused with
another bone. These types of fused bones are called atavistic, e.g., the coracoid process of
the scapula, which has been fused in humans.
a) Axillary
b) Radial
c) Median
d) Ulnar
“Pointing finger” deformity is caused due to injury to the median nerve in the mid-forearm by
paralysis of flexor digitorum superficialis.
The carpal tunnel contains the median nerve, the four tendons of the flexor digitorum
profundus, the four tendons of the flexor digitorum superficialis, and the flexor
pollicis longus (FPL) tendon. Carpal tunnel syndrome (CTS) is the result of median nerve
compression within the carpal tunnel.
a) Pulmonary trunk
b) Thymus
c) Oesophagus
d) Arch of aorta
a) Bronchial artery
b) Abdominal aorta
c) Right gastric artery and inferior phrenic artery
d) Left gastric artery and inferior phrenic artery
The abdominal esophagus is supplied by the left gastric artery (a branch of the coeliac
trunk) and left inferior phrenic artery.
a) Esophagus
b) Stomach
c) Duodenum
d) Liver
Secondary ossification centers develop at later times. Ossification of the distal end of the
femur, to form the condyles and epicondyles, begins shortly before birth.
10. A 44 year old male came in to your clinic post hernia repair and wants you to
evaluate his condition. While eliciting the cremasteric reflex, you notice that the
scrotum did not ascend on the side of the surgery, suggesting an iatrogenic injury
to the:-
a) Sacral plexus
b) Femoral branch of genitofemoral nerve
c) Genital branch of genitofemoral nerve
d) Femoral nerve
11. In the cremasteric reflex, it is noticed that the scrotum did not ascend on the side
of the surgery, suggesting an iatrogenic injury to the Genital branch of
genitofemoral nerve.
The cremasteric reflex is a superficial reflex found in human males that is elicited when
the inner part of the thigh is stroked. Stroking of the skin causes the cremaster muscle to
contract and pull up the ipsilateral testicle toward the inguinal canal.
a) Superficial peroneal
b) Deep peroneal
c) Sural
d) Posterior tibial
The superficial peroneal nerve innervates the peroneus longus and peroneus brevis muscles
and the skin over the antero-lateral aspect of the leg along with the greater part of the
dorsum of the foot (with the exception of the first web space, which is innervated by the
deep peroneal nerve).
a) Superficial peroneal
b) Deep peroneal
c) Common peroneal
d) Tibial
The common fibular nerve is most commonly damaged by a fracture of the fibula, or the
use of a tight plaster cast. The anatomical course of the common fibular nerve causes it to
wrap round the neck of the fibular, and so any fractures of the fibular neck can cause nerve
palsy.
a) Nasion
b) Glabella
c) Pterion
d) Lambda
15. A 43-year-old male came in with complaints of headache, vomiting, blurring of
vision, and drowsiness. His son claimed that he had a fall a few hours ago. If you
suspect an increased intracranial pressure as the root cause of this, which cranial
nerve do you first check since it is commonly injured in cases of raised ICP?
a) 3rd
b) 4th
c) 5th
d) 6th
a)3rd
b)4th
c)5th
d) 6th
The cornea is one of the most sensitive tissues of the body, as it is densely innervated with
sensory nerve fibres via the ophthalmic division of the trigeminal nerve by way of 70–80
long ciliary nerves.
a) Mandibular
b) Facial
c) Glossopharyngeal
d) Hypoglossal
Along with the other three muscles of mastication (temporalis, medial pterygoid, and lateral
pterygoid), the masseter is innervated by the anterior division of the mandibular division
(V3) of the trigeminal nerve.
18. Deepest nucleus of cerebellum:-
a) Dentate
b) Emboliform
c) Fastigial
d) Globose
The four deep cerebellar nuclei are in the center of the cerebellum, embedded in the white
matter. Dentate, Emboliform, Globose, Fastigial from lateral to medial location in the
cerebellar white matter.
An easy mnemonic device to remember their names and positions relative to the midline is
the phrase "Don't Eat Greasy Food", where each letter indicates the lateral to medial
location in the cerebellar white matter.
1. A 25 year old young male was travelling from abroad to India. At the airport he was
diagnosed with some disease condition for which India has quarantine. Disease
usually not seen in a country but brought from abroad is called as?
a) Endemic
b) Epidemic
c) Zoonotic
d) Exotic
Exotic: Disease usually not seen in a country but brought from abroad.
Zoonotic: A disease which is transmitted to man from animals e.g. Rabies, plague.
Epidemic: when disease has exceeded its normal expectancy.
Endemic: Constant or continuous presence of a disease in a population.
We do fellow up, useful parameters obtained e.g. Incidence, relative risk, Attributable risk,
population attributable risk.
Case control - odds ratio
Cross sectional - prevalence
Ecological study Group characteristic
All screening & diagnostic tests comes under secondary level of prevention.
Hence, Screening for hypertension is a secondary level of prevention. It aims to reduce
the prevalence of a disease by shortening its duration.
All other options given protects against development of MI. Hence, primary level of
prevention.
a) Health promotion
b) Specific protection
c) Early diagnosis and treatment
d) Disability limitation and rehabilitation
a) Normal saline
b) Distilled water
c) Dextrose
d) Colloids
a) X-ray positive
b) Sputum positive
c) Mantoux positive
d) History of cough
Case of tuberculosis:
A definite case of TB (defined below) or one in which a health worker (clinician or other
medical practitioner) has diagnosed TB and has decided to treat the patient with a full
course of TB treatment. Note. Any person given treatment for TB should be recorded as a
case.
TB is diagnosed by sputum smear
CBNAAT
ALC, LPA, LED - FM
CXR, TST
All test used in revised national tuberculosis control programme.
7. Cyclopropagative life cycle is defined as the disease agent which undergoes
cyclical change, and multiplies in the body of the arthropod. This type of life cycle
is seen in which of the following disease?
a) Malaria
b) Plague
c) Filarial
d) Yellow fever
a) Leishmania donovani
b) Leishmania tropica
c) Leishmania braziliensis
d) None of the above
Perinatal mortality rate: its death occur during 28 weeks to till day post delivery
11. Number of live births per 1000 women in the reproductive age reproductive age
group in a year refers
General fertility rate is defined as number of live births per thousand women in the age
group (15-49 years) in a given year.
Total fertility rate → Total children born to a women or couple during there life span
Gross reproduction rate → Total number of girl children born to a women in entire
reproduction
Net reproduction → Total girl children produced during entire reproduction life span taking
into a amount mortality
a) Pulses
b) Soyabean
c) Milk
d) Egg
Egg – 94
Milk – 84
Soya bean – 73
13. Mode of disposal of sewage ground water:
The activated sludge process is still the most widely used biological treatment
process for reducing the concentration of organic pollutants in sewage. It's aerobic
oxidation process used as a secondary treatment. Heart of this method its Aeration tank.
Well-established design standards based on empirical data have evolved over the years.
a) 0.2 mg/L
b) 0.3 mg/L
c) 0.4 mg/L
d) 0.5 mg/L
a) Umbrella scheme includes Pradhan Mantri Jan Aarogya Yojna and subcenters.
b) ↑Benefit to 40 % of poor and vulnerable population
c) Covers only tertiary health care centres
d) Cashless ,paperless service by only government hospitals.
Ayushman Bharat
a)Median
b) Mode
c) Standard deviation
d)Mean
The mode is the value that appears most frequently in a data set.
Median is middle value
The mean is the average or the most common value in a collection of numbers.
Standard deviation measure of dispersion.
18. A 25 year old female is using vaginal sponge as a barrier contraceptive during
sexual intercourse. Which of the following Spermicide is used in contraceptive
TODAY?
a)NET-EN
b)DMPA
c)Nonoxynol - 9
d)Nor-ethinosterol
a)Silica fibers.
b)Carbon particles
c)Sugarcane fibers
d)Cotton
a) Yellow bag
b) Red bag
c) Blue bag
d) Black bag
Yellow bag
Human anatomical waste
Animal waste
Soiled waste ( cotton /cloths)
Chemical waste
Discarded medicine ( expired medicine, cytotoxic drug)
Liquid chemical waste
Micro biological, lab waste
Red category → Solid contaminated (plastic / rubber)
White category → wasted sharp (needle, Sx blades, and scalpels)
Blue category → glasses, metallic body implants
21. A 24 year old reproductive age female visited to the rural PHC for IUCD insertion
to prevent pregnancy. Which of the following is the most common side effect of
IUCD insertion?
a) Ectopic pregnancy
b) Pelvic infection
c) Pain
d) Bleeding
a) 42 hours/week
b) 48 hours/week
c) 54 hours/week
d) 60 hours/week
The Factories Act. 1948 is a social legislation which has been enacted for occupational
safety, health and welfare of workers at work places.
Its 9 hours a day & maximum 48 hours. week.
60 hours , week then paid extra.
23. A 36 year old male presented to the OPD with difficulty in breathing while walking,
back pain, unable perform physical activity and excessive laziness. On
examination his body weight is 106 kgs. Which of the following is height
independent obesity index?
a) Corpulence index
b) Quetlet index
c) Brocca index
d) Ponderal index
a) Diet program
b) Child and maternal health
c) National vector borne disease control programme
d) Contraception
Family planning & contraception - "Cafeteria approach" has new name "GATHER"
G: Grade the couples
A: Ask them their contraception requirement
T: Tell them about available methods
H: Help them choose best method for them
E: Explain
R: Returns
25. A 42 year old obese female presented to the rural PHC. Which of the following is
the appropriate statistical test to find out obesity as a significant risk factor for
breast cancer?
a) Incidence rate
b) Case fatality rate
c) Prevalence rate
d) Secondary attack rate
Confounding factor Any factor associated with both exposure, outcome and has an
independent effect in causation of outcome is confounder.
BEST METHOD TO REMOVE CONFOUNDING:-
Case Fatality :
Refers to proportion of fatal cases among those who have the disease.
Provides an index of the deadliness of a particular disease within a specific population.
CFR = ( Number of deaths due to disease / Number of cases of disease ) x 100
29. An epidemic is the rapid spread of disease to a large number of people in a given
population within a short period of time. A well of contaminated water resulting in
an epidemic of acute watery diarrhoea is a typical example for which of the
following?
a)Non union
b)Ischemic necrosis
c)Malunion
d)Pulmonary complications
The given clinical scenario depicts the Fracture neck of femur (intertrochanteric)
2. 8 years old boy sustained fracture to the hand while playing with his sister. In
which of the following condition, the Triangular relation of the elbow is
maintained?
a) Fracture ulna
b) Anterior dislocation of elbow
c) Posterior dislocation of elbow
d) Supracondylar fracture of humerus
Triangular relation of elbow includes medial epicondyle, lateral epicondyle and tip of
olecranon.
The relationship is destroyed by dislocations of elbow
It is maintained in supracondylar fracture of Humerus (occurs above elbow joint)
3. Which of the following statement about tubercular osteomyelitis is not true?
a) Diaphysis
b) Metaphysis
c) Epiphysis
d) Growth plate
a) Hip Joint
b) Knee joint
c) Distal interphalangeal joint
d) Metacarpophalangeal joint
Enrichment media -
a) Pseudomonas aeruginosa
b) Clostridium perfringenes
c) Bacillus stearothermophilus
d) Salmonella typhi
There are three indicators that may be used to detect the efficacy of the
autoclave process:
(1) Physical: pressure and temperature recording devices,
(2) Chemical: indicators that change color after being exposed to specific
temperatures, such as temperature sensitive tape. The color change upon exposure
to the given temperature, and
(3) Biological: Bacillus stearothermophilus spores are used, due to its
resistance to heat, for the testing that measures the biological performance of
the autoclave process.
a) T.Pertunae
b) T. Pallidum
c) T. Caraetum
d) Hemophilus
Yaws is an infectious disease caused by a spiral-shaped bacterium (spirochete) known
as Treponema pertenue. Yaws is usually transmitted by direct contact with the infected skin
sores of affected individuals.
a) Pigs
b) Rats
c) Sand fly
d) Ticks
The deer tick (Ixodes dammini or scapularis) is the most common vector that transmits
babesiosis. In extremely rare cases, babesiosis may be transmitted following a blood
transfusion with blood that is contaminated with the microorganism.
a) Pneumococcus
b) Gonococcus
c) Staphylococcus
d) All of the above
The quellung reaction, also called the Neufeld reaction, is a biochemical reaction in which
antibodies bind to the bacterial capsule of Streptococcus pneumoniae, Klebsiella
pneumoniae, Neisseria meningitidis, Bacillus anthracis, Haemophilus influenzae,
Escherichia coli, and Salmonella.
a) Bacteria
b) Fungus
c) Protozoa
d) Virus
a) Measles
b) Mumps
c) Rubella
d) Rabies
Subacute sclerosing pan encephalitis (SSPE) is a very rare, but fatal disease of the central
nervous system that results from a measles virus infection acquired earlier in life. SSPE
generally develops 7 to 10 years after a person has measles, even though the person seems
to have fully recovered from the illness.
a) HPV 16&18
b) HPV 6&11
c) HPV 31&32
d) HPV 1&2
Fungal stains
10% KOH
LPCB (lactophenol cotton blue)
GMS (Gomori Methenamine Silver)
Calcofluor white (Fluorescent stain)
Mucicarmine stain (used for capsule of cryptococcus neoformans )
Indian ink staining ( used for capsule of cryptococcus neoformans )
H&E (hematoxylin/eosin stain)
PAS -Periodic acid Schiff.
a) Secondary syphilis
b) Lupus vulgaris
c) Acanthosis nigricans
d) Scrofuloderma
Acanthosis nigricans is a skin condition that causes a dark discoloration in body folds
and creases. It typically affects the armpits, groin and neck. Acanthosis nigricans is a skin
condition characterized by areas of dark, velvety discoloration in body folds and creases.
The affected skin can become thickened
In the given image:
a) Ceftriaxone
b) Erythromycin
c) Tetracycline
d) Benzathine penicillin
a) Hair
b) Nail
c) Scalp
d) All of the above
a) 320nm
b) 360nm
c) 400nm
d) 480nm
A traditional Wood lamp is a low-output mercury arc covered by a Wood filter (barium silicate
and 9% nickel oxide) and emits wavelength 320–450 nm (peak 365 nm). The lamp was
invented in 1903 by a Baltimore physicist, Robert W. Wood.
a) Sweat glands
b) Pilosebaceous glands
c) Sebaceous glands
d) Apocrine glands
Acne vulgaris is a common chronic skin disease involving blockage and/or inflammation
of pilosebaceous units (hair follicles and their accompanying sebaceous gland). Acne
can present as noninflammatory lesions, inflammatory lesions, or a mixture of both, affecting
mostly the face but also the back and chest.
OPTHAL JUNE 2018
1. Cornea derives its nutrition chiefly from:
a) Aqueous humour
b) Mucous layer
c) Corneal vessels
d) Perilimbal vessels
Unlike most tissues in the body, the cornea contains no blood vessels to nourish or protect it
against infection. Instead, the cornea receives its nourishment from the tears and aqueous
humor (a fluid in the anterior portion of the eye) that fills the chamber behind it.
a) Limbus
b) Fovea centralis
c) Macula lutea
d) Blind spot
The fovea centralis is located in the center of the macula lutea, a small, flat spot located
exactly in the center of the posterior portion of the retina. As the fovea is responsible for
high-acuity vision it is densely saturated with cone photoreceptors.
a) Field of Vision
b) Pattern of retina
c) Malignant melanoma
d) Squint
a) Erythromycin
b) Ganciclovir
c) Clindamycin
d) Azithromycin
Azithromycin is a macrolide antibiotic and is the drug of choice for trachoma. Plasma
concentrations are low, but tissue concentrations are higher, giving it value in treating
intracellular organisms. It has a long tissue half-life. A single dose is recommended.
a) Vernal conjunctivitis
b) Atopic conjunctivitis
c) Gonococcal conjunctivitis
d) Chlamydial conjunctivitis
Herbert's pits are a characteristic and pathognomonic finding of trachoma, and are caused
by Chlamydia trachomatis.
a) Night blindness
b) Conjunctival xerosis
c) Bitot's spots
d) Corneal xerosis
World Health Organization Re- Classification of Xerophthalmia Signs
a) Painless
b) Affects cornea
c) Sudden loss of vision
d) Bilateral in majority of cases
Mooren's ulcer:
Affects cornea
Painful condition
Slow loss of vision
Mostly unilateral
Peripheral corneal ulceration
Clear cut overhanging edge seems to be melting from periphery
9. Argon laser trabeculoplasty is done in:
The most common type of laser surgery performed for open-angle glaucoma is called
Argon Laser Trabeculoplasty (ALT). The objective of the surgery is to help fluids drain out of
the eye, reducing intra-ocular pressure that can cause damage to the optic nerve and loss of
vision.
a) Normal
b) Dilated
c) Constricted
d) Vertically oval
In acute iritis (inflammation of iris) iris is irritated and pupil constricts due to
stimulus of irritation.
Vertically oval, mid-dilated, non-reacting pupil is seen in acute angle closure glaucoma.
a) Neovascular glaucoma
b) Central retinal artery occlusion
c) Vitreous hemorrhage
d) None of the above
a) Type I pneumocyte
b) Type II pneumocyte
c) Sertoli cell
d) Leydig cell
The classic stretch reflex involves the stretch receptor in the skeletal muscle spindle and
its afferent Ia fiber in the posterior nerve root, which directly or indirectly excites the alpha
motor neuron in the spinal cord.
3. Resting membrane potential of nerve fibre is close to isoelectric potential of:
a) Potassium ions
b) Sodium ions
c) Chlorideions
d) Magnesium ions
a) Thyroxine
b) Parathormone
c) Insulin
d) Calcitonin
a) Stomach
b) Duodenum
c) Jejunum
d) Ileum
Enterokinase, also called Enteropeptidase, proteolytic enzyme (q.v.), secreted from the
duodenal mucosa, that changes the inactive pancreatic secretion trypsinogen into trypsin,
one of the enzymes that digest proteins.
6. Which of the following is the best stimulus for the release of vasopressin?
a) SA node
b) AV node
c) Endocardium
d) Epicardium
The electrical signal starts in a group of cells at the top of your heart called the
sinoatrial (SA) node. The signal then travels down through your heart, triggering first your
two atria and then your two ventricles.
a) Leydig cells
b) Sertoli cells
c) Interstitial cells
d) Peg cells
Inhibin is a protein secreted by the Sertoli cells in men and by the granulosa cells in
women. It inhibits the synthesis and release of the follicle-stimulating hormone in the
pituitary gland and reduces the hypothalamic LH - releasing hormone content.
a) Apneustic centre
b) Pneumotaxic centre
c) Area Postrema
d) Hypothalamus
Area postrema is the center responsible for controlling vomiting it is located outside the
blood brain barrier. and part of the chemoreceptor trigger zone, hence chemicals in blood
can directly stimulate this area and induce vomiting.
a) Tubular cells
b) Macula densa
c) JG cells
d) All of the above
Renin is an enzyme secreted by the juxtaglomerular cells of the kidney. It interacts with
aldosterone in a negative-feedback loop.
11. The rate at which the stomach empties into the duodenum depends on the type of
food ingested. Food rich in carbohydrates leaves the stomach in a few hours.
Protein-rich food leaves more slowly, and emptying is slowest after a meal
containing fat. The rate of emptying also depends on the osmotic pressure of the
material entering the duodenum. Hyperosmolality of the duodenal contents is
sensed by "duodenal osmoreceptors" that initiate a decrease in gastric emptying,
which is probably neural in origin. Which factor is responsible for increasing
gastric motility?
Chemical → Gastrin
Mechanical → Gastric Distension.
12. The adrenal cortex is derived from mesoderm and begins posteromedial to the
urogenital ridge. The adrenal medulla is derived from neural crest cells. These
neural crest cells originate from the dorsal aorta. Clusters of chromaffin cells
become the distinct medulla after birth. The adrenal gland first appears at 28-30
days post-conception. Adrenal medulla secretes?
a) Sex hormones
b) Thyroid stimulating hormones
c) Epinephrine
d) Glucocorticoid
The adrenal medulla, the inner part of an adrenal gland, controls hormones that initiate the
flight or fight response. The main hormones secreted by the adrenal medulla include
epinephrine (adrenaline) and norepinephrine (noradrenaline), which have similar
functions.
a) Incised wound
b) Abrasions
c) Lacerated wound
d) Contusion
A laceration is the result of a shearing force and causes deeper skin tearing, through the
epidermis and sometimes through the dermis and subcutaneous tissues.
a) A police officer
b) Magistrate
c) Panchayat officer
d) District attorney
a) 1 year
b) 10 years
c) 20 years
d) No limit
In India: No time limit - for exhumation [176 (3) CRPC]
Exhumation :
a) Maggot formation
b) Putrefaction
c) Greenish discoloration of right iliac fossa
d) Mummification
a) Skull
b) Pelvis
c) Femur
d) Ribs
a) Brachycephalic
b) Mesaticephalic
c) Dolicocephalic
d) All of the above
Dolichocephalic → Aryans, Negroids Cephalic index (70. 75)
Mesaticephalic → Europeans, Chinese -Cephalic index (75 - 80)
Brachy cephalic → Mongoloids Cephalic index (80 - 85)
a) Hanging
b) Poisoning
c) Strangulation
d) Drowning
A test that traditionally was used is the Gettler Chloride Test, where the amounts of
chloride on each side of the heart were compared to determine if the person drowned
in fresh or salt water.
Gettler test not useful in-
Dry drowning
Hydrocution
Patent foramen oval
Putrefaction.
a) Arsenic poisoning
b) Phosphorous poisoning
c) Mercury poisoning
d) All of the above
Arsenic poisoning :
A - Anemia / Aldrich mee's line / Arsenophagist (Tolerate up to 300mg)
R - Rain drop pigmentation / Reinsch test / Red velvety mucosa
S - Sub endocardial hemorrhages / Sensory neuropathy
E - Eruptions
N - NAA (neutron activation analysis)
I - Imbibition of arsenic (Arsenic imbibed from surrounding soils after death)
C - Cumulative poison / Cholera like symptom / Chelation for Rx
9. A 40-year-old man has committed suicide by consuming some chemical he was
able to procure from the paint-making factory he works at. The doctor notices the
garlicky odor from the tissues and decided to preserve the hair for further
investigation. Poisoning with which one of the following requires preservation
of the hair?
a) Arsenic
b) Manganese
c) Phosphorous
d) Alcohol
a) Endothelial injury
b) Stasis of blood flow
c) Hypercoagulability
d) Platelet thrombus
a) A, B, C
b) A, B, D
c) A, C, D
d) B, C, D
Hemophilia :
Christmas disease :
a) Renal carcinoma
b) Pancreatic carcinoma
c) Prostatic carcinoma
d) Hepatic carcinoma
Serum alpha fetoprotein (AFP) is the most widely used tumor marker in detecting patients
with hepatocellular carcinoma. When elevated, the AFP is 75-91% specific, and values
greater than 400 ng/mL are generally considered diagnostic of HCC in the proper clinical
context, including appropriate radiologic findings.
a) Tetanus
b) Rabies
c) Polio
d) AIDS
Negri bodies are seen in the Pyramidal neurons within Ammon's horn of the Hippocampus
and Purkinje cells in the cerebellar cortex on postmortem of the rabid person.
a) Granuloma
b) Langerhan's Giant cells
c) Lymphocytosis
d) Caseation
Ingestion of mycobacterium tuberculosis inside the lung will cause stimulation in the no. of
lymphocytes, that will cause lymphocytosis [earliest feature of TB]
First cell in the lung which will be affected by myco. TB infection is alveolar macrophage
Transmission of myco. TB - cough & sneeze (droplet infection)
8. Which of the following is not a component of Jones criteria for diagnosing acute
rheumatic fever?
a) Pancarditis
b) Migratory polyrthritis
c) Erythema nodosum
d) Sydenham chorea
Jones criteria :
Major criteria
Migratory polyarthritis
Carditis
Erythema marginatum
Syndenham chorea
Subcutaneous nodules
Minor criteria
Arthralgia
Fever
First degree heart block
Elevated inflammatory markers (ESR, CRP).
a) Liver
b) Caecum
c) Transverse colon
d) Sigmoid colon
Amebiasis may involve any part of the bowel, but the cecum and the ascending colon are
predilection sites. The most common extra intestinal manifestation is liver abscess
caused by hematogenous spread from the GI tract.
a) FSGS
b) MGN
c) RPGN
d) All of the above
a) Neurogenic cyst
b) Thymoma
c) Neurofibroma
d) Sarcoma
13. A 9 Yr old male child came with complaints of an Upper respiratory tract infection.
On General examination, he was found to be anemic. The Pediatrician explained to
his parents about the causes of Anemia in children. Which of the following is the
most sensitive and specific initial laboratory test tc diagnose iron deficiency?
In iron deficiency anemia, the earliest parameter that is effected is → decrease in stored iron
[ferritin] → measured by serum ferritin levels.
Hence, estimation of Serum ferritin levels is most sensitive and specific test to diagnose iron
deficiency anemia in the given options.
a) Good-pasture syndrome
b) Membranous GN
c) Minimal change disease
d) Focal segmental glomerulosclerosis
Primary causes of nephrotic syndrome are diseases that affect only the kidneys. The most
common primary cause of nephrotic syndrome in adults is a disease called focal segmental
glomerulosclerosis (FSGS).
a) Myoglobin
b) Cardiac specific Troponins
c) CK-MB
d) All of the above
Cardiac troponins T and I are the preferred markers for myocardial injury as they have the
highest sensitivities and specificities for the diagnosis of acute myocardial infarction.
17. A known case of peptic ulcer disease presented with repeated episodes of
vomiting’s. The condition is associated with hypochloremic hypokalemic
metabolic acidotic state. What is the most likely part involved in the given
condition?
a) Antrum
b) Duodenum
c) Pylorus
d) Fundus
Peptic ulcer disease , Carcinoma of the stomach → can result in Gastric outlet
obstruction
Peptic ulcer disease, MC site of obstruction - Duodenum [1st part]
Carcinoma of stomach, MC site of obstruction - Pyloric region of stomach
Most common cause of Gastric Outlet Obstruction in adult patient - Carcinoma of the
stomach.
Endoscopy with duodenal biopsy showing villous atrophy is the current gold standard
for diagnosing celiac disease
19. Most common site of intracranial metastasis is from primary carcinoma of:-
a) Breast
b) Lungs
c) Stomach
d) Testes
Some differences are seen in the types of primary malignancies responsible for the brain
metastasis in the two genders. Lung cancer is the most common source of brain metastasis
in males, whereas breast cancer is the most common source in females.
a) Arteriitis
b) Polycystic kidney disease
c) Pyelonephritis
d) Glomerulonephritis
HTN
Hematuria
Oliguria
21. Deficiency of which vitamin causes subacute combined degeneration of spinal
cord?
a) Vitamin B1
b) Vitamin B6
c) Vitamin B2
d) Vitamin B12
Subacute combined degeneration of the spinal cord can be caused by inadequate oral
intake of vitamin B12, poor absorption of vitamin B12, or by the use of medications such as
metformin, proton pump inhibitors, and nitrous oxide.
a) Autograft
b) Allograft
c) Isograft
d) Xenograft
23. Patients with Polycystic disease of kidney presents with all except:-
a) Hematuria
b) Hypertension
c) Renal failure
d) Erythrocytosis
Hematuria
Due to activation of Renin - Angiotensin - Aldosterone System (RAAS) the patient will
have HYPERTENSION.
Over period of time, renal mass will be reduced so patient will develop Renal
failure, as the renal mass is reduced. There will be reduction in the secretion of
Erythropoietin, leading to anemia NOT Erythrocytosis.
Adult PCKD has Autosomal dominant inheritance
Child PCKD has Autosomal Recessive inheritance
a) Breach of dura
b) Onset of headache is usually 12-72 hours following procedure
c) Commonly occipito - frontal in location
d) Headache is relieved in sitting standing position
Pathophysiology → CSF leak → ICP ↓es → Traction on nerve fibres originating from
pia mater
Site → Occipital > Frontal > Retrorbital
Duration → 7-10 days
Pathognomic sign → Change in posture aggravates headache
Predisposing conditions -
Dura cutting needle (spinal needle)
Most commonly seen with 22G Quincke needle.
Wide bore needle Multiple attempts.
Pregnancy (In pregnancy dura mater is very fragile venous engorgement fragility
increases chances of PDPH is more).
Note- Early ambulation never predispose to PDPH ***
a) Morphine
b) Thiopentone sodium
c) Ketamine
d) Halothane
a) Methoxyflurane
b) Sevoflurane
c) Desflurane
d) Isoflurane
Sevoflurane is less soluble than isoflurane and is very pleasant to breathe, which makes it
an excellent choice for inhalational induction of anesthesia, particularly in children. The
respiratory and cardiovascular effects of sevoflurane are very similar to isoflurane, but
sevoflurane does not cause 'coronary steal'.
a) 3:1
b) 15:2
c) 30:2
d) 100:8
Hyperkalemia: Potassium gets released from stored blood after day 5 from the date
of collection.
Metabolic Alkalosis: Citrate in blood is converted to bicarbonate in liver ,which
causes metabolic alkalosis.
Hypocalcemia: Calcium in patient's blood binds to the chelating agent contained
within the stored blood.
a) Heroin
b) Opium
c) Alcohol
d) Cocaine
COCAINE -
Intoxication-
a) Autism
b) Automatism
c) Association Disturbances
d) Ambivalence
The fundamental symptoms, which are virtually present through all the course of the
disorder, are also known as the famous Bleuler's four A's
4. A girl normal language milestones spends her time seeing hel own hands and
does not interact with others. Likely diagnosis is:-
a) ADHD
b) Autism
c) Asperger's syndrome
d) Rett's syndrome
a) Digoxin
b) Propranolol
c) Adenosine
d) Dilitiazem
Adenosine is the first-line medical treatment for the termination of paroxysmal SVT.
a) Antidepressants
b) Anxiolytics
c) Antipsychotics
d) Anti-epileptics
a) Climetidine
b) Omeprazole
c) Misoprostol
d) Aluminium hydroxide
6. The drug aspirin has been used for secondary prevention of ischemic heart
disease because, it:-
Aspirin produces its effects through inhibition of thromboxane A2 (TXA2) production, while
P2Y12 antagonists attenuate the secondary responses to ADP released by activated
platelets. The anti-platelet effects of aspirin and a P2Y12 antagonist are often considered to
be separately additive.
a) Insulin
b) Desmopressin
c) Diazoxide
d) Chlorpropamide
In patients with central DI, desmopressin is the drug of choice. A synthetic analogue of
antidiuretic hormone (ADH), desmopressin is available in subcutaneous, IV, intranasal, and
oral preparations.
a) Lubiprostone
b) Streptomycin
c) Ebastine
d) Cisapride
Erythromycin is inhibitor of microsomal enzyme - CYP3A4
CAT drugs increase QT interval (metabolized by CYP3A4).
Cisapride
Astemizole
Terfenadine
If administered with Erythromycin, metabolism is inhibited, drug concentration rises.
At high plasma concentration, these drugs block K* channel in the heart leading to QT
prolongation →Torsades de pointes.
a) Melphalan
b) Cyclophosphamide
c) Busulfan
d) 5-Fluorouracil
a) Isoniazid
b) Hydralazine
c) Phenytoin
d) Procainamide
a) Potency
b) Safety
c) Efficacy
d) All of these
The therapeutic index (TI, also referred to as therapeutic ratio) is a quantitative
measurement of the relative safety of a drug. It is a comparison of the amount of a
therapeutic agent that causes the therapeutic effect to the amount that causes toxicity.
a) Essential tremors
b) AV block
c) Angina pectoris
d) Migraine prophylaxis
13. A 45 year old male was admitted to the ICU after being diagnosed with Sub
arachnoid hemorrhage secondary to a ruptured aneurysm. The patient is a known
case of stage 2 hypertension and was taking Ramipril for the same. Which of the
following calcium channel blockers can be used in this case, which might help
relieve the cerebral vasospasm?
a) Nimodipine
b) Felodipine
c) Amlodipine
d) Nitrendipine
Nimodipine (DHPs)- Blocks Ca2+ channel only in cerebral blood vessels which leads
to Cerebral vasodilation is it used in subarachnoid hemorrhage (SAH).
Dry cough
Increased potassium levels in the blood (hyperkalemia)
Fatigue
Dizziness from blood pressure going too low
Headaches
Loss of taste
a) Cyclooxygenase inhibitor
b) 5-lipooxygenase inhibitor
c) 5-phosphodiesterase inhibitor
d) Prevents mast cell degranulation
16. In the management of diabetes mellitus, lactic acidosis is caused by which of the
following?
a) Tolbutamide
b) Glipizide
c) Pioglitazone
d) Metformin
A specific class of oral diabetes medication, called biguanides, can cause a buildup of
lactic acid levels. Metformin (Glucophage) is one of these drugs. It's used to treat
diabetes and may also be prescribed for other conditions, such as renal insufficiency.
a) Right atrium
b) Pulmonary trunk
c) Aortic knuckle
d) Superior vena cava
The aortic knob or knuckle refers to the frontal chest x-ray appearance of the distal
aortic arch as it curves posterolaterally to continue as the descending thoracic aorta. It
appears as a laterally-projecting bulge, as the medial aspect of the aorta cannot be seen
separate from the mediastinum.
2. Floating water lily sign is seen in:
a) Lung hydatid
b) Bronchial adenoma
c) Lung abscess
d) Aspergilloma
The water-lily sign, also known as the camalote sign, is seen in hydatid infections when
there is detachment of the endocyst membrane which results in floating membranes within
the pericyst that mimic the appearance of a water lily.
a) Crohn's disease
b) TB of ileocecal region
c) Idiopathic hypertrophic pyloric stenosis
d) All of the above
The gastrointestinal string sign (also known as the string sign of Kantor) refers
to the string-like appearance of a contrast-filled bowel loop caused by its severe
narrowing.
Originally used to describe the reversible narrowing caused by spasms in Crohn
disease, it is now used for any severe narrowing of the bowel lumen, including that
seen in hypertrophic pyloric stenosis, gastrointestinal tuberculosis, carcinoid
tumor and colon cancer.
a) More radio-opaque
b) Radiolucent
c) Less radio-opaque
d) Not seen at all
In chronic osteomyelitis there is dead dense bone in center called sequestrum , which
is more radio opaque.
a) Acute leukemia
b) Thalassemia
c) Multiple myeloma
d) Ca prostate
a) Maturation
b) Thelarche
c) Pubarche
d) Menarche
In most girls, breast budding is the first visible sign of sexual maturation –“Thelarche”
means "the beginning of breast development, followed closely by the initiation of the
growth spurt. Shortly thereafter, pubic and axillary hair appears. Menarche generally occurs
about 2 years after onset of breast development and when growth in height slows after
reaching its peak.
a) Copies Cross
b) Bridge with blocks
c) Copies Circle
d) Copies triangle
a) Pitting edema
b) Weight loss
c) Flag sign
d) Muscle wasting
Kwashiorkor Marasmus
Because of protein deficiency Because of both protein and calorie deficiency
Mc in 6 months to 3 yrs. of age Mc in infancy
Subcutaneous fat preserved Not preserved
Edema present Edema Absent
Ribs are not very prominent Ribs are very prominent
Lethargic Alert and irritable
Mild or absent muscle wasting Sever muscle wasting
Poor appetite Voracious feeder
Fatty liver is common Fatty liver is Uncommon
Skin and hair changes are more common Skin and hair changes are Less common
a) Erythroblastosis fetalis
b) Congenital hyperbilirubinemia
c) Biliary atresia
d) Physiological jaundice of newborn
Erythroblastosis fetalis is hemolytic anemia in the fetus (or neonate, as
erythroblastosis neonatorum) caused by transplacental transmission of maternal
antibodies to fetal red blood cells. The disorder usually results from incompatibility
between maternal and fetal blood groups, often Rho(D) antigens.
Destruction of the red blood cells (hemolysis) can be rapid in a fetus. As a result, the
fetus will not receive enough oxygen, which may lead to anemia, other illnesses, or even
death.
As hemolysis continues, the fetus will rapidly attempt to produce more red blood cells.
However, these cells new red blood cells are often immature and are unable to function
fully.
Because the body produces red blood cells in the liver and spleen, this overproduction
can sometimes cause these organs to enlarge.
When a newborn has this condition, it is known as hemolytic disease of the newborn.
As the immature red blood cells continue to break down, bilirubin, which is a by-product
of the breakdown of red blood cells, builds up. The excess amounts of bilirubin
circulating in the newborn’s body will lead to jaundice, where the skin and eye whites of
the infant turn yellow.
a) PDA
b) ASD
c) Atrioventricular septal defect
d) VSD
a) Retinoblastoma
b) Leukemia
c) Wilms tumor
d) Neuroblastoma
a) Wilms tumor
b) Neuroblastoma
c) Adrenal gland tumors
d) Granulosa cell tumor of ovary
10. A 2 year old male child was brought by the parents to the pediatric department
with loose stools from last 3 days. On examination she has signs of severe
dehydration. The intern on duty was asked by the JR to calculate the volume of
fluid to be given in the first 3 hrs if her weight is 10 kg.
a) 1000 ml
b) 100 ml
c) 200 ml
d) 500 ml
In a 2 year old child with severe dehydration, fluid given over 1st 30 minutes is 30mL/kg,
over next 2.5 hrs is 70mL/kg: So fluid given over 3 hrs is 100mL/kg. In this case, as weight of
child is 10 kg, amount of fluid to be given is 10x100=1000mL.
11. A 1-year-old male child present to the hospital with a fever associated with neck
stiffness. You are suspecting meningitis. Which of the following is the most
common cause of meningitis in this age group?
a) Klebsiella
b) Streptococcus pneumoniae
c) Staphylococcus aureus
d) Mycobacterium tuberculosis
IN INDIA :
IN WORLD :
a) Western blot
b) ELISA
c) PCR
d) All
a) 4x2x1
b) 3x2x1
c) 4x3x1
d) 5x4x2
UTERUS :
a) Accidental pregnancies
b) Population
c) Fertility rate
d) Abortions
a) Micronized progesterone
b) Norgesterone
c) Levonorgesterol
d) DMPA
Only the progestin levonorgestrel has been studied for use as emergency contraception. It
is marketed as Plan B.
4. Most likely to be associated with vaginal pH of 4:-
a) Atrophic vaginitis
b) Trichomanas vaginatis
c) Candidal vaginitis
d) Gardenella vaginitis
Bacterial Vaginosis :
a) Premature menopause
b) Sheehan syndrome
c) PCOD
d) Turner's syndrome
a) Mullerian agenesis
b) Turner syndrome
c) Sheehan's syndrome
d) Turner's syndrome
Mullerian agenesis/hypoplasia
Imperforate hymen
Vaginal agenesis
Cervico vaginal atresia
Endocrinological causes
Hypogonadotropic hypogonadism
Primary ovarian failure
Congenital adrenal hyperplasia
Constitutional delay
Hypothyroidism
Genetic causes
Turner syndrome
Pure gonadal agenesis
Androgen insensitivity syndrome
8. A 23 year old woman presents with irregular cycles. Her cycles come at intervals
of 35-45 days, last for 3 to 4 days with mild to moderate flow, no passage of clots
and no dysmenorrhea. Which of the following describes this condition?
a) Amenorrhoea
b) Oligomenorrhoea
c) Polymenorrhoea
d) Menorrhagia
Ovarian tumors:
a) Pelvic mass
b) Amenorrhoea
c) Infertility
d) Menstrual irregularity
Symptoms of fibroid:
a) HSV
b) HBV
c) HPV
d) HIV
Condyloma acuminate (genital warts ) → HPV 6,11
Cancer cervix HPV 16, 18
Prevention → Gardasil vaccine prevents HPV types 6,11,16,18 (Quadrivalent)
RX → Podophyllin resin + Cryotherapy
12. Ferning pattern of drying cervical mucus suggests the action of:
a) Estrogen
b) Progesterone
c) Prolactin
d) 17-ketosteroids
Ferning occurs due to the presence of sodium chloride in mucus under estrogen effect.
When high levels of estrogen are present, just before ovulation, the cervical mucus forms
fern-like patterns due to crystallization of sodium chloride on mucus fibers. This pattern is
known as arborization or 'ferning'.
a) Amikacin
b) Metronidazole
c) Chloramphenicol
d) Ampicillin
Infections with the sexually transmitted protozoan Trichomonas vaginalis are usually treated
with metronidazole, a 5-nitroimidazole drug derived from the antibiotic azomycin.
Metronidazole treatment is generally efficient in eliminating T. vaginalis infection and has a
low risk of serious side effects.
14. A liquid media used to suppress commensal bacteria while allowing the pathogen
to remain viable and grow. Which of the following is an enrichment media?
Enrichment media
a) Competitive
b) Non-competitive
c) Irreversible
d) Uncompetitive
a) Zinc
b) Copper
c) Molybdenum
d) Selenium
a) Creatinine
b) Uric acid
c) Glutamine
d) Urea
a) Methionine
b) Cysteine
c) Homocysteine
d) Leucine
HOMOCYSTINURIA :
Homocysteine accumulation lead to stroke, atherosclerosis, MI, Pulmonary embolism.
Homocysteine binds with the endothelium of the blood vessels and activates signaling
pathways which leads to the release of pro-inflammatory molecules. This could result in the
following:
Extensive atheroma formation at a young age affecting many arteries but not the
coronary arteries
Intravascular thrombosis
a) G-6-P dehydrogenase
b) G-6-phosphatase
c) Aldolase
d) Glucokinase
Muscles lack the enzyme Glucose-6-phosphatase, that's why muscles cannot maintain
blood glucose levels.
Liver cells have this enzyme. And therefore liver can only maintain blood glucose.
a) Amino acids
b) Glycerol
c) Acetoacetate
d) Lactic acid
a) Glycogen synthetase
b) Glucose-6 phosphatase
c) Amylo-1, 4-1, 6-transglycolase
d) Glycogen Phosphorylase
Classic galactosemia (type 1) - the most common and severe type, caused by mutations in
the GALT gene, and characterized by a complete deficiency of an enzyme called galactose-
1-phosphate uridyl transferase (GALT).
10. Essential fatty acid:
a) Citric acid
b) Linoleic acid
c) Stearic acid
d) Palmitic acid
a) Glucose
b) Lactose
c) Fatty acid
d) Glycogen
a) Apo E
b) Apo C1
c) Apo A2
d) Apo A1
APOPROTEIN - PROTEIN PRESENT IN LIPOPROTEIN
Lipoprotein Protein
Chylomicron APO B48
Chylomicron remnant APO B48 + APO E
VLDL APO B100
IDL APO B100 APO E
LDL APO B 100 + APO
HDL APO A, C, E
a) Carbohydrate
b) Triglyceride
c) Proteins
d) Fibres
The major energy reserve in the body is found stored as fat (triglyceride) in the adipose
tissue.
POST-TRANSITIONAL CHANGE
They facilitate and favour the interactions on the polypeptide surfaces to finally give the
specific conformation of a protein.
a) Hydroxylation
b) Chelation
c) Transamination
d) Carboxylation
Vitamin K :
a) Epistaxis
b) Headache
c) Nasal obs
d) Cervical lymphadenopathy
2. A 14 year old Male presented with history of fever since 2 days, unable to swallow
the food with muffled voice. On examination it is noted right tonsil is shifted to
midline. What is the diagnosis:
a) Quincy
b) Acute tonsillitis
c) Para pharyngeal abscess
d) Acute retropharyngeal abscess
If tonsil is enlarged / hypertrophied and reaching midline and soft palate is moved on other
side it is a case of quincy but, if tonsil itself is pushed to the midline it is a case of
parapharyngeal abscess.
a) Hyoid bone
b) Tracheal rings
c) Thyroid cartilage
d) Cricoid cartilage
a) Tinnitus
b) Vertigo
c) Fullness of ear
d) Ear Pain
Meniere's disease also called endolymphatic hydrops is a disorder of the inner ear where
the endolymphatic system is distended with endolymph.
It is characterized by
Vertigo
Sensorineural hearing loss
Tinnitus
Aural fullness
5. 34 yrs old female presents with tinnitus, vertigo and fullness in ear. Likely
Diagnosis:
a) Meniere's Disease
b) Otosclerosis
c) Ototoxicity
d) Noise Induced Hearing Loss
Meniere's disease also called endolymphatic hydrops is a disorder of the inner ear where
the endolymphatic system is distended with endolymph.
It is characterized by
Vertigo
Sensorineural hearing loss
Tinnitus
Aural fullness
a) Labyrinthitis
b) Presbycusis
c) Meniere's disease
d) Vestibular Schwannoma
Presbycusis :
a) Supraglottic
b) Subglottic
c) Anterior Glottis
d) Posterior Glottis
Laryngeal web
a) Posterior cricoarytenoid
b) Lateral cricoarytenoid
c) Transverse arytenoid
d) Thyroarytenoid
a) Aspiration
b) Stridor
c) Bowed vocal cords
d) Loss of pitch
Voice is weak and pitch cannot be raised with decreased ability to sing.
Anaesthesia of the larynx on one side may pass unnoticed or cause occasional
aspiration.
Presence of both paralysis and bilateral anaesthesia. Causes inhalation of food and
pharyngeal secretions gives rise to cough and choking fits. Voice is weak and husky.
–
Stridor is not seen In superior laryngeal nerve palsy is seen in B/L recurrent
laryngeal nerve palsy.
Bell's palsy is the most common cause of facial paralysis, although its exact cause is
unknown. Generally, Bell's palsy affects only one side of the face; however, in rare cases, it
can affect both sides.
a) Nasopharyngeal polyp
b) Nasopharyngeal carcinoma
c) Cleft palate
d) Nasopharyngeal angiofibroma
Rhinolalia clausa-
a) Actinomycetes
b) Aspergillus niger
c) Mucor
d) Candida albicans
Aspergillus fumigatus
Candida albicans
14. A 3 yrs old child presents with c/o fever, barking cough and stridor for 2days, what
is the diagnosis
a) Acute tonsillitis
b) Acute Epiglottitis
c) Croup
d) Adenoiditis
a) Cribriform plate
b) Petrous temporal bone
c) Parietal bone
d) Tympanic membrane
CSF otorrhea - CSF coming from ear due to trauma to petrous temporal bone.
CSF Rhinorrhea - CSF watery discharge from nose
a) Acantholysis
b) Hemoglobinuria
c) Ketosis
d) Lateral sinus thrombosis
The Tobey–Ayer test is used for lateral sinus thrombosis by monitoring cerebrospinal
fluid pressure during a lumbar puncture.
19. Laryngocele arises from ?
a) Subglottis
b) Anterior commissure
c) Saccule
d) True glottis
a) Atrophic rhinitis
b) Vasomotor rhinitis
c) Antrachonal polyp
d) Allergic rhinitis
Young's operation is a surgery designed for the treatment of atrophic rhinitis, first
described by Austen Young in 1967.
a) Rhinoscleroma
b) Rhinosporidiosis
c) Rhinophyma
d) Rhinitis
The Mikulicz cell is a large macrophage with clear cytoplasm that contains the bacilli; this
cell is specific to the lesions in rhinoscleroma.
23. MC malignancy of maxillary antrum:
a) Muco-epidermoid Carcinoma
b) Adeno-cystic Carcinoma
c) Adeno Carcinoma
d) Squamous cell Carcinoma
M/c malignancy of paranasal sinus and nasal cavity → Squamous cell Carcinoma
1. A 44 year old female presenting to med opd with complaints of fatigue, weakness,
and shortness of breath with minimal activity. Her friends and family have told her
she appears pale, and combined with her recent symptoms she has decided to get
checked out. She also states that she has noticed her hair and fingernails
becoming extremely thin and brittle, causing even more concern. The patient first
started noticing these symptoms a few months ago and they have been getting
progressively worse. Upon initial assessment, her mucosal membranes and
conjunctivae are pale. Considering the above scenario the most sensitive and
specific test to diagnose this particular type of deficiency anemia is:
a. Hyper-segmented neutrophils
b. Microcytosis precedes hypochromia
c. MCHC < 50%
d. Commonest cause of anemia in India
Hyper segmented neutrophils (more than 5 lobes of neutrophils) is seen in vitamin B12
deficiency.
Iron deficiency anemia is the most common cause of anemia in India.
It is characterized by microcytic hypochromic peripheral smear.
MCHC < 50% Low MCHC values occur if you have anemia due to iron deficiency.
Since there is mitral stenosis, more time for blood to enter LV.
Therefore, increased transient time, so increase duration of murmur
***Intensity of S1 sound can be loud or soft. If sound is soft it tells us about calcified
mitral valves but not of severity.
5. Malignant hypertension can lead to all of the following EXCEPT
a. Hypertensive retinopathy
b. Respiratory failure
c. Renal failure
d. Stroke
Emergencies include
Atrial fibrillation is the most common sustained arrhythmia, increases with age, and
presents with a wide spectrum of symptoms and severity Paroxysmal, persistent, and
permanent forms require very individualized approaches to management.
a. Myoglobin
b. Cardiac specific Troponins
c. CK-MB
d. All of the above
Cardiac troponin I and T is the preferred markers of myocardial injury. It is highly specific to
cardiac tissue and accurately diagnoses myocardial infarction with a history of ischemic pain
or ECG changes reflecting ischaemia.
8. On ECG, ST segment elevation is seen in all of the following conditions EXCEPT:
a. Uremia
b. Infection
c. Cardiovascular disease
d. Malnutrition
Cardiovascular disease is the leading cause of death in dialysis patients and sudden death
(SD) represents a significant proportion of overall mortality in both hemodialysis (HD) and
peritoneal dialysis (PD) patients.
10. True statements regarding Gluten sensitive enteropathy is/ are -
11. In gastric outlet obstruction in a peptic ulcer patient, the site of obstruction is
most likely to be:
a. Antrum
b. Duodenum
c. Pylorus
d. Fundus
Gastric ulcer leads to gastric outlet obstruction causing pyloric stenosis showing HOUR
GLASS STOMACH.
a. Sporozoite
b. Schizont
c. Hypnozoite
d. Gametocyte
14. A 39 year old male presents with fever, cough and anorexia. He is sputum positive
for AFB. In order to consider this as a case of XDR-TB, which of the following
should he be resistant to ?
a. Myopathy
b. Peripheral neuropathy
c. Optic atrophy
d. Myelopathy
Peripheral neuropathy
Optic atrophy
Myelopathy But Not Myopathy
SACD spinal cord Sub-acute combine degeneration of spinal cord).
17. You are an intern, posted in the department of Internal Medicine ward. You are
seeing the reports of a 47-year-old female who had history of AGE & having
neurological manifestations. Investigations revealed significant elevation of
protein & other normal values in CSF. This characteristic dissociation in CSF is a
feature of ?
a. Multiple sclerosis
b. GB syndrome
c. TB meningitis
d. Multiple myeloma
18. A 34-year-old woman presented with complaints of sudden onset headache which
she described as a characteristic 'thunderclap headache'. She has no significant
medical history. On examination, her pupils were dilated bilaterally. These findings
are consistent with the diagnosis of?
a. Meningitis
b. Brain stem encephalitis
c. Acute ischemia of midbrain
d. Acute aneurismal hemorrhage
Thunderclap headache (TCH) is a term initially introduced to describe the apoplectic onset
of a headache that begins suddenly, without warning, and peaks with severe intensity
within seconds.
The most likely diagnosis in the given clinical scenario isAmyotrophic lateral sclerosis
(ALS).
Amyotrophic lateral sclerosis (ALS)is the Most common form of motor neuron disease.
The diagnosis is based on clinical criteria in which there is bothUpper and lower motor
neuron signs with progressive disease.
Spasticity is the velocity-dependent increase in muscle tone due to the exaggeration
of stretch reflex. It is only one of the several components of the upper motor neuron
syndrome.
a. FVC is high
b. FEV1 is high
c. FEV1/FVC is high
d. All of the above
The FEV1/FVC ratio can be higher than normal, This is because it is easy for a person with
a restricted lung (e.g fibrosis) to breathe out quickly, because of the high elastic recoil of the
stiff lungs.
21. A 49 year old man, presented with 10 days history of progressive worsening
dyspnea on exertion, A chest radiograph confirmed a large left pleural effusion.
Which of the following feature and finding you won't see in this case ?
a. Vitamin B1
b. Vitamin B6
c. Vitamin B2
d. Vitamin B12
a. Obesity
b. Kyphoscoliosis
c. Pleural effusion
d. Asthma
COPD
Asthma
Bronchiectasis
α1-antitrypsin deficiency.
24. In which of the following conditions there is an increase in lung diffusion
capacity?
a. Emphysema
b. Idiopathic pulmonary fibrosis
c. Alveolar haemorrhage
d. Pulmonary oedema
DLCO(diffusing capacity of the lungs for carbon monoxide )is increased in alveolar
hemorrhage whereas it decreases in pulmonary edema, idiopathic pulmonary fibrosis,
emphysema.
25. In the given ECG widespread concave ST elevation and PR depression is present
throughout the precordial (V2-6) and limb leads (I, II, aVL, aVF).There is reciprocal
ST depression and PR elevation in aVR. What is the diagnosis ?
a. Breast
b. Lungs
c. Stomach
d. Testes
a. DIC
b. Hypothermia
c. Hypercalcemia
d. Thrombocytopenia
a. Brachial artery
b.Popliteal artery
c. Radial artery
d. Femoral artery
a. X-ray
b. Nucleotide scan
c. VMA excretion
d. Clinical examination
IOC for extra adrenal pheochromocytoma is MRI > MIBG scan(aka-nucleotide scan)
a. Acute appendicitis
b. Acute pancreatitis
c. Acute cholecystitis
d. Acute hepatitis
In acute pancreatitis, bleeding into fascial planes can produce bluish discoloration of
the flanks (known as Grey turner's sign) or umbilicus (Cullen sign)
a. Lipoma
b. Glomus tumor
c. Leiomyosarcoma
d. Leioblastoma
a. Colocolic
b. Ileoileal
c. Ileocolic
d. Ileal
a. Mallory-Weiss syndrome
b. Esophageal carcinoma
c. Achalasia cardia
d. Boerhaave's syndrome
a. Herniotomy
b. Herniorrhaphy
c. Truss
d. Hernioplasty
a. Osteosarcoma
b. Prostatic carcinoma
c. Paget's disease
d. Hyperparathyroidism
Elevated serum levels of acid phosphatase are seen in patients with carcinoma of the
prostate that has extended beyond the prostatic capsule. Patients with prostatic
carcinoma still confined within the capsule usually have a normal serum acid phosphatase
level.
12. A 61 year old patient undergoes TURP, after 3 days patient develops altered
sensitiveness and drowsiness. Most probable diagnosis is:-
a. Hypernatremia
b. Hyponatremia
c. Stroke
d. Meningitis due to spinal anesthesia
13. 54 year old female known case of multinodular goitre now complaints of sudden
increase in size of swelling and mild pain over the swelling. There is no history of
dysphagia, dyspnea. The patient complaints of mild hoarseness of voice. No
lymph nodes palpable. Malignancy in this patient is most often:-
a. Follicular carcinoma
b. Papillary carcinoma
c. Anaplastic carcinoma
d. Medullary carcinoma
Follicular ca: -
a. Infection
b. Dermatitis
c. Neoplasia
d. Hypopigmentation
a. Metastasis
b. Adenoid cystic carcinoma
c. Warthin's tumor
d. Pleomorphic tumor
The most common tumor of the parotid gland is the pleomorphic adenoma,
which represents about 60% of all parotid neoplasms.
MC neoplasm of salivary gland in children: Hemangioma
MC neoplasm of salivary gland: Pleomorphic adenoma
MC malignant tumor of salivary gland: Mucoepidermoid carcinoma
MC malignant tumor of salivary gland in children: Mucoepidermoid carcinoma
MC malignant tumor of minor salivary glands: Adenoid cystic carcinoma
Best diagnostic modality for parotid swelling: FNAC
Open incisional biopsy is contraindicated due to tumor cell implantation and formation
of parotid fistula
Best imaging investigation for salivary gland neoplasms: MRI
A branchial cleft cyst (BCC) commonly presents as a solitary, painless mass in the neck of a
child or young adult. They are most commonly located along the anterior border and the
upper third of the sternocleidomastoid muscle in the anterior triangle of the neck.
a. Intracerebral hematoma
b. Acute subdural hemorrhage
c. Chronic subdural hemorrhage
d. Epidural hemorrhage
THE LUCID interval following head trauma and unconsciousness is described classically
in epidural hematomas. The historic emphasis placed on the lucid interval in cases of
extradural hematoma has made this one of the best-remembered signs of the syndrome.
18. Marjolin's ulcer is:-
A Marjolin ulcer is a rare and aggressive type of skin cancer that grows from burns, scars,
or poorly healing wounds. It grows slowly, but over time it can spread to other parts of your
body, including your brain, liver, lungs, or kidneys. Most Marjolin ulcers are cancerous
and form squamous cancer cells in the upper layers of your skin. Some Marjolin ulcers
may also form as basal cell tumors which form in deeper layers of your skin.
a. Repair to tendons
b. Repair of skin cover
c. Repair of nerves
d. All
a. Hair follicle
b. Sweat gland
c. Subcutaneous tissue
d. Epidermis
The most common type of staph infection is the boil, a pocket of pus that develops in
a hair follicle or oil gland. The skin over the infected area usually becomes red and
swollen.
21. Pendred syndrome is due to a defect in:-
a. Chromosome 7p
b. Chromosome 7q
c. Chromosome 8p
d. Chromosome 8q
a. Papilloma
b. Infected sebaceous cyst of scalp
c. Cylindroma
d. Squamous cell carcinoma
a. Mile's procedure
b. Frey's procedure
c. Scanlon's modification surgery
d. Anderson-Hynes operation
Oliguria is defined as urinary output less than 400 ml per day or less than 20 ml per
hour and is one of the earliest signs of impaired renal function.
FMGE DEC 2018
ANATOMY
1. A 55-year-old male is brought to the Neurology OPD by his wife. She complains
that something is wrong with her husband and they were referred to this center by
a local doctor. The doctor's note read:- Patient unable to repeat words spoken,
though is able to understand what is told & nods head accordingly. When asked to
name people and write, the patient found difficulty. Referred higher center for CT
brain. You being the intern on duty, would inform the CMO it could be a lesion in
which area of the brain?
a) Arcuate fasciculus
b) Anterior commissure
c) Corpus callosum
d) Fornix
Arcuate fasciculus connects Wernicke's area present in the posterior superior temporal
gyrus of the dominant hemisphere (sensory area of speech) to Broca's speech area
present in the prefrontal & premotor facial region of the cerebral cortex to integrate
language understanding & skilled motor function.
Damage to Wernicke's area = Wernicke's aphasia (lack meaning)
Damage to Broca's area = motor aphasia (vocal system couldn't emit words)
Damage to arcuate fasciculus = conduction aphasia
a) Corpus callosum
b) Thalamus
c) Pituitary gland
d) Fornix
3. The parotid gland is a paired salivary gland contributing serous secretions. Its
duct opens at the level of the:.
4. Trismus is due to spasm of which of the following muscle (FMGE Dec 2018)
a) Buccinator
b) Lateral pterygoid
c) Medial pterygoid
d) Mentalis
5. Which branch of facial nerve supplies muscles of lower lip (FMGE Dec 2018)
a) Temporal
b) Cervical
c) Buccal
d) Mandibular
The marginal mandibular branch of the facial nerve passes forward beneath the platysma
and depressor anguli oris, supplying the muscles of the lower lip and chin, and
communicating with the mental branch of the inferior alveolar nerve.
a) Manubrium
b) Clavicle
c) First rib
d) Second rib
a) Serratus anterior
b) Trapezius
c) Subscapularis
d) Supraspinatus
Retractors of scapula:
Trapezius
Rhomboid major
Rhomboid minor
Protraction of scapula:
Pectoralis minor
Serratus anterior (damage causes winging of scapula)
a) Trapezius
b) Serratus anterior
c) Latissimus dorsi
d) Pectroralis major
The most common cause of scapular winging is paralysis of the serratus anterior muscle
due to the injury of the long thoracic nerve.
10. During the embryological development, the midgut rotates a total of _____in the
intrauterine life.
11. Vasa brevia is the other name for which of the following arteries carried by the
gastrosplenic ligament?
Splenic artery (branch of celiac trunk) supply greater curvature of stomach by it following
branches:
Short gastric arteries which are called as vasa brevia. (carried by Splenogastric
ligament)
Long gastric artery called left gastroepiploic artery.
Jejunal vasa recta (long arteries) & ileal vasa recta (short) are straight arteries coming
from arcades in mesentery of jejunum & ileum.
12. In a healthy adult female, the tail of pancreas lies in lienorenal ligament derived
from peritoneum. This is another name for which of the following ligaments?
a) Hapato-gastric
b) Hepato- duodenal
c) Gastro- splenic
d) Spleno- renal
Splenorenal ligament, also known as the lienorenal ligament is a peritoneal ligament. It
represents the dorsal most part of dorsal mesentery and forms part of the lateral border
of the lesser sac. It is continuous with the gastrosplenic and phrenicocolic ligaments
Tail of pancreas lies in Spleno renal ligament.
Gastro splenic short gastro and left gastro-omental vessels.
External hemorrhoids:
Hemorrhoids occurring at anal verge (distal boundary of anal canal below pectinate
line) Normally have low tendency to bleed on straining at stool.
Bleeding comes from inferior rectal vein.
Internal hemorrhoids:
Occur inside the rectum (above pectinate line) have high tendency to bleed.
Bleeding from "superior rectal vein"
Prolapsed hemorrhoids are internal hemorrhoids that pass outside anal canal & form
lump, which may undergo thrombosis & become painful.
a) Extensor digitorum
b) Peroneus longus
c) Tibialis anterior
d) Tibialis posterior
a) Gracilis
b) Biceps femoris
c) Adductor longus
d) Adductor magnus
Biceps femoris is a muscle of the posterior compartment of the thigh, and lies in the
posterolateral aspect. It arises proximally by two 'heads', termed the 'long head'
(superficial) and the 'short head' (deep).
Nerve Supply
1. The ICF volume represents the fluid contained within the body's cells. This volume
cannot be measured directly but is calculated as the difference between the
measured TBW and the measured ECF volume. Potassium provides the osmotic
skeleton for the ICF and because water is freely diffusible into and out of the cell,
changes in the tonicity of the ECF are rapidly reflected by similar changes in ICF
tonicity. Which of the following has the least concentration in intracellular fluid?
a) Calcium
b) Magnesium
c) Potassium
d) Protein
Calcium is kept very LOW in cytoplasm, hence intracellular fluid doesn't contain much
Ca2+
Calcium is mostly stored in organelles inside cell
E.g. In sarcoplasmic reticulum of muscle cell etc.
a) Tone
b) Spasticity
c) Rigidity
d) Paratonia
Muscle tone is defined as the continuous and passive-partial contraction of the muscle or
the muscle's resistance to passive stretch during the resting state.
Spasticity:
Condition in which certain muscles are continuously contracted
This contraction causes stiffness/tightness of muscle & interfere with movement
Occurs in lesions of upper motor neurons.
Rigidity:
increase in muscle tone leading to resistance to passive movement throughout range of
motion
E.g. Cogwheel rigidity in parkinsonism
Paratonia:
Inability to relax muscles during muscle tone assessment.
3. The formation of the active endopeptidases from their inactive precursors occurs
only when they have reached their site of action, secondary to the action of the
brush border hydrolase. enterokinase. The powerful protein-splitting enzymes of
the pancreatic juice are secreted as inactive proenzymes. Enterokinase is an
activator of?
a) Trypsinogen
b) Trypsin
c) Chymotrypsin
d) Antitrypsin
When first synthesized in pancreatic cells, the proteolytic digestive enzymes like
trypsinogen, chymotrypinogen & pro-carboxypeptia are in inactive forms( to prevent auto
proteolysis of pancreas)
After secreted into intestine, they are activated:
Trypsinogen is activated by enterokinase (secreted by intestinal mucosa)
Trypsinogen → can also be autocatalytically activated by trypsin
Chymotrypsinogen is activated by trypsin to form chymotrypsin.
4. Which structure connects the Broca's area and Wernicke area?
a) Arcuate fasciculus
b) Anterior commissure
c) Corpus callosum
d) Fornix
5. Calcium binding proteins are proteins that participate in calcium cell signaling
pathways by binding to Ca2+, which plays an important role in many cellular
processes. Calcium-binding proteins have specific domains that bind to Ca2+ and
are known to be heterogeneous. One of the functions of calcium-binding proteins
is to regulate the amount of free (unbound) Ca2+ in the cytosol of the cell. The
cellular regulation of calcium is known as Calcium homeostasis. Which of the
following is not a calcium-binding protein?
a) Calbindin
b) Calmodulin
c) Troponin
d) Clathrin
On inside of cell membrane beneath coated pits, latticework of fibrillar protein is present
called clathrin
Helps in pinocytosis
Calbindin
Present in brush border of intestinal epithelial cells & transports Cat2into cell cytoplasm
Amount of calcium absorption is directly proportional to quantity of calcium-binding
proteins
Troponin
a) Enterochromaffin cell
b) Parietal cell
c) Oxyntic cell
d) Chief cell
“Enterochromaffin-like (ECL) cells” are a population of cells that are found in the gastric pits
of the stomach luminal epithelium and secrete histamine. In response to gastrin released
by neighbouring G-cells, secreted histamine from ECL cells acts on parietal cells to stimulate
the release of gastric acid.
a) Amygdala
b) Thalamus
c) Hippocampus
d) Ventral tegmental area
Reward center contains ventral tegmental area connected with nucleus accumbens
(through dopamine NT)
Previously major reward centers are thought to be located along course of "medial
forebrain bundle"
Substances like glucose, AA, Na+2, H,0 is absorbed maximum from upper part of PCT
As proximal tubule epithelial cells have:
Large number of mitochondria to support powerful active transport processes (highly
metabolic)
Have brush borders & provide extensive surface area for absorption
As glucose is reabsorbed by secondary active transport, it is absorbed almost
all by proximal convoluted tubule.
a) 24
b) 48
c) 72
d) 96
a) Golgi tendon
b) Muscle spindle
c) Unmyelinated C fibres
d) Dorsal Column
The Inverse Myotatic Reflex Involves Sensors of Muscle Force in the Tendon. Stretch
receptors called Golgi tendon organs are found within the collagen fibers of tendons and
within joint capsules. They are generally located in series with the muscle rather than the
parallel arrangement of the intrafusal muscle fibers.
a) Mitochondria
b) Golgi complex
c) SER
d) RER
The three organelles that contain DNA are the nucleus, mitochondria and
chloroplasts.
Organelles are membrane-bound subunits within a cell -- analogous to organs in
the body -- that perform specific functions.
The nucleus is the control center of the cell, and houses genetic information.
The mitochondria and chloroplasts both produce energy, in animal and plant cells,
respectively.
12. All are Glucogenic hormones except?
a) ADH
b) Glucagon
c) Thyroxine
d) Glucocorticoids
Glucogenic hormones are those which are capable of increasing blood glucose levels
1.Glucagon
2. Thyroxine
3. Glucocorticoids
4. Catecholamines,
5. Growth hormone etc.
The all-or-none law is a principle that states that the strength of a response of a nerve cell
or muscle fiber is not dependent upon the strength of the stimulus. If a stimulus is
above a certain threshold, a nerve or muscle fiber will fire.
14. Which part of thalamus is related to motor control?
a) Ventrolateral thalamus
b) Ventral posteromedial
c) Ventral posterolateral
d) Superior and inferior colliculi
Ventrolateral thalamus
Ventroanterior thalamus
Centro median thalamus
a) Calcitriol
b) Calciferol
c) Calcidiol
d) Ergocalciferol
Calcitriol is the active form of vitamin D, normally made in the kidney. It is also known
as 1,25-dihydroxycholecalciferol. It is a hormone which binds to and activates the vitamin
D receptor in the nucleus of the cell, which then increases the expression of many genes.
Calcitriol increases blood calcium (Ca2+) mainly by increasing the uptake of calcium from
the intestines.
16. Tyrosine kinase receptors are membrane-spanning proteins with large amino-
terminal extracellular domains bearing the ligand binding site, a juxtamembrane
domain, a protein kinase catalytic domain, and carboxyl-terminal tail. Which of the
hormone has 4 subunits and 2 units for tyrosine kinase receptor
binding?
a) Insulin
b) Glucagon
c) T3
d) ADH
The insulin receptor is an enzyme-linked receptor.
Insulin receptor
combination of 4 subunits
(held by disulfide linkages)
2 α subunits 2 β subunits
a) Hypersensitivity
b) Causalgia
c) Hyperalgesia
d) Allodynia
yperalgesia:
An increased sensitivity to feeling pain and an extreme response to pain. Hyperalgesia may
occur when there is damage to the nerves or chemical changes to the nerve pathways
involved in sensing pain.
a) Mitochondria
b) Cytoplasm
c) Both
d) None
Heme synthesis
Urea cycle
Gluconeogenesis
a) UAA
b) UAG
c) UGA
d) AUG
a) Tyrosine
b) Tryptophan
c) Phenyl alanine
d) Threonine
Melanin is a highly irregular heteropolymer consisting of monomeric units derived from the
enzymatic oxidation of the amino acid tyrosine. The process of melanin formation takes
place in specialized acidic organelles (melanosomes) in melanocytes.
a) Keratan sulfate
b) Dermatan sulfate
c) Hyaluronic acid
d) Chondroitin sulfate
Hyaluronic acid (hyaluronan) is a key component of the extracellular matrix and is known to
be involved in several mechanisms of the wound healing process. It has been shown to
improve and accelerate the healing process of chronic wounds.
a) CO
b) Antimycin A
c) Malonate
d) Thermogenin
Natural uncouplers:
Protein synthesis
Immune response
Cell growth & differentiation
Regulation intestinal transport & absorption of water and electrolytes
Improve intestinal epithelium lining and increase enzyme in brush border epithelium
Cell membrane transport and function
For polyribosome
More than 100 metalloenzyme require zinc example carbonic anhydrase alkaline
phosphates , lactate dehydrogenase
Deficiency lead to acrodermatitis enteropathica - skin affected around cheek elbow
knee anus and mouth
In diarrhea zinc is given along with ORS to reduce the risk , duration and severity.
7. Which is cardio protective?
a) HDL
b) LDL
c) IDL
d) VLDL
High-Density Lipoprotein :
a) Cysteine
b) Methionine
c) Arginine
d) Alanine
Cysteine is unique among coded amino acids because it contains a reactive sulph-
hydryl group. Therefore, two cysteine residues may form a cystine (disulfide link)
between various parts of the same protein or between two separate polypeptide chains.
No other amino acid given as options can form disulfide bond due to absence of -SH
group
PCR is based on three simple steps required for any DNA synthesis reaction:
(1) denaturation of the template into single strands;
(2) annealing of primers to each original strand for new strand synthesis; and
(3) extension of the new DNA strands from the primers.
10. Genetic material is transferred from one bacterial to another by all EXCEPT:
a) Conjugation
b) Transduction
c) Transformation
d) Transfection
a) Early PCT
b) Henle loop
c) Collecting duct
d) Distal convoluted tubule
a) Easily digestible
b) Increases plasma glucose
c) Has slower absorption
d) Increase glycogen deposits
GLYCEMIC INDEX -glycemic index which measures the time course of postprandial glucose
concentrations from a graph
1. Which of the following drugs is considered as first line therapy for the prophylaxis
of motion sickness in adults?
a) Promethazine
b) Ondansetron
c) Metoclopramide
d) Domperidone
As hyoscine is not given in the options, the answer is first generation Antihistaminics
Best Drug/ DOC for prophylaxis of motion sickness - Hyoscine (scopolamine).
First generation Antihistamines can cross the blood brain barrier and also have strong Anti-
cholinergic properties.
Example of a first generation Antihistamines – Promethazine.
2. Exenatide is a new drug used in diabetes mellitus. Mechanism of action of this
drug is:
a) Inhibition of DPP - 4
b) Release of insulin via acting as agonist of GLP-1 receptors
c) Inhibiting intestinal absorption of carbohydrates
d) Stimulation of PPAR-gamma
Mechanism of Action:
Exenatide is a GLP-1 receptor agonist released from the gut and acts to increase glucose-
dependent insulin secretion from pancreatic beta cells, suppress glucagon secretion,
delay gastric emptying, and reduce food intake. The binding of the drug to pancreatic GLP-1
receptors mediates these actions.
a) Stearic acid
b) Arachidonic acid
c) Linoleic acid
d) Linolenic acid
a) Erythromycin
b) Amoxicillin
c) Ceftriaxone
d) Streptomycin
Some antibiotics are known to be teratogenic and should be avoided entirely during
pregnancy. These include streptomycin and kanamycin (which may cause hearing loss)
and tetracycline (which can lead to weakening, hypoplasia, and discoloration of long bones
and teeth).
5. Which of the following is the mechanism of action of methyl dopa?
Mechanism of Action
Alpha-methyldopa is converted to methyl norepinephrine centrally to decrease the
adrenergic outflow by alpha-2 agonistic action from the central nervous system, leading
to reduced total peripheral resistance and decreased systemic blood pressure.
6. A 12 months old baby is brought to your clinic. If you have to give a cyclopegic
agent for this pediatric patient, which of the following drugs would you prescribe?
a) Atropine oral
b) Atropine eye ointment
c) Tropicamide eye drops
d) Phenylephrine eye drops
In children the tone of ciliary muscle is very high, so we have to give drug which is a strong
cycloplegic drug-Atropine.
Atropine is given in the form of eye ointment because it would then have only
localised effect in the eyes. If we give it in form of eye drops, it can be absorbed through
nasolacrimal duct, and there will be risk of hyperthermia.
7. Pilocarpine produces:-
a) Active miosis
b) Active mydriasis
c) Passive miosis
d) Passive mydriasis
Pilocarpine is a drug that acts as a muscarinic receptor agonist. It acts on a subtype of
muscarinic receptor (M3) found on the iris sphincter muscle, causing the muscle to contract -
resulting in pupil constriction (miosis). Pilocarpine also acts on the ciliary muscle and causes
it to contract.
a) Apixaban
b) Argatroban
c) Fondaparinux
d) Aspirin
The factor Xa inhibitors that are currently commercially available include rivaroxaban,
apixaban, betrixaban, and edoxaban. These drugs bind to factor Xa and prevent the
formation of thrombin by interrupting the extrinsic and intrinsic coagulation cascades.
a) Linezolide
b) Nevirapine
c) Indinavir
d) Ceftriaxone
a) Ciprofloxacin
b) Ceftriaxone
c) Cefalexin
d) Rifabutin
Important points to remember about Meningococcal Meningitis
a) 3 mg / kg once daily
b) 20 mg once a day
c) 20 mg twice day
d) 20 mg thrice day
13. Which of the following is the shortest acting muscle relaxant and is considered for
rapid sequence induction?
a) Tubocurarine
b) Succinylcholine
c) Pancuronium
d) Atracurium
MR
- Depolarizing
- Non Depolarizing
14. The drug that causes fall in elderly patients with postural hypotension is:
a) Metformin
b) Prazosin
c) Acarbose
d) Nor-adrenaline
The side effect postural hypotension AkA "First Dose Hypotension" is observed with drug
ending with zosin - Prazosin, Terazosin, Doxazosin.
These drugs cause blocking of a1 receptors, leading to vasodilation, that can Cause
Postural Hypotension.
15. A 23 year old female presented with burning pain during micturition. On
investigations, she is found to be suffering from co-infection with chlamydia and
gonococcus. Drug of choice for treatment of this female is:.
a) Azithromycin
b) Ceftriaxone
c) Fluconazole
d) Clindamycin
16. A patient was given anti-cancer therapy and developed neutropenia. Which of the
following drug can be used to prevent neutropenia in next cycle of chemotherapy?
a) Filgrastim
b) Prednisolone
c) Vitamin B12
d) Folic acid
CSFs help your body make more white blood cells. This lowers your risk for febrile
neutropenia. CSFs include Neupogen (filgrastim), Neulasta (pegfilgrastim), and Leukine
and Prokine (sargramostim). They are usually given as shots 24 hours after a chemotherapy
treatment.
↓ WBC → Neutropenia
↓ Platelets → thrombocytopenia
17. Bacillus Anthrax is:
Gram positive bacilli with spherical ends → Drumstick app seen in cl. Tetani.
Corona virus
Enveloped, carrying petal , club, crown shaped peplomer like spike giving appearance
of solar corona
120-160 nm helical symmetry
Linear positive sense single stranded RNA (Non segmented genome)
Mostly infect animals except gamma coronavirus
There are 6 types of coronavirus to infect humans
Human corona virus 229E alpha
Human corona virus NL 63 alpha
Human corona virus OC 43- beta
Human corona virus HK U1- beta
SARS - Cov- beta
MERS- beta
SARS CoV-2 (COVID-19)
a) Conjugation
b) Transduction
c) Transformation
d) Translation
Transduction is the process by which a virus transfers genetic material from one bacterium
to another. ... Later, when one of these bacteriophages infects a new host cell, this piece of
bacterial DNA may be incorporated into the genome of the new host. There are two types of
transduction: generalized and specialized.
20. Sporulation occurs in this phase of bacterial growth curve:
a) Stationary phase
b) Lag phase
c) Log phase
d) Decline phase
1.Lag phase- Adaptation phase to culture media decreased Accumulate nutrient size
2. Log Phase - Exponential increases in number
3. Stationary phase - Start of Accumulation of toxic metabolites = Dead
Live -Sporulation occur in stationary phase (S-S)
4. Decline - Complete accumulation of Toxin metabolites
21. Which of the following is the most likely cause in a case of granuloma?
a) Cat scratch disease
b) Trench fever
c) Leprosy
d) Syphilis
Cat scratch disease is the most likely cause in a case of granuloma.
Various granulomatous diseases and their causative agents.
Cat-scratch disease - Bartonella henselae (Gram-negative bacillus): The necrotic foci
enlarge and coalesce to form necrotizing granulomas called stellate micro abscesses.
EHEC
a) C1-C4 deficiency
b) C5-C9 deficiency
c) C3 deficiency
d) C2 deficiency
a) C1
b) C3 Convertase
c) lgA
d) Ag - Ab complex
The complement pathway. Complement can be activated through three pathways: classical,
lectin, and alternative. The classical pathway is activated when C1q binds to antibody
attached to antigen, activating C1r and C1s, which cleave C4 and C2.
a) Organo-Phosphosphate poisoning
b) Hydrocarbons
c) Bicarbonate
d) PCM toxicity
2. Forensic experts investigated the crime scene. what confirmatory test will be used
to prove a bloodstain?
a) Spectroscopic test
b) Kastle meyer test
c) Benzidine test
d) Orthotolidine test
3. The charred body has been recovered from the dense vegetation close to the
highway. The police were led to the spot by a sniffer dog that was tracking a 25-
year-old IT employee who had been missing since the previous evening. The team
has to collect sample to identify the victim. A sample of which tissue from a burnt
body, is best for identification?
a) Hair
b) Bone
c) Teeth
d) Blood
Hair: singed
Blood: burned
Bone: difficult to extract DNA from charred bones
Teeth: DNA can be extracted from pulp of the teeth even in a burnt body.
4. Death registration in India done within?
a) 21 days
b) 28 days
c) 30 days
d) 40 days
According to 'registration of birth & death act 1969 every birth / still birth / death to be
registered with the covered state / UT within 21 days.
Birth - 21 days
Deaths - 21 days
Marriages - 30/60/90 days
Birth registration is the responsibility of hospital
Child born to NRI couple abroad , birth registration done within 60 days of arrival
5. Estimation of age at death and determination of sex of the victim or remains are
important factors in the identification of an individual in forensic odontology.
Teeth are among the most reliable tools in the process of identification of age.
Teeth are one of the most durable parts of our body, which can withstand more
assaults than any other part of the body. This is particularly useful in the
identification of bodies in mass disasters and natural calamities. The first
scientific technique for age estimation in adults was presented by Gustafson
{1950}. It was based on longitudinal sections of teeth cut through the central area.
The technique consisted of attributing scores from 0-3 for the presence and
amount of age-related changes such as attrition, periodontal ligament retractions,
secondary dentin formations, root translucency, and root resorption. The most
reliable criteria among Gustafson's criteria is?
a) Root resorption
b) Paradentosis
c) Translucency of root
d) Attrition
Attrition
P. Paradentosis
S. Secondary dentin 2nd most reliable
R. Root resorption
T. Transparency of root most reliable
Cementum opposition
SPM DEC 2018
a) Anopheles
b) Culex
c) Aedes
d) Mansonia
Rest parallel to water surface (No Rest at an angle to water Rest at an angle to water surface
Larvae siphon tube) surface (Slender Siphon tube present)
(Short Siphon tube present)
a) 200
b) 350
c) 500
d) No cut-off
Any age
Any stage
Any group
Start ART
Normally CD4 count is 900 - 1400 & if it starts going below 500 opportunistic infections start.
Most common opportunistic infection HIV world → Pneumocystis carinii Pneumonia
a) 0.8 in males
b) 0.85 in females
c) 0.90 males
d) < 0.90 males
Waist to hip ratio used as indirect marker of obesity and direct marker of cardiovascular risk
to the body.
Cutoffs :
Male ≤ 0.90
Female ≤ 0.85
According to new WHO guidelines waist hip ratio has been replaced by a new indicator that
is "Waist Height Ratio"
WHO SAYS, this is now the best indicator for cardiovascular risk.
Age and sex independent
Single cutoff for al age and sex → 0.5
Flourescent Antibody Test : This is a highly reliable and the best single test currently
available for the Rapid diagnosis of Rabies viral Antigen in Infected specimen.
Corneal impression & Mouse inoculations are also used for diagnosis.
a) 1 Lac IU once
b) 1 Lac IU three times
c) 2 Lac IU once
d) 2 Lac IU three times
a) 20 mg OD X 5 days
b) 12 mg BD X 5 days
c) 25 mg BD X 5 days
d) 20 mg BD X 5 days
Oseltamivir (Tamiflu ) is the drug of choice for swine flu H1N1, But it's dose is given
according to the age and weight of the person.
<15 kg →g 30 mg BD x 5 Days
Weight 15-23 kg → 45 mg BD x 5 Days
24-40 kg → 60 mg BD x 5 Days
>40 kg → 75 mg BD x 5 Days
8. Beri-Beri seen in
a) Sorghum eaters
b) Wheat esters
c) Polished rice eaters
d) Maize eaters
10. The proposal to control malaria in towns named as Urban Malaria Scheme.
According to Urban Malaria Scheme, Which of the following is not true?
11. A 30 year old male truck driver visited to the clinic with fever, skin rash, swollen
glands and fatigue. The physician referred him to suraksha clinic. Which among
the following is included in provision of services a Suraksha Clinic ?
a) Immunization services
b) Blood transfusion safety services
c) RTI/STI services
d) Diarrhea control services
Services expected at suraksha clinic (STD clinic) are related to RTI/STI. It includes :
Diagnostic services
Treatment care
Preventive services
12. Ujjawala scheme is for prevention of
a) Child abuse
b) Child trafficking
c) Child labour
d) Child marriage
a) Female MPW
b) ANM
c) Medical office
d) Male MPW
14. A government school in rural village was checking all students for their health
conditions. Under Vision 2020, to check visual acuity, teacher will refer school
children's to which of the following centre?
a) Service centre
b) Training centre
c) Centre for excellence
d) Vision centre
The goal of Vision 2020 is to eliminate needless blindness by the year 2020. The role played
by Vision Centre in achieving this target by providing primary eye care in the community
therefore assumes significant importance.
Vision2020 has developed certain policies for Vision Centre:
15. A school has a small outbreak were children are showing symptoms like runny
nose, cough, fever and skin rash, The doctors identified it as measles outbreak.
Which of the following is the correct time to give post exposure measles vaccine?
a) 3-4 days
b) 7-10
c) 10-14 days
d) 1-6 months
a) +300 kcal
b) +600 kcal
c) +350 kcal
d) +520 kcal
a) Spermicidal methods
b) Implant
c) Oral contraceptive pills
d) IUD
18. While launching a new national health programme and policy population
attributable risk is calculated. Which of the following best defines population
attributable risk?
It is the incidence of disease (or death) in total population minus incidence of disease
(or death) among those who were not exposed to suspected causal factor.
It is useful as it provides an estimation of amount by which incidence of disease could
be reduced in that population if the suspected factor was eliminated or modified.
PAR is useful for National health programmes and policies.
19. Human anatomical wastes are disposed in ___ BMW (Bio Medical Waste) category
a) Red
b) Yellow
c) White
d) Blue
Yellow category
Human anatomical waste
Animal waste
Soiled waste
Chemical waste
Discarded medicine
Liquid chemical waste
Microbiological waste
Red category: Solid contaminated (Recyclable) (plastic/rubber)
White category: Wasted sharp
Blue category:
Glassware
Metallic body implants
20. A disaster is a sudden, calamitous event that seriously disrupts the functioning of
a community or society and causes human, material, and economic or
environmental losses. Which among the following covers Disaster management?
a) Ministry of Health
b) Ministry of HRD
c) Ministry of Home
d) Ministry of Agriculture
a)Disaster response
b)Medial relief and rehabilitation
c)Mitigation and preparedness
d)Rehabilitation
a) Vaccine carrier
b) ILR
c) Walk in Cold rooms
d) Cold box
At the PHC (Primary health centre) level, all vaccines are kept in the ILR for a period of one
month at temperature of +2oC to +8oC
23. A specific package of intervention and services which are given to population
those who have either high risk of HIV or those who have reported high risk of HIV
IS known as targeted intervention. Which of the following group is not involved in
this intervention?
a) Migrant labourer’s
b) Street children
c) Long distance truckers
d) Doctors and nursing professionals
Targeted intervention :
It is a specific package of intervention & services which are
given to population those who have either high risk of HIV or those who have reported high
risk of HIV.
Groups with high risk of HIV are:
24. A patient from rural village was given ORS + Zinc. What does ORS plus Zinc do?
a) Reduces infection
b) Antispasmodic
c) Reduces duration of diarrhea
d) Enhance sodium absorption
Duration of diarrhea
Severity of acute and persistent diarrhea
Volume of diarrhea
ENT DEC 2018
1. Mainstay of treatment of glue ear:-
a) Radical Mastoidectomy
b) Myringotomy + aeration to middle ear
c) Temporal bone resection
d) Tonsillectomy & adenoidectomy
Mainstay treatment of glue ear should be treat the cause, like if its adenoids or tumor so
removing it would be the treatment. But the cause is not mentioned in the question. So the
mainstay treatment should be myringotomy in the anterio-inferior quadrant with
grommet tube insertion.
a) Quinsy
b) Parapharyngeal abscess
c) Retropharyngeal abscess
d) Tonsillitis
In the history of the patient it's given that the tonsil is pushed to the midline and there is a
swelling in the neck, this is the typical clinical feature of parapharyngeal abscess.
a) Airway obstruction
b) Headache
c) Neck swelling
d) Epistaxis
Picket fence fever:- fever never comes back to normal baseline. Also known as remittent
fever. (intermittent fever is malaria).
This is the typical presentation of lateral sinus thrombosis/sigmoidal sinus thrombosis.
Syringing
6. Which of the following muscles originates from the first pharyngeal arch:-
a) Stapedius muscle
b) Anterior belly of digastric muscle
c) Posterior belly of digastric muscle
d) Stylopharyngeus
The first arch gives rise to the muscles of mastication and the mylohyoid, anterior belly of
digastric, tensor veli palatani and tensor tympani – all of which are innervated by the
branches of the trigeminal nerve.
7. Dip at 4000hz in pure Tone audiometry indicates:-
a) Otosclerosis
b) Meniere's disease
c) Noise induced hearing loss
d) Age related hearing loss
8. Which of the following structures is used as a landmark for facial nerve in parotid
gland surgery?
a) Tragal pointer
b) Posterior border of mylohyoid
c) Tip of helix
d) Styloid process
Surgical landmarks:
9. Continuous watery discharge from nose after trauma is most likely a feature of::
a) CSF otorrhea
b) Common colds
c) CSF rhinorrhea
d) Anterior epistaxis
Premalignant lesion:
a) Palatoglossus
b) Styloglossus
c) Genioglossus
d) Hyoglossus
12. After head trauma a patient in unable to smell coffee but can smell ammonia.
Which of the following statements is true about the patient?
a) Buccinator
b) Masseter
c) Medial pterygoid
d) Lateral pterygoid
Masseter
Temporalis
Pterygoids (Lateral & Medial)
These are Supplied by the mandibular nerve
a) Inflammation of cornea
b) Fatty deposition of sclera
c) Conjunctival invasion making a flap over cornea
d) Dead epithelial debris accumulation
Pterygium:
Wing shapes fold of conjunctiva encroaching upon the cornea from the either side
Etiology-
Hot climates
Degenerative & hyperplastic condition of conjunctiva
Symptoms
a) Argon laser
b) Nd YAG laser
c) Excimer laser
d) Co2 Laser
3. A newborn is hypermetropic by
a) + 2.5 D
b) +5D
c) + 10 D
d) +1D
a) Neuroectoderm
b) Neural crest cells
c) Mesoderm
d) Surface ectoderm
Retinoscopy
a) Maxillary fracture
b) It is a blowout fracture
c) Zygomatic bone is most likely injured
d) Frontal bone fracture
MC fracture of orbit
Inferior wall M/C involved
Entrapment of inferior rectus in inferior wall fracture - not able to look up
Features
Enophthalmos
Diplopia
Infra orbital nerve anesthesia
Hanging drop or tear drop sign on imaging
Blow out orbital fracture result from trauma to orbit by a relatively large often rounded
object such as tennis ball, cricket ball, human fist, or part of automobile.
The force of blow causes a backward displacement of the eyeball and increase in the
intraorbital pressure.
9. Sarcoidosis is associated with
Cataract
Ectopia lentis
Anterior uveitis
Keratitis
10. 10 years old child brought to your clinic with developmental delay associated with
extrapyramidal symptoms. On further examination, dislocation of the lens towards
the inferonasal area is revealed. which of the following is the most probable
diagnosis?
a) Homocystinuria
b) Refsum's disease
c) Wilson's disease
d) Sarcoidosis
a) Motor obstacles
b) Sensory obstacles
c) Central obstacles
d) All the above
CONCOMITANT SQUINT - a type of manifest squint which the amount of deviation in the
squinting eye remains constant in all directions of gaze, with no limitation of ocular
movements. Any obstacles to the development of these processes result in concomitant
squint.
Refractive errors
Prolonged use of incorrect spectacles
Anisometropia
Media opacities - Corneal/Lenticular opacities
Motor obstacles - hinder the maintenance of eyes in the correct positional relationship in
primary gaze or during ocular movements.
E.g.-
Hydrops fetalis -
Excessive destruction of the fetal red cells leads to severe anemia, tissue anoxemia
and metabolic acidosis
It is accumulation of fluid in the body of the fetus in two out of three compartments:
ascites, pleural effusion, pericardial effusion
Most common cause of immune hydrops fetalis - Rh incompatibility.
a) Precipitated labour
b) Prolonged 2nd stage
c) Premature bearing down
d) Episiotomy
a) Perineorrhaphy
b) Shirodkar Procedure
c) Pessary
d) Le Fort Repair
Shirodkar procedure
Surgical treatment of prolapse Perineorrhaphy
temporary Rx
Le fort repair
Woman beyond 40 years and multipara Vaginal hysterectomy and pelvic floor repair
4. Cut off point for 50 gram glucose challenge test at 1 hour mg%
a) 100
b) 140
c) 180
d) 200
Glucose challenge test :
50 grams of glucose to non-fasting women :
After 1 hour
Value:
3 hr - < 145
5. 28 year old lady (G2 P1) who does not want to conceive after abortion, which is the
preferred mode of contraception:
a) OCP
b) IUD
c) Barrier
d) Implant
6. A 30 year old primigravida delivers a full term baby. However she chooses to
formula feed her newborn. After how much duration can she start using combined
oral contraceptives for avoiding pregnancy?
a) 2 weeks
b) 6 weeks
c) 12 weeks
d) Immediately after delivery
Please note that COC should never be started immediately post-partum in any scenario
since post-partum period in itself is high risk state for thromboembolism, maximum risk
being in the first week. Progesterone only pills can be safely used in lactation, and can be
started as soon as contraception is required.
HPV
16: Most common
18: most malignant
HSV 1 and.
HIV 1 and 2
Commercial sex worker
Women with many sex partners
Partner with STD
Early intercourse (< 16 yrs)
Low Socio Economic status
Smokers
a) Young's rule
b) Naegele's formula
c) Cardiff Formula
d) Hadlock Formula
Expected date of delivery is calculated by Naegele's formula
From the 1st day of last menstrual cycle → 9 months + 7 days.
9. If a zygote divides 2-3 days after fertilization which of the following type of twining
is seen:
a) Monochorionic, Monoamniotic
b) Monochorionic, Diamniotic
c) Diamniotic, Dichorionic
d) Dichorionic, Monoamniotic
Complete mole
A complete mole may be formed if.
23 , - X bearing haploid sperm
penetrates a empty ova paternal
chromosome then duplicate to create
46 XX diploid complement Solely of
paternal origin
11. A woman is diagnosed with an unruptured tubal ectopic. She is admitted for
medical management with methotrexate while a strict watch is kept for signs of
tubal rupture. An ectopic sac in which of the following parts of the tube is likely to
rupture the earliest in case of failed medical management?
a) Interstitial
b) Isthmus
c) Ampulla
d) Infundibulum
12. A pregnant woman with a known history of fibroid uterus is being planned for an
elective cesarean section for breech presentation. She insists her fibroid be
removed with the surgery, however the gynaecologist counsels her the regarding
disadvantages of myomectomy at the time of cesarean and tells her that he would
remove it only if it is ?
a) Pedunculated subserosal
b) In the broad ligament
c) Posterior in location
d) Small in size
13. After 28 weeks POG major amount of amniotic fluid is secreted from:
a) Fetal skin
b) Fetal urine
c) Placental cell membrane
d) Plasma
In early pregnancy Ultrafiltrate of maternal plasma
Beginning of 2nd trimester Extra cellular fluid which diffuses through
fetal skin
After 20 weeks Fetal urine > fetal lung
a) Chlamydia
b) Trichomoniasis
c) Candidiasis
d) Bacterial vaginosis
15. A young couple presents to fertility clinic with inability to conceive since years.
They are living together,have been married for 4 years and not using any
contraception at present. The initial tests that should be advised to this couple
are?
16. A G4P2 lady presented with history of two abortions at 16 weeks and 20 weeks
POG. Which of the following could be the most likely reason for these abortions?
a) Chromosomal abnormality
b) Cervical incompetence
c) Placenta previa
d) Thyroid abnormality
Abortions- MC causes
Endometriosis
18. A 29 year old female presented with infertility. There is history of abdominal pain,
dyspareunia, dysmenorrhea, menorrhagia. Most likely cause:
a) Adenomyomatosis
b) Endometriosis
c) Myomas
d) Cervicitis
Symptoms of Endometriosis :
Dysmenorrhea
Abdominal pain
Dyspareunia
Infertility
Menorrhagia
Chronic pelvic pain
19. A 32 year old pregnant woman presents with 36 week pregnancy with complaints
01 pain abdomen and decreased fetal movements. Upon examination PR= 96/min,
BP = 156/100 mm Hg, FHR = 128 bpm. On per-vaginum examination there is altered
blood seen and cervix is soft 1 cm dilated. What is the preferred management?
a) Tocolytics
b) Induce labour
c) Immediate LSCS
d) Wait and watch
20. A 30 year old woman presents to the clinician with excessive growth of facial hair
associated with hairgrowth on her upper back, arms and thighs. Which of the
following is the most likely cause?
a) Drug induced
b) PCOS
c) Endometriosis
d) Adenomyosis
21. 52 year old lady presents to the physician with complains of episodes of feeling of
intense warmth followed by profuse sweating. She stopped having her periods
since the past 3 years. Which of the following is correct regarding her condition?
a) Yolk sac
b) Liver
c) Syncytiotrophoblast
d) Umbilical cord
a) Zinc
b) Copper
c) Selenium
d) Iron
X- linked recessive disorder caused by mutations in genes coding for the copper
transport protein ATP7A.
Copper level is low in liver & brain, high in enterocytes & fibroblasts
Clinical features:-
a) Gaucher disease
b) Alkaptonuria
c) Hartnup disease
d) Phenylketonuria
Hartnup disorder:
Defect in transport of neutral amino acids by intestinal mucosa & renal tubules:
Autosomal recessive inheritance
Defect in SLC6A19 gene on chr 5p15
Most children remain asymptomatic
Cutaneous photosensitivity and pellagra like rash.
a) Gumma
b) Hutchinson's teeth
c) Olympian brow
d) Interstitial keratitis
Syphilitic rhinitis
Maculopapular rash
Condyloma lata
Symptom/sign Descriptions
Olympian brow Bony prominence of the forehead caused by persistent or
recurrent periostitis.
Higoumenakis sign Unilateral or bilateral thickening of the sternoclavicular third of
the clavicle
Saber shins Anterior bowing of the midportion of tibia
Hutchinson teeth Peg-shaped upper central incisors with a notch along the biting
surface
Mulberry molars Abnormal 1st lower molars with excessive number of cusps
a) X chromosome
b) Y chromosome
c) Chromosome 21
d) Chromosome 22
Mucopolysaccharidoses:
a) Phenylketonuria
b) Alkaptonuria
c) Albinism
d) Von Gierke disease
Infectious mononucleosis:
9. In an eye camp doctor's try to make people aware of the causes of blindness.
Vitamin A is the leading cause for preventable childhood blindness. A parent
brings her 18 month old child to the doctor for routine eye check up. Doctor
notices thinned out cornea and from old records it was seen that the child also
has iron deficiency anemia. Weight of the child is 10kg, what should be the dose
of vit. A to be given in this case if doctor marks it as keratomalacia.
a) 50,000 IU
b) 1,00,000 IU
c) 2,00,000 IU
d) 5,00,000 IU
Vitamin A DEFICIENCY
It is characterized by night blindness, conjunctival xerosis. Bitot spots, Keratomalacia and
fundus changes in severe cases
Treatment:
a) Head
b) Abdomen
c) Palms & soles
d) Neck
A new born loses maximum heat from ‘Head’ because of greater surface area.
a) 120-200 μg
b) 90-120 αg
c) 30-60 μg
d) 500 μg
2. Xenograft is:
a) Fibronectin
b) Laminin
c) Integrin
d) Syndecan
5. Cell that can form the maximum number of other cells in the body is called?
a) Totipotent
b) Multipotent
c) Pluripotent
d) Lineage stem cells
a) Hurler Syndrome
b) Thalassemia
c) Hereditary spherocytosis
d) G6PD deficiency
X-LINKED RECESSIVE DISORDERS: Oblivious Female Will Give Her Boys Her X-Linked
Disorders"
Ocular albinism
Fabry disease
Wiskott-Aldrich syndrome
G6PD deficiency
Hunter syndrome
Bruton agammaglobulinemia
Hemophilia A & B
Lesch - Nyhan syndrome
Duchenne muscular dystrophy
a) Autosomal recessive
b) Autosomal dominant
c) X - linked recessive
d) X - linked dominant
CONSANGUINITY condition in which closely related individuals are going to have the
marriage.
This can lead to expression of even those genes which are recessive in nature.
Therefore, it is a risk factor for expression of Autosomal Recessive disorder.
a) Liver
b) Adrenals
c) Kidney
d) Brain
9. A 57-year-old male came with complaints of swelling over the scalp region which
gradually developed over 6 months. Now complaints of pus discharge several
times in the past month associated with pain. On examination, swelling sized 3x2
cm, Firm, Fixed; Tenderness(+), Warmth(+), Punctum(+), Discharge(+). Incision &
Drainage was done, the wound was left open to heal. Which of the following is a
mechanism for healing in this case?
a) Granuloma formation
b) Scab formation
c) Granulation tissue
d) Neovascularization
Healing intention: Wounds by secondary are left open and allowed to heal by
granulation, contraction and epithelialization.
In healing by second intention, large amounts tissue are formed of granulation to fill a
bigger gap caused by the larger area of deficit.
10. A 47 years old male, active smoker with a known case of Bronchial Asthma now
presents with difficulty in breathing with the breath sounds being audible
externally. Which of the following is not a feature you will expect to find in this
patient?
a) Seconds to minutes
b) Days to weeks
c) Weeks to months
d) Months to years
Hyperacute rejection: type 2 HSR, happens over span of few minutes to hours. Eg: kidney
transplant
Acute rejection: Few weeks to months(< 6 months)
Chronic rejection: After 6 months.
Graft vs host disease (Runt Disease):
Iron deficiency typically causes low serum iron, high iron-binding capacity, and low serum
ferritin levels. Anemia occurs when you have a decreased level of hemoglobin in your red
blood cells (RBCs).
a) Nor-epinephrine
b) Secretin
c) Vasoactive intestinal polypeptide
d) Somatostatin
Follows rule of 10 -
10% are extra adrenal
10% are bilateral
10% are children
10% are w/o hypertension
Tx: Surgical Resection
α + β blocker
SURGERY DEC 2018
a) Hemorrhage
b) Fistula
c) Bleeing from gastric mucosa
d) Pancreatitis
Complications of splenectomy:
Early:
Haemorrhage
Injury to stomach, splenic flexure, pancreas
Delayed:
Fistula from stomach, pancreas
Subdiaphragmatic collection
Left basal atelectasis and pleural effusion
Thrombocytosis- thrombotic complications
All the given options increase the risk of carcinoma of oral cavity but, oral Lichen
planus has the least risk as it is Low-risk or Equivocal lesion.
3. A 6 year old child is brought with high fever with rigors for 5 days with pain
examination patient s anicteric and tenderness is noted in right upper quadrant.
What is best investigation for this case?
a) USG
b) Serology
c) SGOT/LFT
d) CECT
The patient is having pain in the right hypochondrium which suggests the involvement of
the liver.
An abscess can be suspected as the patient is suffering from fever accompanied with
rigors & pain and tenderness in the right upper outer quadrant
Abscess be of two types:
Pyogenic liver abscess
Amoebic liver abscess
On examination the patient is anicteric. So, the patient could be suffering from Amoebic
liver abscess.
In developing countries like India, Amoebic liver abscess is more common because of
the feco-oral and poor sanitation barrier
Initial investigation for the diagnosis of amoebic liver abscess can be done by USG and
CT-Scan.
Investigation of choice or the best investigation for this patient is Serology i.e.
ELISA which detects the anti-bodies formed by the parasite Entamoebahistolytica.
The amoebic liver abscess can be treated by metronidazole empirically.
Ulcero proliferative growth or Cauliflower like growth on the lower lip suggestive of
squamous cell carcinoma of the lower lip
In total 95%of carcinomas of lip arise the lower lip and 5% arise in central one-third
and commissures.
Aetiological feature include UV radiation exposure.
Tumors tend to spread laterally over the mucosal surface.
Lymph node metastases, usually to submental or submandibular nodes which reflect
aggressive disease behavior then approach.
5. Best treatment strategy for carcinoma anal canal:
a) Chemoradiation
b) Surgery
c) Radiation
d) Chemotherapy
6. 32 year old female presented to the OPD with complaints of abdominal pain,
diarrhea and weight loss. On further evaluation, patient was found to have multiple
strictures of small bowel with interspersed areas of normal mucosa in between.
Which of the following is false regarding the given condition?
Recurrences occur after surgery in Crohn's disease. Fistula formation, skip lesions and
aphthous ulcers all seen in Crohn's disease.
a) Superficial
b) Cystic
c) Nodular
d) Pigmented
Rodent ulcer is also known as basal cell carcinoma of the skin, or tear cancer which is a
slow-growing type of cancer.
Types of Rodent ulcers:
a) ESWL
b) PCNL
c) Antegrae pyeloplasty
d) Retrograde pyeloplasty
Generally, ESWL is a non-invasive method used for removal of renal calculi but the
maximum size of stone that can be removed by ESWL is up to 1.5 cm
If the size of the stone is more than 2 cm then the preferred treatment option is
Percutaneous nephrolithotomy (PCNL)
Other contraindications to ESWL are obese patient, pregnancy, & patients taking oral
anti-coagulants.
a) HCC
b) RCC
c) Oncocytoma
d) Chordoma
a) Choriocarcinoma
b) Seminoma
c) Sertoli cell tumour
d) Teratoma
Seminoma
Non-Seminomatous GCT
Yolk sac tumor
Teratoma
Embryonal carcinoma
Choriocarcinoma
11. Dye for Senitnel Lymph Node Biopsy is injected in which of the following sites?
a) Axilla
b) Tail of spence
c) Nipple
d) Areola
a) 1 hour
b) 2 hours
c) 3 hours
d) 4 hours
Priapism: It is defined as a persistent painful erection lasting longer than 4 hours irrespective
of any sexual stimulation & beyond reaching of orgasm.
It is an emergency situation
It is of two types:
1.Ischemic priapism
2.Non-ischemic priapism
14. The image given below indicates which of the following procedures?
a) Biopsy
b) pH - metry/monitoring
c) CT chest
d) PET scan
a) Torsion testis
b) Varicocele
c) Cancer of testis
d) Hydrocele
A bell clapper deformity is a predisposing factor in testicular torsion in which the tunica
vaginalis has an abnormally high attachment to the spermatic cord, leaving the testis free to
rotate. Bell clapper deformity predisposes to intravaginal torsion of the testis.
17. A 29 yr old man with a history of varicose veins since the age of 12 presented with
c/o pain, heaviness, and swelling on the right leg. The patient underwent below-
knee stripping of the Long saphenous vein after which the symptoms completely
resolved along with visible varicose veins. The most common complication that
can occur to this patient is:
a) Hemorrhage
b) Thromboembolism
c) Neuralgia
d) Infection
Ideally, the long saphenous vein should not be stripped below the knee as it causes
injury to the saphenous nerve which is the most common serious complication.
The incidence of saphenous nerve neuralgia up to 7 % following long saphenous vein
stripping to the knee.
And the, most commonly injured nerve while stripping the short saphenous vein is sural
nerve whose incidence of injury may be as high as 20%.
a) Fibroadenoma
b) Fibroadenosis
c) DCIS
d) Phyllodes tumour
Fibroadenoma is the most common benign breast tumor which is most commonly
seen in young females.
Fibroadenoma is also known as "Breast Mouse" due to its characteristic high mobility.
On the other hand, fibro adenosis is a benign breast diseaseso it is excluded
On Mammography (if performed on older female) there is popcorn calcification.
The name DCIS(Ductal carcinoma in-situ) suggests its malignancy and hence can be
excluded.
Phylloides tumor or Cystosarcoma Phylloides is a malignant tumor.
a) Back
b) Soles
c) Scalp
d) Scrotum
Sebaceous cyst arises from sebaceous glands, commonly related to hair follicles
They usually occur on the scalp, face, ear, back & scrotum
A Sebaceous cyst consists of a punctum usually which discharges on pressing
The punctum is absent in the sebaceous cyst o1 scalp & scrotum.
Infection acquired from the environment or the staff following surgery or after admission to
the hospital is termed as Hospital-acquired infection (HAI) which is also referred to as
Nosocomial infections.
There are four main groups of HAI’s
1. Respiratory infections → including Ventilator-associated pneumonia (10-15%)
a) <15%
b) 15-30%
c) 30-40%
d) >40%
Class 3: Volume loss from 30% to 40% of total blood volume, from 1500 mL to 2000
mL. A significant drop in blood pressure and changes in mental status occurs. Heart
rate and respiratory rate are significantly elevated (more than 120 BPM). Urine
output declines. Capillary refill is delayed.
Class 1: Volume loss up to 15% of total blood volume, approximately 750 mL.
Heart rate is minimally elevated or normal. Typically, there is no change in
blood pressure, pulse pressure, or respiratory rate.
Class 2: Volume loss from 15% to 30% of total blood volume, from 750 mL to
1500 mL. Heart rate and respiratory rate become elevated (100 BPM to 120
BPM, 20 RR to 24 RR). Pulse pressure begins to narrow, but systolic blood
pressure may be unchanged to slightly decreased.
Class 4: Volume loss over 40% of total blood volume. Hypotension with
narrow pulse pressure (less than 25 mmHg). Tachycardia becomes more
pronounced (more than 120 BPM), and mental status becomes increasingly
altered. Urine output is minimal or absent. Capillary refill is delayed.
22. Comment on the diagnosis of al film shown of a 65-year -old man with acute
abdomen:
a) Ileocolic intussusception
b) Sigmoid volvulus
c) Toxic megacolon
d) Colocolic intussusception
Notice the significant Clawsign in the given film of a Ba. Enema which is suggestive of
intussusception.
INTUSSUSCEPTION:
M/C type → "ileocolic
Since in the question the age of the patient was clearly mentioned as 65 years old. The
diagnosis will be colocolic intussusception.
a) Clinical examination
b) Axillary sampling
c) USG
d) FNAC and biopsy
Triple assessment, as the name indicates, includes three modalities, physical examination,
imaging (mammography and/or ultrasound), and biopsy (FNAC and core biopsy).
a) Wet gangrene
b) Dry gengrene
c) Frost bite
d) Ainhum
From the given image we can figure out the lesion is a gangrene due to the presence of
black necrotic areas.
Putrefaction Limited due to less blood supply Marked due to congestion of organ
with blood
Line of demarcation Present No clear-cut line
Bacteria Bacteria fail to survive Numerous bacteria present
Prognosis Better due to septicemia Poor due to profound toxemia
25. 30 year old male presents to the OPD with complaints of lower abdominal pain and
bleeding per rectum. On further evaluation , patient was found to have multiple
superficial ulcers in the rectum and extending upward to involve entire colon. On
histology, only mucosa and submucosa were found to be involved. Surgery is
indicated for the above disease in all of the following conditions except:
a) Toxic megacolon
b) Colonic polyp
c) Dysplasia
d) Massive colonic bleeding
26. Patient complains of painless swelling over the face with difficulty in swallowing.
The appearance of face is shown. The probable diagnosis is:
a) Acute parotitis
b) Cancer of parotid gland
c) Angioedema of face
d) Acute sialadenitis
27. A 35-year-old lady the presented with a 6-month painless fluctuant, non-
transilluminant swelling with a thin watery discharge. Clinical diagnosis is?
a) Brachial cyst
b) Secondaries
c) TB
d) Lymphoma
Among the given options discharge can only be seen with the TB which has
characteristic undermined edges, which can be identified easily from the given image.
The branchial fistula may, however have discharge but around the anterior border of SCM
and doesn't have undermined edges.
Lymphoma has a lump, anorexia, weight loss but no discharge
28. The following patient has presented after chest trauma. On examination crepitus is
felt. The clinical diagnosis is?
a) Subcutaneous emphysema
b) Gas gangrene
c) Acute tubular necrosis
d) SVC syndrome
From the given image, you can notice that the patient has puffy eyes and
swelling around the face and crepitus is felt when palpated.
This type of appearance is usually noticed in Sub-cutaneous emphysema.
It is advised not to confuse between gas gangrene and subcutaneous emphysema as
both the conditions have crepitus.
Gas gangrene is a condition that is caused by CI. Perfiringens which has a
gangrenous type of appearance with crepitus due to clostridial myonecrosis and
takes more than 24 hours to appear.
a) Treadmill test
b) Electrophysiological studies
c) Oral beta blocker
d) Procainamide
3. In fever of unknown origin, blood sample should be drawn how many times?
a) 2
b) 3
c) 4
d) 5
Septic Shock :
Sepsis with hypotension despite adequate fluid resuscitation for at least 1 hr, with perfusion
abnormalities. or Need of vasopressors to maintain the blood pressure.
a) 0.80 in male
b) 0.80 in female
c) 0.85 in male
d) 0.85 in female
Waist to hip ratio that increases risk of heart disease for male is > 1
Waist to hip ratio that increases risk of heart disease for female is > 0.85
a) Abducens nerve
b) Trochlear nerve
c) Trigeminal nerve
d) Facial nerve
Abducens= Longest subarachnoid course (so whenever ICP will raise abducens nerve
will affect first )
Trochlear=Longest intracranial route, thinnest, only nerve has dorsal origin
Trigeminal=Thickest nerve
Facial= Longest intraosseous route
8. After trauma a patient is unable to smell coffee and asafetida but can smell
ammonia. Which of the following statement is true about the patient?
a) ESWL
b) PCNL
c) Antegrade Pyeloplasty
d) Retrograde pyeloplasty
10. A new vaccine which was launched in 2018 in Thailand with the name CYD-TDV. In
which of the following condition it is used?
a) Dengue
b) Malaria
c) Yellow fever
d) Japanese encephalitis
11. A 45 year old male who had a history of domestic animal bite has come for follow
up post 90 days after receiving rabies post-exposure prophylaxis. Now what will
you use for Rapid diagnosis of rabies in this case ?
a) Acoustic neuroma
b) Optic glioma
c) Café-Au-Lait macules
d) Meningioma
13. A 7-year-old with fever for days with pain in right hypochondrium. On examination
tenderness is noted in Right upper quadrant. What is the next best investigation?
a) USG
b) Serology
c) SGOT/LFT
d) CT abdomen
a) Mydriasis
b) Ptosis
c) Anhidrosis
d) Enophthalmos
Horner syndrome is characterized by damage to sympathetic chain
Features are – “SAMPLE”
Sympathetic Nerve Fiber Injury
Anhidrosis
Miosis
Ptosis
loss of ciliospinal reflex
Enophthalmos
a) Achalasia cardia
b) Diffuse oesophageal spasm
c) Carcinoma Oesophagus spasm
d) Schatzki's Ring
17. Which of the following features are true about Allergic Broncho Pulmonary
Aspergilloma?
1)Distal bronchiectasis
2)Serum precipitins to Aspergillus
3) Increased IgE Levels
4)Seen in asthmatics
a) 1,2,3 only
b) 2,3,4 only
c) 1,3,4 only
d) All of the above
a) SGOT
b) Bilirubin
c) Albumin
d) Prothrombin time
The Child-Pugh (CP) classification is the standard to assess preoperative liver function and
is determined by five factors: serum bilirubin and albumin levels, prothrombin time,
ascites, and encephalopathy.
a) Diabetic ketoacidosis
b) Coxsackie B virus
c) Non ketonic hyperosmolar coma
d) Autoimmune pancreatitis
a) Infective endarteritis
b) Infectious mononucleosis
c) Inflammatory myopathy
d) Infective endocarditis
The Modified Duke Criteria for the Clinical Diagnosis of Infective Endocarditis
Duke's staging - Cancer of rectum
Duke's score - Chronic stable angina
23. Which of the following shall be seen with use of a small size BP cuff?
a) False elevation of BP
b) Falsely low value of BP
c) Cancels the effects of calcified arteries
d) Increases trans arm impendance
SMALL SIZED CUFF: It leads to false elevation of BP as Blood vessels are squeezed
so there is an increase in the peripheral resistance
BIG SIZED CUFF: It leads to falsely low measure of BP
a) Cysticercosis cellulose
b) Larva currens
c) larva migrans
d) Visceral larva migrans
Cysticercosis is a common helminthic infection caused by the larval stage of pork tapeworm,
Taenia solium. The larvae, also called as cysticercosis cellulose, reside within the
muscles and other tissues of pigs that act as their intermediate hosts.
25. On putting an internal jugular vein catheter, a patient has developed sudden onset
severe respiratory distress. Clinical diagnosis is?
a) Pneumothorax
b) Sepsis
c) ARDS
d) Infective endarteritis
The catheter penetrated the jugular vein and traumatized the lung parenchyma
damaging the lung tissues leading to PNEUMOTHORAX.
26. You are an intern, posted in the neurology ward in the Department of Internal
Medicine. A 48-year-old woman was admitted with a history of muscle weakness.
Investigations & examination revealed normal nerve conduction but flaccidity is
seen. Which of the following would be the possible diagnosis?
a) Myasthenia gravis
b) GBS
c) Transverse myelitis
d) Traumatic neuritis
Myasthenia Gravis- Problem in Neuromuscular junction due to improper functioning of Ach
receptor, so nerves are not damaged. neuromuscular junction is affected leading to muscle
weakness.
In option “b,c,d” the nerve conduction is always abnormal .
27. Which of the following is the most sensitive test for the diagnosis of
Organophosphate poisoing ?
a) Plasma acetylcholinesterase
b) Organophosphate level in blood
c) Organophosphate level in plasma
d) RBC transaminase levels
a) Gower Sign
b) Homan Sign
c) Oppenheim Sign
d) Gordon Sign
On squeezing or pinching the calf muscles (gastrocnemius), the Big toe goes away -
GORDON SIGN
a) SACD
b) AD
c) CJD
d) Pick's disease
Vit B12 ↓
a) Intraventricular bleed
b) Massive Epidural hemorrhage
c) Subdural hemorrhage
d) Subarachnoid hemorrhage
a) H.I.D.A Scan
b) USG
c) CT Scan
d) Biopsy
a) CT and MRI
b) CT and X-ray
c) USG and HIDA Scan
d) MRI and PET Scan
Principle of:
a) Pneumoperitoneum
b) Pneumothorax
c) Eventration of diaphrgram
d) Pneumomediastinum
Pneumoperitoneum - "Air below Diaphragm" is the characteristic feature
a) Mallampati score
b) Cormack lehane
c) ASA grading
d) AHA grading
a) Sevoflurane
b) Isoflurane
c) Halothane
d) Procaine
Halothane
Chloroform
Isoflurane, Enflurane & Desflurane
Nitrous oxide.
a) Ketamine
b) Medetomidine hydrochloride
c) Midazolam
d) All of the above
Any I/M injectable drug which we are using in human can be used in canines as well.
4. Shortest action local anaesthetic:
a) Procaine
b) Cocaine
c) Dibucaine
d) Chloroprocaine
5. During laryngoscopy, best method for checking whether the intubation is correct
or not:
a) Capnography
b) Air entry on auscultation
c) Ultrasonography
d) Chest movement
a) Proximal tibia
b) Distal tibia
c) Proximal femur
d) Distal femur
a) Osteoarthritis
b) Rheumatoid arthritis
c) Psoriasis
d) Hyperparathyroidism
a) Radius
b) Humerus
c) Femur
d) Tibia
Epithelial tumors are classified into Ameloblastoma and adamantinoma, though both the
tumors show same type of histopathology, they are not interchangeable.
Ameloblastoma Adamantinoma
Site Mandible Tibia
5. A lady had a trauma to the neck. X-ray is attached. What is the diagnosis?
a) Hangman's fracture
b) Jefferson's fracture
c) Clay shoveler fracture
d) Sciworal
Thumb abduction leads to ulnar collateral ligament damage. It is the most common injury
of metacarpal phalangeal joint. Game keeper's thumb occurs due to forceful abduction of
thumb as seen in gamekeepers who twist necks of small animals.
a) Pronation
b) Supination
c) Mid prone
d) Any position
8. A 80 yr old lady had slip and fall, the following X-ray was taken. Which of the
following is treatment?
a) Hemiarthroplasty
b) Hip spica
c) Total Hip Replacement
d) Nailing
X-ray shows fracture in the neck of Femur. The treatment for neck of femur fracture is
based on age:
9. A child comes with limp and limb shortening. X-ray is below, what is given the
diagnosis?
a) Perthes
b) SCFE
c) DDH
d) Fracture neck of femur
X-ray show normal right side of hip joint. In left side of hip joint, the epiphysis is smaller,
shallow acetabulum and the hip joint is dislocated- indicates Developmental Dysplasia of Hip
(DDH).
a) BZD
b) CBT
c) ECT
d) Haloperidol
CBT (cognitive behavioral therapy)takes longer i.e. months of therapy SO can't be used
in emergency management of a violent patient. Cognitive behavioral therapy (CBT) is
usually short-term approach, typically with 12 to 20 sessions. CBT is collaborative,
structured, and goal-oriented intervention that targets core components of a given
disorder.
a) Depression
b) Schizophrenia
c) Delirium
d) Dementia
a) Over activity
b) Anhedonia
c) Alogia
d) Apathy
Negative symptoms of Schizophrenia :
a) Delirium
b) Dementia
c) Amnesia
d) Anxiety disorder
Delirium: Always has an acute onset, there's disturbance of consciousness, has fluctuating
course.
Dementia: Always has an insidious onset, and there's no disturbance of consciousness, has
a progressing course.
a) Impetigo
b) Chickenpox
c) Herpes Simplex
d) Molluscum contagiousm
Impetigo is a common and highly contagious skin infection that mainly affects infants
and young children. It usually appears as reddish sores on the face, especially around the
nose and mouth and on the hands and feet. Over about a week, the sores burst and develop
honey-colored crusts.
a) Lymphatic development
b) Dermatomes
c) Blaschko lines
d) Vascular development
Blaschko lines are consistently V-shaped on the upper spine, S-shaped on the abdomen,
inverted U-shaped from the breast area to the upper arm, and perpendicular down the front
and back of the lower extremities. They never cross the anterior truncal midline but run along
it.
a) Retinoids
b) Systemic steroids
c) Psoralen PUVA
d) Nd-YAG laser
Vitiligo is an autoimmune disease, can only be treated with immunosuppressants.
Treatment of choice PHOTOTHERAPY
Target Phototherapy is used in vitiligo with Excimer laser.
PUVA Given with UV radiation.
a) Acanthosis
b) Acantholytic cells
c) Smudge cells
d) Anaplasia
Acanthocytes - Pemphigus
Multi nucleated giant cells - Herpes
Acanthocytes/Acantholytic cells:
Oval
Single cell with nucleus
Perinuclear halo
ANATOMY JUN 2019
a) Hydranencephaly
b) Rachischisis
c) Anencephaly
d) Spina bifida
Anencephaly is the absence of a major portion of the brain, skull, and scalp that
occurs during embryonic
development. It is a cephalic disorder that results from a neural tube defect that
occurs when the rostral (head) end of the neural tube fails to close, usually between
the 23rd and 26th day following conception.
a) T2
b) T4
c) T6
d) T8
5. What is the structure highlighted ?
a) Supraspinatus
b) Infraspinatus
c) Teres major
d) Teres minor
Muscle highlighted in this picture is TERES MINOR
a) Serratus Anterior
b) Pectoralis major
c) Pectoralis minor
d) Subclavius
Muscle Action
Pectoralis major Adducts and medial rotation of arm
Pectoralis minor Stabilizes the scapula
Subclavius anchors and depresses the scapula
Serratus Anterior protracts the scapula
laterally rotates the scapula
overhead abduction
7. One of the nerves supplying the upper limb, the upper lateral cutaneous
nerve of arm, supplying the skin overlying the posterior border of the
deltoid muscle as well as covering the long head of the triceps, is a branch
of:-
a) Musculocutaneous nerve
b) Radial nerve
c) Ulnar nerve
d) Axillary nerve
Axillary Nerve
This nerve originates from the posterior cord of the brachial plexus with contributions
from the C5 and C6 ventral rami. The axillary nerve travels through the inferior-
lateral axilla, and around the surgical neck of the humerus with the posterior
circumflex humeral artery. After providing motor innervation to the deltoid, the
axillary nerve gives off the lateral superior cutaneous nerve branch which
innervates the lateral skin of the shoulder
a) Supinator
b) Pronator quadratus
c) Pronator teres
d) Brachioradialis
a) Lunate
b) Scaphoid
c) Hamate
d) Trapezium
Anatomical snuff box: The given picture shows anatomical snuff box which Is a
depression formed at the
posterolateral side of the wrist and metacarpal by the extensor tendons passing into
the thumb. Base of the triangle is at the wrist and apex is directed into thumb. It is
most apparent when thumb is extended.
Boundaries:
Lateral: Abductor pollicis longus and extensor pollicis brevis tendons.
Medial :Extensor pollicis longus tendon
Floor: Scaphoid and trapezium the distal ends of the tendons of the extensor carpi
radialis longus and extensor carpi radialis brevis
Scaphoid Fracture:
The commonest carpal injury is a fracture across the waist of the scaphoid bone It is
uncommon to see other injuries. In approximately 10% of individuals, the scaphoid
bone has a sole blood supply from the radial artery, which enters through the distal
portion of the bone to supply the proximal portion. When fracture occurs across the
waist of the scaphoid, the proximal portion therefore undergoes avascular necrosis.
10. A patient presents with hypothenar muscle wasting and loss of sensation
of medial one and a half digits. Which nerve is involved ?
a) Musculocutaneous
b) Radial
c) Ulnar
d) Median
11. Testicular artery is a branch of -
a) Abdominal aorta
b) Common iliac artery
c) External iliac artery
d) Internal iliac artery
Segment Branches
Ventral branches:
Celiac trunk -T12
Superior mesenteric artery - L1
Inferior mesenteric artery - L3
Dorsal branches
4 pairs of lumbar artery
Median sacral artery
Lateral branches
Inferior phrenic artery (1t branch)
Middle suprarenal artery
Renal artery
Gonadal artery
Terminal branches Common iliac arteries
The deep (internal) ring is found above the midpoint of the inguinal ligament. which is
lateral to the epigastric vessels, The ring is created by the transversalis fascia, which
invaginates to form covering of the contents of the inguinal canal.
a) Cremaster muscle
b) External oblique aponeurosis
c) Internal oblique aponeurosis
d) Transversus abdominis
The inguinal ligament(also known as Poupart's ligament or the groin ligament) is a
fibrous band extending from the anterior superior iliac spine to the pubic tubercle.
It is formed by the external oblique aponeurosis and is continuous with the fascia lata
of the thigh.
a) Gluteus maximus
b) Gluteus medius
c) Obturator internus
d) Quadratus femoris
Gluteus medius is the prime mover of abduction at hip joint. Anterior portion of
Gluteus medius abduct, assist in flexion and medial rotation of hip. Posterior portion
of Gluteus medius abduct, assist in external and lateral rotation of hip.
ANASTHESIA JUN-2019
Depth:
Adult - 5-6 cm
Pediatric - 4-5 cm
Infants - At least 4 cm
a) T12-L1
b) L1-L2
c) L3-L4
d) L5-S1
Spinal cord ends at lower border of L, in adults .Hence spinal anesthesia can be
given at levels below L1. L2-L3 or L4-L5 are considered best in adults.
a) Blue
b) Blue body with white shoulder
c) White
d) Black
Gas State in cylinder International USA
Air Gas Black body and Yellow
shoulders with
black and white.
Oxygen Gas Black body and Green
white shoulders
Nitrous oxide Gas + liquid(below Blue Blue
98oF)
Carbon oxide Gas + liquid(below Grey Grey
88oF)
Helium Gas Brown Brown
Entonox Blue body and
shoulders with blue
and white quarters
Cyclopropane Orange Orange
Nitrogen Gas Black Black
a) Dose
b) Height
c) Age
d) Total body fat
The duration of spinal anesthesia primarily depends upon the dose, intrinsic
properties of local anesthetic
used, use of additives
The level and duration is directly based on dose of the drug
a) Isoflurane
b) Halothane
c) Sevoflurane
d) Ether
a) Ketamine
b) Thiopentone
c) Propofol
d) Etomidate
1. Thiopentone
2. Halothane
3. Morphine
4. Long acting neuromuscular blockers
5. Diazepam / Lorazepam
Bull's neck appearance seen in diphtheria is due to cervical edema and cervical
lymphadenopathy.
a) Conjugation
b) Transduction
c) Transformation
d) Translation
Transformation
There is direct uptake of donor DNA by the recipient cell, which may be natural
or forced.
Transduction - donor DNA is carried in a bacteriophage and is transferred
to the recipient by the mechanism used for phage infection , two type
lytic and lysogenic
Lysogenic conversion, bacteriophages exhibit two types of cycles, virulent or
lytic cycle where the host bacterium is lysed and the temperate or non-lytic
cycle where host bacterium is unharmed. Eg diphtheria
Conjugation- The donor cell contributes energy and building blocks to the
synthesis of a new DNA strand, which is physically transferred into the
recipient cell, usually by tube called sex pilus.
Transferable drug resistance mediated by the factor is the most important
method of drug resistance
Mutational resistance is of great importance in tuberculosis. If a patient is
treated with streptomycin alone, initially the appear and multiply unchecked. If
two or more anti-re-population by resistant mutants does not occur, as a
mutant resistant to one drug will be destroyed by the other drug.
a) Stationary phase
b) Lag phase
c) Log phase
d) Decline phase
1.Lag phase- Adaptation phase to culture media decreased Accumulate nutrient
size
2. Log Phase - Exponential increases in number
3. Stationary phase - Start of Accumulation of toxic metabolites = Dead
Live -Sporulation occur in stationary phase (S-S)
4. Decline - Complete accumulation of Toxin metabolites
Bacillus Anthrax-
Gram positive rods with square ends ( non bulging spore) (bulging spore seen in
cl. tetani)
GPB, Non Motile, capsulated.
Mc fadyeans reaction
M - Mc'fadyean reaction
Medusa Head appearance
I- Inverted fir tree appearance
S - String of pearl reaction
Bamboo - Bamboo stick appearance ( on Gram stain)
5. Which of the following is the most likely cause in a case of granuloma with
positive AFB?
a) Respiratory disease
b) U.T.I
c) Retroperitoneal fibrosis
d) Acute gastroenteritis
a) 1st week
b) 2nd week
c) 3rd week
d) 4th week
Widal test
Highly sensitive but poorly specific.
Also biological false positive in -
Infectious mononucleosis
Malaria
SLE [ auto immune Disease]
a) E. coli
b) Malaria
c) Parvovirus B19
d) Bartonella henselae
EHEC
Entero hemorrhagic E.coli
-Cause HUS (Hemolytic Uremic Syndrome)
-Shiga like toxin or Verotoxin (VT-1& VT-2)
Aka verotoxigenic E.coli
Culture media is Sorbitol Mac-Conkey agar (SMAC) and Rainbow agar used
Diarrhoea causing E coli
1. EPEC -Entero Pathogenic Attaching/Effacing lesions to villi
2. EIEC - Entero Invasive resemble shigellosis & sereny test +ve 3.
3. ETEC-> Traveler’s diarrhea CFA (colonization factor antigen 4.
4. EAEC -Aggregative persistent type of diarrhea (EAST-1 Toxin)
Parvo Virus B19: Single standard DNA Virus cause Aplastic Crisis
a) Rubella
b) Measles
c) Rabies
d) EBV
a) Adenovirus
b) Flavi virus
c) Rubi virus
d) Pox virus
a) Histoplasmosis
b) Chromoblastomycosis
c) Coccidioidomycosis
d) Candida albicans
Dematiaceous Fungi
Involves skin & sub cutaneous tissues
Sclerotic bodies seen in tissue biopsy Brown colored globose bodies Aka medlar
bodies Show copper penny appearance
Examples -
1) Phialophora
2) Cladosporium
a) Larva currens
b) Cysticercosis cellulose
c) Cutaneous larva migrans
d) Visceral larva migrans
a) C1-C4 deficiency
b) C5-C9 deficiency
c) C3 deficiency
d) C2 deficiency
a) C1
b) C3 convertase
c) IgA
d) Ag-Ab complex
The complement pathway. Complement can be activated through three pathways:
classical, lectin, and alternative. The classical pathway is activated when C1q
binds to antibody attached to antigen, activating C1r and C1s, which cleave C4
and C2.
a) Acute psychosis
b) Panic attack
c) Depression
d) Mania
Panic Disorders: -
Recurrent & unexpected panic attacks
Anticipatory anxiety in between the attacks
a) Denial
b) Anger
c) Bargaining
d) Agitation
PHARMACOLOGY JUNE-2019
a) Dexamethasone
b) Hydrocortisone
c) Triamcinolone
d) Deflazacort
Short acting steroids → Cortisone
Hydrocortisone
Intermediate acting → Prednisone
Prednisolone
Triamcinolone
Long acting → Dexamethasone
Betamethasone
Paramethasone
a) Valproate
b) Carbamazepine
c) Phenytoin
d) Lamotrigine
3. You, being an ardent researcher in the field of vaccine production, are the
in-charge of preparing samples to make a vaccine against the new variant
of SARS-CoV 2. After getting satisfactory results in pre-clinical studies, you
seek approval to do clinical studies, which requires healthy volunteers.
Which of the following phase does this refer to?
a) Phase 1
b) Phase 2
c) Phase 3
d) Phase 4
a) Salbutamol
b) Propofol
c) Betaxolol
d) Timolol
a) Salbutamol
b) Terbutaline
c) Adrenaline
d) Formoterol
a) Desmopressin
b) Vasopressin
c) Terlipressin
d) Conivaptan
Halothane causes hepatitis .It can also cause arrhythmia by making adrenaline more
adrenogenic, i.e, in conditions Adrenaline Shows more adrenogenic action when
halothane is used. Hence it should not be used where adrenaline is high ,i.e,
pheochromocytoma
a) Digoxin
b) Propranolol
c) Diltiazem
d) Lignocaine
9. Which of the following drugs mentioned below is the first choice for
chronic gout?
a) Allopurinol
b) Febuxostat
c) Probenecid
d) Sulfinpyrazone
a) Ketamine
b) Thiopentone
c) Propofol
d) Etomidate
Dr -Desflurane
Manmohan- Midazolam
Singh -Sevoflurane
Is -Isoflurane
A –Alfentanyl
Prime -Propofol- mc drug used in day care surgery
Minister- Mivacurium
11. A female with 20 weeks of pregnancy presents with fever and dysuria. A
preliminary diagnosis of cystitis was made. Which of the following drugs
will be safe to use for this patient?
a) Ciprofloxacin
b) Gentamicin
c) Cotrimoxazole
d) Amoxicillin
12. A 66 year old known diabetic, on poor compliance, was referred to your
institution as her had symptoms of DKA. Which of the following insulin
would you prefer for the treatment of this condition?
a) Regular Insulin
b) NPH insulin
c) Insulin glargine
d) Insulin lispro
a) Insulin lispro
b) Insulin aspart
c) Insulin glulisine
d) Insulin glargine
Insulin glargine:
Long acting insulin.
Subcutaneous glargine given, most of it stay in subcutaneous area, very small
amounts comes into the blood, gradually it keeps coming into the blood
(maintains basal level)
Never become high in plasma
Single injection work for 24 hrs
Glargine is called as peakless insulin.
Rapid acting insulins (insulin Lispro, insulin aspart, insulin glulsine)
a) Strongyloidiasis
b) Trichomoniasis
c) Schistosomiasis
d) Rhinosporidiosis
15. A young male presents with reduced sleep, hyperactivity and elevated
mood. He has a family history of mania. Which of the following drug should
be used for long term treatment of this patient?
a) Sodium valproate
b) Lithium carbonate
c) Carbamazepine
d) Barbiturates
a) Vancomycin
b) Teicoplanin
c) Mupirocin
d) Linezolid
a) Venlafaxine
b) Topiramate
c) Bupropion
d) Amitriptyline
a) Ketorolac
b) Etoricoxib
c) Piroxicam
d) Nimesulide
19. Aerosol drug used for treatment of RSV infection in a child is?
a) Indinavir
b) Amantadine
c) Ribavirin
d) Tenofovir
a) Donepezil
b) Atropine
c) Physostigmine
d) Fluoxetine
Organophosphates
ACHE inhibitors (acetylcholine cannot be broken)
Level of acetyl choline increases
Stimulate nicotinic & muscarinic receptors
Over activity of M1- acidity
M2-- bradycardia
M3-- bronchoconstriction, secretion, diarrhea & urination
a) Dopamine
b) Nor - adrenaline
c) Adrenaline
d) Serotonin
a) Reassure
b) USG
c) HSG
d) Hormonal study
In the question the girl is 15 yrs with normal secondary sexual characteristics, she is
a case of primary amenorrhea and needs evaluation (reassurance could have been
given had the patient been younger than 15 years)
The next important step would be an USG to check whether she has normal
mullerian structures (uterus and tubes) and ovaries or not
Hormonal studies and karyotyping are done subsequently to find but the cause.
Management -
1. Bed Rest
2. Avoid Coitus
3. Avoid Lifting weights
4. Progesterone is given to compliment the placental function
a) Repeat USG
b) Hysterosalpingography
c) Endometrial biopsy
d) Urine culture and sensitivity
A past history of PID and normal pelvic USG and hormonal assays is most likely
suggestive of tubal block (secondary to adhesions due to PID) as a cause of the
patient's infertility.
HSG
a) Placenta previa
b) Vasa previa
c) Abruptio placenta
d) Placenta percreta
Presentation
Painful bleeding
Protraction of labour
PPH
5. Risk of endometrial cancer is least in:
1) Late menopause
2) Nulliparity
3) Unopposed estrogen therapy
4) Obesity, diabetes, hypertension
5) History of persistent anovulation (PCOS)
6) History of irregular & excessive premenopausal bleeding
7) Atypical endometrial hyperplasia
8) Tamoxifen therapy
9) Radiation menopause.
a) >2
b) >3
c) >4
d) ≥5
7. A 26 weeks pregnant female presented with HTN for the first time. There is
no proteinuria. Diagnosis of such condition
a) Chronic hypertension
b) Eclampsia
c) Gestational Hypertension
d) Preeclampsia
Gestational hypertension - bp > 140/90, > 20weeks of gestation
Pre eclamptic toxemia - gestational hypertension + proteinuria >300mg
Eclampsia - above 2 + convulsions
Chronic hypertension - hypertension before she is pregnant
a) 5 million/mL
b) 10 million/mL
c) 15 million/mL
d) 20 million/mL
9. A 26 year old woman presents with infertility. She has regular cycles of 28
days with no dysmenorrhea. Her LMP was on 8th of May. When should her
endometrial biopsy be planned ?
a) 22nd May
b) 8th-12th May
c) Anytime during cycle
d) 29th May
Endometrial biopsy
Best done preferably around 7 days before the onset of menstruation (that is around
21st day of a 28 day cycle)
Secretory changes prove that the cycle has been ovulatory
Can predict the functional integrity of the corpus luteum
Can detect luteal phase defect
10. In this normal menstrual cycle graph, the mark X represents levels of which
hormone?
a) LH
b) FSH
c) Estrogen
d) Progesterone
11. A patient who was using CuT for contraception, presented with a 20 weeks
pregnancy. The IUCD is placed at fundus, tail visible at os and she wants to
continue the pregnancy. What is your next step this patient:
a) Leave IUD in-situ, continue pregnancy
b) Do medical termination of Pregnancy
c) Remove IUCD and continue pregnancy
d) Remove IUCD and do MTP
With intrauterine pregnancy, if the tail is seen, it should be grasped and the IUD
removed by gentle outward traction This action reduces complications such as
abortion, chorioamnionitis and preterm birth.
Specifically, in one cohort, a 54% abortion rate and 17% preterm delivery rate
was noted if the device remain insitu
a) Multiparity
b) Twins
c) Renal agenesis
d) Macrosomia
Oligohydramnios causes:
1) Fetal chromosomal or structural anomalies
2)Renal agenesis
3) Obstructed uropathy
4) Spontaneous rupture of membranes
5) Intrauterine infections
6) Postmaturity
7) lugr
8) Amnion nodosum
2. Wide open posterior fontanelle large tongue ,rough dry skin with
constipation is seen in?
a) Pellagra
b) Down syndrome
c) Hypothyroidism
d) Nutritional rickets
a) 2 mg%
b) 3 mg%
c) 4 mg%
d) 5 mg%
a) Size of V.S.D
b) Overriding of aorta
c) Degree of RVH
d) Degree of pulmonary stenosis
The more severe the pulmonic stenosis, the less flow into the pulmonary artery and
bigger the right to left shunt. Thus, the severity of cyanosis is directly proportional to
the severity of pulmonic stenosis, but the intensity of the systolic murmur is inversely
related to the severity of pulmonic stenosis.
6. Infant is admitted with respiratory distress and prolonged expiration with
rhonchi in chest. CXR shows hyperinflation. What is the diagnosis?
a) Pneumonia
b) Croup
c) Asthma
d) Bronchiolitis
Acute Bronchiolitis:
Age group 6 months to 2 years.
MC agent is RSV
Clinical features: Viral prodrome followed by tachypnea, retractions and
hypoxemia, wheeze and crepitation heard on examination
Chest x-ray: Hyperinflated chest
Treatment:
Moist Oxygen
In immunocompromised children and those on ventilator: Nebulized Ribavirin is
indicated
Pneumonia: h/o Fever, cough, tachypnea, retractions, sick looking child, CXR
shows consolidation
Asthma: recurrent h/o wheezing present, cough which worsens at night or early
morning, family h/o atopy present.
a) Double
b) Triple
c) Quadruple
d) Same
Breast Milk Cow Milk
Proteins 1 g/dl 3.5 g/dl
Carbohydrates 7 g/dl 4.5 g/dl
Lipids Richer in PUFA Poorer
Carbohydrates:
BM contains more lactose (7g/dI) compared to cow's milk (CM), which contains 4.5
g/dl
Proteins:
BM contains lesser protein (1g/dl vs 3.5 g/dl) as compared to CM → lesser solute
load on
BM contains adequate cysteine, taurine → help in brain development
BM is richer in whey proteins (lactalbumin), which are easily digestible as compared
CM, which is richer in casein.
Lipids:
Breast milk is richer in PUFA (poly unsaturated fatty acids)
It contains adequate amounts of DHA (Docosa Hexaenoic Acid) = helps in brain &
eye development
It is more easily digestible due to presence of lipase in breast milk.
Minerals:
Calcium : Phosphate ratio in breast milk favours calcium absorption
Iron present in breast milk is more easily absorbable than in cow's milk
Vitamins:
Breast milk contains adequate amounts of all vitamins except vitamin D. K(& B12 in
strictly vegan mothers).
a) Erythema Infectiosum
b) Exanthema subitum
c) Erythema marginatum
d) Erythema nodosum
Terminology Other names for the disease Etiology
MEDICINE—JUNE 2019
a) 1,2,3 only
b) 2,3,4 only
c) 1,2,4 only
d) All of the above
a) Ankle
b) Knee
c) Shoulder
d) Foot
Charcot's joint –
Painless, Paralyzed joint
Neuropathy
Delayed wound repair
Angiopathy
Non-heading ulcer on soul of test
4. A 70 yrs old male patient was diagnosed of having heart failure.His ejection
fraction was found to be <40%. Which of the following drug does not
reduce the mortality for this patient?
a) Metaprolol
b) Digoxin
c) Captopril
d) K*sparing diuretics
ACE inhibitors
Angiotensin receptor blockers (ARBS)
Angiotensin receptor blocker with Neprilysin Inhibitor (ARNI)
Beta blockers
Mineralocorticoid receptor antagonists
—Spironolactone
—Eplerenone
Isosorbide dinitrate + Hydralazine
Continuous murmurs begin in systole, peak near the second heart sound, and
continue into all or part of diastole. Their presence throughout the cardiac cycle
implies a pressure gradient between two chambers or vessels during both systole
and diastole
a) Trisomy 16
b) Trisomy 21
c) Monosomy
d) Trisomy 18
10. The following serological status is noted in a patient: HbsAg positive and
HbeAg positive. Diagnosis is?
a) Carrier
b) Chronic viral hepatitis
c) Acute viral hepatitis with infectivity
d) Remote infection
Hbs Ag positive
IgM anti Hbc Ag Acute Hepatitis + Infective
Hbe Ag positive
IgG positive
Remote infection . Hb Ag negative
IgG anti Hbc Ag positive
a) HBV
b) HCV
c) HDV
d) HEV
a) Few hours
b) <1 month
c) < 6 months
d) 6-12 months
Hypersensitivity Time
reaction
1.Hyperacute II Minutes
2.Acute II <6 months
3.Chronic IV >6 months
Pernicious anemia
Characterized by antibodies against parietal cells of stomach
Results in vitamin B12 deficiency ---- Anemia
Serum Sickness/ Arthus reaction are example of Type III HSN
Pathergy phenomenon
For Behcet disease
Type IV Hypersensitivity
a) Thermophilic Actinomycetes
b) Aspergillus Fumigatus
c) Actinobacter
d) Aspergillus Flavus
a) Malaria
b) Hereditary spherocytosis
c) G6PD Deficiency
d) Thalassemia
The above image shows Banana shaped gametocytes which are characteristic of
Plasmodium falciparum malaria.
Hereditary spherocytosis --- Spherocytes (absence of central pallor)
G6PD --- Bite cells
Thalassemia Target cells
Whipple's disease
Rx
IV - Ceftriaxone
Oral - Cotrimoxazole
Whipple's triad -seen in Insulinoma
Whipple's Sx- is performed in Ca of head pancreas.
Extra Edge: The most common type of cardiovascular presentation in Whipple's
disease is culture-negative endocarditis.
17. Most common site of Brain metastasis is?
a) Lung cancer
b) Head and neck cancer
c) Prostate cancer
d) Breast cancer
a) Hypophosphatemia
b) Hypocalcemia
c) Hyperuricemia
d) Hyperkalemia
19. Comment on the diagnosis for SAP normal, PTH normal, Vitamin D3 normal
with elevated serum Calcium values?
a) Vitamin D intoxication
b) Hyperparathyroidism
c) Multiple myeloma
d) Nutritional rickets
Multiple myeloma:
SAP Negative
PTH Negative
D3 Negative
↑ Sr Cal
a) Vitamin
b) Phytates
c) Oxalate
d) Myoglobin
a) Troponin
b) Cytokeratin
c) Myoglobin
d) CPK- MM
Berry aneurysm due to defect in tunica media and intimal elastic lamina layer leading
to weakening of vessel wall and out-pouching in anterior portion of circle of willis.
Saccular Aneurysms: is a 2nd most common cause of clinically significant
subarachnoid hemorrhage is rupture of a saccular ("berry") aneurysm in cerebral
artery following trauma. Patient will describe the pain as worst headache of his life.
Bloody tap on lumbar puncture is seen. Nimodipine is a cerebro-selective
vasodilatory drug helpful for treatment of symptoms associated with subarachnoid
hemorrhage. Berry aneurysm is associated with adult polycystic kidney disease
(ADPKD).
2. A 45-year patient working in a factory for past 20 years presents with
breathlessness. HRCT chest shows pleural thickening and fibrosis, What is
the person suffering from
a) Asbestosis
b) Coal worker pneumoconiosis
c) Silicosis
d) Berylliosis
Caused by dust particle with range of 1-5 micron and it mainly involves the upper
lobe of lung.
HRCT will show pleural thickening and fibrosis.
a) High ferritin
b) Increased percentage of saturation of transferrin
c) High transferrin
d) High serum iron
S: Sideroblastic
I: Iron deficiency
T: Thalassemia
A: Anemia of chronic disease
6. Chronic viral hepatitis is seen with all of the following viruses, except?
a) HBV
b) HCV
c) HDV
d) HEV
a) Few minutes
b) Few hours
c) < 6 months
d) 6-12 months
Hyper acute rejection: type 2 HSR, happens over span of few minutes to hours. Eg:
kidney transplant
Acute rejection: Few weeks to months(< 6 months)
Chronic rejection: After 6 months.
Graft vs host disease (Runt Disease):
Acute GVH: 100 days
Chronic GVH: > 100 days
a) Pernicious anaemia
b) Serum sickness
c) Arthus phenomenon
d) Pathergy phenomenon
My : Myasthenia gravis,
Blood : Blood transfusion (mismatch)
Group : Graves, good pasture
Is: Immune hemolytic anemia
Rh: Rheumatic heart disease
Positive: Pernicious anemia/ pemphigous vulgaris
S: Serum sickness/SLE
H: Henoch schonlein purpura
A: Arthus reaction
R: Reactive arthritis
P: Post strep glomerulonephritis
Pathergy phenomenon (Type 4 HSR) is useful in diagnosis of Behcet
syndrome(oculo-oro-genital syndrome): patient complains of recurrent oral ulcer eye
lesion, skin lesion and genital lesion.
9. A stable boy admitted with acute onset of fever. cough and dyspnea. Chest
radiograph showed findings of pneumonia for which he was treated with
antibiotics; showed good response. After month he again presented in
hypoxemia state with progressive dyspnea. CT chest showed diffuse
ground glass changes. Transbronchial biopsies showed numerous small
granulomas and marked lymphocytosis. Which of the following is a
causative agent for given disease?
a) Thermophilic Actinomycetes
b) Aspergillus Fumigatus
c) Actinobacter
d) Aspergillus Flavus
Pathogenesis C/F
Acute Phase 1st exposure Ab formation Cough / Dyspnea /Fever
Repeat exposure - Ag-Ab immune
complex formation ~ type lll
hypersensitivity
a) Serum Ferritin
b) TIBC
c) Percentage saturation of transferrin
d) Bone marrow iron
Most specific test for ron deficiency anemia is bone marrow iron. But bone
marrow iron test is invasive procedure
Most sensitive indicator for iron deficiency anemia: serum ferritin.
Lab finding in iron deficiency anemia:
Serum ferritin : decrease (early indicator)
Serum iron: decrease
% Tf saturation: decrease
TIBC: increase
11. Whipple's disease is caused by ?
a) Bacteria
b) Virus
c) Protozoa
d) Helminths
12. Most common primary for brain metastasis is which of the following?
a) Lung cancer
b) Head and neck cancer
c) Prostate cancer
d) Breast cancer
13. 12-year-old Boy came with complaints of Severe Abdominal pain and
headache for the past 2 days, H/0 On & off for we the past 1 week,
associated with myalgia, and several episodes vomiting self-medicated
with Tab Paracetamol with no relief. Presents with Dengue NS -1 antigen
(+). What is the first line of defense of the body in the given scenario?
a) NK cell
b) T cell
c) Histiocyte
d) Macrophage
The major components of innate immunity are epithelial barriers that block entry of
microbes, phagocytic cells (mainly neutrophils and macrophages), dendritic cells,
natural killer cells and other innate lymphoid cells and several plasma proteins,
including the proteins of the complement system.
NK cells
Ability to kill virus-infected cells and tumor cells
NK cells act as an early line of defense against viral infections and some tumors.
Two cell surface molecules, CD16 and CD56, are commonly used to identify NK
cells.
CD16 is an Fc receptor for IgG, and it confers on NK cells the ability to lyse lgG-
coated target cells. This phenomenon is known as antibody-dependent cell-
mediated cytotoxicity (ADCC).
The functional activity of NK cells is regulated by a balance between signals
from activating and inhibitory receptors
NK cell inhibitory receptors recognize self-class I MHC molecules, which are
expressed on all healthy cells. The inhibitory receptors prevent NK cells from
killing normal cells.
Virus infection or neoplastic transformation often enhances expression of ligands
for activating receptors and at the same time reduces the expression of class I
MHC molecules.
a) Vitamin
b) Phytates
c) Oxalate
d) Myoglobin
15. Most sensitive and specific marker for myocardial infarction is?
a) Troponin
b) Cytokeratin
c) Myoglobin
d) CPK-MM
a) ASO
b) CRP
c) ESR elevation
d) Low C3 levels
a) Thymoma
b) Neurofibroma
c) Pericardial cyst
d) Bronchogenic cyst
Diabetic Nephropathy Three lesions are encountered: (1) glomerular lesions; (2)
renal vascular lesions, principally arteriolosclerosis; and (3) pyelonephritis, including
necrotizing papillits.
19. A 23 year old individual with serum alkaline phosphatase normal, PTH
normal, Vitamin D3 normal with elevated serum Calcium values what will be
the diagnosis ?
a) Vitamin D intoxication
b) Hyperparathyroidism
c) Multiple myeloma
d) Nutritional rickets
Multiple myeloma is a plasma cell neoplasm commonly associated with lytic bone
lesions, hypercalcemia, renal failure, and acquired immune abnormalities.
Clinical Features.
The clinical features of multiple myeloma stem from
(1) the effects of plasma cell growth in tissues, particularly the bones
(2) the production of excessive Igs, which often have abnormal physicochemical
properties; and
(3)The suppression of normal humoral immunity.
Bone resorption often leads to pathologic fractures and chronic pain. The attendant
hypercalcemia can give rise to neurologic manifestations, such as confusion,
weakness, lethargy. constipation, and polyuria, and contributes to renal dysfunction.
Decreased production of normal lgs sets the stage for recurrent bacterial infections.
Cellular immunity is relatively unaffected. Of great significance is renal insufficiency,
which trails only infections as a cause of death. The pathogenesis of renal failure
,which occurs in up to 50% of patients, is multifactorial. However, the single most
important factor seems to be Bence Jones proteinuria, as the excreted light chains
are toxic to renal tubular epithelial cells. Certain light chains (particularly those of the
λ6 and λ3 families)are prone to cause amyloidosis of the AL type, which can
exacerbate renal dysfunction and deposit in other tissues as well.
a) Mycoplasma pneumoniae
b) Klebsiella pneumoniae
c) Hemophilus influenzae
d) Chylamydia
MC cause of Atypical pneumonia- Mycoplasma Pneumonia aka "Eaton's agent".
This organism does not have cell wall which makes them resistant to the treatment
by Penicillin's.
The infection will be treated by the use of Macrolide antibiotic (Azithromycin).
a) NaF
b) Fluoroacetate
c) iodoacetate
d) Potassium oxalate
Potassium oxalate--anti-coagulant
BCAA b6 BCKAL
Transamination α-keto acid decarboxylase
Branched chain amino acid - valine, leucine, isoleucine this amino acid are
essential amino acid
Maple syrup urine disease - defect in branched chain amino acid metabolism
Enzyme - alpha keto acid dehydrogenase alpha- keto acid decarboxylase
C/F- burnt sugar like odour in urine
Ketosis
Mental retardation
Abnormal muscle tone
Coma & death
a) Glycine
b) Arginine
c) Phenyl - alanine
d) Tyrosine
Collagen is a structural protein that contains high amounts of glycine. In fact, every
third to fourth amino acid in collagen is glycine. Collagen is the most abundant
protein in your body. It provides strength for your muscles, skin, cartilage, blood,
bones and ligaments.
The triple-helical structure of collagen arises from an unusual abundance of three
amino acids: glycine, proline, and hydroxyproline.
a) Arginine
b) Histidine
c) Glycine
d) Aspartate
Nitric oxide (NO) is a gaseous molecule with autocrine and paracrine effects on
many cell types. NO is synthesized from the amino acid l-arginine by NO
synthase (NOS) and is involved in a myriad of cellular functions, including muscle
relaxation, neuronal signaling, and immune function.
a) NADPH
b) TPP
c) FAD
d) NAD
a) NADP
b) FAD
c) NAD
d) SAM
a) Mc Ardle's disease
b) Von- Gierke's disease
c) Her's disease
d) Pompe's disease
Glycogen glucose
glycogen phosphorylase
Anaerobic Glycolysis
Lactate
(PH ↓ )
8. Most potent lipid phase antioxidant:
a) Vitamin A
b) Vitamin E
c) Vitamin C
d) Vitamin K
Vit E delays the progression of atherosclerotic plague because its convert oxidased
LDH to Normal LDH
Vit E act simultaneously with selenium & selenium is part of glutathione peroxidase
which is involved in the oxidant Pathway in cell
a) Vit B1
b) Vit B2
c) Vit B7
d) Vit B12
Vit B1 deficiency seen in chronic alcoholics & lead to Wernicke's Korsakoff psychosis
and patient has memory loss / confabulation ,cerebral hemorrhage, muscles
paralysis
Vit B2 → riboflavinosis → 2C
Corneal Vascularization
Cheilosis
a) VMA
b) Catecholamine
c) HIAA
d) Serotonin
Vanillylmandelic acid (VMA) and other catecholamine metabolites (homovanillic
acid: HVA and dopamine) are typically elevated in patients with catecholamine-
secreting tumors example: neuroblastoma, pheochromocytoma, and other neural
crest tumors.
a) Mast cells
b) Eosinophils
c) T-cells
d) NK cells
1. Grounds of divorce:
a) Sterility
b) Frigidity
c) Impotence developing after the marriage
d) Poverty
a) 312
b) 313
c) 314
d) 315
IPC - Abortion
Section Defines/Punishes
Sec. 312 1.P.C. whoever voluntarily causes criminal abortion is liable for
imprisonment up to three years, and/or fine; and if the woman is quick with child the
imprisonment may extend up to seven years. It is necessary that the woman should
be pregnant and that abortion should be carried with her consent. Both the person
causing the abortion and the woman are liable for punishment.
Sec. 313 IPC: if the miscarriage is caused without the consent of the woman, the
imprisonment may be up to ten years.
Sec. 314 PC: if a pregnant woman dies from an act intended to cause miscarriage,
the offender is liable to be punished with rigorous imprisonment which shall not be
less than ten years and also fine up to two lakh rupees.
Sec. 315 IPC: a person doing an act intended to prevent the child from being born
alive or to cause to die after its birth, is liable to be punished with imprisonment up to
ten years.
Sec. 316 IPC: causing death of quick unborn child by any act amounts to culpable
homicide, and the punishment may extend up to ten years imprisonment.
Toxic substances may be held in high concentrations in the rugae and crypts of the
mucosa in the actual stomach wall.
The pathway of acids and alkalis in food-filled stomach starts along the lesser
curvature of the stomach and leads to the pylorus, which explains the location of
greatest damage in stomach.
a) Bagging
b) Spraying
c) Huffing
d) Sniffing
Huffing: Applying the chemical to a cloth/rag and then inhaling by covering nose and
mouth with the cloth/rag.
Bagging: Placing the chemical in a bag and then putting it over the face.
5. One of the victims of the fire which had destroyed 6 huts in a nearby slum
has decided to give a dying declaration. The doctor was called in as part of
the team to record the statement. Which among the following is the role of
the doctor ?
The law does not provide who can record dying declaration, nor is there any
prescribed form, format or procedure for the same.
The doctor should certify that the person is conscious and his mental faculties are
normal, i.e. he is in compos mentis.
1. Inertization is:
Inertization technique is used when large volume of toxic Bio medical waste is
converted to non- toxic waste.
It is achieved by mixing,
15% lime and
15% cement with
65% biomedical waste
5% water
a) Pig
b) Dogs
c) Cats
d) Birds
Japanese Encephalitis
a) Carrot
b) Halibut liver oil
c) Cod fish oil
d) Cow milk
Halibut Liver Oil has the maximum natural content of Vitamin A & also it has the
richest source of the Vitamin D.
Among fruits Ripe Mango & among vegetables carrot have high content of
Vitamin A.
Vitamin A deficiency causes Xerophthalmia.
a) Quetelet index
b) Chandler index
c) Pearl index
d) Sullivan's index
BMI is still referred to as the Quetelet index, for which the formula is: BMI (kg/m2) =
mass (kg) / height (m)2.
a) Yellow fever
b) Polio
c)HIV
d) Malaria
The International Health Regulations (WHO IHR) does not cover HIVAIDS
Immediately Notifiable Diseases: Smallpox, Human Influenza, Wild Polio, SARS
Potentially Notifiable Diseases:
Public health importance: Cholera, Plague, Yellow fever, Viral hemorrhagic
fevers, West nile ever, Dengue, Rift valley fever, Meningococcal disease
Biological/Chemical/Radiological events
Serious illnesses of Unknown origin
a) Acute gastroenteritis
b) Pneumonia
c) Leptospirosis
d) Malnutrition
Extra edge:
Chairman of National Disaster Management Authority (NDMA) of India is Prime
Minister of India
a) 65 mmol/L
b) 75 mmol/L
c) 20 mmol/L
d) 100 mmol/L
10. As compared to cow milk, the protein content of human milk is:
a) Normal saline
b) Distilled water
c) Ringer lactate
d) Distilled water
BCG is available in Powder form, which is diluted with Normal saline and it should be
used within 4 hours,
Otherwise Toxic shock syndrome can occur.
Strain →Danish - 1331 •
Dose → 0.1 ml intradermally in left deltoid in newborns within 28 days. -
Efficacy of dose →0 to 80%
Protective duration → 20 years
QUARANTINE ISOLATION
Is done for Healthy contact of a case Cases until recovery
with maximum Insulation
period
Level of prevention Primary Secondary
14. A 26 year old pregnant female visited the sub centre for her check-up.
Which of the following test is done at Subcentre during pregnancy?
a) USG
b) Haemoglobin
c)OGTT
d) Triple test
15. 25 year old patient with diarrheal episodes presented to the hospital. The
physician started his treatment with ORS. In preparation of ORS sugar is
added, Which of the following is the correct reason to add sugar in ORS?
a) Enhance acceptability
b) Enhance salt absorption
c)Enhance shelf life
d) Enhance taste
Osmolar (mmol/liter)
concentration
Sodium 75
Potassium 20
Chloride 65
Citrate 10
Glucose 75
anhydrous
Total 245
a) Vitamin A
b) Vitamin B
c) Vitamin C
d) Vitamin E
a) Thiamine
b) Biotin
c)Niacin
d) Hydroxycobalamin
Thiamin (Vitamin B1) deficiency leads to Beri- Beri seen in polished rice eaters.
In polished rice outer layer of rice is removed which contains Thiamine, it is
more commonly in Andhra Pradesh & Telangana States.
Thiamine deficiency also causes Wernicke's encephalopathy & Wernicke's
Korakoff psychosis (seen in alcoholics)
a) Phase1
b) Phase 2
c)Phase 3
d) Phase 4
Phase 1: Healthy volunteers are re elevated to establish safety & Non- toxicity of
drugs.
Phase 2: Patients are used to record the efficiency of the drug.
Phase 3: Patients are used mainly for compassion of existing drugs.
Phase 4:
Done on patients where post marketing surveillance of rare and long term side
effects is evaluated.
This is the longest phase of trail with minimum 10 to 25 years preferably till
lifelong.
Phase 1 → tell the Maximum Tolerated Dose of the drug.
Phase 2 → Maximum Drug Failure occurs.
Phase 3 → Is a Randomized control trail
Phase O → Used before phase with healthy volunteers with 1/10th of the dose
(micro dosing) with very less volunteers
19. A 29 year old male patient with abdominal pain, fatigue, dark colored urine,
clay colored stools and yellow sclera presented to the hospital. On
serological examination the patient is showing HBsAg positive and HBeAg
positive. Which of the following is the correct clinical diagnosis ?
a) Thermophilic Actinomycetes
b) Aspergillus fumigatus
c) Aspergillus flavus
d) Aspergillus niger
Farmers lung-exposure to mouldy hay contaminated with Thermophilic
Actinomycetes/ Micropolyspora Faeni .
Aspergillus fumnigatus and flavus are opportunistic pathogens and common causes
of infections in humans.
Pneumoconiosis occurs with particles of size 0.5- 3μ diameter.
a) Vitamin C
b) Phytates
c)Hemoglobin
d) Myoglobin
a) 2 million / mL
b) 15 million /mL
c)10 million / mL
d) 20 million ml
a) Cataract
b) Refractive error
c)Trachoma
d) Glaucoma
The dicrotic notch is a small and brief increase in arterial blood pressure that
appears when the aortic valve closes. The dicrotic notch would therefore be the
result of a short period of backward flow of blood immediately before the aortic valve
closes.
ISOVOLUMETRIC RELAXATION:
Heart:
At the beginning of this phase the AV valves are closed.
Heart sounds:
The second heart sound (S2, "dup") occurs when the semilunar (aortic and
pulmonary) valves close. S2 Is normally split because the aortic valve closes slightly
earlier than the pulmonary valve.
a) Hypoxic
b) Anaemic
c)Stagnant
d) Histotoxic
Histotoxic hypoxia (also called histoxic hypoxia) is the inability of cells to take up
or use oxygen from the bloodstream, despite physiologically normal delivery of
oxygen to such cells and tissues but the oxygen delivery to tissue is normal.
a) 8 times
b) 16 times
c)32 times
d) 256 times
Vessel resistance (R) is directly proportional to the length (L) of the vessel and the
viscosity (n) of the blood, and inversely proportional to the radius to the fourth power
(r4)
So when radius is increased by 2 times blood flow decreases by 16 times.
a) 4-6
b) 6-12
c) 12-24
d) 24-50
The cation normally measured is Nat, and the anions are usually Cl® and HCO3 The
"anion gap" is the difference between unmeasured anions and unmeasured cations
and is estimated as
1. Biot breathing
2. Apneustic breathing
3. Cheyne stokes breathing
4. Kussmaul breathing
9. A 7-day old infant presents with bilious vomiting and gross abdominal
distention with absent bowel sounds. X ray abdomen shows multiple gas
filled loops. Diagnosis is?
a) Hirschsprung disease
b) Congenital Hypertrophic pyloric stenosis
c)Duodenal atresia
d) Malrotation of gut
Malrotation of gut:
Surgical emergency
Bilious vomiting - first sign of volvulus
Abdominal distention
Multiple gas filled loops
10. Newborn infant presents with abdominal distension and bilious vomiting.
There is failure to pass meconium beyond 24 hours. X-ray shows dilated
small bowel loops with no air fluid levels and soap bubble appearance in
right lower quadrant. This condition is initially treated by:
a) Paul mikulicz ileostomy
b) Bishop ileostomy
c) Contrast enema
d) Barium enema
Infant presents with signs and symptoms of small bowel obstruction (abdominal
distension, bilious vomiting and inability to pass meconium). X ray shows typical
features, suggestive of meconium ileus
Uncomplicated meconium ileus is initially treated by contrast enema failure of
which surgical management is advocated.
11. A full-term, 2.0 Kg male baby, delivered at home presented with excessive
frothing within a few hours of birth. The antenatal period was unsupervised
and the delivery, uncomplicated. At presentation, the baby had mild
respiratory distress. The first investigation to be done in this case IS:
Two problems
Collection of saliva in the mouth
Air coming inside the mouth
Therefore, saliva is not going into the esophagus
Esophageal atresia with tracheoesophageal fistula
Diagnosis:
The inability to pass a NG int the stomach is a cardinal feature for the diagnosis
of EA
If gas is present in the GIT below the diaphragm, an associated TEF is confirmed
Inability to pass a NG tube with absent radiographic evidence for gastrointestinal
gas is virtually diagnostic of an isolated EA without TEF
For H-type: tracheobronchoscopy + endoscopy should be performed
12. Calculate the GCS of a patient exhibiting eye opening on pain, conscious
but confused and cannot tell time and exhibits flexion on painful noxious
stimuli to the arm.
a) 8
b) 9
c) 10
d) 11
Treatment of PCT:
Total thyroidectomy (TOC)
Radioactive iodine (RAI)
Contraindications of RAI:
Pregnancy
Lactation
a) Excisional surgery
b) Intramarginal excision followed by radiation
c)Cryosurgery
d) Silicone Gel Sheeting
Keloids rarely regress with time, often refractory to medical and surgical
intervention
First line treatment: silicone in combination with pressure therapy and
intralesional corticosteroid injection
Refractory cases (after 12 months of therapy): excision + post op radiotherapy
(external beam or
brachytherapy)
New treatment modalities: internal cryotherapy and 5% imiquimod
a) Splenic laceration
b) Splenic contusion
c) Mesenteric tear
d) Mesenteric adenitis
Mesenteric adenitis is a condition that affects the lymph nodes in the membrane
which connects bowel to the abdominal walll (mesentry). A viral infection is the usual
cause.
a) Neuralgia
b) Deep vein thrombosis
c)Acrocyanosis
d) Telangiectasia
a) Suction drainage
b) Varicocelectomy
c)Jaboulay procedure
d) Herniotomy
VARICOCELE
Bag of worm like sensation
Dilated tortuous pampiniform plexus of vein
Treatment- Varicocelectomy/ high inguinal ligation of pampiniform plexus of veins
Jabuloy procedure is eversion of sac performed for median sized hydrocele
Herniotomy is preferred for Hernia
Prophylactic antibiotic to minimize SSI are given 30- 60 minutes before skin incision /
at the time of induction
Pheochromocytoma
a) Artery forceps
b) Kocher forceps
c)Allis forceps
d) Babcock forceps
Artery forceps are used to compress an artery to stem bleeding, because of this
they belong to a group of surgical instruments called hemostats.
a) Tongue depressor
b) Doyen retractor
c)Self-retaining retractor
d) Langenbeck's retractor
Tongue depressor
Used in surgery department for evaluation of oral cavity
Used in ENT department generally
a) Sebaceous cyst
b) Implantation dermoid
c)Angular dermoid
d) Lipoma
a) Lipoma
b) Encephalocele
c)Cystic hygroma
d) Lymphadenopathy
26. 40 year old male presented with complaints of dysphagia ,regurgitation and
foul smelling mouth. On further evaluation he was diagnosed to have
Zenker's diverticulum. This diverticulum is due to herniation of esophagus
through the Killian's dehiscence which is located:
a) Cornea
b) Gut
c)Liver
d) Skin
Graft vs Host disease
Maximum chance of Graft vs Host disease is GUT transplantation/small bowel
transplantation because Donor's small bowel- presence of leucocytes in the lumen
a) Kidney
b) Heart
c)Lung
d) Liver
a) Vitamin D toxicity
b) Hyperparathyroidism
c) Osteoporosis
d) Osteomalacia
Bio-Chemical Changes
Osteoporosis Osteomalacia Hyperparathyroidism
Calcium Normal ↓/ Normal Increased
Phosphate Normal ↑ Low (phosphateuria)
ALP Normal ↑ -
PTH - ↑ -
a) Elbow fracture
b) Monteggia fracture
c)Galeazzi fracture
d) Colles' fracture
4. Management of olecranon fracture as seen in the image is:
Given X ray shows olecanon fracture (type 2)- Tension band wiring
TREATMENT:-
Type I → above-elbow plaster slab in 30 degree of flexion
Type II → open reduction and internal fixation using tension-band wiring
Type III→ plating or tension-band wiring or excision of the fragments
5. The diagnosis of the given image
The given x-
ray shows
the Terry-
Thomas sign
and the
image shows
comedian
Terry-
Thomas,
after whom
this sign has
been named:
6. Diagnosis of Gout is confirmed by which test?
a) Cataract
b) Refractive error
c)Trachoma
d) Glaucoma
a) Ipsilateral mydriasis
b) Contralateral mydriasis
c) Ipsilateral miosis
d) Contralateral miosis
Foster-Kennedy syndrome.-
It is associated with olfactory or sphenoidal meningioma and frontal lobe
tumors.
In this condition, there occurs pressure optic atrophy on the side of lesion and
papilledema on the other side (due to raised intracranial pressure).
Ipsilateral central scotoma
Ipsilateral anosmia
Evisceration:
The contents of the eye are removed, but the outer layer of the eyeball (sclera) is left
intact.
Enucleation
The entire eye, including the globe, is removed but the orbital contents are left in
place.
Exenteration
The contents of the eye socket are removed, including the muscles, lacrimal gland,
optic nerve and various bones of the orbit.
5. The following image shows ?
a) Bacterial keratitis
b) Fungal keratitis
c) Viral keratitis
d) Syphilitic keratitis
Fluorescein staining
May reveal dendritic lesions (linear branching patterns with terminal bulbs)
If the lesion enlarges, it can form a geographic ulcer
Dendritic ulcer
Finger like projections
Feathery margins
Satellite lesions may be seen
Hypopyon
a) Tear drops
b) Steroids
c)Cycloplegics
d) Timolol
a) Arteriolar attenuation
b) Neovascularization
c)Retinal artery thrombosis
d) Papilledema
Ishihara chart
Most commonly used for color vision testing
9. Where is the lesion that produces the visual field defect shown in the image
?
a) Craniopharyngioma
b) Meningioma
c)Pituitary adenoma
d) Temporal lobe lesion
LESIONS
10. Which of the following field defects occurs due to a lesion at the optic
chiasma?
a) Bitemporal hemianopia
b) Unilateral blindness
c) Pie on the floor
d) Bilateral homonymous macular defect
BITEMPORAL HEMIANOPIA
Nasal fibres gets damaged
Caused by optic chiasmal lesion → pituitary adenoma (mc)
LESIONS
11. In which of the following condition there is ipsilateral 3rd nerve palsy with
contralateral hemiplegia and facial palsy of upper motor neuron?
a) Weber syndrome
b) Terson syndrome
c) Millard-Gubler syndrome
d) Foville's syndrome
Weber's syndrome
Ipsilateral 3rd nerve Palsy
Contralateral Hemiplegia
Facial palsy of upper motor neuron
12. All among the following are incorrect statements regarding lens induced
glaucoma except :
a) Phacomorphic glaucoma is an open angle glaucoma
b) In phacolytic glaucoma lens capsule is intact.
c) Phacomorphic glaucoma has a deep anterior chamber
d) Mature cataract is an example of phacolytic glaucoma
a) Circumcorneal congestion
b) Hypopyon
c) Retrolental flare
d) Iris nodule
1. A 25-year-old female presents with history of fever and oral ulcers and has
developed erythematous lesions on her face. Comment on the diagnosis.
a) SLE
b) Dermatomyositis
c)Melasma
d) Rosacea
Systemic Manifestation
Fever
Joint pain
Oral ulcer
Photosensitivity
Melasma
Brownish
Malar area and Nose
Rosacea
Erythematous lesion + Telangiectasia
Convex areas
Dermatomyositis → Heliotrope rash → Violaceous → eye
2. The following lesion was noticed in a patient with history of involuntary
weight loss. What is the diagnosis?
a) Acanthosis nigricans
b) Leser-Trélat sign
c)Actinic keratosis
d) Intertriginous Candida
Herpes zoster
Caused by Varicella zoster virus
Meningoencephalitis can be due to varicella zoster virus
In Ramsay hunt syndrome, painful grouped fluid filled lesions →vesicles →Herpes
zoster oticus.
a) Intralesional steroids
b) Surgery
c)Cryotherapy
d) Electrocoagulation
SURGERY:
Treatment of keloid:
Silicon dressing
Mechanical compression
Intralesional steroids [Doc- keloids]
a) Genital wart
b) Pearly penile papule
c) Balanitis
d) Sebaceous gland prominence
Pearly papules :
Asymptomatic
Seen in uncircumcised patients
Etiology unknown
Not infective
Normal lesions
No treatment required
a) Molluscum contagiosum
b) Chicken pox
c)Pemphigus
d) Common warts
a) Aortic dissection
b) Pulmonary embolism
c)Cardiac myxoma
d) Aortic aneurysm
NORMAL CT
Aortic dissection
Aortic dissection:
a) MUGA
b) SPECT using thallium 201
c)PET myocardial perfusion imaging
d) Sestamibi scan with pharmacological stress
3. Refer to the image given below and give the correct diagnosis?
a) Diaphragmatic hernia
b) Intestinal obstruction
c)Pleural effusion
d) Pneumonia
Diaphragmatic hernia is a birth defect where there is a hole in the diaphragm (the
large muscle that separates the chest from the abdomen). Organs in the abdomen
(such as intestines, stomach, and liver) can move through the hole in the diaphragm
and upwards into a baby's chest.
a) X-ray abdomen
b) Chest X ray
c)IV pyelography
d) Barium Enema
X-ray abdomen- Single spot film, therefore comparatively less radiation exposure.
Chest X ray- Single spot film, therefore comparatively less radiation exposure.
IV pyelography- Sequential/ multiple imaging but covers less area, therefore.
comparatively less radiation
exposure.
PET CT - Has highest radiation exposure.
5. In the shown lateral view, the structure marked with arrow head is?
a) Esophagus
b) Pulmonary artery
c) Trachea
d) Left atrium
6. A patient presented with complaints with dyspnea. The shown X- ray is
suggestive of:-
a) Consolidation
b) Exudative Pleural Effusion
c)Pnemothorax
d) Hydropnemothorax
ELLIS CURVE
150-200 ml on PA view
5 ml lateral view
15-20 ml on lateral decubitus view
Damoiseau-Ellis line is a clinical sign, that refers to upper limit of dullness
to percussion caused by pleural effusion. It has characteristic shape of
curved line with highest point at middle axillary line
a) Supraglottis
b) Glottis
c) Subglottis
d) Epiglottis
2. A 15 year old male patient presented with nasal mass reaching upto cheek
and causing unilateral obstruction with intermittent epistaxis. Most likely
diagnosis is:
a) Angiofibroma
b) Nasal polyp
c)Nasopharyngeal Carcinoma
d) Inverted Papilloma
a) Hearing Aids
b) BAHA
c) Cochlear implant
d) Auditory Brainstem Implant
Bilateral severe to profound H.L. that is not adequately treated with standard
hearing aids.
Congenital H.L./ prelingual deafness
Acquired H.L./ post lingual deafness
Severe hearing loss that can be aided and that deteriorates to profound loss in
childhood, adolescence, or adulthood (prelingual).
8. Which of the following muscles is the lifesaving muscle of larynx that pulls
back vocal cords in order for breathing to take place
a) Cricothyroid
b) Posterior Cricoarytenoid
c)Lateral Cricoarytenoid
d) Transverse Arytenoid
Nasopharyngeal carcinoma
Due to obstruction of eustachian tube there is serous otitis media and leads to
hearing loss.
M/C site of origin → Fossa of Rosenmuller
M/C type → Squamous cell carcinoma
M/C Presentation →cervical lymphadenopathy (Neck Mass)
a) Cervical lymphadenopathy
b) Hearing Loss
c) Airway Obstruction
d) Epistaxis
Nasopharyngeal carcinoma
Due to obstruction of eustachian tube there is serous otitis media and leads to
hearing loss.
M/C site of origin → Fossa of Rosenmuller
M/C type → Squamous cell carcinoma
M/C Presentation →cervical lymphadenopathy (Neck Mass)
a) Anterior Commissure
b) Posterior Commissure
c) Epiglottis
d) False Vocal Cord
Anterior commissure can't be seen because of bulge of anterior wall of larynx
which hides the anterior commissure
a) Superior Constrictor
b) Middle constrictor
c) Inferior Constrictor
d) Thyroepiglottis
a) Cough
b) Change of voice
c) Fatigue voice
d) Pain
a) Facial Nerve
b) Glossopharyngeal
c) Vagus Nerve
d) Hypoglossal Nerve
a) Genioglossus
b) Hyoglossus
c) Palatoglossus
d) Styloglossus
The genioglossus arises from the mandible and protrudes the tongue. It is also
known as the tongue's "safety muscle" since it is the only muscle that propels the
tongue forward. The hyoglossus, arises from the hyoid bone and retracts and
depresses the tongue.
Embryologically, the trigone of the bladder is derived from the caudal end of
mesonephric ducts, which is of mesodermal origin (the rest of the bladder is
endodermal).
a) Biceps femoris
b) Quadriceps femoris
c) Addactur magnus
d) Quadratus
The knee-jerk reflex, also known as the patellar reflex, is a simple reflex that
causes the contraction of the quadriceps muscle when the patellar tendon is
stretched.
5. Celiac artery is seen at which vertebral level?
a) T9
b) T10
c) T12
d) L2
The celiac trunk, also known as the celiac artery, is a short vessel that arises
from the aorta and passes below the median arcuate ligament, just as the aorta
enters the abdomen at the level of the T12 vertebra.
a) L3
b) L4
c) S1
d) S2
The top of the iliac crests also marks the level of the fourth lumbar vertebral
body (L4), above or below which lumbar puncture may be performed.
7. 45-year-old policeman comes from duty with complain of pain in right leg.
On examination, pulse in right popliteal fossa was absent and pain in right
buttock, whereas in left side it was normal. The block is at which level?
a) Femoral
b) Iliac
c) Aortoiliac
d) Popliteal
Aortoiliac occlusive disease is a type of vein condition in the legs. It occurs when
the iliac artery that brings blood to your legs becomes narrow or blocked by plaque.
Symptoms can include pain, numbness, or cramping in the lower limbs, gangrene in
the feet, and erectile dysfunction (ED) in men.
a) Ciliary ganglia
b) Otic ganglia
c) Pterygopalatine
d) Submandibular
The postganglionic axons are the output of the pterygopalatine ganglion project to
the lacrimal gland and supply the parasympathetic innervation of the lacrimal gland
a) Facial nerve
b) ICA
c) Branch of ECA
d) Retromandibular vein
The facial nerve and its branches pass through the parotid gland, as does the
external carotid artery and retromandibular vein. The external carotid artery
forms its two terminal branches within the parotid gland: maxillary and superficial
temporal artery
a) Serratus anterior
b) Pectoralis major
c) Pectoralis minor
d) Lateral pterygoid
a) Maxillary artery
b) Mandibular
c) Facial artery
d) Ophthalmic artery
The eustachian tube opens upon swallowing or yawning by contraction of the tensor
veli palatini muscle.
a) Radial nerve
b) Ulnar nerve
c) Median nerve
d) Musculocutaneous nerve
The musculocutaneous nerve innervates the three muscles of the anterior
compartment of the arm: the coracobrachialis, biceps brachii, and brachialis. It is
also responsible for cutaneous innervation of the lateral forearm.
a) Pterion
b) Glabella
c) Asterion
d) Nasion
The pterion is the H-shaped formation of sutures on the side of the calvarium
representing the junction of four skull bones: the greater wing of the sphenoid bone,
squamous portion of the temporal bone, frontal bone, parietal bone.
The posterior tibial artery (PTA) is one of the 2 branches of the tibioperoneal
trunk in the lower leg and provides oxygenated blood to the posterior
compartments of the leg and plantar surface of the foot. It is accompanied by the
posterior tibial vein, along its course.
a) Thymoma
b) Lymphoma
c) Neurogenic tumor
d) Neuroblastic tumor
The most common mediastinal masses are neurogenic tumors (20% of mediastinal
tumors), usually found in the posterior mediastinum, followed by thymoma (15–20%)
located in the anterior mediastinum. Lung cancer typically spreads to the lymph
nodes in the mediastinum.
a) Fibrous astrocytoma
b) Neuroglioma
c) Ependymoma
d) Medulloblastoma
Medulloblastomas. The most common type of embryonal tumor, these fast-growing
cancerous brain tumors start in the lower back part of the brain, called the
cerebellum. The cerebellum is involved in muscle coordination, balance and
movement.
a) Hereditary spherocytosis
b) Sickle cell anaemia
c) Osteogenesis imperfecta
d) VWD
21. A patient has MCV < 80, MCH < 23. Which type of anemia shall be
classified?
a) Normocytic hypochromic
b) Microcytic hypochromic
c) Hyperchromic macrocytic
d) Normocytic normochromic
Erythrocytes that have a normal size or volume (normal MCV) are called
normocytic,
When the MCV is high, they are called macrocytic.
When the MCV is low, they are termed microcytic.
Erythrocytes containing the normal amount of hemoglobin (normal MCHC) are
called normochromic.
When the MCHC is abnormally low they are called hypochromic, and when
the MCHC abnormally high, hyperchromic.
MCV MCHC
Female: 80-95 fl 30-34
gHb/100ml
Male: 80-95 fl 30-34
gHb/100ml
a) Calcium
b) Sodium
c) Potassium
d) Chloride
a) Microcytic, hypochromic
b) Macrocytic, Hypochromic
c) Normocytic Normochromic
d) Macrocytic, Normochromic
a) Totipotent
b) Pluripotent
c) Multipotent
d) None of above
Totipotent stem cells can differentiate into embryonic and extraembryonic cell types.
These cells are produced from the fusion of an egg and sperm cell, and can
construct a complete, viable organism. Cells produced by the first few divisions of
the fertilized egg are also totipotent.
25. 16-year-old girl with short height, amenorrhea with widely spaced nipple.
karyotyping is;
a) 45 X0
b) 46 XO
c) 47 XXX
d) 46XY
a) Coagulative
b) Liquefactive
c) Fat
d) Fibrinoid
a) Loud S1
b) Loud S2
c) Systolic murmur
d) S4
a) Mumps
b) Measles
c) Rubella
d) Varicella
Koplik spots are seen with measles. They are small, white spots (often on a
reddened background) that occur on the inside of the cheeks early in the course of
measles.
29. NITI AAYOG Represents;
National Institution for Transforming India, better known as NITI Aayog, was formed
via a resolution of the Union Cabinet on 1 January 2015. NITI Aayog is the premier
policy think tank of the Government of India, providing directional and policy inputs.
30. On republic day, a camp was organized and people were screened for
Hypertension by checking BP and for diabetes by checking their BMI and
Blood sugar level, which level of prevention is this?
a) Primary
b) Primordial
c) Secondary
d) Tertiary
31. A person comes with right eye 6/60 and left eye 3/60. He shall be
categorized into which type of blindness?
a) Manifest
b) Social
c) Economical
d) No blindness
Types of Blindness:
a) Red
b) Blue
c) Yellow
d) White
a) Child health
b) Nutritional health
c) Mental health
d) Social health
UNICEF is mandated by the United Nations General Assembly to advocate for the
protection of children's rights, to help meet their basic needs and to expand their
opportunities to reach their full potential.
Subacute sclerosing panencephalitis (SSPE) is a very rare, but fatal disease of the
central nervous system that results from a measles virus infection acquired earlier
in life. SSPE generally develops 7 to 10 years after a person has measles, even
though the person seems to have fully recovered from the illness.
a) Gossypol
b) Nonoxynol-9
c) Centchroman
d) Clomiphene
a) Water sanitization
b) Food and nutrition
c) To reduce greenhouse gas emission
d) Water chlorination
a) UNICEF
b) United Nations
c) WHO
d) Government of India
a) Chloroquine
b) Quinine
c) Lumefantrine
d) Pyrimethamine
39. Worms, bacteria and bugs attached in clothing of a person. This is;
a) Infection
b) Infestation
c) Contamination
d) Contagion
Target couples :
The term target couples was applied to couples who have had 2-3 living children, or
even newly married couples to make them accept the idea of family planning from
the earliest possible stage.
a) Life long
b) 10 years
c) 5 years
d) 20 years
As of 11 July 2016, a certificate of vaccination against yellow fever (ICVP) is valid
for the lifetime of the person vaccinated. Prior to 2016, yellow fever vaccination
certificates were valid for a period of 10 years, commencing 10 days after the date of
vaccination.
a) Specific protection
b) Health promotion
c) Early diagnosis and treatment
d) Rehabilitation
SPECIFIC PROTECTION:
Idea is to avoid disease altogether. Currently available interventions include:
Immunization
Use of specific nutrients
Chemoprophylaxis
Protection against occupational hazards
Protection against accidents
Protection from carcinogens
Avoidance of allergens
Better environmental conditions e.g. control of air and noise pollution
Control of consumer product quality and safety of foods, drugs, cosmetics.
a) Triage
b) Tagging
c) Mitigation
d) Surge capacity
Triage is the process of quickly examining sick or injured people, for example
after an accident or a battle, so that those who are in the most serious condition can
be treated first.
44. Vision 2020 - which was added recently in India?
a) Cataract
b) Childhood blindness
c) Glaucoma
d) Refractive errors and low vision
Glaucoma is now regarded as a sort of “missing link” and VISION 2020 India
successfully advocated for the inclusion of glaucoma in the next phase by the Indian
government.
a) Kala azar
b) Oroya Fever
c) Chagas disease
d) Cutaneous leishmania
a) Pox virus
b) Toga virus
c) Coronavirus
d) Lyssa virus
Corona viruses are a large family of viruses which may cause illness in animals or
humans. In humans, several coronaviruses are known to cause respiratory infections
ranging from the common cold to more severe diseases such as Middle East
Respiratory Syndrome (MERS) and Severe Acute Respiratory Syndrome (SARS)
.
a) Airborne Precautions
b) Droplet Precautions
c) Contact Precautions
d) None of the Above
a) Mean
b) Range
c) Variance
d) Standard deviation
Measures of dispersion describe the spread of the data. They include the range,
interquartile range, standard deviation and variance. The range is given as the
smallest and largest observations.
a) Silica
b) Asbestos
c) Coal dust
d) Sugar cane waste
50. Breast feeding should be continued for minimum how many months?
a) 6 months
b) 2 months
c) 12 month
d) 5 months
The World Health Organization recommends that all babies be exclusively breastfed
for 6 months, then gradually introduced to appropriate foods after 6 months while
continuing to breastfeed for 2 years or beyond. Stopping breastfeeding is called
weaning.
51. Components of Kuppuswamy scale-
The modified Kuppuswamy scale is commonly used to measure SES in urban and
rural areas. This scale was devised by Kuppuswamy in 1976 and consists of
a composite score which includes the education and occupation of the Family
Head along with income per month of the family, which yields a score of 3–29.
a) Transformation
b) Transduction
c) Translation
d) Conjugation
a) Fasciola hepatica
b) Schistosoma japonicum
c) Schistosoma mansoni
d) Schistosoma hematobium
54. A patient presented with following findings. what will be the media used to
diagnose this condition?
Membrane filtration and the Knott concentration method are more sensitive than
routine thick blood smears for microfilaria detection, because they can be used to
concentrate microfilariae from 1 mL or more of blood.
a) Pseudomonas
b) Staph aureus
c) Streptococcus
d) Influenza
Malignant external otitis (MEO) is an infection that affects the external auditory canal
and temporal bone. The causative organism is usually Pseudomonas aeruginosa,
and the disease commonly manifests in elderly patients with diabetes
a) H. nana
b) Schistosoma
c) T. saginata
d) T. Solium
Hymenolepis nana, the dwarf tapeworm, is the smallest and a common tapeworm
in humans worldwide. H. nana infection occurs more frequently in warm climates and
temperate zones such as Asia, Central and South America, and Eastern Europe.
a) IgM
b) IgG
c) IgE
d) IgD
60. A patient who was recently got dentature fixed, found with oral thrush. It
bleeds on scraping. Causative agent for this condition
a) Diphtheria
b) Candida
c) Strep Mutans
d) Staph Aureus
Oral thrush (mouth thrush)
Also called: oral candidiasis
An infection in which the fungus Candida albicans accumulates in the mouth.
It's common in babies, people with immune deficiency and those who use
steroid sprays for asthma.
Oral thrush causes white lesions on the tongue or inner cheeks.
Unsweetened yoghurt or an anti-fungal medication can help.
a) Gardenerella
b) Mobilincus
c) Proteus
d) Coccobaccillus
a) 400 meters
b) 100 meters
c) 600 meters
d) 200 meters
a) Thermophilic
b) Mesophilic
c) Cycrophilic
d) Halophilic
a) Shigella
b) E.Coli
c) H. Pylori
d) Lactobacilli
a) Influenza virus
b) Corona virus
c) Rheo virus
d) Rabies virus
Influenza is a viral infection that attacks your respiratory system — your nose, throat
and lungs. Influenza is commonly called the flu, but it's not the same as stomach "flu"
viruses that cause diarrhea and vomiting.
66. Which of the following is cardio protective?
a) HDL
b) LDL
c) VLDL
d) CHYLOMICRON
a) Glucagon
b) Insulin
c) Cholecystokinin
d) 5 alpha hydroxylase synthase
68. Which structure of Protein is not denatured after heating upto 100 degree?
a) Primary
b) Secondary
c) Tertiary
d) Quaternary
Primary structure, such as the sequence of amino acids held together by covalent
peptide bonds, is not disrupted by denaturation.
GLUT4 expression is highest in adipose tissue and skeletal muscle, but GLUT4 is
also found in other organs such as brain, kidney and intestine
a) Hyaluronic acid
b) Chondroitin sulphate
c) Keratan sulphate
d) Dermatan sulphate
a) Tyrosine
b) Tryptophan
c) Phenylalanin
d) Alanine
a) adenine-guanine-thiamine- uracil
b) adenine-guanine- cytosine-uracil
c) adenine- guanine-cytosine-thiamine
d) None of the above
The bases in DNA are adenine (A), thymine (T), guanine (G) and cytosine (C).
RNA consists of four nitrogenous bases: adenine, cytosine, uracil, and guanine.
a) Oligomycin
b) Antimycin
c) Cyanide
d) CO2
a) Carbonic anhydrase
b) Lysyl oxidase
c) Carboxylase
d) Kinase
a) Xanthine oxidase
b) Kinase
c) Lysyl oxidase
d) Carboxylase
a) 1 Kcal
b) 4 Kcal
c) 8 Kcal
d) 2 Kcal
1 gram of protein (1 gram times 4 calories = 4 calories from protein)
a) Glutamine
b) Glycine
c) Cyteine
d) Urea
Much of the waste nitrogen produced in tissues such as skeletal muscle and brain is
packaged as glutamine. Glutamine synthase catalyzes the conversion of ammonia
and glutamate to glutamine
a) Retinol < 10
b) Beta carotene < 10
c) Bitot spot
d) Night blindness
According to the World Health Organization (WHO). One of the first signs of
vitamin A deficiency is night blindness, Young children and pregnant women are
most commonly affected. Vitamin A testing is used to help diagnose a vitamin A
deficiency in people with signs and symptoms, such as night blindness, or in people
who are at risk of vitamin A deficiency.
a) PDH
b) Pyruvate kinase
c) Transketolase
d) Kinase
a) Glycine
b) Cysteine
c) Valine
d) Lysine
The triple-helical structure of collagen arises from an unusual abundance of three
amino acids: glycine, proline, and hydroxyproline. These amino acids make up the
characteristic repeating motif Gly-Pro-X, where X can be any amino acid.
Joint fluid test (Synovial fluid, also known as joint fluid, is a thick liquid located
between your joints). Your doctor may use a needle to draw fluid from your
affected joint. Urate crystals may be visible when the fluid is examined under a
microscope.
a) Syncytiotrophoblast
b) Yolk sac
c) Cytotrophoblast
d) Placenta
84. Lady presents with infertility and diagnosed with bilateral cornual block
on hysterography. What is the next step?
a) IVF
b) Tuboplasty
c) Laproscopy and hysteroscopy
d) USG
a) Bacterial vaginosis
b) Candidiasis
c) Trichomoniasis
d) Chlymadia infection
a) Leydig cell
b) Sertoli cell
c) Theca lutein
d) Granulosa cell
a) Haemorrhage
b) Uterine inversion
c) Uterine rupture
d) Placenta retraction
89. A 32-year lady with uterine fetal death after normal vaginal delivery
continues passing of urine from vagina. What is the most probable
diagnosis?
a) Bladder rupture
b) Vesicovaginal fistula
c) Urge incontinence
d) Stress incontinence
A vesicovaginal fistula is an abnormal opening that forms between the bladder
and the wall of the vagina. A fistula can be a complication after vaginal or bladder
surgery. The only way to repair this opening is through surgery, which is very often
successful.
90. A 16 years old girl with acute bleeding per vaginal was presented to
the clinic. What is the immediate management?
a) Uterine ablation
b) Uterine artery block
c) Progesterone
d) Hysteroscopy examination
91. A 67 years old female with HTN and diabetes comes with heavy
bleeding per vaginal. What is the next management?
a) Uterine ablation
b) Uterine artery block
c) Hysterectomy
d) Laser
Indications of Hysterectomy:
Fibroids 30%
Endometriosis
Uterine prolapse
Cancer of the uterus, cervix, or ovaries
Vaginal bleeding, DUB 20%
Uncontrollable PPH.
92. A lady with amenorrhea and galactorrhea;
a) Pituitary adenoma
b) Adrenal hyperplasia
c) 70 alpha hydroxylase deficiency
d) None of above
a) Only Labetalol
b) Only MgSO4
c) MgSO4 + Labetalol
d) Clonidine
labetalol and hydralazine intravenous infusion regimens are well tolerated and
effective in controlling severe hypertension in pregnant women with severe
preeclampsia in combination with magnesium sulfate.
94. A 34 years old lady with 4 children, after 5th normal vaginal delivery,
excessive bleeding after placenta removal is due to;
a) Estrogen
b) Progesterone
c) FSH and LH
d) PPH
After the placenta is delivered, these contractions help put pressure on the bleeding
vessels in the area where the placenta was attached. If the uterus does not contract
strongly enough, these blood vessels bleed freely. This is the most common cause
of postpartum hemorrhage.
95. 32 years old lady with twin pregnancy dichorionic diamniotic, first baby
breech presentation and second baby cephalic presentation. What is the
management?
a) C – Section
b) Assisted breech
c) Instrumental delivery
d) Normal vaginal delivery
Indications of caesarean section in Twins
Obstetrics Specific for twins
Placenta previa Both the fetus or even the 1st
Severe preeclampsia fetus with non-cephalic
Post C/S presentation.
Cord prolapse of the 1st Conjoined twins.
baby Collision of both heads at the brim
Abnormal uterine preventing engagement of either
contraction head.
Contracted pelvis Monochorionic twins with Twin to
twin transfusion syndrome (TTTS)
96. A lady with 36-week pregnancy with previous C section comes with low
BP, tachycardia and on USG fluid present in peritoneum. What is
diagnosis and next management?
A complete uterine rupture is a tear through the thickness of the uterine wall at
the site of a prior cesarean incision. It is a potentially life threatening condition for
both the mother and/or the baby and requires immediate surgical intervention.
Treatment of uterine rupture is immediate laparotomy with cesarean delivery and,
if necessary, hysterectomy.
97. 32 years old lady presented with pregnancy, mild bleeding and pain. On
examination uterus is tender and fetal heart sound is absent. What is
diagnosis?
a) Abruptio placenta
b) Fetal distress
c) Ectopic pregnancy
d) Placenta Previa
Placental abruption occurs when the placenta separates from the inner wall of
the uterus before birth. Placental abruption can deprive the baby of oxygen and
nutrients and cause heavy bleeding in the mother
98. A lady with 12-week pregnancy with bleeding and on examination vagina
is normal, Internal OS is closed and live birth sign on USG and fundal
height is 13 weeks. Diagnosis;
a) Threatened abortion
b) Complete abortion
c) Incomplete abortion
d) Inevitable abortion
Threatened Abortion
It is a clinical entity where the process of abortion has started but has not
progressed to a state from which recovery is impossible.
C/o - Slight bleeding
O/E - Uterus - corresponds to gestational age, Cx - Internal os is
closed
USG - live fetus with subchorionic haemorrhage
99. A newly married couple, the woman is having irregular menstruation. What
is the contraceptive of choice?
a) Barrier method
b) OCP
c) Calendar method
d) Progesterone only pills
a) OCP
b) Barrier method
c) Billing’s method
d) IUD
Viable fetus
• Immediate cesarean
Dead fetus
• Perform a destructive procedure and relieve the obstruction
• If the obstetrician is not well versed with destructive procedures then
immediate cesarean
• Immediate cesarean even in a dead baby because even a dead baby
causing obstruction can lead to rupture or severe infections
In a mediolateral episiotomy, the incision begins in the middle of the vaginal opening
and extends down toward the buttocks at a 45-degree angle.
a) CT
b) MRI
c) USG
d) Laproscopy
a) Atonic
b) Traumatic PPH
c) Clotting factor deficiency
d) None of the above
Atonic PPH
• It is the most common cause of primary PPH accounting for 90% of cases.
• The bleeding occurs as the blood vessels are not obliterated by contraction
and retraction of uterine muscle fibres
a) 600 mg
b) 200 mg
c) 400 mg
d) 100 mg
The double decidual sac sign (DDSS) is a useful feature on early pregnancy
ultrasound to confirm an early intrauterine pregnancy (IUP) when the yolk sac or
embryo is still not visualized.
It consists of the decidua parietalis (lining the uterine cavity) and decidua
capsularis (lining the gestational sac) and is seen as two concentric rings
surrounding an anechoic gestational sac.
a) Nesseria meningitis
b) GardenellaVaginalis
c) Bacteroids
d) Chalamydia
Most often, PID is caused by infection from two common STIs: gonorrhea and
chlamydia. The number of women with PID has dropped in recent years. This may
be because more women are getting tested regularly for chlamydia and gonorrhea.
109. Late rupture of tubal pregnancy is seen in;
a) Interstitium
b) Isthmus
c) Ampula
d) Fimbriae
An ectopic pregnancy occurs when a fertilized egg grows outside of the uterus.
Almost all ectopic pregnancies—more than 90%—occur in a fallopian tube. As the
pregnancy grows, it can cause the tube to burst (rupture). A rupture can cause
major internal bleeding. Very late rupture of tubal pregnancy is seen when the
implantation is at Interstitium.
a) OCP
b) IUCD
c) Mifepristone
d) Progesterone only pills
112. 25-year-old male had RTA after that he is fearful of driving vehicle and
going to that place since 6 weeks and he is having flashbacks of
incidents and he is not going for his office also. What is diagnosis?
113. 20-year-old female scared of elevator since birth. She prefers stairs
instead elevator. Diagnosis
a) Agoraphobia
b) Specific phobia
c) Social phobia
d) Acrophobia
a) Alexithymia
b) Depression
c) Anhedonia
d) Derailment
An inability to experience pleasure from activities usually found enjoyable.
COMMON CAUSES
Lack of pleasure can have causes that aren't due to underlying disease. Examples
include overwork, recent tragedy, financial problems, bad weather and boring
activities.
116. A clinician asked the patient “how is your mood”, patient replied
“up and down”, what could be the diagnosis?
a) Neologism
b) Tangentiality
c) Clanging
d) Derailment
Tangentiality is the tendency to speak about topics unrelated to the main topic of
discussion. While most people engage in tangentiality from time to time, constant
and extreme tangentiality may indicate an underlying mental health condition,
particularly schizophrenia
a) Anterior uveitis
b) Glaucoma
c)Optic Neuritis
d) Keratoconus
Anterior uveitis is an inflammation of the middle layer of the eye. This middle
layer includes the iris (colored part of the eye) and adjacent tissue, known as the
ciliary body.
a) Zeis gland
b) Meibomian glands
c) Mohl gland
d) Lacrimal gland
usually caused by Staphylococcus that infects the eyelash hair follicle. The external
hordeolum is caused by a blockage of the sebaceous (Zeis) glands or sweat (Moll)
glands. The blockage occurs at the lash line and presents as a painful red swollen
area that develops into a pustule. The internal hordeolum is caused by a blockage of
the Meibomian glands, and the pustule forms on the inner surface of the
eyelids. Hordeola may present on both the upper and the lower eyelids.
HSV (Herpes Simplex Virus) keratitis is an infection of the cornea—the clear dome
that covers the colored part of the eye—that is caused by HSV. The infection usually
heals without damaging the eye, but more severe infections can lead to scarring of
the cornea or blindness. The hallmark of HSV keratitis is the presence of multiple
small branching epithelial dendrites on the surface of the cornea, although
often times it first presents as a coarse, punctuate epithelial keratitis, which may be
mistaken for a viral keratitis
In a myopic eye, the image of a distant object is formed in front of the retina and
not at the retina itself. This defect may arise due to (i) excessive curvature of the eye
lens, or (ii) (ii) elongation of the eyeball. This defect can be corrected by using a
concave lens of suitable power.
a) Keratoconus
b) Limbus
c) Keratomalacia
d) None
A condition in which the clear tissue on the front of the eye (cornea) bulges outward.
With keratoconus, the clear, dome-shaped tissue that covers the eye (cornea) thins
and bulges outward into a cone shape. Its cause is unknown.
Symptoms first appear during puberty or the late teens and include blurred vision
and sensitivity to light and glare.
Vision can be corrected with glasses or contact lenses early on. Advanced cases
may require a cornea transplant.
129. Vision 2020, which disease has added recently for India?
a) Cataract
b) Childhood blindness
c) Glaucoma
d) Refractive errors and low vision
VISION2020 aims to eliminate avoidable blindness in the world by 2020 and targets
the world's leading causes of avoidable visual impairment: cataract, trachoma,
onchocerciasis, childhood blindness (including vitamin A deficiency), and refractive
error and low vision.
Glaucoma is now regarded as a sort of “missing link” and VISION 2020 India
successfully advocated for the inclusion of glaucoma in the next phase by the Indian
government.
130. Diabetic patient sudden loss of vision, 6/60 vision, fundus picture is
given here. What is the diagnosis?
a) CRVO
b) CRAO
c) Haemorrhage
d) Hypertensive retinopathy
Contact lenses move with your eye, allow a natural field of view, have no frames to
obstruct your vision and greatly reduce distortions. Unlike glasses, they do not fog up
or get splattered by mud or rain. Contact lenses are excellent for sports and other
physical activities. since the contact lens are extremely close to the eye. they reduce
the prismatic effect which is normal while wearing spectacles due to the distance
between the glasses and the eye
Alpha blockers are a type of blood pressure medication. They lower blood
pressure by preventing a hormone called norepinephrine from tightening the muscles
in the walls of smaller arteries and veins. As a result, the blood vessels remain open
and relaxed. This improves blood flow and lowers blood pressure. When you start
taking an alpha blocker, you might develop low blood pressure and dizziness,
which can make you faint when you rise from a sitting or lying position.
Mechanism of Action
Allopurinol and oxypurinol both inhibit xanthine oxidase, an enzyme in the purine
catabolism pathway that converts hypoxanthine to xanthine to uric acid.
Zero order kinetics is a way of describing how the body uses and breaks down
some medicines. While the rate at which the body eliminates most drugs is
proportional to the concentration administered, known as first order kinetics, drugs
that work by zero order kinetics work at a predictable, constant rate
a) Phenytoin
b) Heparin
c) Aspirin
d) Metformin
137. A lady has taken medication for ameiobiasis infection. She drank
alcohol in a party. She has nausea vomiting and dizziness. Which anti
amoebic drug could have led to interaction with alcohol to produce these
symptoms?
a) Metronidazole
b) Nitazoxanamide
c) Paromomycin
d) Diloxanide
a. Fentanyl
b. Morphine
c. Pentazocine
d. Pethidine
142. All of the following drugs are used for prophylaxis of migraine except;
a) Propranolol
b) Amitriptyline
c) Flutamide
d) Flunarizine
143. Antidote for opioid poisoning;
a) Naloxone
b) Pethidine
c) Flumazenil
d) Physostigmine
Opioid use can lead to death due to the effects of opioids on the part of the brain
which regulates breathing. An opioid overdose can be identified by a combination of
three signs and symptoms:
pinpoint pupils;
unconsciousness; and
difficulties with breathing.
Death following opioid overdose is preventable if the person receives basic life
support and the timely administration of the drug naloxone. Naloxone is an antidote
to opioids that will reverse the effects of an opioid overdose if administered in time.
Naloxone has virtually no effect in people who have not taken opioids.
Hydrochlorothiazide acts on the distal convoluted tubules and inhibits the sodium
chloride co-transporter system. This action leads to a diuretic action that lowers
blood pressure, but there is also a potassium loss in the urine.
Exenatide is a GLP-1 receptor agonist released from the gut and acts to increase
glucose-dependent insulin secretion from pancreatic beta cells, suppress
glucagon secretion, delay gastric emptying, and reduce food intake. The binding of
the drug to pancreatic GLP-1 receptors mediates these actions
152. A farmer with pinpoint pupils, increased secretions and urination. What
is the most likely diagnosis?
a) Organophosphate poisoning
b) Opioid poisoning
c) Alcohol poisoning
d) Atropine poisoning
A posthumous birth is the birth of a child after the death of a biological parent. A
person born in these circumstances is called a posthumous child or a posthumously
born person
155. After injury hair bulbs are seen crushed under microscopy. Probable
type of wound;
a) Abrasion
b) Laceration
c) Stab injury
d) None
Lacerations
o These wounds are commonly known as ‘gashes, tears or cuts’ of the skin
o It is an open wound.
o Breach in the epidermis, dermis or underlying tissue as a result of blunt force
Features
o Tissues not cleanly divided and is bridged across base of the wound.
o Foreign material may be found as well
o Hair and hair bulbs are crushed
o Margins are irregular, ragged and uneven and their ends are pointed or
blunt
o Lacerations produced without excessive skin crushing may have
relatively regular sharp margins.
o Bruising is seen either in the skin or subcutaneous tissue around the wound
o Hair bulbs are crushed
o Hair and epidermal tags may be driven deeply into the wound
o Less haemorrhage
o Presence of foreign matter in the wound
o Shape and size of the wound does not correspond to the weapon used
156. A patient was found to have pink coloured skin and mucosa, bitter
almond smell from breath and frothy discharge. Cause of death in this
patient;
a) Cyanide poisoning
b) CO poisoning
c) H2S poisoning
d) Dhatura poisoning
Entrance wounds: point where the bullet enters the biological target
o General features:
Abrasion ring: a reddish brown abrasion on the skin surrounding the
gunshot wound of entrance
Greasy rim or bullet wipe is a dirt collar due to the lubricant carried on
the bullet
Inverted edges: due to the bullet movement inside the body
Smaller than bullet caliber (usually): due to the elastic recoil of the
affected tissues
158. A lady comes to OPD after fall from scooty. Her vitals are
stable. She is having continuous, clear watery discharge from
nose after 2 days. These are most likely the feature of;
a) CSF rhinorrhoea
b) Acute respiratory infection
c) Middle cranial fossa fracture
d) Rhinitis
Cerebrospinal fluid (CSF) rhinorrhea is a condition where the fluid that surrounds
the brain leaks into the nose and sinuses. Head trauma, surgery, or even birth
defects can make a hole in the membranes that hold this fluid. It then leaks into your
nose or ear, causing a watery, runny nose
Frontal sinusitis is inflammation or infection of the sinuses located just behind the
eyes and in the forehead. The sinuses are a system of connected hollow cavities in
the face that contain air and a thin layer of mucus. All sinuses produce mucus that
moisturizes the airways and drains into the nasal passages.
If the frontal sinuses are inflamed or infected, they cannot drain mucus efficiently,
and this can make breathing difficult. It can also lead to a feeling of increased
pressure around the eyes and forehead.
Since most cases of acute frontal sinusitis are caused by a viral infection, treatment
would be to recommend taking a nasal spray or decongestant to decrease
inflammation, assist with mucus drainage, and relieve pressure in the frontal sinuses
The Adam's apple is made up of the thyroid cartilage. Cartilage is the same tissue
that makes up your nose, ears, and windpipe (i.e., trachea). The voice box and
windpipe have several kinds of cartilage, which work together to make sure your
airways stay clear and you're able to speak.
Bulla ethmoidalis
This is the largest and most prominent anterior ethmoid air cell. It is
related laterally to the lamina papyracea. It may fuse with the skull
base superiorly and with the lamella basalis posteriorly
EBV-associated diseases
Nasopharyngeal carcinoma (NPC) ...
EBV-associated gastric cancer...
Breast cancer. ...
HL and NHL. ...
Post-transplant lymphoproliferative carcinoma (PTLD) ...
Infectious mononucleosis (IM) ...
Chronic active EBV infection (CAEBV) ...
EBV-associated hemophagocytic lymphohistiocytosis (EBV-HLH)
164. All of the following is true about Meniere’s disease except;
a) Idiopathic
b) Low frequency hear loss
c) Increase in endolymph
d) Destroying the cochlea is mandatory
Diagnosing a CSF leak includes an analysis of the nasal fluid for a protein called
beta-2 transferrin which is most only found in cerebrospinal fluid. X-ray,CT and MRI
scans may also be requiring to determine the location and severity of the leakage.
The main treatment for NPC is radiation therapy. It is often given in combination
with chemotherapy. This approach may be called concomitant chemoradiotherapy.
Surgery for NPC is occasionally used, mainly to remove lymph nodes after
chemoradiotherapy or to treat NPC that has come back after initial treatment.
a) Impetigo contagiosa
b) Contact dermatitis
c) Molluscum contagiosum
d) HSV
a) Alopecia areata
b) Androgenic alopecia
c) Anagen effluvium
d) Telogen effluvium
Sudden hair loss that starts with one or more circular bald patches that may overlap.
Alopecia areata occurs when the immune system attacks hair follicles and may be
brought on by severe stress.
The main symptom is hair loss.
Treatment may address any underlying conditions and includes topical scalp
medication.
The characteristic diagnostic finding of alopecia areata is the exclamation point hair.
These can be found in areas of hair loss and are short broken off hairs that are
narrower closer to the scalp and therefore mimic an exclamation point. In some
cases a biopsy is necessary for diagnosis.
a) Burn
b) Castle neck appearance
c) Shawl sign
d) Gottron's papule
The Casal collar or Casal necklace is a clinical sign in which there is an
erythematous pigmented skin rash in the distribution of a broad
collar (dermatomes C3 and C4). It is seen in patients with pellagra, as a result of
vitamin b3 deficiency. The sign is named after Gaspar Casal
172. A lady with 50% burns of dermis & subcutaneous tissue came to
emergency department. Burns will be classified as;
a) 1st degree
b) 2nd degree superficial
c) 2nd degree deep
d) 3rd degree burn
173. Identify the abnormal condition given below;
a) Testicular torsion
b) Hydrocele
c) Varicocele
d) Testicular atrophy
A twisting of the male organ that makes hormones and sperm (testicle).
When the testicle rotates (testicular torsion), it twists the cord supplying blood to the
loose bag of skin (scrotum) beneath the penis. This may occur after vigorous activity,
a minor injury to the testicles or sleep.
Sudden, severe pain and swelling in the testicle are symptoms.
Surgery is required. Treated promptly, the testicle can often be saved. A longer wait
may affect fertility.
Sister Mary Joseph nodule or Sister Mary Joseph Sign refers to a palpable nodule
bulging into the umbilicus as a result of metastasis of a malignant cancer in the
pelvis or abdomen. Gastrointestinal malignancies account for about half of the
underlying sources (gastric, colonic, pancreatic cancer), gynecologic (ovarian,
uterine cancer), unknown primary tumors and rarely bladder or respiratory
malignancies cause umbilical metastasis.
Rodent ulcers are a type of skin cancer. Also known as basal cell carcinoma,
rodent ulcers occur in the lower most layer of your skin. They usually develop in skin
areas exposed to the sun. Rodent ulcers are a very slow growing cancer that usually
does not spread to other parts of your body.
Squamous cell carcinoma is the most common type of mouth cancer, accounting
for 9 out of 10 cases. Squamous cells are found in many areas of the body, including
the inside of the mouth and in the skin. Less common types of mouth cancer include:
adenocarcinoma, which is cancers that develop inside the salivary glands.
The main function of the ileum is to absorb vitamin B12, bile salts, and whatever
products of digestion were not absorbed by the jejunum. ... They absorb fatty acid
and glycerol, the products of fat digestion.
178. 62-year-old female has kidney stone and treated with PCNL. After 2
days she again comes to OPD with chills and fever. What is the
complication?
a) Bacterial sepsis
b) Acute pancreatitis
c) Splenic injury
d) Ureteric stricture
179. Cellulitis is
A port-wine stain is a type of birthmark. It got its name because it looks like maroon
wine was spilled or splashed on the skin. Though they often start out looking pink at
birth, port-wine stains tend to become darker (usually reddish-purple or dark red) as
kids grow.
Port-wine stains are caused by an abnormal formation of tiny blood vessels in
the skin. In rare cases, port-wine stains are a sign of Sturge-Weber syndrome or
Klippel-Trenaunay-Weber syndrome.
182. Abdominal pain, fever and jaundice. This triad is known as;
a) Charcot’s triad
b) Saint’s triad
c) Virchow triad
d) Renault’s triad
183. 40 years old patient with femur fracture with pulmonary infiltration and
respiratory distress
a) Fat embolism
b) Pulmonary embolism
c) Air embolism
d) Obstruction
185. 33-year man presented with fever and pain in upper right
hypochondrium after food intake. Investigation of choice;
a) CECT
b) Ultrasound
c) MRI
d) HRCT
Ultrasonography is the most common test used in the emergency department for
the diagnosis of biliary colic and acute cholecystitis This imaging modality may be
diagnostic for biliary disease, help exclude biliary disease, or may reveal alternative
causes of the patient's symptom
186. Long term diabetic patient with blisters walked barefoot few miles on
hot sand. He presented with this clinical condition. What is the most
probable diagnosis?
a) Diabetic foot
b) Burn
c) Necrotizing Fasciitis
d) Elephantiasis
Noncontrast head CT is the most useful study for the evaluation of patients after
trauma, focal neurologic symptoms, change in mental status, and severe
headaches to rule out life-threatening pathology. Clinicians in all fields must be able
to readily identify a number of different abnormalities.
Early morning urine (EMU) is accepted as a useful method for improving the
concentrations of a number of antigens and analytes found in human urine. The
strategy has been shown to improve the yield of urine culture for TB diagnosis
Urine is easily obtainable and can be tested using either a lateral flow assay
(LFA) for TB-lipoarabinomannan (TB-LAM) or the Xpert MTB/RIF assay (Xpert;
Cepheid, Sunnyvale, CA). Tuberculosis-LAM is emerging as the key urine-based TB
diagnostic
A chronic inflammatory bowel disease that affects the lining of the digestive tract.
Crohn's disease can sometimes cause life-threatening complications.
Crohn's disease can cause abdominal pain, diarrhoea, weight loss, anaemia and
fatigue. Some people may be symptom-free most of their lives, while others can
have severe chronic symptoms that never go away.
Crohn's disease cannot be cured. Medications such as steroids and
immunosuppressants are used to slow the progression of disease. If these aren't
effective, a patient may require surgery. Additionally, patients with Crohn's disease
may need to receive regular screening for colorectal cancer due to increased risk.
Early dumping syndrome happens when the sudden influx of food into your
intestine causes a lot of fluid to move from your bloodstream into your
intestine as well. This extra fluid causes diarrhea and bloating. Your intestines also
release substances that speed your heart rate and lower your blood pressure.
193. In an accident case, after the arrival of medical team, all should be done
in early management except;
a) Stabilization of cervical vertebrae
b) Check BP and Pulse
c) Check Respiration
d) Glasgow coma scale
The Glasgow Coma Scale (GCS) is used to objectively describe the extent of
impaired consciousness in all types of acute medical and trauma patients. The scale
assesses patients according to three aspects of responsiveness: eye-opening,
motor, and verbal responses.
Traditionally the right iliac fossa is the standard fossa for a kidney transplantation
procedure and the left iliac fossa is the preferred site for simultaneous kidney-
pancreas transplantation.
At birth people usually have 20 baby (primary) teeth, which start to come in (erupt)
at about 6 months of age. They fall out (shed) at various times throughout childhood.
By age 21, all 32 of the permanent teeth have usually erupted.
202. 40-year-old male presents with fever and abdominal pain and
diagnosed with HIV and TB. How will you give treatment?
a) ATT and AIDS treatment simultaneously.
b) First ATT and then ART
c) ATT only
d) First ART and then ATT
a) IgA nephropathy
b) HSP
c) Buerger’s disease
d) PSGN
a) Antigliadin antibody
b) Antimitochondrial antibody
c) Anticentromere antibody
d) Antitopoisomerase antibody
A gliadin antibodies test is used to help doctors diagnose celiac disease. Celiac
disease is an autoimmune disorder in which the body's immune system mistakenly
thinks that gluten — a protein in wheat, barley, rye, and oats
Craniopharyngiomas are benign tumors that grow near the pituitary gland. They
can develop as solid tumors or cysts (hollow sacs filled with fluid). Approximately 10
percent to 15 percent of pituitary tumors are craniopharyngiomas.
Craniopharyngioma is a rare type of noncancerous (benign) brain tumor.
Craniopharyngioma begins near the brain's pituitary gland, which secretes hormones
that control many body functions. As a craniopharyngioma slowly grows, it can affect
the function of the pituitary gland and other nearby structures in the brain
False Contraindications
diarrhea.
minor upper respiratory illnesses with or without fever.
mild to moderate local reactions to a previous dose of vaccine.
current antimicrobial therapy.
the convalescent phase of an acute illness.
210. Unconscious patient, most common cause of respiratory obstruction;
a) Tongue fall
b) Head injury
c) Respiratory center block
d) Hyoid fracture
The tongue is the most common cause of upper airway obstruction, a situation seen
most often in patients who are comatose or who have suffered cardiopulmonary
arrest. Other common causes of upper airway obstruction include edema of the
oropharynx and larynx, trauma, foreign body, and infection.
A pulmonary shunt often occurs when the alveoli fill with fluid, causing parts of the
lung to be unventilated although they are still perfused. Intrapulmonary shunting is
the main cause of hypoxemia (inadequate blood oxygen) in pulmonary edema and
conditions such as pneumonia in which the lungs become consolidated.
212. A 32 years’ lady present with shoulder tip pain and diagnosis is
pan coast tumour and presents with meiosis. Diagnosis-
a) Horner syndrome
b) Thoracic outlet syndrome
c) Aberrant right subclavian artery
d) Upper trunk of brachial plexus injury
A disrupted nerve pathway on one side from the brain to the face and eye.
Horner's syndrome is the result of another medical condition, such as a stroke,
tumour or spinal cord injury.
Symptoms include a smaller pupil, drooping eyelid and little or no sweating on the
affected side.
There's no specific treatment for Horner's syndrome, but treatment for the underlying
cause may restore normal nerve function.
213. An 85 year old patient was brought to the ER, BP: 180/100, right
hemiparesis was seen. What is the next best step in management?
a) Reduce BP
b) NCCT
c) MRI
d) Aspirin 300mg and anticoagulants
Causes of hypercalcemia
Overactive parathyroid glands (hyperparathyroidism). This most common
cause of hypercalcemia can stem from a small, noncancerous (benign) tumor
or enlargement of one or more of the four parathyroid glands.
Cancer. Lung cancer and breast cancer, as well as some blood cancers, can
increase your risk of hypercalcemia. Spread of cancer (metastasis) to your
bones also increases your risk.
Other diseases. Certain diseases, such as tuberculosis and sarcoidosis, can
raise blood levels of vitamin D, which stimulates your digestive tract to absorb
more calcium.
Hereditary factors. A rare genetic disorder known as familial hypocalciuric
hypercalcemia causes an increase of calcium in your blood because of faulty
calcium receptors in your body. This condition doesn't cause symptoms or
complications of hypercalcemia.
Immobility. People who have a condition that causes them to spend a lot of
time sitting or lying down can develop hypercalcemia. Over time, bones that
don't bear weight release calcium into the blood.
Severe dehydration. A common cause of mild or transient hypercalcemia is
dehydration. Having less fluid in your blood causes a rise in calcium
concentrations.
Medications. Certain drugs — such as lithium, used to treat bipolar disorder
— might increase the release of parathyroid hormone. Thiazides have several
metabolic effects contributing to higher serum calcium levels, but increased
renal tubular reabsorption of calcium resulting in reduced urine calcium
excretion is the most likely cause
Supplements. Taking excessive amounts of calcium or vitamin D
supplements over time can raise calcium levels in your blood above normal.
219. Lady exposed to cold, extremities turn to blue associated with which
antibody;
a) Anti scl 70
b) ANA
c) anti ssb
d) anti ssa
Raynaud's disease causes smaller arteries that supply blood flow to the skin to
narrow in response to cold or stress. The affected body parts, usually fingers and
toes, might turn white or blue and feel cold and numb until circulation improves,
usually when you get warm
In a patient presenting with Raynaud phenomenon, a positive ANA test (even in the
absence of other symptoms) warrants more frequent follow-up, urinalysis and
perhaps referral to a rheumatologist.
The PR interval is the time from the onset of the P wave to the start of the QRS
complex. It reflects conduction through the AV node
Ependymal cells are mostly known as the cell type lining the brain ventricles. As
non-neuronal cells in the brain and derived from neuroectoderm, they are clearly
defined as a subtype of glial cells.
When the neurons in the substantia nigra are damaged in large numbers, the loss
of dopamine prevents normal function in basal ganglia and causes the motor
symptoms of PD: tremor, rigidity, impaired balance, and loss of spontaneous
movement.
227. 20-year-old male came to clinic and following findings were seen, what
will be the diagnosis?
a) AML
b) ALL
c) CML
d) CLL
Acute myeloid leukemia (AML) is a cancer of the myeloid line of blood cells,
characterized by the rapid growth of abnormal cells that build up in the bone marrow
and blood and interfere with normal blood cell production.
Signs and symptoms of acute myelogenous leukemia include:
Fever.
Bone pain.
Lethargy and fatigue.
Shortness of breath.
Pale skin.
Frequent infections.
Easy bruising.
Unusual bleeding, such as frequent nosebleeds and bleeding from the gums.
Head tilt, chin lift, jaw thrust is the correct sequence of jaw thrust method
a) Osteosarcoma
b) Ewings sarcoma
c) Osteoid osteoma
d) Osteochondroma
Osteosarcoma is a type of bone cancer that begins in the cells that form bones.
Osteosarcoma is most often found in the long bones — more often the legs, but
sometimes the arms — but it can start in any bone. In very rare instances, it occurs
in soft tissue outside the bone.
The sunburst appearance occurs when the lesion grows too fast and the
periosteum does not have enough time to lay down a new layer and instead
the Sharpey's fibers stretch out perpendicular to the bone. It is frequently
associated with osteosarcoma but can also occur with other aggressive bony lesions
232. What is the diagnosis of the given image?
a) Extradural haemorrhage
b) Subdural haemorrhage
c) Subarachnoid haemorrhage
d) Intracerebral haemorrhage
that if one side of the chest cavity has an increase in pressure (such as in the case
of a pneumothorax) the trachea will shift towards the opposing side. The trachea
is the tube that carries air from the throat to the lungs.
234. A boy was riding bicycle, he fell down forward and injury to shoulder
was seen, which nerve injury will be seen in the condition as given in X-
ray?
a) Radial
b) Ulnar
c) Median
d) Axillary
all nerves of the brachial plexus are at risk of injury during glenohumeral dislocation,
the most commonly injured is the axillary nerve
a) Colles fracture
b) Smith fracture
c) Monteggia fracture
d) Galeazzi fracture
the Colles fracture is defined as a distal radius fracture with dorsal comminution,
dorsal angulation, dorsal displacement, radial shortening, and an associated fracture
of the ulnar styloid. The term Colles fracture is often used eponymously for distal
fractures with dorsal angulation
A common injury in which forceful twisting causes certain tissue in the knee to tear.
A meniscus tear occurs in the rubbery knee cartilage that cushions the shinbone
from the thighbone. The meniscus can tear with forceful twisting or rotation of the
knee.
Pain, swelling, stiffness and difficulty extending the knee are symptoms.
Treatment includes rest, ice, pain relievers and physiotherapy. Less commonly,
surgery may be required.
The medial circumflex femoral artery (internal circumflex artery, medial femoral
circumflex artery) is an artery in the upper thigh that helps supply blood to the neck
of the femur. Damage to the artery following a femoral neck fracture may lead to
avascular necrosis (ischemic) of the femoral neck/head.
The Phemister triad was named after American orthopedic surgeon Dallas Burton
Phemister (1882–1951). The three features are juxta-articular osteopenia or
osteoporosis, peripheral osseous erosions, and gradual narrowing of joint spaces—
classically seen with tuberculous arthropathy.
A hormonal disorder causing enlarged ovaries with small cysts on the outer edges.
The cause of polycystic ovary syndrome isn't well understood, but may involve a
combination of genetic and environmental factors.
Symptoms include menstrual irregularity, excess hair growth, acne and obesity.
Treatments include birth control pills to regularise periods, medication called
metformin to prevent diabetes, statins to control high cholesterol, hormones to
increase fertility and procedures to remove excess hair.
a) CMV
b) Herpes Simplex
c) Herpes Zoster
d) Measles
Giant cell pneumonia: A deadly but fortunately rare complication of measles that
tends to strike children who are immunodeficient from leukemia or AIDS. The lung
tissue shows multinucleated giant cells lining the alveoli (air sacs) of the lungs. Also
known as Hecht's pneumonia.
a) X Ray
b) CT
c) MRI
d) NCCT
coxa vara usually presents with a limp, a leg length difference, and limited
ability to bring the thigh out to the side. X-rays are used to diagnose coxa
vara. Depending on what caused the condition and how severe the
deformity, reconstructive surgery may be needed to improve the hip
alignment and biomechanics.
245. A 35-year-old anemic female patient came for family planning, she
has 2 living children and history of ectopic pregnancy what is your next
step:
a) Lippe’s Loop
b) Hysterectomy
c) T Piece
d) Progestasert
The Progestasert system combines the advantageous features of IUDs and oral
minidose progestogen preparations. An internal device continuously delivers
progesterone for 1 year to the uterine lumen and endometrium. A T-shaped
Progestasert which releases 65 mcg/day has been selected for wide scale clinical
use.
a) Lyme disease
b) Chagas Disease
c) Anaplasmosis
d) Powassan Virus Disease
a) Ampulla
b) Isthmus
c) Infundibulum
d) Interstitium
An ectopic pregnancy occurs when a fertilized egg grows outside of the uterus.
Almost all ectopic pregnancies—more than 90%—occur in a fallopian tube. As the
pregnancy grows, it can cause the tube to burst (rupture). A rupture can cause
major internal bleeding. Very late rupture of tubal pregnancy is seen when the
implantation is at Interstitium.
a) Polyarteritis nodosa
b) Fibromuscular dysplasia
c) Atherosclerosis
d) Takayasu Arteritis
Fixed drug eruption (FDE) is a distinctive type of cutaneous drug reaction that
characteristically recurs in the same locations upon reexposure to the offending
drug.
Fixed drug eruption typically presents as a single (or small number of) well-
defined, round or oval red or violaceous patch or plaque which may blister or
ulcerate. It is usually asymptomatic but can be itchy or painful.
250. A young male presents with recurrent bouts of diarrhoea, two hours
after drinking milk. Diagnosis?
a) Gastrocolic Reflex
b) Lactase deficiency
c) Inflammatory Bowel Disease
d) IBS
a) Actinomycosis
b) Toxoplasma Gondii
c) Pneumocystis Jiroveci
d) Isospora Belli
a) Atrial regurgitation
b) Atrial Myxoma
c) Pulmonary regurgitation
d) Pulmonic Stenosis
Late diastolic (presystolic) murmurs start after S2 and extend up to S1 and have a
crescendo configuration. They can be associated with AV valve narrowing. They
include mitral stenosis, tricuspid stenosis, myxoma, and complete heart block.
253. A 3-year-old patient is having loose motions with weight loss. Small
intestinal biopsy was done. Which of the following tests is best suited for
clinical diagnosis of the patient?
a) Anti-Mitochondrial AB
b) Anti-Gliadin AB
c) Anti-SCL 70 AB
d) Anti-Microsomal AB
A gliadin antibodies test is used to help doctors diagnose celiac disease. Celiac
disease is an autoimmune disorder in which the body's immune system mistakenly
thinks that gluten — a protein in wheat, barley, rye, and oats
254. A 40-year old smoker patient has been diagnosed with infra-renal
aorta aneurysm of 4 cm. What is the best treatment?
a) Gastrinoma
b) Primitive Neuroectodermal Tumour
c) Carcinoid Syndrome
d) Autoimmune Gastritis
256. Patient presents with MI, STK (streptokinase) was given. What should be
checked in order to monitor toxicity?
a) Bleeding Time
b) Fibrin degradation products
c) Platelet Count
d) Thrombin Time
Careful monitoring for arrhythmia is recommended during and immediately following
administration of Streptase, Streptokinase, for acute myocardial infarction.
Occasionally, tachycardia and bradycardia have been observed.
a) Pericardial Effusion
b) Cardiac Tamponade
c) Aortic Dissection
d) Constrictive Pericarditis
Elevated jugular venous pressure (JVP) is present in virtually all patients that are not
hypovolemic. Pericardial constraint results in the inability of the right heart to
accommodate inspiratory abdominal venous return, translating to an inspiratory
increase in the JVP (Kussmaul's sign)
258. What is the best treatment for a 35-year old AIDS positive patient with
Cryptosporidium Diarrhoea?
a) Nitazoxanide
b) Metronidazole
c) Parmomycin
d) Cotrimoxazole
a) Withdrawal Reflex
b) Crossed Extensor Reflex
c) Peristaltic Reflex
d) Stretch Reflex
The monosynaptic stretch reflex, or sometimes also referred to as the muscle stretch
reflex, deep tendon reflex, is a reflex arc that provides direct communication
between sensory and motor neurons innervating the muscle. ... This contraction
allows the muscle to resist the force that initially caused the reflex.
FMGE AUG 2020
During the emission phase, the muscles around the epididymis and ductus
deferens (the tube extending from the epididymis) contract to push the sperm into
the prostate and urethra. During ejaculation, the semen is expelled by strong
spasmodic contractions of the bulbocavernosus muscle, which encircles the corpus
spongiosum (the structure in the penis that encloses the urethra). The whole process
of ejaculation is accomplished by nerve impulses received from the penis; once
ejaculation is started it becomes a reflex reaction that cannot be voluntarily
interrupted.
The deep (internal) inguinal ring is a defect in the transversus abdominis fascia that
allows the contents to enter the canal and leave the abdomen before extending
obliquely, medially and inferiorly through the canal to exit at the superficial (external)
inguinal ring, a defect in the external oblique fascia.
3 . Name the skin receptor responsible for detecting rapid vibration sense:
a ) Meissner corpuscle
b ) Paccinian corpuscle
c ) Merkel cell
d ) Ruffini's corpuscle
4 . Which type of joint is present at the arrow mark shown in the diagram:
a) Primary cartilaginous
b) Secondary cartilaginous
c) Fibrous
d) Synovial
Secondary cartilaginous joint
symphysis pubis between the right and left pubic bones.
manubriosternal joint between the sternal body and the manubrium.
intervertebral discs.
sacrococcygeal symphysis.
For example, a left corticobulbar lesion results in paralysis of the muscles that
control the lower right quadrant of the face. By contrast, a lower motor neuron lesion
to the facial motor nucleus results in paralysis of facial muscles on the same side
of the injury.
The liver develops in the ventral mesogastrium; the spleen develops in the dorsal
mesogastrium. The liver grows rapidly, pressing against the body wall, and
obliterating these layers of peritoneum. These changes produce this almost separate
pocket behind the stomach, the lesser sac.
a) Latissimus dorsi
b) Trapezius
c) Serratus anterior
d) Supraspinatus
Scapular winging is almost always caused by damage to one of three nerves that
control muscles in your arms, back, and neck: the long thoracic nerve, which controls
the serratus anterior muscle.
The posterior tibial artery enters the sole of the foot by passing deep to the flexor
retinaculum. Its pulsation can be felt midway between the medial malleolus and
the medial border of the tendocalcaneus. The pulsation of the peroneal artery is
felt in front of the lateral malleolus at its medial border.
Ductus arteriosus itself arises from the left 6th pharyngeal arch artery.
13. An Alcoholic patient presented with mental confusion, gait problem and
ophthalmoplegia. Which vitamin deficiency is seen?
a) B1
b) A
c) B9
d) B12
15. Macrocytic anaemia case with normal methyl-malonyl with increased level
of homocysteine. What is the diagnosis?
a) B9 deficiency
b) B12, 9 deficiency
c) Iron deficiency
d) Anaemia of chronic ds
Vitamin B12 or B9 (commonly called folate) deficiency anaemia occurs when a lack
of vitamin B12 or folate causes the body to produce abnormally large red
blood cells that cannot function properly. Red blood cells carry oxygen around the
body using a substance called haemoglobin. Additional testing with homocysteine
and MMA determinations may help distinguish between B12 and folate deficiency
states. In folate deficiency, homocysteine levels are elevated and MMA levels are
normal. In vitamin B12 deficiency, both homocysteine levels and MMA levels are
elevated.
16. A heart patient suddenly has chest pain. Which isoenzyme of LDH enzyme is
elevated?
a) LDH -1
b) LDH -5
c) LDH -2
d) LDH -4
20. Which of the following is not given in a case of fatty liver disease?
a) Choline
b) Ethanol
c) Folic acid
d) Methionine
Recent studies indicate that additional effects of ethanol impair fat oxidation as well
as stimulate lipogenesis. The effects of ethanol on lipid metabolism result
from inhibition of PPAR-α and stimulation of SREBP-1, resulting in metabolic
remodelling of the liver toward a fat-storing, rather than fat oxidizing organ.
21. 16 Years old girl was kidnapped. Police caught the accused and brought to
the police station. After some time, the accused was found dead in the station.
Inquest in this case should be done by?
a) Executive Magistrate.
b) Judicial Magistrate.
c) Police.
d) No inquest needed.
a) Loop
b) Whorl
c) Arch
d) Composite
24. Identify the test that shows the following crystals.
a) Teichman
b) Takayama
c) Florence
d) Barberios
Takayama test is a confirmation test used to detect blood spots. Based on the
research results this test can still be used to identify dried blood spots on clothing
aged 20 years, and able to detect positively the presence of blood with the formation
of pink crystals.
A dying declaration is considered credible and trustworthy evidence based upon the
general belief that most people who know that they are about to die do not lie. ... A
person who makes a dying declaration must, however, be competent at the time he
or she makes a statement, otherwise, it is inadmissible.
26. In regard to a fetal death classification, a test was done to determine the
cause which includes dipping the lung in to the water. Identify the test.
a) Foderes
b) Ploquets
c) Hydrostatic
d) Wredins
The hydrostatic test, or floatation test, has historically been used to determine
whether a newborn infant has breathed. It is performed by placing the lungs, still
attached to the heart, in water and seeing whether they float.
27. A patient is brought to casualty with history of snake bite with features
as shown. During course of Treatment, the patient expires. The accused
snake’s photo was captured by relatives as shown in image. What could be the
cause of death in this case?
a) Shock
b) Respiratory paralysis
c) Circulatory failure
d) Rhabdomyolysis
The venom of common kraits contains 3 major neurotoxins (α, β and κ-bungarotoxin)
that can induce failure of neuromuscular transmission resulting in respiratory
muscles paralysis.
28. Identify the method of solvent abuse done with use of a plastic bag
applied to mouth & nose.
a) Bagging
b) Huffing
c) Glading
d) Sniffing
"bagging" — sniffing or inhaling fumes from substances sprayed or deposited inside
a plastic or paper bag
30. A person was found dead in his apartment and the forensic team
observed the scene and gave these noted details regarding injury that caused
death. The injury over the vertebra has clean cut margins but the bones
appeared to be crushed. What could be the correct option in this scenario?
a) Incised
b) Chop
c) Lacerated
d) Stab
33. A man came for health check-up after his father had a cerebrovascular
accident who died last month. He was having the history of hypertension.
Which type of prevention is this?
a) Primordial
b) Primary
c) Secondary
d) Specific protection
The date of the midyear population is July 1. Note that the midyear month is
different from the census month. These differences are taken into account when
calculating.
a) Histogram
b) Bar diagram
c) Frequency polygon
d) Scatter diagram
Histogram is a diagram consisting of rectangles whose area is proportional to the
frequency of a variable and whose width is equal to the class interval.
a) AGMARK
b) PFA standards
c) Food Standards and Safety Authority of India (FSSAI)
d) Bureau of Indian Standards (BIS)
BIS is the National Standard Body of India established under the BIS Act 2016 for
the harmonious development of the activities of standardization, marking and
quality certification of goods and for matters connected therewith or incidental
thereto.
40. A Researcher did study on staff, nurses and junior resident doctors
doing yoga (Divided into male and female). Yoga shows 25% decreased risk of
infections. Which is the best test to test the significance of the result?
a) ANOVA
b) Chi square test
c) Z test
d) Correlation
chi-square test
relating to or denoting a statistical method assessing the goodness of fit between
observed values and those expected theoretically. Here the researcher has 2
groups where 25% decreased risk of infections when they do yoga i.e. 25% less
than the risk expected theoretically.
The secular trend describes the occurrence of disease over a prolonged period,
usually years; it is influenced by the degree of immunity in the population and
possibly nonspecific measures such as improved socioeconomic and nutritional
levels among the population and are usually represented by line diagram.
Case fatality rate, also called case fatality risk or case fatality ratio,
in epidemiology, the proportion of people who die from a specified disease among all
individuals diagnosed with the disease over a certain period of time. Case fatality
rate typically is used as a measure of disease severity and is often used for
prognosis (predicting disease course or outcome)
47. A 8-month-old child reported to a PHC with runny nose and fever since
last 3 days. Rash started on face and then covered the trunk part of body.
What's the most likely diagnosis?
a) Measles
b) Chicken pox
c) Mumps
d) Rubella
Measles symptoms appear 7 to 14 days after contact with the virus and typically
include high fever, cough, runny nose, and watery eyes. Measles rash appears 3 to
5 days after the first symptoms.
a) CuT 7
b) CuT 220B
c) CuT 380A
d) NOVA-T
CuT 380A - It is a T shaped device with a polyethylene frame holding 380 mm2 of
exposed surface area of copper. The IUD frame contains barium sulfate thus making
it radio-opaque.
51. If you are posted as a Medical officer in an area under NVBDCP, and you
will be advised to spray Malathion for malaria prophylaxis. What could be the
probable decision you will take regarding frequency of Malathion spray?
a) Once very fortnight
b) Once every 3 months
c) Twice every3 months
d) Once every month
Frequency of malathion spray under NVBDCP is once every 3 months.
52. A country is suspecting a severe disease. What's the probable time
within which one should report it to WHO?
a) 6 hours
b) 12 hours
c) 24 hours
d) 48 hours
the probable time within which one country should report it to WHO is 24 hrs
Both cow's milk and breastmilk are low in iron (around 0.2-0.5 mg iron per litre),
however the iron in breastmilk is bonded to lactoferrin and offers greater
bioavailability.
54. In an operation theatre, the nurse spilled blood on the floor. What can be
used to disinfect it?
a) Cresol
b) 70% Ethyl alcohol
c) Glutaraldehyde
d) 1% Sodium hypochlorite
Wipe the area with water and detergent until it is visibly clean. Saturate the area
again with sodium hypochlorite 1% (10 000 ppm available chlorine). This is a 1:10
dilution of 5.25% sodium hypochlorite bleach, which should be prepared daily. Rinse
off the tongs, brush and pan, under running water and place to dry.
56. A healthy person get in contact with an infected case, and they have to
separate him not more than the incubation period of disease. This is called as;
a) Absolute quarantine
b) Isolation
c) Interruption of transmission
d) Serial interval
57. A male underwent Vasectomy and after 3 months his wife got pregnant.
What advice should have been given to him post-vasectomy?
a) Abstinence for 3 months after the procedure
b) Usage of Barrier methods for 3 months after the procedure
c) OCPS usage for I month
d) Female condom usage for 1 month
Sperms can be seen in semen for up to 3 months’ post ligation, so repeat semen
analysis after 3 months and if azoospermia recorded then he can have
unprotected intercourse
Composed of glucose and sodium and other electrolytes, ORS promote sodium and
thus water absorption in part via passive sodium-coupled glucose transport in
intestinal villi.
Aflatoxin Alert: Moldy Nuts and Corn Increases Liver Cancer Risk 60-Times If You
Have Hepatitis B. One of the biggest health threats to people living with chronic
hepatitis B is a toxic, nearly invisible mold called aflatoxin found in corn, peanuts,
peanut butter, almonds, Brazil nuts, walnuts and pistachios.
long time workers in mines have high risk of TB and can be suspected to have
silicosis, So TB screening should be done at regular intervals.
All prophylactic drugs should be taken with unfailing regularity for the duration of the
stay in the malaria risk area, and should be continued for 4 weeks after the last
possible exposure to infection since parasites may still emerge from the liver during
this period.
Buffalo milk is rich in vitamin A, has a higher protein value and contains more iron,
calcium and phosphorus than the cow's milk. ... It is better to introduce buffalo milk
slowly.
Long term changes of a disease are seen in secular trend. The secular trend
describes the occurrence of disease over a prolonged period, usually years; it is
influenced by the degree of immunity in the population and possibly nonspecific
measures such as improved socioeconomic and nutritional levels among the
population.
65. A 5-year-old child came into OPD with fever, rashes on the body. There
were rashes on the axilla and flexor surface with various macules, papules and
vesicles. Rashes and blisters are in different stages and non- uniform. Most
probable diagnosis is?
a) Chickenpox
b) Smallpox
c) Epstein Barr
d) Measles
Chickenpox is an infection caused by the varicella-zoster virus. It causes an itchy
rash with small, fluid-filled blisters. Chickenpox is highly contagious to people who
haven't had the disease or been vaccinated against it
All individuals who have taken hydroxychloroquine for greater than 5 years should
receive annual screening for retinopathy. All individuals who have taken
chloroquine for greater than 1 year should receive annual screening for retinopathy
Gemifloxacin and moxifloxacin are not effective for the treatment of UTIs because
they do not achieve adequate concentrations in the urine. 5-7 The fluoroquinolones
indicated for UTI are excreted in the urine as greater than 40% unchanged drug
while gemifloxacin is less than 35% and moxifloxacin is only 20%.
68. A patient with heart disease presented with breathlessness the features
of the patient shown in images. Which of the following is used to treatment of
oedema for this patient?
a) ACE Inhibitor
b) Na – K – 2Cl symporter inhibitor in loop of Henle
c) Na – Cl symporter inhibitor in DCT
d) Aldosterone in DCT
Furosemide is a loop diuretic (water pill) that prevents your body from absorbing
too much salt. This allows the salt to instead be passed in your urine. Furosemide
is used to treat fluid retention (edema) in people with congestive heart failure, liver
disease, or a kidney disorder such as nephrotic syndrome.
Chronic opioid use leads to tolerance, defined as a decrease of the drug response.
It's possible to reproduce in vitro such phenomenon when cellular models expressing
OR are exposed to agonists; in that situation, a decrease of signaling is observed
and is designated as OR desensitization. Some reports distinguish the OR
desensitization from the cellular tolerance.
70. A female patient presented with pain and redness in great toe. Serum
uric acid level is 9.4 mg/dL. The physician prescribed a drug for the treatment.
Which of the following enzyme is inhibited by this drug?
a) Xanthine oxidase
b) Thymidylate synthase
c) phosphoribosyl transferase
d) DHFR
Xanthine oxidase inhibitors are primarily used in the clinical prevention and
treatment of gout associated with hyperuricemia. The archetypal xanthine
oxidase inhibitor, Allopurinol has been shown to have other beneficial effects such as
a reduction in vascular reactive oxygen species and mechano-energetic uncoupling.
72. A person presents with acute chest pain. Nitroglycerine was given
sublingually and the pain relieved within 5 minutes. what is the mechanism of
the drug?
a) Release of nitric oxide
b) Release of Endothelin
c) Calcium channel Blocker
d) Beta channel Blocker
nitroglycerin converts to nitric oxide (NO) in the body. NO then activates the enzyme
guanylyl cyclase, which converts guanosine triphosphate (GTP) to guanosine 3',5'-
monophosphate (cGMP) in vascular smooth muscle and other tissues. cGMP then
activates many protein kinase-dependent phosphorylations, which enhances the
reuptake of calcium into the sarcoplasmic reticulum, increases extracellular calcium,
and opens the calcium-gated potassium channel. This ultimately results in the
dephosphorylation of myosin light chains within smooth muscle fibers. This activity
causes the relaxation of smooth muscle within blood vessels, resulting in the desired
vasodilatory effect.
73. A 60-year-old man present to OPD with low backache. His PSA levels are
100 ng/ml. Which of the following drug is indicated?
a) Somatostatin
b) Goserelin
c) Terlipressin
d) Testosterone
Rivastigmine is now widely approved for the treatment of mild to moderately severe
dementia in Parkinson's disease (PDD). However, since anticholinergic drugs have a
role in the management of tremor in patients with Parkinson's disease (PD),
concerns have been raised that the use of cholinergic drugs might worsen PD
76. A patient has pain in facial nerve distribution while chewing and
speaking, which of the following drug should be given for treatment?
a) Lamotrigine
b) Phenobarbitone
c) Haloperidol
d) Carbamazepine
Carbamazepine is a medicine used to treat epilepsy. It can also be taken for nerve
pain caused by diabetes (peripheral neuropathy) or if you have a painful condition of
the face called trigeminal neuralgia.
79. A 20-year-male is on low dose of oral steroid therapy for the last 10
years, now he develops progressive loss of vision in both the eyes. What is
the reason?
a) Glaucoma
b) Cystoid macular edema
c) Cataract
d) Retinal detachment
Taking steroids can cause a cataract type called posterior sub-capsular cataracts. It
causes a small, cloudy area to form underneath the eye's lens. While cataracts are a
known side effect for some people when taking steroids, they're highly treatable.
80. For the treatment of acute migraine attack, a patient took some
medicines. After that he develops change in colour vision and numbness in
the tip of the thumb. What is likely drug responsible and its treatment?
a) Sumatriptan
b) Dihydroergotamine
c) Propranolol
d) Phenoxybenzamine
82. A Person from non-endemic area visited India and was given
prophylaxis for malaria. How long the person should continue after travelling
from India?
a) 7days
b) 15days
c) 4weeks
d) 4 months
All prophylactic drugs should be taken with unfailing regularity for the duration of the
stay in the malaria risk area, and should be continued for 4 weeks after the last
possible exposure to infection since parasites may still emerge from the liver during
this period.
83. A child took several iron tablets. What is the antidote for poisoning?
a) BAL
b) EDTA
c) Penicillamine
d) Desferrioxamine
Desferrioxamine injection is used to remove excess iron from the body in anemia or
thalassemia patients who have many blood transfusions. It is also used with other
medicines to treat acute iron poisoning, especially in small children.
Atenolol belongs to a group of medicines called beta blockers. It's used to treat
high blood pressure and irregular heartbeats (arrhythmia). It can also be used to
prevent chest pain caused by angina. If you have high blood pressure, taking
atenolol helps prevent future heart disease, heart attacks and strokes
86. A boy fell down while playing and the nose is deviated but septum was
found normal, what is the next step done?
a) Lower the edema then close reduction after 7 days
b) Closed reduction and automatically swelling decreases
c) Septo-rhinoplasty
d) Open reduction
Initially medications given to treat the edema and close reduction done after 7 days.
If your lingual nerve sustains an injury, you'll most likely experience any of these
nerve damage symptoms: Changed sensation in the tongue, chin, or lower lip areas
(similar to sensations you feel when your oral cavity is numbed for a dental
procedure or as the anesthesia slowly wears off) Altered ability to taste. but not
tongue deviation.
89. A professional trumpet blower presents with left sided neck swelling.
Following is the radiographic findings of the patient. What is the most likely
diagnosis?
a) Laryngocele.
b) Laryngeal cyst
c) Thyroglossal cyst
d) Epiglottits
Laryngoceles can be congenital or acquired, though the majority fall into the latter
category. They occur as a result of increased intralaryngeal pressure such as that
in excessive coughing, playing blowing instruments or due to obstructing lesions
within the larynx.
Extravasation is not common and its transluminant from the given picture. but it is
not usually a retention test
96. A 2-month old baby is having watery discharge / fluid leakage from
umbilicus, this is due to
a) Failure of midgut to recanalize
b) Failure of involution of allantoic duct
c) Failure of involution of omphalomesenteric duct
d) Failure of rotation of midgut
Patent urachus refers to a spectrum of umbilical disorders that result from the failure
of involution of normal embryologic tissues that connect the developing bladder to
the umbilical cord. Similarly, presentation is variable and can be readily evident at
birth with umbilical cord abnormalities or diagnosed later in adolescence with the
complaint of persistent umbilical wetness or recurrent umbilical infection.
98. A mother delivered 33 weeks old baby, via normal vaginal delivery with
no complications at birth. What is the preferred method of feeding the
neonate?
a) OGT feeds
b) Direct breast feeding
c) Paladai feeds
d) Parenteral nutrition
The Paladai is a cup-like utensil with a narrow tip that has been used traditionally to
feed babies in India1 when the mother cannot breast feed. It is used to hold
expressed breast milk or other types of liquid or semi-solids. The baby is held
reclining while being fed.
Portal vein thrombosis is blockage or narrowing of the portal vein (the blood
vessel that brings blood to the liver from the intestines) by a blood clot. Most people
have no symptoms, but in some people, fluid accumulates in the abdomen, the
spleen enlarges, and/or severe bleeding occurs in the esophagus.
100. A 1-year-old child presents with 1st episode of wheeze and breathing
difficulty. Chest X-ray shows bilateral hyperinflation. Which of the following
can be a causative organism for this case?
a) RSV
b) S. pneumoniae
c) H. influenza
d) Coxsackie virus
The early phase of RSV in babies and young children is often mild, like a cold. In
children younger than age 3, the illness may move into the lungs and cause
coughing and wheezing. In some children, the infection turns to a severe respiratory
disease.
Buffalo milk is rich in vitamin A, has a higher protein value and contains more iron,
calcium and phosphorus than the cow's milk. ... It is better to introduce buffalo milk
slowly.
102. A 3-month old child presents with complaints of fever, cough, not able to feed
and breathing difficulty. On examination, respiratory rate was 58 / min, chest
retractions were present. What would be your next step of Management?
a) Severe pneumonia - Give antibiotics and refer immediately
b) Very severe pneumonia and refer urgently
c) Pneumonia -Start antibiotics and send child home
d) No pneumonia
105. A patient's mother while bathing her child noticed there was no testes in
the scrotum and on examination- one right testes was in Inguinal region and
left was in perineum. What is the DIAGNOSIS?
a) bilateral undescended testes
b) bilateral ectopic testes
c) right undescended and left ectopic testes
d) left undescended and right ectopic testes
from the examination above we can easily deduct that the right testes are
undescended and left testes had become ectopic.
106. A 4-year-old child presented with fever, running nose and following
lesions. Probable aetiology?
a) HSV 1
b) HSV 2
c) Coxsackie virus
d) Streptococci pneumonia
Coxsackievirus can produce a wide variety of symptoms. About half of all kids with
an infection have no symptoms. Others suddenly get a high fever, headache, and
muscle aches, and some also develop a sore throat, abdominal discomfort, or
nausea. a type of coxsackievirus syndrome, causes painful red blisters in the throat
and on the tongue, gums, hard palate, inside of the cheeks, and the palms of hands
and soles of the feet.
107. A patient of 23 years came to the clinic with a known history of myopia
with -2 D in left and -3 D in the right eye. He is sent for retinoscopy. What is the
movement of retinal reflex in the right eye?
a) It moves in the same direction of the retinoscope
b) It moves in the opposite direction of the retinoscope
c) Vertical movement is having wider reflection and moves in the opposite
direction
d) There is no movement of the reflex
it moves in the opposite direction because of the myopia negative i.e. -2D and -3D
respectively
108. Identify the clinical condition from the skin finding as given below:
a) Chicken pox
b) HSV
c) HPV
d) Measles
As babies, children have a C-shaped spine. Secondary curves in the cervical and
lumbar spine develop as infants become able to lift their heads, sit up, crawl,
stand, and walk. As children grow, their spine continues to develop natural curves
into a normal, mature spine.
110. A mother complains that her baby is not drinking properly from breast
and now complaints of pain and swelling of the breast. Probable condition?
a) Breast abscess
b) Mastitis
c) Breast engorgement
d) Sore nipple
Breast engorgement means your breasts are painfully overfull of milk. This
usually occurs when a mother makes more milk than her baby uses.
Once engorgement occurs gentle massage, frequent feeding, correct positioning
and warm compresses to the breast have been advocated to relieve symptoms
along with analgesia to relieve pain.
Other Causes:
Sore and cracked Nipples
Inverted Nipples
Engorged breasts
Mastitis
Previous surgeries on breast
Neonatal causes
Insufficient/Unsuitable Milk
112. A 54-year-old female patient with the history of Amenorrhea for last 16
months. What should be the hormone
levels identified in this patient?
a) Increased LH, decreased FSH
b) Increased LH and FSH
c) High FSH, low LH
d) LOW LH AND FSH
Menopause
• Permanent cessation of menses dated 12 months of amenorrhea
• The avg. age - 51 years (range of 43-57 years)
Perimenopause
• 2-8 years preceding + 1 year after last menses - fluctuating hormone levels
irregular menstrual cycle & symptoms
When estrogen deficiency occurs in the menopause LH levels increase. Later the
FSH is raised and remains high for the rest of life. This raised FSH and low estrogen
levels appear to cause the characteristic hot flushes.
25 - 30 Cardiac arrest
116. A mother brings her 18-year-old child with complaints of not attained
menarche. GPE shows normally developed breast and pubic hair. On
examination vaginal ending is blind and uterus not palpable. What is the next
line of investigation?
a) USG
b) T3, TSH
c) Estrogen and Progesterone
d) FSH and LH
117. A Female patient came with c/o pruritis, greenish discharge per
vaginum. What is the causative organism?
a) Trichomonas
b) Candida - curdy white discharge + itching.
c) Bacterial vaginosis - Foul smelling greyish white + no itching
d) Chlamydia - mucopurulent endocervical discharge
TRICHOMONAS VULVOVAGINITIS :
• M/c vaginitis
Etiology - Trichomonas vaginalis –flagellated protozoan
Mode of Spread: Sexual & non Sexual
Symptoms - Nature of discharge: Profuse frothy, Colour of discharge:
Yellowish green, Pruritis: Present
Signs - Discharge: serous & frothy & greenish, pH: > 5, Vulva: inflamed,
Vagina: Strawberry Vagina
Ix: Wet mount: motile flagellated trichomonas, Clue cells: absent, Whiff test:
negative
• Definitive Dx: culture on Feinberg – Whittington media or Diamonds TYM
media
• Rx - Metronidazole 400mg TID x 5days ‘or’ Metronidazole 2gm single dose
(Mode : both partners)
118. A 14-year-old girl with well-developed secondary sexual characteristics
but presents with amenorrhea. What is the treatment you will prefer next?
a) Wait and Watch
b) Estrogen
c) Give ADH
d) GnRH
Amenorrhea can occur for a variety of reasons. Some are normal, while others may
be a side effect of medication or a sign of a medical problem.
Causes of 20 Amenorrhea
Pregnancy
Breastfeeding
Menopause
Contraceptives
Medications
Lifestyle factors
Hormonal imbalance
Structural problems
120. A female in labor is having occipital posterior position. The cervix gets
8cm dilated, forceps applied in occipitomandibular direction and tried to
rotate, but doctor is not able to rotate. In which type of pelvis, the fetal head
cannot be rotated?
a) Android pelvis
b) Anthropoid pelvis
c) Platypelloid pelvis
d) Gynecoid pelvis
Contracted pelvis: rotation of the head cannot easily occur in android pelvis due to
projection of the ischial spines and convergence of the side walls.
Anthropoid: The elongated shape of the anthropoid pelvis makes it roomier from
front to back than the android pelvis. But it’s still narrower than the gynecoid pelvis.
Some pregnant women with this pelvis type may be able to have a vaginal birth, but
their labor might last longer.
IMPORTANT POINTS
• M/c pelvis – Gynecoid*
• Least common pelvis – Platypelloid pelvis*
• The only pelvis with AP > Tr – Anthropoid pelvis*
• Face to pubis is M/c in – Anthropoid pelvis*
• OP position is M/c in – Anthropoid pelvis*
• DTA is M/c in – Android pelvis*
• Flat bowl shaped pelvis is – Platypelloid pelvis*
• Marked posterior asynclitism is seen in – Platypelloid pelvis*
122. A 35yr old Female presented with 14 week of gestation with bleeding per
vaginum, doctor started misoprostol with dose 200 mcg. How much maximum
dose can be given?
a) 3
b) 4
c) 5
d) 6
a) Polyhydraminos
b) Endometriosis
c) Endometritis
d) Mucinous ovarian adenoma
Ovarian mucinous cystadenoma is a benign tumour that arises from the surface
epithelium of the ovary. It is a multilocular cyst with smooth outer and inner
surfaces. It tends to be huge in size.
125. A Young Female is having inserted IUCD 6 months ago. Now she has
presented with the complaints of amenorrhea and fainting since two cycles.
What may be the cause for this condition?
a) PID
b) Endometriosis
c) Ectopic pregnancy
d) Failed contraception
The main risk factor for contraceptive-failure ectopic pregnancy is IUD failure. IUDs
do not increase the absolute risk of ectopic pregnancy, but a pregnancy
occurring with an IUD is more likely to be ectopic than intrauterine.
a) Clostridium difficile
b) Clostridium perfringens
c) Clostridium tetani
d) Corynebacterium diphtheria
127. A primigravida is in labor from last 12 hours with tender and rigid
uterus. Fetal head not palpable, loss of Fetal movements. what is the most
appropriate diagnosis?
a) Abruptio placenta
b) placenta previa
c) polyhydramnios
d) Prom
Placental abruption is a complication of pregnancy that happens when the placenta
separates from your uterus before your baby is born. Symptoms or signs can also
include:
Abdominal pain.
Uterine contractions that are longer and more intense than average labor
contractions.
Uterine tenderness.
Backache or back pain.
Decreased fetal movement.
128. A female with C-Section delivery, the baby was seeming to be normal
but the placenta was morbidly adherent, what could be the reason for this
condition?
a) absence of nitabuchs membrane
b) absence of decidua parietalis
c) absence of syncytiotrophoblast
d) absence of cytotrophoblast
129. A 37 weeks of pregnant female came with the complaint of bleeding per
vaginum associated with pain. the fetal heart rate was absent and uterus is
tender. what is the diagnosis and management for this patient?
a) Placental abruption and normal vaginal delivery
b) Placental abruption and LSCS
c) Placenta previa and normal vaginal delivery
d) Placenta previa and LSCS
bleeding per vagina with pain indicates placental abruption and since the fetal heart
rate was absent LSCS would be the best management.
The two primary hormones that are needed for lactation are prolactin and oxytocin.
Prolactin stimulates milk biosynthesis within the alveolar cells of the breast and
oxytocin stimulates contraction of the myoepithelial cells that surround the alveoli,
causing the milk to be ejected into the ducts leading to the nipple.
131. A pregnant lady delivered a child and after 24 hrs later she came with
complaint of heavy bleeding. What might be the cause for the bleeding in this
patient?
a) Battledore placenta
b) Succenturiate lobe
c) Velamentous placenta
d) Circumvallate placenta
Complications
increased incidence of type II vasa previa
increased incidence of postpartum hemorrhage due to retained placental tissue
132. Most common symptom seen in endometriosis is;
a) Polymenorrhea
b) Dysmenorrhea
c) HMB
d) Amenorrhea
Puerperal sepsis was defined as infection of the genital tract occurring at any time
between the onset of rupture of membranes or labour, and the 42nd day postpartum
in which two or more of the following are present:
Zinc deficiency occurs in infants when its demand exceeds its supply. It presents
with cutaneous signs which, in severe cases, are associated with diarrhea, alopecia,
and irritability. Genetic and acquired forms of zinc deficiency have been described
and often have overlapping clinical features.
a) Topical steroid
b) Topical antibiotics
c) Benzyl peroxide
d) Topical retinoids
Topical retinoids such as tretinoin or adapalene are effective in many patients with
comedonal acne. Patients with inflammatory lesions benefit from treatment with
benzoyl peroxide, azelaic acid or topical antibiotics. Frequently, the use of
comedonal and antibacterial agents is required
138. Identify the condition shown in the image?
a) Malignant melanoma
b) BCC
c) Squamous cell carcinoma
d) Nevus
a) Streptococcus
b) Staphylococcus
c) Bacteroides
d) Fungus
Erysipelas is an infection of the upper layers of the skin (superficial). The most
common cause is group A streptococcal bacteria, especially Streptococcus
pyogenes. Erysipelas results in a fiery red rash with raised edges that can easily be
distinguished from the skin around it.
140. Cutaneous horn as shown in the image is associated with which of the
following skin cancer?
Squamous cell carcinoma (SCC) is the second most common form of skin
cancer. It's usually found on areas of the body damaged by UV rays from the sun or
tanning beds. Sun-exposed skin includes the head, neck, chest, upper back, ears,
lips, arms, legs, and hands. SCC is a fairly slow-growing skin cancer.
142. Patient suffering from ulcerative colitis presents with ulcer on anterior
leg as shown in the image. Diagnosis?
a) Pyoderma gangrenosum
b) Venous ulcer
c) Tubercular ulcer
d) Chancroid
PSGN is a kidney disease that can develop after infections caused by bacteria called
group A Streptococcus (group A strep). These infections include throat and skin
infections like strep throat, scarlet fever, and impetigo. PSGN is not a group A strep
infection of the kidneys.
144. A tourist guide in Mt. Everest, after one of his trips developed blisters on
the feet.
Which is not useful for his treatment?
a) Hyperbaric Oxygen
b) Aspirin
c) Phenylephrine
d) Pentoxifylline
Phenylephrine is used to relieve nasal discomfort caused by colds, allergies,
and hay fever. It is also used to relieve sinus congestion and pressure.
Phenylephrine will relieve symptoms but will not treat the cause of the symptoms or
speed recovery.so it cannot be helpful for the patient with the above condition.
145. Patient presents with history of severe sunburn after only a few minutes
in the sun (photo sensitivity), freckling in sun
exposed areas, dry skin and changes in skin
pigmentation. Diagnosis?
a) Xeroderma pigmentosum
b) Bloom syndrome
c) Warts
d) Melanocytes nevus
Xeroderma pigmentosum (XP) is a very rare skin disorder where a person is highly
sensitive to sunlight, has premature skin ageing and is prone to developing skin
cancers. Xeroderma pigmentosum is caused by cellular hypersensitivity to ultraviolet
(UV) radiation, as a result of a defect in the DNA repair system.
146. Child presents with sores in the mouth and a rash on the hands and feet
as shown in the image below. What is the causative organism for it?
a) Coxsackievirus A16
b) Pox virus
c) Herpes virus
d) Human papilloma virus
Coxsackievirus can produce a wide variety of symptoms. About half of all kids with
an infection have no symptoms. Others suddenly get a high fever, headache, and
muscle aches, and some also develop a sore throat, abdominal discomfort, or
nausea. a type of coxsackievirus syndrome, causes painful red blisters in the throat
and on the tongue, gums, hard palate, inside of the cheeks, and the palms of hands
and soles of the feet.
147. Child presents with fever and pleomorphic rash as shown in the image.
Diagnosis?
a) Chicken pox
b) Small pox
c) Measles
d) Rubella
149. A patient has been diagnosed with a solid cancer of the bowel. He also
experienced massive proteinuria after a few months of initial diagnosis of
cancer. which of the following is a likely cause for the development of the
urinary finding?
a) Focal segmental glomerulosclerosis
b) Minimal change disease
c) Membranous Glomerulopathy
d) MPGN
With each cell division, telomeres shorten until eventually they become too short to
protect the chromosomes and the cell dies. Cancers become immortal by reversing
the normal telomere shortening process and instead lengthen their telomeres
151. A patient died of myocardial infarction and his heart was taken out and
immersed in a solution. The normal area of the heart turned brick red whereas
infarcted portion turned white. Which of the following can be likely component
of the solution?
a) Formalin
b) Triphenyl tetrazolium chloride
c) Ethanol
d) Glutaraldehyde
152. A patient presents with the presence of a thyroid swelling and weight
loss the physician found presence of cervical lymphadenopathy in the same
patient and decided to go in for a Thyroid biopsy. Histopathology report of the
patient suggested the presence of cells with orphan Annie eye nuclei. Which
of the following is a likely diagnosis for this patient?
a) Follicular thyroid cancer
b) Medullary thyroid cancer
c) Papillary thyroid cancer
d) Anaplastic thyroid cancer
The Orphan Annie-eyed clear nucleus, defined as a large, optically clear nucleus,
devoid of chromatin strands, with sharp chromatin rim, is a more specific feature
than are nuclear grooves or intranuclear cytoplasmic inclusions in papillary thyroid
carcinoma.
Hepatic angiosarcoma (HAS) is a rare type of liver cancer that is often fatal, and
arsenic and vinyl chloride monomer (VCM) are two major causal agents.
155. An old man underwent a bone marrow examination which revealed the
presence of the finding shown in the image.
What is the likely diagnosis of the affected
individual?
a) Multiple myeloma
b) ALL
c) Tuberculosis
d) Sickle cell disease
People with smoldering myeloma have some signs of multiple myeloma, such as any
of the following: Plasma cells in the bone marrow between 10% and 60% High
level of monoclonal immunoglobulin (M protein) in the blood. High level of light
chains in the urine (also called Bence Jones protein)
158. Which of the following is the molecule required for presentation of the
antigens by the antigen presenting cell to the TH2 cell?
a) Toll like receptor
b) MHC
c) G protein coupled receptor
d) Lectin receptor
159. Which of the following is the mode of inheritance in the patient suffering
from color blindness?
a) Autosomal dominant
b) Autosomal recessive
c) X linked dominant
d) X linked recessive
160. A patient has been diagnosed with a condition due to a DNA repair gene
defect. He is having hyper pigmentation on the skin and high risk for
development of skin cancers as per the physician. which of the following is a
likely enzyme defect in the patient?
a) Helicase
b) Nucleotide excision repair defect
c) ATM gene
d) MSH gene
Nucleotide excision repair (NER) is the main pathway used by mammals to remove
bulky DNA lesions such as those formed by UV light, environmental mutagens, and
some cancer chemotherapeutic adducts from DNA. Deficiencies in NER are
associated with the extremely skin cancer-prone inherited disorder xeroderma
pigmentosum.
161. Which of the following is a likely diagnosis of the patient looking at the
karyotype given in the image as Shown
a) Down syndrome
b) Turner syndrome
c) Klinefelter syndrome
d) Edward syndrome
162. Which of the following amino acids is required for the formation of nitric
oxide in the blood vessels?
a) Citrulline
b) Arginine
c) Histidine
d) Tryptophan
In the body, the amino acid arginine changes into nitric oxide (NO). Nitric oxide is a
powerful neurotransmitter that helps blood vessels relax and also improves
circulation. Some evidence shows that arginine may help improve blood flow in the
arteries of the heart.
a) Preseptal Cellulitis
b) Impetigo
c) Orbital cellulitis
d) Conjunctivitis
Orbital cellulitis is an infection of the soft tissues of the eye socket behind the orbital
septum, a thin tissue which divides the eyelid from the eye socket. Infection isolated
anterior to the orbital septum is considered to be Preseptal cellulitis. Orbital cellulitis
most commonly refers to an acute spread of infection into the eye socket from either
extension from periorbital structures (most commonly the adjacent ethmoid or frontal
sinuses (90%), skin, dacryocystitis, dental infection, intracranial infection),
exogenous causes (trauma, foreign bodies, post-surgical), intraorbital infection
(endopthalmitis, dacryoadenitis), or from spread through the blood (bacteremia with
septic emboli).
a) Simple Myopia
b) Irregular astigmatism
c) Regular astigmatism
d) Simple Hyperastigmatism
Each meridian in the regular astigmatic eye has a uniform curvature at every point
across the entrance of the pupil. This is the most common type of astigmatism in
which the symptoms included are blurry vision, headaches, and light sensitivity
169. In which chamber of the eye, given keratome is used in Phaco surgery?
a) Anterior chamber
b) Posterior chamber
c) Sclera
d) Lens
Acute angle closure is an urgent but uncommon dramatic symptomatic event with
blurring of vision, painful red eye, headache, nausea, and vomiting. Diagnosis is
made by noting high intraocular pressure (IOP), corneal edema, shallow anterior
chamber, and a closed angle on gonioscopy. Medical or surgical therapy is directed
at widening the angle and preventing further angle closure. If glaucoma has
developed, it is treated with therapies to lower IOP.
171. The patient was brought to the emergency. The patient was hit by a
tennis ball…. And noted to have hyphema. What is the source of bleed in this
patient?
a) Anterior ciliary artery
b) Posterior ciliary artery
c) Major arterial circle of iris
d) Minor arterial circle of iris
Hyphema is the collection of blood in the anterior chamber of the eye. The most
common cause of hyphema is blunt trauma, though spontaneous hyphemas can
occur in the setting of sickle cell disease or other increased bleeding states.
Hyphemas are graded based on the degree of blood obscuring the cornea. major
arterial circle of iris is the source of bleed.
172. A 2-year-old Child presented with leucocoria in the right eye since 2
months. On examination a total retinal detachment was the present in the
same eye. Ultrasound B scan revealed a heterogonous sub- retinal mass with
calcification. The most probable clinical diagnosis is:
a) Coats – Disease
b) Retinoblastoma
c) Toxocariasis
d) Retinal tuberculoma
Retinoblastoma is an eye cancer that begins in the retina — the sensitive lining on
the inside of your eye. Retinoblastoma most commonly affects young children, but
can rarely occur in adults. Your retina is made up of nerve tissue that senses light as
it comes through the front of your eye
a) LMA PROSEAL
b) Tight Seal
c) High Seal
d) Classical LMA
ProSeal LMA has a gastric drainage tube, placed lateral to the main airway tube.
The gastric drainage tube forms a channel for regurgitated gastric contents and
prevents gastric insufflation and pulmonary aspiration.
a) Univalent tube
b) Single Endo tracheal Lumen
c) Double Lumen Endo tracheal Tube
d) Ryles tube
Double lumen tubes play a crucial role in:
1. Airway management during thoracic surgery.
2. Anatomical lung separation or isolating normal from the diseased lung in
situations such as: Massive hemorrhage from one lung. ...
3. Differential lung protective ventilation in case of lung transplantation patients.
176. A lady was taken for C-section operation. the duration of surgery was
more than anticipated and the patient lands into post op hyperventilation.
which of the following could be the reason for that?
a) High Spinal Anaesthesia
b) Malingering
c) Hypoxic Encephalopathy
d) Cranial nerve palsy
177. A person was admitted into hospital for tibial fracture and a cast was
applied. After 3 weeks the cast was removed and another cast was applied.
The patient was given crutches and asked to walk. What is the stage of healing
of the bone?
a) Inflammation Phase
b) Remodeling Phase
c) Proliferative Phase
d) Infiltration Phase
Remodeling or also known as maturation phase is the fourth and final phase in
wound healing and lasts from 21 days up to 2 years. In this final and longest phase,
collagen synthesis is ongoing in order to strengthen the tissue. Remodeling occurs
as wound continues to contract and fibers are being reorganized.
178. Most common Associated Site of Tb Infection?
a) 1
b) 2
c) 3
d) 4
Bone tuberculosis is simply a form of TB that affects the spine, the long bones, and
the joints
In all cases of DDH, the socket (acetabulum) is shallow, meaning that the ball of the
thighbone (femur) cannot firmly fit into the socket. Sometimes, the ligaments that
help to hold the joint in place are stretched. The degree of hip looseness, or
instability, varies among children with DDH.
180. A patient had trauma and was applied cast. On removal of the cast the
doctor pressed the fracture site and described as movable or soft. True among
the following?
The callus holds the bone together, but isn't strong enough for the body part to be
used. Over the next few weeks, the soft callus becomes harder. By about 2–6
weeks, this hard callus is strong enough for the body part to be used.
181. All are true except
Paronychia is the inflammation of the tissue that immediately surrounds the nail. It is
the most common infection of the hand and it occurs after disruption of the seal
between the nail fold and the nail plate due to penetrating trauma, nail biting,
manicures or a hangnail
182. A parents brought child with the following image of thigh (image has
protruded bone with discharge). Parents confirmed there is history of bone
pain and discharge from the site prior to the protrusion. what could be the
diagnosis?
a) Acute Osteomyelitis
b) Chronic Osteomyelitis
c) Involucrum with Cloaca
d) Osteosclerosis
Chronic osteomyelitis, defined as continuous infection of a low-grade type or of a
recurrent type, is characterized predominantly by bony sclerosis, periosteal new
bone formation, and the presence of sequestra and/or draining sinuses.
a) GCT
b) Chondrosarcoma
c) Osteosarcoma
d) Ewings Sarcoma
a) P. Vivax
b) P.falciparum
c) P.malariae
d) P.ovale
a) Teniasolium
b) Trichuris spiralis
c) Trichuris trichiura eggs
d) Wuchereria bancrofti
Trichuris trichiura eggs are 50-55 micrometers by 20-25 micrometers. They are
barrel-shaped, thick-shelled and possess a pair of polar “plugs” at each end. The
eggs are unembryonated when passed in stool. Trichuris trichiura eggs are 50-55
micrometers by 20-25 micrometers. They are barrel-shaped, thick-shelled and
possess a pair of polar “plugs” at each end. The eggs are unembryonated when
passed in stool.
187. A 30-year-old patient of valvular heart disease is having fever for last 2
weeks. On physical examination the following features are noticed in this
patient. Which of these will be the abdomen examination finding of this
patient?
a) Ascites
b) Portal Hypertension
c) Splenomegaly
d) Renal artery Bruit
An enlarged spleen is the result of damage or trauma to the spleen from any of
several different medical conditions, diseases, or types of physical
trauma. Infections, liver problems, blood cancers, and metabolic disorders can
all cause your spleen to become enlarged, a condition called splenomegaly.
188. A 70-year-old man collapsed in his house and was rushed to the
hospital. On arrival his ECG tracing was as shown below. Which of the
following is the best intervention for this patient?
a) Use of Automated external defibrillator
b) Injection of Vasopressin 40 IU
c) Injection of adrenaline 1 mg iv
d) Injection of atropine 0.5mg iv
189. A 60-year-old woman presents with weakness in right arm for four
hours’ duration. The weakness gradually became lesser and resolves. She is a
known case of Hypertension and BMI is elevated. Which of the following is the
probable diagnosis of this case?
a) Transient ischemic attack
b) Compressive Neuropathy
c) Transverse Myelitis
d) Stroke
weakness in the right arm for 4 hours, which self resolves for a patient with
elevated BMI and Hypertension is suggestive of stroke
a) Fluid thrill
b) Shifting dullness
c) Puddle sign
d) Hingorani sign
In medicine, the fluid wave test or fluid thrill test is a test for ascites (free fluid in the
peritoneal cavity). It is performed by having the patient (or a colleague) push their
hands down on the midline of the abdomen. The examiner then taps one flank, while
feeling on the other flank for the tap.
192. An elderly patient with Hypertension fell down in his bathroom. His BP
was 220/110 on admission. Which of the following statements is correct about
this patient?
a) Start anti platelet drugs
b) Platelets are transfused even if the platelet count is not low
c) Reduce the BP to <120:
d) Putamen is the most common site
The putamen is the most common site for hypertensive ICH. Hemorrhages may
remain localized to the putamen; enlarge to involve the internal capsule, corona
radiata, centrum semiovale, or temporal lobe; or rupture into the ventricular system
193. A 70-year-old man has recurrent episodes of exertional syncope and
recurrent episodes of chest pain on climbing stairs. He was diagnosed as a
case of valvular aortic stenosis. Which of the following is the radiological
finding of XRC of this patient?
a) Dilated aortic Root:
b) Widening of aortic knob
c) Widening of vascular pedicle
d) Post stenotic dilatation of aorta
194. Elderly Chronic alcoholic patient has presented to OPD today. The
following findings were noted in the patient. What is the best investigation to
be done in this patient?
HBeAg negative shows that the virus is not actively replicating and the patient is not
infectious
HbsAg negative shows the patient is not currently infected with the virus
Anti HBs, IgG Anti HBc both are positive showing that the antibodies are formed for
the previous infection
197. A 30-year-old female patient presented with pain in joints of hand with
rash on dorsum of hand with extensive skin induration and skin tightening.
Comment on the diagnosis.
a) Rheumatoid arthritis
b) Psoriatic arthritis
c) Osteoarthritis
d) Scleroderma
Psoriatic arthritis is a form of arthritis that affects some people who have
psoriasis — a disease that causes red patches of skin topped with silvery scales.
Most people develop psoriasis years before being diagnosed with psoriatic arthritis.
a) Hepatolenticular degeneration
b) Alcoholic Cirrhosis
c) Portal Hypertension
d) Budd Chiari Syndrome
The patient condition is getting better and since there is no complaints of any
respiratory distress and equal air entry is heard in bilateral auscultation, we just
continue the observation of the patient and provide oxygen if needed.
201. A 55-year-old man has chest pain on rest for last 6 hours. ECG was
performed on admission and showed findings of myocardial ischemia. His
biomarkers report was sent to lab for diagnostic workup. Which of the
following will help in diagnosis?
a) LDH1
b) LDH2
c) LDH3
d) LDH4
Atenolol belongs to a group of medicines called beta blockers. It's used to treat
high blood pressure and irregular heartbeats (arrhythmia). It can also be used to
prevent chest pain caused by angina. If you have high blood pressure, taking
atenolol helps prevent future heart disease, heart attacks and strokes
203. A 30-year-old man has presented with complaints of fever for 3 days
with SOB. On examination on right mammary area bronchial breathing is heard
with normal air entry bilaterally. Comment on the diagnosis.
a) Right lower lobe pneumonia
b) Right middle lobe pneumonia
c) Right sided hydro-pneumothorax
d) Right sided Hydrothorax
Bronchial sounds are high pitched & usually heard over the trachea. Timing includes
an inspiratory phase that is less than the expiratory phase. If bronchial sounds are
heard in the actual lung fields, this may indicate consolidation.
a) Chemo-port device
b) Pacemaker
c) Artifact
d) Travel tract for bypassing accessory pathway
A pacemaker is a device that sends small electrical impulses to the heart
muscle to maintain a suitable heart rate or to stimulate the lower chambers of the
heart (ventricles). A pacemaker may also be used to treat fainting spells (syncope),
congestive heart failure and hypertrophic cardiomyopathy.
205. Prophylactic CNS radiation is given for which of the following cancers?
a) Liver cancer
b) Small cell cancer of lung
c) Prostate cancer
d) Seminoma
206. An elderly patient fell in the bathroom in early morning hours and was
rushed to the casualty. On arrival GCS is 9/15 and urgent NCCT head was
performed. Comment on the diagnosis?
a) Extradural hemorrhage
b) Subdural hemorrhage
c) Diffuse axonal injury
d) Intracerebral haemorrhage
208. Comment on the diagnosis of the Flow volume curve shown below
a) COPD
b) Interstitial lung disease
c) Retrosternal Goiter
d) Tracheal stenosis
210. Rabies virus will use which kind of molecular motor to cause
involvement of Central nervous system from the site of inoculation into the
host?
a) Dynein
b) Kinesin
c) Kinin
d) Actin
The patient is suspected to have hemothorax due to the contusion and so first we go
for elective intubation and PPV
a) AV intranodal conduction
b) Septal Activation of bundle of His
c) Antegrade conduction from Bundle of His to bundle branches
d) Antegrade conduction from Bundle to His to Purkinje fibers
HV interval measures the conduction time through the distal His- Purkinje
tissue and is measured from the onset of the His-bundle deflection to the earliest
ventricular activation.
215. A 60-year-old patient has right sided homonymous hemianopia with
macular sparing. Locate the neurological site of lesion in this patient.
a) Optic chiasma
b) Optic tract
c) Medial geniculate body
d) Occipital cortex
216. A 60-year-old man has presented with chronic projectile vomiting and
significant weight loss. Which of the following electrolyte abnormality is seen
in this patient?
a) Hypokalemic hypochloremic Metabolic alkalosis with Hyponatremia
b) Hypokalemic hypochloremic Metabolic alkalosis with Hypernatremia
c) Hypokalemic hypochloremic Metabolic alkalosis with Hypercalcemia
d) Hypokalemic hypochloremic Metabolic alkalosis with Hypomagnesemia
Pyoderma gangrenosum is a rare condition that causes large, painful sores (ulcers)
to develop on your skin, most often on your legs. The exact causes of pyoderma
gangrenosum are unknown, but it appears to be a disorder of the immune system
218. A patient presented with back pain. Labs show a grossly elevated ESR,
X- Ray skull shows multiple punched out lesions. Which is the most important
investigation to be done for this case?
a) Serum acid phosphatase
b) CT head with Contrast
c) PET scan
d) Serum Electrophoresis
back pain, elevated ESR X-ray showing multiple punched out lesions leading to
multiple myeloma. Serum protein electrophoresis is used to identify patients with
multiple myeloma and other serum protein disorders. Electrophoresis separates
proteins based on their physical properties, and the subsets of these proteins are
used in interpreting the results. Plasma protein levels display reasonably predictable
changes in response to acute inflammation, malignancy, trauma, necrosis, infarction,
burns, and chemical injury.
219. An AIDS Positive 30-year-old male patient has developed fever, vomiting
and meningismus. Which of the following test: will have help in rapid
diagnosis of Cryptococcal meningitis?
a) India Ink preparation of CSF
b) Blood Lateral flow assay for Cryptococcal antigen
c) Blood culture
d) CSF Cultures
220. A 30-year-old man who works in a glass factory presents with complaints of
progressive shortness of breath on exertion. He is predisposed to development of
which of the following?
a) Histoplasmosis
b) Tuberculosis
c) Mesothelioma
d) Pleural calcification
since the patient was under steroid therapy for Immune thrombocytopenic purpura
(ITP). Since there is no petechiae or any bleeding other than less platelet count ,
most probable next step should be to wait and observe as acute ITP is self-resolved
in most cases of children.
Delirium tremens (DTs) is the most severe form of ethanol withdrawal, manifested
by altered mental status (global confusion) and sympathetic overdrive (autonomic
hyperactivity), which can progress to cardiovascular collapse. Minor alcohol
withdrawal is characterized by tremor, anxiety, nausea, vomiting, and insomnia
224. A 25-year-old male patient is having loose stools 18 times per day and
his urine frequency is reduced. What will be the correct finding of this patient?
a) Urine sodium >40
b) Kidney biopsy showing Glomerular damage
c) Hyperosmolar urine
d) Post renal azotemia
Both high and low urine osmolality have several causes. Some of these, such as
dehydration, are relatively easy to treat. Others can be more serious or ongoing.
Here due to severe dehydration, hyperosmolar urine can be found.
Pitting edema is a specific type of edema that is associated with pitting or indentation
in the affected areas, and is often caused by conditions that lead to the
pooling of blood in the feet or legs. The excess fluid that builds up in pitting edema is
mainly composed of water.
226. A 6-year-old boy has presented with multiple episodes of hematemesis.
On admission his vitals are stable and splenomegaly is noted. The mother tells
that at birth the boy had developed jaundice and an exchange transfusion
procedure had been performed in the same hospital. Diagnosis is
a) Budd Chiari Syndrome
b) Sinusoidal fibrosis
c) Portal vein thrombosis
d) Non cirrhotic portal fibrosis
Portal vein thrombosis is blockage or narrowing of the portal vein (the blood
vessel that brings blood to the liver from the intestines) by a blood clot. Most people
have no symptoms, but in some people, fluid accumulates in the abdomen, the
spleen enlarges, and/or severe bleeding occurs in the esophagus.
227. A 6-year-old child weighs 12 kg. The urine output of the child is only
0.5ml / kg / hour for last 12 hours and serum creatinine has shown a doubling
over the baseline. What is stage of AKI by KDIGO criteria?
a) 1
b) 2
c) 3
d) 4
228. A girl is having honey crust injuries on face which was diagnosed by her
paediatrician as impetigo. She has now been brought your hospital with facial
puffiness and not eating well. When you work up the patient, which of the
following will be the most common finding encountered in this case:
a) RBC in urine
b) Nephrotic range proteinuria
c) Derangement of liver enzymes
d) Focal neurological deficit
Transient proteinuria and hematuria may occur during impetigo and resolve before
renal involvement develops. Antibiotic treatment does not prevent the development
of glomerulonephritis, but it limits the spread of the disease to other individuals.
Other complications may include the following: Scarlet fever.
230. A 35-year-old patient presents with high grade fever, chills and cough
with copious production of purulent sputum. An x-ray examination was made
as shown in the image, what is the most likely diagnosis?
232. Pt with History of Recurrent UTI, the IVP image is given, Identify the
location of insertion of ureter arising from left superior pelvis?
a) Optic Nerve
b) Optic Chiasma
c) Optic tract
d) P Superior Ophthalmic Artery
234. 5yr old boy is unable to pronate and supinate since childhood?
a) Radioulnar synostosis
b) Radial head dislocation
c) Monteggia fracture
d) Myositis ossificans
a) Pons
b) Medulla
c) Midbrain
d) Spinal cord
236. 45 year old Male with Heart Disease undergoes a chest X-ray, what is the
diagnosis?
a) Prosthetic aortic valve
b) Prosthetic mitral valve
c) Pericardial effusion
d) Implantable pacemaker
a) Hemolytic anemia
b) Multiple myeloma
c) Increased ICT
d) Osteopetrosis
Hemolytic anemia is a disorder in which red blood cells are destroyed faster
than they can be made. The destruction of red blood cells is called hemolysis. Red
blood cells carry oxygen to all parts of your body. If you have a lower than normal
amount of red blood cells, you have anemia.
238. X-ray of a 2-year-old child is given, what is the possible diagnosis?
a) Pneumastosis intestinalis
b) Pneumoperitoneum
c) Subcutaneous emphysema
d) Duodenal atresia
240. A 45-year-old female came for post-surgery follow up, A CXR was taken
what is the most likely diagnosis?
a) Malignancy
b) Fibroadenoma
c) Emphysema
d) Left sided mastectomy
Unicornuate uterus is a rare genetic condition in which only one half of a girl's
uterus forms. A unicornuate uterus is smaller than a typical uterus and has only one
fallopian tube. This results in a shape often referred to as “a uterus with one horn” or
a “single-horned uterus.
242. A female patient with dysphagia, Barium swallow image is shown, what is the
likely diagnosis?
Achalasia Cardia is a rare disorder that makes it difficult for food and liquid to pass
from the swallowing tube connecting your mouth and stomach (esophagus) into your
stomach. Achalasia Cardia occurs when nerves in the esophagus become damaged.
The hallmark of the achalasia pattern is a gradually tapering, smooth, conical
narrowing (bird's beak or rat-tail appearance) of the distal esophageal segment
which extends about 1-3 cm in length
243. A 65-Year-old female fell down in the bathroom &sustained head injury, Ct
scan was performed, what is likely diagnosis?
a) Extradural hematoma
b) Intracranial bleed
c) Thrombosis
d) Subdural hematoma
A ureterocele is a swelling at the bottom of one of the ureters. Ureters are the
tubes that carry urine from the kidney to the bladder. The swollen area can block
urine flow. A ureterocele is a birth defect
a) Medial epicondyle
b) Trochlea
c) Capitulum
d) Olecrenon
In human anatomy of the arm, the capitulum of the humerus is a smooth, rounded
eminence on the lateral portion of the distal articular surface of the humerus.
247. 33 yr. old female with fever, weight loss, elevated levels of ACE levels
with hilar lymphadenopathy on x ray, Probable diagnosis?
a) TB
b) Sarcoidosis
c) Silicosis
d) Small cell cancer
Sarcoidosis is an inflammatory disease that affects one or more organs but most
commonly affects the lungs and lymph glands. As a result of the inflammation,
abnormal lumps or nodules (called granulomas) form in one or more organs of the
body.
248. Identify the structure being pointed by the Arrow?
a) Hepatic artery
b) Portal vein
c) Hepatic vein
d) Inferior venacava
The portal vein (PV) is the main vessel of the portal venous system (PVS),
which drains the blood from the gastrointestinal tract, gallbladder, pancreas,
and spleen to the liver.
249. A patient met with a RTA, is unconscious and the CT scan does not
show any findings except for few petechial hemorrhages, what is the probable
diagnosis?
a) Diffuse Axonal injury
b) Concussion
c) Contusion
d) Subarachnoid hemorrhage
251. A 35 yrs old patient from Bihar comes with generalized bone pain and
was diagnosed to have Fluorosis. Fluorosis of bone is commonly associated
with?
a) Osteosclerosis
b) Osteomalacia
c) Osteopetrosis
d) Osteochondrosis
Exposure to very high fluoride over a prolonged period results in acute to chronic
skeletal fluorosis Vague, diffuse aches, muscle weakness, chronic fatigue and
stiffness of joints with decreased range of motion are common initial symptoms.
These symptoms may be dismissed as functional but may, in fact, be early signs of
fluoride damage to tendinous insertions and ligaments as well as joint capsules.
During later stages, calcification of the bones takes place, osteoporosis in long
bones and symptoms of osteosclerosis where the bones become denser and
develop abnormal crystalline structure develop.
252. Patient with Right Hypochondrial pain, Water Lily Sign is seen on
Ultrasound, what is the diagnosis?
a) Entamoeba
b) Schistosoma
c) Ascariasis
d) Hydatid cyst
Aspirated foreign bodies most commonly are lodged in the right main stem and
lower lobe. Aspiration has been documented in all lobes, including the upper lobes,
though with less frequency
255. A Patient with Brain Metastasis had to undergo Head and Neck
Irradiation, what is the most common delayed symptom with Head and Neck
Irradiation?
a) Xerostomia
b) Dysphagia
c) Dysgeusia
d) Anhidrosis
a) Hydropneumothorax
b) Pneumothorax
c) Pneumo-mediastinum
d) Lower lobe pneumonia
a) Staphylococcus
b) Streptococcus
c) Histoplasmosis
d) Miliary TB
a) Microsporium gypseum.
b) Microsporium canis
c) Trichophyton Epidermidis.
d) Trichophyton tonsurans
261. A boy fell down while playing and the nose is deviated but septum was
found normal, what is the next step done?
Initially medications given to treat the edema and close reduction done after 7 days.
262. Where is the marked structure placed in the heart to activate the device
present here?
a) AVN
b) SAN
c) Bundle of His
d) Purkinje fibres
263. A 15-year-old boy playing football has an injury to the leg but now
complains of deep bone pain and high grade fever. A x ray was taken. What is
the diagnosis?
a) Osteochondroma
b) Ewings sarcoma
c) Osteosarcoma
d) Osteochondroma
264. A 43-year-old male returned from abroad and after a week develops
breathing difficulty with loss of smell and taste. Which is the next best line of
investigation?
a) Sputum examination
b) Nasopharyngeal swab
c) CT chest
d) Bone marrow biopsy
The patient shows the symptoms of SARS COVID-19, the next appropriate step for
the investigation would be nasopharyngeal swab
265. All the following measures are useful against COVID 19 except?
a) 2% Glutaraldehyde
b) 70% ethanol
c) Face mask
d) Hand wash
Hand wash, Face mask, 70% ethanol are all effective against the spread of COVID-
19.
2% glutaraldehyde is the disinfectant used for hospital instruments
266. Identify the image given below?
a) 1 and 2
b) 2 and 3
c) 1 and 3
d) All
A. Thalamus
B. Caudate nucleus
C. Lentiform nucleus
D. Claustrum
Answer: C
Explanation:
This is the transverse section of cerebrum
2. Which of the following nerve is involved in the given condition?
A. Axillary nerve
B. Ulnar nerve
C. Median nerve
D. Radial nerve
Answer: B
Explanation: from the following images of nerves related to humerus, we can
clearly see that the ulnar nerve passes through the medial epicondyle.
A. Gluteus maximus
B. Gluteus medius
C. Gluteus minimus
D. Iliopsoas
Answer: D
Explanation: the muscle attached to lesser trochanter is Iliopsoas whereas in
the greater trochanter , Gluteus medius
5. Identify the type of joint that is marked by the arrow?
A. Syndesmosis
B. Primary Cartilaginous joint
C. Secondary Cartilaginous joint
D. Synchondrosis
Answer: C
Explanation: Secondary Cartilaginous joint are present in the midline of the
body except Symphysis Menti
A. Cuboid
B. Talus
C. Navicular
D. Cuneiform
Answer: C
Explanation:
A. Radial nerve
B. Ulnar nerve
C. Median nerve
D. Musculocutaneous nerve
Answer: B
Explanation:
from the following images of nerves related to humerus, we can clearly see
that the ulnar nerve passes through the medial epicondyle.
11. Tingling sensation and paresthesia on medial side of hand. Which structure
most likely to be affected?
A. A
B. B
C. C
D. D
Answer: A
Explanation:
A is Brachial plexus that is marked here
B is Subclavian artery and vein
C is Trachea
D is Clavicle
BIO CHEM
1. Enzyme deficiency in von gierke’s disease?
(a) Glucose-6-phosphatase
(b) Branching enzyme
(c) Acid maltase
(d) Glycogen Phosphorylase
2. Patient presents with drying of skin and appears as shown below. Identify the
vitamin deficiency?
(a) Biotin
(b) Niacin
(c) Riboflavin
(d) Ascorbic acid
Ans. (b) Niacin deficiency
(a) Oligomycin
(b) Malonate
(c) Cyanide
(d) Rotenone
Ans. (c) Cyanide
(a) Alkaptonuria
(b) Phenylketonuria
(c) Maple syrup urine disease
(d) Hartnup’s disease
(a) Zinc
(b) Copper
(c) Calcium
(d) Iodine
(a) Glutamine
(b) Urea
(c) Uric acid
(d) Nitric oxide
Nitrogenous waste is excreted from the body in the form of uric acid. The
nitrogenous waste that is generated in the body tends to produce toxic ammonia if
remained in the body it must be excreted.
Urea and uric acid are the most common nitrogenous waste products in terrestrial
animals; freshwater fish excrete ammonia and marine fish excrete both urea and
trimethylamine oxide.
FORENSIC
1. Patient’s relatives are complaining about patient’s death and filed a case against
the doctor. Which IPC section do you think will be appropriate to punish the
medical practitioner?
Causing death by negligence. --Whoever causes the death of any person by doing
any rash or negligent act not amounting to culpable homicide, shall be punished with
imprisonment of either description for a term which may extend to two years, or with
fine, or with both.
(a) Datura
(b) Hyoscine
(c) Thebaine
(d) Tetrahydro-cannabinol
(a) Magistrate
(b) Police
(c) Village headman
(d) Judge
4. A dead fetus was found in water. The crown heel length was found to be 25 cm.
Light hairs were present on the skull with ossification center. Approximate age of
fetus is –
(a) 5 months
(b) 7 months
(c) 9 months
(e) 3 months
RULE OF HASSE
Rule of hasse is used to determine the age of the foetus from crown to heel
length in cm
During first five months , length in cm = ( age in months ) 2
After age of > 5months, Age in months = ( length in cm )/5
5. A man was found to be agitated and shouting at the airport. Later on he fell down.
On further checking, his bag was found to have enema apparatus and laxatives.
All of the following are true regarding this case except?
A body packer is someone who carries drugs such as heroine or cocaine, packed in
rubber or plastic, in his/her body in order to smuggle them. These people can
present with symptoms that vary from mild abdominal complaints to respiratory
insufficiency and even death. Physical examination and additional radiology tests are
helpful for the diagnosis. Any packages can usually be seen on a plain abdominal X-
ray.
6. A person was found dead in bushes with tied arms and legs, cyanosis of the
nails, scratches over face, neck and chin, laceration near lips was noticed. Which
of the following is not the probable mode of death in this case?
(a) Throttling
(b) It’s a Homicide
(c) Asphyxia
(d) Case of a suicide
Suicide is death caused by injuring oneself with the intent to die. A suicide
attempt is when someone harms themselves with any intent to end their life
7. A gunshot case brought for autopsy shows an entry wound with inverted margins
& surrounded by blackening and tattooing. Burning & singeing of hairs is noted.
What is the range of firearm weapon?
(a) Near
(b) About 2 meters
(c) Contact
(d) Distant
Depending on the distance of fire, the wound can also classify as contact wounds –
again divided into firm-contact and loose contact – near-contact or close-range
wounds, mid-range or intermediate-range wounds, distant or far-range wounds, and
indeterminate wounds.
Contact wounds are self-explanatory and are diagnosable by the presence of
muzzle-imprint on the skin. Near-contact or close-range wounds are identifiable
by the presence of flame burns and singeing of hair. Mid-range or intermediate-
range wounds do not have flame burns or singeing of hair but will show the presence
of smoke, gas, and unburnt particles. Distant or far-range wounds will be embedded
with any accompanying components other than the lubricant forming a grease collar.
Indiscriminate wounds are atypical wounds beyond the range of the weapon with an
irregular shape due to yawing (changing in the axis and direction of flight of the
projectile).
8. A piece of cloth was obtained from a crime scene with some spots. On
examination, yellow colored needle shaped crystals were found. Which body
secretions it can be?
(a) Serum
(b) Semen
(c) Saliva
(d) CSF
Ans. (b) Semen
Barberio test was invented by Barberio in the year 1905. When the questioned
stain is allowed to react with picric acid it leads to the formation of yellow
needle shaped spermine picrate crystals, including the presence of seminal stain.
Scorpion sting
- More than > 10 Species are present
- Indian red scorpion is highly venomous
- Scientific name → Mesobuthus tumulus
- Nature of venom → resembles snake venom
Action:
- Scorpion venom acts on Na+& K+ Channels
↓
(Autonomic Storm)
1. Pain → the tap sign
2. Paresthesia
3. Systemic Symptoms → Vomiting, Sweating, Salivation, chest pain,
anxiety, cardiac arrhythmias
Rx:-
1. Immobilise
2. Pain relief
3. Prazosin
4. Scorpion antivenom (If available)
10. Dead body of a 12yr old child was found. Which bone would be best for gender
assessment?
(a) Skull
(b) Femur
(c) Pelvis
(d) Mandible
Ans. (c) Pelvis
Krogman’s Table
%age of accuracy in determination of sex from bone.
Pelvis 95%
Skull 90-92%
Long Bones 80%
Pelvis + Skull 98%
All 206 bones 100%
• Best bone for determination of sex is Pelvis
11.A case of RTA was brought for autopsy with stiffness of neck, hands, both upper
and lower limbs. What is this condition called?
Rigor mortis: Literally, the stiffness of death. The rigidity of a body after death. ...
Rigor mortis is due to a biochemical change in the muscles that occurs several hours
after death, though the time of its onset after death depends on the ambient
temperature
12. A woman tried to fake the pregnancy & delivery. After sometime she brought a
baby claiming it to be her neighbor’s child and trying to blackmail him for getting
benefit in property. Such kind of child is called as?
(a) Suppositious child
(b) Spurious
(c) Superfecundation
(d) Superfetation
Ans. (a) Suppositious child
Supposititious children are fraudulent offspring. These arose when an heir was
required and so a suitable baby might be procured and passed off as genuine.
PATHO
(a) AL
(b) AA
(c) Beta 2 microglobulin
(d) ATTR
2. A person develops pruritic rashes every time on eating sea food within one hour.
Which type of hypersensitivity is this?
(a) I
(b) II
(c) III
(d) IV
Ans. (a) I
3. A nurse got needle prick injury from a HIV positive patient while doing blood
sampling. Which test would be the confirmatory test to rule out the infection?
(a) ELISA
(b) P24 assay
(c) Western blot
(d) Blood culture
The p24 test identifies actual HIV virus particles in blood (p24 is a capsid
structural protein which makes up a protein 'shell' on the surface of the HIV virus).
However, the p24 test is generally only positive from about two to three weeks after
infection with HIV.
4. A patient comes for organ transplantation. He has a twin brother who is a match
donor for
him. Which type of grafting it would be considered as?
(a) Isograft
(b) Allograft
(c) Autograft
(d) Xenograft
5. A 83-year-old female suffered with heart disease and with cardiac atrophy. On
H/E, perinuclear brown colored deposits were seen. What could be the pigment
responsible for this?
(a) Hemosiderin
(b) Melanin
(c) Lipofuscin
(d) Iron
Thalassemia is an inherited blood disorder that causes your body to have less
hemoglobin than normal. Hemoglobin enables red blood cells to carry oxygen.
Thalassemia can cause anemia, leaving you fatigued.
Numerous target cells are present in this patient with hemoglobin E and beta
thalassemia trait. Target cells, or codocytes, have an excess of cell membrane
relative to cell volume. Macrocytic target cells can be seen in liver disease, and
microcytic target cells may be seen in thalassemia.
7. The Image below shows origin from which of the following cells?
(a) B cells
(b) CD4 T cell
(c) NK cell
(d) CD8 T cell
Reed–Sternberg cells (also known as lacunar histiocytes for certain types) are
distinctive, giant cells found with light microscopy in biopsies from individuals with
Hodgkin lymphoma. They are usually derived from B lymphocytes, classically
considered crippled germinal center B cells.
(a) IDA
(b) Thalassemia major
(c) Myelodysplastic syndrome
(d) Myelofibrosis
A marked increase of dacrocytes is known as dacrocytosis. These tear drop cells are
found primarily in diseases with bone marrow fibrosis, such as: primary
myelofibrosis, myelodysplastic syndromes during the late course of the disease, rare
form of acute leukemias and myelophthisis caused by metastatic cancers.
CMV is a herpesvirus, has double-stranded DNA, and renders infected cells 2–4
times the size of surrounding cells. These cytomegalic cells contain an eccentrically
placed intranuclear inclusion surrounded by a clear halo, with an “owl's-eye”
appearance.
10. A 50-year-old man presented with gum bleeding. Peripheral smear shows
marked leukocytosis with 70% cells showing MPO positivity. What could be the
diagnosis?
(a) AML
(b) ALL
(c) CML
(d) CLL
Ans. (a) AML
Nucleus
Cytoplasm
Mitochondria
Ribosomes
Ans. Cytoplasm
13. A 10 year old boy presented with a swollen knee after suffering a trauma. Which
of the following is the likely condition seen in this patient?
Hemophilia is a disease that prevents blood from clotting properly. A clot helps stop
bleeding after a cut or injury. In factor VIII deficiency (hemophilia A), the body
doesn't make enough factor VIII (factor 8), one of the substances the body needs
to form a clot.
SPM
1. Identify the logo given in the image below:
A. Sikkim
B. Kerala
C. Jammu & Kashmir
D. Andaman & Nicobar
Answer: D
Explanation: Andaman & Nicobar Islands and Lakshadweep Island from our country
are currently Rabies-free. As import of animals from these are avoided and hence it
was easy to eradicate rabies in these states. Remember water is an effective barrier.
3. You are a medical officer and now you have to check an ANM worker’s ability
to do immunization who has been given training. She is having cotton,
syringe-needle and lunch box after immunization of a child. Which of the
following statement is correct?
Answer: D
Explanation: NITI (National Institute Transforming India). It is actually new
name of pre-existing NPC (National Planning Commission). It was established
in 1950 but in 1st January 2015 NITI Aayog was formed. Comprising of Prime
Minister as the Chair Person and along with him the chief ministers of all the
states, Lieutenant Governor of Union territories.
5. In a population, a screening test was used for a disease, from the image
below what does B-C represent:
A= Disease onset
B= First possible
detection
C= Critical point
A. Lead time D= Usual time of
B. Screening time diagnosis
C. Incubation time E= final outcome
D. Latent period
Answer: B
Explanation: Screening time
B-D = lead time also known as advantage gained by a screening test.
A. 3000
B. 5000
C. 1000
D. 2000
Answer: C
Explanation: ASHA (Accredited Social Health Activist). A worker under
National Rural Health Mission (NRHM) 2005-2012. From 2013 called as NHM
(national Health Mission)
Was designated for
1 per village
1 per 1000
From 2013, 2 per village of a population of 1000
7. Calculate Neonatal mortality rate from the data given below. Total live births
were 4000
Total death among day 0-7 :40
Total death among day 7-28 :40
Total death >than 1 month :40
Perinatal death :40
A. 10
B. 20
C. 30
D. 40
Answer: B
Explanation: A neonate is one who is having an age of 0-28 days. And is
Divided into 2
i. Early Neonatal Period = 0-7 days (40 from question)
ii. Late Neonatal Period = 8-28 days (40 from question)
Therefore, neonatal period = early neonatal period + late neonatal period
Answer: B
Explanation:
Total fertility rate (TFR) = total no of children born to a woman in her
entire reproductive life.
Gross Reproductive rate (GRR) = total no of girl children born to a
woman in her entire reproductive life.
Net reproduction rate (NRR) = total no of girl children born to a woman
in her entire reproductive life taking in account their mortality
Completed family size is simply a synonym for TFR.
A. Anatomical waste
B. Sharp waste
C. Glassware waste
D. Plastic waste
Answer: B
Explanation: A white container, puncture proof and sometimes translucent:
12. For a female with Vaginal discharge, STD color kit code would be?
A. Kit 1 Grey
B. Kit 2 Green
C. Kit 3 White
D. Kit 4 Blue
Answer: B
Explanation
14. After which phase of clinical trials, a new drug is introduced in the market?
A. Phase 1
B. Phase 2
C. Phase 3
D. Phase 4
Answer: C
Explanation:
Phase 1 For safety and toxicity profile of a Done in HHV (healthy human
drug volunteers)
Phase 2 For effectiveness Done in patients
Phase 3 For comparison trial with existing Also done in patients
drug
Phase 4 For long term, rare side effects Done in patients
Phase 4 is also called as Post-Marketing Surveillance and is the
longest phase of a trial
Phase 3 is RCT
Maximum failure is reported from phase 2
Maximum tolerated dose (MTD) of a drug is assessed in Phase 1
Under new guidelines, before phase 1, we have a phase 0, where drug
is tested in very small doses in very small number of healthy human
volunteers to access for its efficacy and non-toxicity profile also known
as phase 0 micro dosing.
15. Parameter used to test for confirmatory diagnosis when a Health worker nurse
got an accidental prick with HIV+ needle would be?
A. ELISA
B. p24 Antigen
C. Western Blot assay
D. PCR
Answer: C
Explanation: when we get needle prick injury, we get 0.1-0.3% of
transmission, it is also a big risk of transmission
-Screening is done by ELISA. It has very high sensitivity, but it lacks
specificity and its main role is to detect antibodies
- if ELISA is positive then confirmatory diagnosis is done by WBA(Western
Blot Assay) according to Indian Program, it is a protein based test and it a
high specificity, it is based on p24 and GP41
-p24 antigen test can be used in window period before seroconversion
-Another test use din window period is HIV RNA Genomic sequencing (NAAT)
Nucleic acid amplification test.
16. One student developed abdominal pain, jaundice and fever. Soon other
students presented with similar symptoms, they are all from same boarding
school and have been eating from the same canteen. Hostel Medical officer
will investigate which of the following parameters to establish diagnosis?
A. IgM for Hepatitis A
B. IgG for Hepatitis A
C. IgM for Hepatitis B
D. IgG for Hepatitis B
Answer: A
Explanation: out of the all types of hepatitis, A & E are the ones with fecal oral
transmission and A is the most common cause of hepatitis in children and
adults, since we are doing immediate test so it has to be IgM antibody of
hepatitis A.
Hepatitis A is caused by Enterovirus 72 which is from Picornaviridae family.
And in children it is sub-clinical. Incubation period of 15-45 days and period of
infectivity is 2 weeks prior to Jaundice till 1 week after. Elisa is used as
confirmatory to find the antibody.
A Patient is presented with rash and diarrhea for 2 months, from the image
given below, find the deficiency
17. A Patient is presented with rash and diarrhea for 2 months, from the image
given below, find the deficiency?
19. A lactating woman came to OPD 6 weeks after delivery for check-up and to
get advice for the contraception, which of the following is not advised?
A. Combined OCP
B. Norplant
C. IUCD
D. Mini-pill
Answer: A
Explanation: Combined OCP suppresses lactation due to its estrogen
component. So, we advise the mother to opt for mini-pill (lactation pill) and it
has only progesterone in the form of LNG (Levonorgestrel).
24. Used to prevent Covid-19 among health workers, Relative risk for two
vaccines, Relative risk for two vaccines A and B is 0.5 and 2.0 respectively.
Identify the graph(s) which represent them correctly?
A. A & B
B. A & C
C. B only
D. None of the above
Answer: A
Explanation: RR risks of 2 vaccines were given. It is used to measure strength
of association in a Cohort study. It can be
RR > 1, Positive association
RR = 1, No Association
RR < 1, Inverse Association
From the options both graphs A & B are correct
25. What is Urban Heart?
A. Urban Health Equity Assessment and Response Tool
B. Urban Health Evaluation assessment and Response Tool
C. Urban Health Equity Assessment and Response Technique
D. Urban Health Evaluation Assessment and Response Technique
Answer: A
Explanation: Tool used by WHO (World Health Organization) to identify and
reduce health inequities in the city
It has 3 components:
Sound Evidence
Intersectoral Action
Community Participation
It is integrated into local planning cycle
26. The agency that collects data and publishes large scale surveys continuously
regarding morbidity, family planning, vital events?
A. AIIMS
B. Central Bureau of Health
C. National Sample Survey
D. Sample registration system
Answer: C
Explanation: National Sample Survey taken up by NSSO
(National Sample Survey Organization)
NPV
29. Newborn birth rate is 25 per 1000 population in a village with total population
of 5000. What is the total number of pregnancies per year?
A. 125
B. 138
C. 145
D. 250
Answer: B
Explanation: First we calculate
Expected births for 1000 = 25
So, for 5000 = 125 births
But in question total no of pregnancies is asked
In India we have about 10% of pregnancy wastage so we add that along with
125 here
30. Under ESI after certification by medical officer, beneficiaries get 70% wages
for 3 months under?
A. Medical Benefit
B. Sickness Benefit
C. Disablement Benefit
D. Dependent’s Benefit
Answer: B
Explanation:
31. In a group of 200 Hypertensive patients. First group of 100 follow medical
advice whereas second group of 100 follow lifestyle advice. There is test for
both the groups after 3 months. Which of the following is best test for
significance to compare Blood pressure levels in both the groups?
A. Paired Student’s T test
B. Unpaired Student’s T test
C. Fischer test
D. Chi-Square test
Answer: B
Explanation: Comparing Quantitative data in two groups, we use Unpaired
Student’s T test
If Comparing Quantitative data in same group before and after an intervention
we use Paired Student’s T test
Answer:
Explanation: Full form National Program for prevention and control of Cancer
Diabetes CVD’s Stroke (NPCDCS)
Through formation of NCD clinics, this was established in 100+ districts in
India
Now 450 districts in country have this and they have included COPD and CKD
33. A doctor is teaching an intern about knee reflex demonstration. What type of
learning is this?
A. Cognitive learning
B. Affective learning
C. Psychomotor learning
D. None of the above
Answer: C
Explanation: skills is associated with psychomotor learning
35. Blood smear for malaria will be collected by which of the following health
worker?
Answer: D
Explanation: Multipurpose health worker male has the main duty of collecting
blood smear for malaria.
ANAESTHESIA
1. Which is the following I.V. anesthetic drug used for day care surgeries?
(a) Ketamine
(b) Propofol
(c) Thiopentone
(d) Etomidate
Explanation: Day care anesthesia is involved in small procedures that are done in
one day, that is he has to be admitted, operated, recovered and discharged on the
same calendar day, so the anesthetic agent should be short acting, easily
metabolized and excreted and that is why we choose propofol. It is the anesthetic
agent of choice for day care. It has anti-emetic and anti-pruritic properties and has
not much side effects.
Opioid of choice for day care: Remifentanil
Inhalational agent of choice in day care: Sevoflurane
(a) Ketamine
(b) Fentanyl
(c) Thiopentone
(d) Halothane
(a) Grey
(b) Orange
(c) Pink
(d) Green
Ans. (b) Orange
Explanation:
a. Field block
b. Nerve block
c. Infiltration
d. All of the above
Answer: D
Explanation: All of the above
2. A person with anxiety has a felling that his parents will die one day and he will
have sleepless nights, which of the following disorders describes his
condition?
A. Generalized anxiety
B. Severe depression
C. PTSD
D. Adjustment disorder
Answer: A
Explanation: Generalized anxiety disorder: in generalized anxiety disorder
We have major and minor criteria for making a diagnosis of generalized
anxiety disorder we need 2 major and 3 minor for 6 months. Treatment would
be SSRI and behavior therapies
Major criteria
Excessive worry
Not able to control
Minor criteria
Restlessness
Muscle tension
Irritability
Sleep disturbance
Easy fatigability
Decreased concentration
3. A kid with normal learning skills but poor mathematical skills. His IQ is
appropriate to his age. He passed the exams with poor marks. What could be
the probable diagnosis?
A. Exam phobia
B. ADHD
C. Specific learning disorder
D. Asperger’s syndrome
Answer: C
Explanation: in Specific learning disorder (Dyslexia)
We have reading, writing, mathematical skills, mixed types
For making a diagnosis of Dyslexia, IQ should be normal for the child
H/o migration should be ruled out
ORTHO
A. Galeazzi fracture
B. Monteggia deformities
C. Monteggia Fracture
D. Green stick fracture
Answer: A
Explanation: this a fracture involving disruption of distal radial-ulnar joint with
damage to the interosseous membrane and triangular Fibro cartilage
component (TFCC). this is Galeazzi Fracture.
-Monteggia fracture is a fracture of ulnar with dislocated radial head
4. A patient comes after injury, from the following X-ray, What is the probable
diagnosis?
A. Tibial Fracture
B. Patellar Fracture
C. Knee loose body
D. Femur fracture
Answer: B
Explanation: the image clearly shows fracture of Patella and the treatment is
Tension Band wiring
A. Osteosarcoma
B. Chondrosarcoma
C. Chondroblastoma
D. Ewing’s sarcoma
Answer: B
Explanation: - since osteosarcoma is a lytic type
- Chondroblastoma occurs in epiphysis
- Ewing’s sarcoma occurs in the diaphysis
- Chondrosarcoma is the answer because it is the calcified
lesion shown in the image and femur is the most common
site
6. A 30-year-old patient has a history of trauma and pain in right hip with flexion,
abduction and external rotation, what is the most likely diagnosis?
A. Hip posterior dislocation
B. Hip anterior dislocation
C. Fracture neck femur
D. Intertrochanteric fracture
Answer: B
Explanation: we conclude Hip anterior dislocation because
o Hip anterior dislocation: From FABER Flexion Abduction
External Rotation is seen
o Hip posterior dislocation: From FADIR Flexion Adduction
Internal Rotation is seen
o Fracture of neck of femur: Intracapsular and will have shortening
and external rotation
o Intertrochanteric fracture: extracapsular and will have more
shortening and more External rotation
7. Patient has multiple fractures along with blue sclera, what is the most likely
diagnosis?
A. Osteogenesis Imperfecta
B. Rickets
C. Scurvy
D. Paget’s disease
Answer: A
Explanation: Osteogenesis Imperfecta
Osteogenesis imperfecta (OI) is an inherited (genetic) bone disorder that is
present at birth. It is also known as brittle bone disease. A child born with OI
may have soft bones that break (fracture) easily, bones that are not formed
normally, and other problems. Blue sclera is also a feature of Osteogenesis
Imperfecta. It is the commonest cause of multiple fractures at birth.
8. 60-year-old female patient complaints of pain in knee bilaterally, the pain
increases while going up and down the stairs. The x-ray shows Bilateral
reduced joint space, what is the most likely diagnosis?
A. Osteoarthritis
B. Rheumatoid Arthritis
C. Ankylosing spondylitis
D. Hemophiliac arthropathy
Answer: A
Explanation: Osteoarthritis
Osteoarthritis is the most common form of arthritis, affecting millions of
people worldwide. It occurs when the protective cartilage that cushions the
ends of the bones wears down over time. Although osteoarthritis can
damage any joint, the disorder most commonly affects joints in your
hands, knees, hips and spine.
A. Winging of scapula
B. Erb’s palsy
C. Klippel Feil Syndrome
D. Fracture Scapula
Answer: A
Explanation: the image clearly shows winging of scapula, The term 'winged
scapula' is used when the muscles of the scapula are too weak or paralyzed,
resulting in a limited ability to stabilize the scapula. As a result, the medial or
lateral borders of the scapula protrudes from back, like wings.
10. A child with widening of bone has the following x-ray, what is the aetiology?
A. Scurvy
B. Rickets
C. Gout
D. Salter Harris type 2 Injury
Answer: B
Explanation: Rickets will have a wide skeleton, the bones have mineralization
defect and cupping and splaying of the bone can be clearly seen and the
margins point out called as flaring/fraying, classical characters of Rickets
11. Which of the following nerve is related to medial epicondyle?
A. Median nerve
B. Ulnar nerve
C. AIN
D. PIN
Answer: B
Explanation: Ulnar nerve
o In upper limb around upper part of humerus= axillary nerve
o Around the shaft =Radial nerve
o Behind the medial epicondyle=Ulnar nerve
o And head of radius=Posterior interosseous nerve
A. Scoliosis
B. Disc prolapses
C. Klippel Feil Syndrome
D. Torticollis
Answer: C
Explanation: short neck, lower hairline, short and high scapula which is a
classical feature of Klippel Feil Syndrome
SURGERY
1. In the hospital, a patient presents with the following swelling on standing
which tends to reduce when he lies down. What could be the probable
diagnosis?
A. Para-umbilical Hernia
B. Umbilical Hernia
C. Epigastric Hernia
D. Omphalocele
Answer: A
Explanation:
2. A 5-year-old child was brought to the clinic with the swelling shown below
which keeps on increasing in size. What should be the appropriate
management for this condition?
A. Intralesional steroids
B. Excision
C. Reassurance
D. Conservative
Answer: B
Explanation: A dermoid cyst is a collection of tissue under the skin. It may
contain hair follicles, oil, and sweat glands. In some cases, it may contain
bone, teeth, or nerves. A dermoid cyst may appear at birth or soon after.
Dermoid cysts are often found on the head, neck, or face, most often around
the eyes.
Surgical Excision is the only mode of treatment for dermoid cyst.
3. A 45-year-old female presents with Rapidly enlarging lump in her breast. Firm
and mobile on examination with Bosselated surface. Lymph nodes are not
involved. What could be the diagnosis?
A. Fibroadenoma
B. Phyllodes tumor
C. Ductal carcinoma
D. Ductal papilloma
Answer: B
Explanation: Phyllodes tumor
A phyllodes tumor is a rare tumor of the breast. Phyllodes tumors grow
in the connective tissue of the breast, called the stroma. This includes
the tissue and ligaments that surround the ducts, blood vessels, and
lymph vessels in the breast.
Most phyllodes tumors are benign. They may look very much like
common benign breast tumors called fibroadenomas. Often, the
pathologist needs to look at the whole tumor under the microscope to
make a diagnosis. This is why surgery to remove a phyllodes tumor is
recommended, even if it is thought to be benign.
4. An Incision was made over the patient’s chest for a surgery and after
sometime, he comes with the scar shown in the
image. What is the diagnosis?
A. Keloid
B. Hypertrophic scar
C. Contracture
D. Abscess
Answer: A
Explanation: Keloid scar
A. Intubation
B. Tracheostomy
C. Thoracotomy
D. Radical neck dissection
Answer: B
Explanation: Tracheostomy
6. A bed ridden patient developed fever, pain and pedal edema, what could be
the diagnosis?
A. Cellulitis
B. DVT
C. DIC
D. Pressure ulcer
Answer: B
Explanation: DVT only can cause low grade fever, remaining all in the options
can only cause high grade fever.
7. A Patient presented with long standing pain in the anal region. On
examination the finding shown is given below. What is the most important
thing to look for on per rectal examination of this patient?
A. Phallen’s sign
B. Tinel’s sign
C. Prayer sign
D. Schamroth sign
Answer: C
Explanation: A positive prayer sign can be is when on examination the patient
is unable to approximate the palmar surfaces of the phalangeal joints while
pressing their hands together
Seen in diabetes
10. A woman was presented with the following lesion in the image through which
purulent discharge was coming out. The most probable diagnosis?
A. Hydradenitis Suppurativa
B. Tuberculous lymphadenitis
C. Branchial cyst
D. Bezold abscess
Answer: B
Explanation: Tubercular Lymphadenitis
Tuberculous lymphadenitis is a chronic, specific granulomatous
inflammation of the lymph node with caseation necrosis, caused by
infection with Mycobacterium tuberculosis or related bacteria.
11. A man came to the hospital presented with vomiting, nausea and fever. He
was admitted for appendectomy after diagnosing appendicitis. During surgery,
surgeons found an incidental finding as shown in the image below which was
present at a distance of 2 feet from ileocaecal junction. What is the most likely
diagnosis?
A. Intussusception
B. Intestinal obstruction
C. Meckel’s diverticulum
D. Diverticulitis
Answer: C
Explanation: Meckel’s diverticulum
Most common congenital anomaly of small bowel
It is remnant of Vitellointestinal duct
Connects between midgut and yolk sac
Normally disappears in 7th week of intrauterine life
Present in 2% of population
Lies in ileum 2 feet from ileocaecal junction
Measures 2 inches in length
Has a separate blood supply
May contain ectopic gastric, pancreatic or colonic mucosa
A. Colostomy
B. Loop ileostomy
C. Ascending colostomy
D. Appendectomy
Answer: B
Explanation: loop Ileostomy
To form a loop ileostomy, a loop of small intestine is pulled out through
a cut in your abdomen. This section of intestine is then opened up and
stitched to the skin to form a stoma. The colon and rectum are left in
place.
13. A cricket player was injured with the ball and suddenly lost consciousness.
When one of the teammate comes for helping, he gets up by himself and
starts playing. During tea break, the person was again unconscious while his
friend saw him, immediately he has been rushed to the hospital. What is the
most probable diagnosis?
A. EDH
B. SDH
C. SAH
D. Intracerebral haemorrhage
Answer: A
Explanation: EDH
The player was in Lucid interval and it is a classical feature of EDH
CT-scan will show Biconvex shaped hematoma
A. Cricoid webs
B. Pharyngeal pouch
C. Achalasia Cardia
D. Diffuse esophagal spasm
Answer: B
Explanation: used in pharyngeal pouch or Zenker’s Diverticulum
15. A young male a chronic smoker, after developing the following condition of
finger tips as seen in the Image coms to the clinic. What could be the
condition?
Answer: B
Explanation: Buerger’s disease
Buerger's disease is a rare disease of the arteries and veins in the
arms and legs. In Buerger's disease — also called thromboangiitis
obliterans — blood vessels become inflamed, swell and can
become blocked with blood clots (thrombi)
This eventually damages or destroys skin tissues and may lead to
infection and gangrene. Buerger's disease usually first shows in
your hands and feet and may eventually affect larger areas of your
arms and legs.
The exact cause of Buerger's disease is unknown. While tobacco
use clearly plays a role in the development of Buerger's disease, it's
not clear how it does so. It's thought that chemicals in tobacco may
irritate the lining of your blood vessels, causing them to swell.
16. A patient presented with the following condition as shown below. what could
be the diagnosis?
A. Arterial ulcer
B. Venous ulcer
C. Neuropathic ulcer
D. Pressure ulcer
Answer: B
Explanation: venous ulcer
Are seen most commonly seen in medial malleolus (Gaiter area)
They are usually sloping ulcers and floor is covered by granulation
tissue
If it bleeds, we ask the patient to lift the limb and we do not compress
17. Identify the Anomaly shown in the picture?
A. Pyloric stenosis
B. Pancreatic Divisum
C. Annular Pancreas
D. Duodenal Atresia
Answer: C
18. A patient has an USG finding of a gall stone abutting the cystic duct with
dilatation of Common hepatic duct. What is the most likely diagnosis?
A. Acute cholecystitis
B. Porcelain GB
C. GB polyp
D. Mirizzi Syndrome
Answer: D
Explanation: Mirrizi Syndrome
Mirizzi syndrome is defined as common hepatic duct obstruction
caused by extrinsic compression from an impacted stone in the cystic
duct or infundibulum of the gallbladder
19. Which of the following fluid is avoided in Intracranial Head Injury?
A. NS
B. RL
C. 5% dextrose
D. Hypertonic solution
Answer: C
Explanation: The ideal fluid is Mannitol, even any hypertonic solutions can be
used.
But we cannot use any hypotonic solution or glucose containing solutions
because they will cause cerebral edema causing an increase in intra-cranial
pressure.
20. A male came to the OPD with an abdominal lump in periumbilical region that
moves at right angle to attachment of mesentry. What is your diagnosis?
A. Chylolymphatic cyst
B. Pancreatic Pseudocysts
C. Retroperitoneal Sarcoma
D. Peritoneal cyst
Answer: A
Explanation: Chylolymphatic cyst
A chylolymphatic cyst is a rare variant of a mesenteric cyst [1,2]. These
cysts present within the mesentery, lined with a thin endothelium or
mesothelium and filled with chylous and
lymphatic fluid.
21. A new born is found to have herniation of bowel and liver through the
Umbilicus, which is covered with a membrane. What is the diagnosis?
A. Gastrochisis
B. Omphalocele
C. Epigastric Hernia
D. Ectopia Vesica
Answer: B
Explanation:
PEDIA
1. A 5-year-old child of 17 kg weight with diarrhea and unable to take the feed.
What is the amount of maintenance fluid required in 24 hours?
(a) 1000 ml
(b) 1700 ml
(c) 1350 ml
(d) 2000 ml
2. A mother with the new born is worried for her child as the newborn hasn’t
passed the meconium till 6 hours of birth. How much time should she wait for
the meconium to pass out?
(a) 48 hours
(b) 24 hours
(c) 12 hours
(d) 72 hours
24 hours – urine
48 hours – meconium
Hemophilia is a disease that prevents blood from clotting properly. A clot helps stop
bleeding after a cut or injury. In factor VIII deficiency (hemophilia A), the body
doesn't make enough factor VIII (factor 8), one of the substances the body needs
to form a clot.
Hemophilia:
X linked Recessive
Causes severe forms of bleeding
M/C joint affected: ANKLE
Characteristic lab feature: increased APTT
MEDICINE
1. A 45-year-old female having arthralgia, swallowing difficulty with dry food and
gritty feeling under eyelid. She is suffering from NCNC(Normocytic normochromic
anemia) with Increased ESR, positive for anti-ANA, Anti-Ro and Rheumatoid
factor. Which of the following could be the diagnosis?
Annular pancreas develops due to failure of the ventral bud to rotate with duodenum,
causing encasement of duodenum.
(a) HCC
(b) Hepatitis
(c) Acute pancreatitis
(d) Gall bladder stones
I: Idiopathic
G: Gallstones
E: Ethanol (alcohol)
T: Trauma
S: Steroids
M: Mumps (and other infections) / malignancy
A: Autoimmune
S: Scorpion stings/Spider bites
H: Hyperlipidemia/Hypercalcemia
E: ERCP
D: Drugs
5. A patient with ptosis, anhidrosis and slight drooping of eyelid is presented to the
clinic. What do you think the patient is suffering from?
a. Horner syndrome
b. Keratoconjunctivitis
c. Dacryocystitis
d. Orbital cellulitis
(a) 5% dextrose
(b) Normal saline
(c) Hypertonic saline
(d) Mannitol
Cushing's syndrome is a disorder that occurs when your body makes too much
of the hormone cortisol over a long period of time. Cortisol is sometimes called
the “stress hormone” because it helps your body respond to stress.
8. A boy presented with blue sclera who has a history of frequent fractures. What
could be the diagnosis?
(a) Scurvy
(b) Rickets
(c) Osteogenesis Imperfecta
(d) Osteomalacia
a. Nail pterygium
b. Clubbing
c. Koilonychia
d. None of the above
Ans. Clubbing
Causes of Nail Clubbing - Mnemonic CLUBBING
Cyanotic Heart Disease, Cystic Fibrosis
Lung Cancer, Lung abscess
Ulcerative Colitis
Bronchiectasis
Benign Mesothelioma
Infective Endocarditis, Idiopathic Pulmonary
Neurogenic Tumors
Gastrointestinal Disease
12. A 68-year-old Diabetic patient develops which of the following macro vascular
complications after 20 years?
(a) Retinopathy
(b) Neuropathy
(c) Nephropathy
(d) Myocardial Infarction
Ans. (d) Myocardial Infarction
13. Tracheal shift towards right with left sided pneumothorax. What is the
management?
When tracheal deviation is present, the trachea will be displaced in the direction of
less pressure. Meaning, that if one side of the chest cavity has an increase in
pressure (such as in the case of a pneumothorax) the trachea will shift towards the
opposing side.
Thoracostomy is a minimally invasive procedure in which a thin plastic tube is
inserted into the pleural space — the area between the chest wall and lungs — and
may be attached to a suction device to remove excess fluid or air. A chest tube may
also be used to deliver medications into the pleural space.
14. A HIV patient presents with following presentation. What is the staging of HIV?
(a) I
(b) II
(c) III
(d) IV
15. JVP Image shows deep x descent and a rapid y descent and elevated over all.
What could be the diagnosis?
(a) Constrictive pericarditis
(b) Dilated cardiomyopathy
(c) HOCM
(d) Tamponade
16. A student who was eating the food of his hostel mess found to have developed
hepatomegaly, jaundice. On the same time all other students who were eating
from the same mess developed similar symptoms. Which investigation is more
appropriate to be done to find out the cause?
Hepatomegaly is an enlarged liver, which means it's swollen beyond its usual size. A
swollen liver usually is a symptom of another health condition, such as hepatitis.
Depending on the cause of your enlarged liver, you may notice symptoms like:
(a) E coli
(b) Listeria
(c) Neisseria meningitides
(d) Cryptococcus
18. A patient who is normal at rest but has discomfort on activities. Normal activities
are slightly affected. The patient comes under which category of NYHA
classification?
(a) NYHA I
(b) NYHA II
(c) NYHA III
(d) NYHA IV
Ans. (b) NYHA II
19. A patient comes to emergency with fever and headache. O/E he has neck
stiffness, CSF analysis was done and opening pressure increased, proteins mild
increased, glucose normal, lymphocytes increased. What is the most likely
diagnosis?
(a) Hyperkalemia
(b) Hypokalemia
(c) Hypernatremia
(d) Hyponatremia
21. Patient with previous history of MI 3 years ago now presenting with tachypnea,
tachycardia, S3 heard. What could be the diagnosis?
(a) MI
(b) Pericarditis
(c) Congestive heart failure
(d) Pulmonary embolism
Results from increased atrial pressure leading to increased flow rates, as seen
in congestive heart failure, which is the most common cause of a S3.
(a) Hypercholesterolemia
(b) Hypertriglyceridemia
(c) Hyper alpha lipoproteinemia
(d) Hyper beta lipoproteinemia
23. A 12-year-old child having proteinuria 4+ and edema and features associating
with nephrotic syndrome without hematuria. Which drug would you prefer?
(a) Prednisolone
(b) Cyclosporine
(c) Cyclophosphamide
(d) Methotrexate
Nephrotic syndrome is a condition where the kidneys leak protein from the blood into
the urine. When untreated, children can suffer from serious infections. In most
children with nephrotic syndrome, this protein leak resolves with corticosteroid drugs
(prednisone, prednisolone) reducing the risk of serious infection
24. A 65-year-old patient presents with dementia and loss of cognition. Which is the
most common cause for this condition?
(a) Alzheimer’s
(b) Pick’s
(c) Parkinson’s
(d) CJD
25. A patient is allergic to wheat and unable to eat it. O/E, he was tested positive with
Anti-Endomysial Antibodies. What could be the diagnosis?
IgA anti-endomysial antibodies are found in 90% coeliac patients. They are a very
specific & sensitive marker for coeliac disease and dermatitis herpetiformis. The
anti-endomysial antibody test has been suggested to be 98% sensitive and 98%
specific for clinical or subclinical coeliac disease.
Celiac disease, also known as celiac sprue or gluten-sensitive enteropathy, is a
chronic disorder of the digestive tract that results in an inability to tolerate gliadin,
the alcohol-soluble fraction of gluten. Gluten is a protein commonly found in wheat,
rye, and barley.
26. A 75-year-old female has a problem in walking long distances. O/E bruit heard in
knee. On X-ray imaging, reduced joint space and osteophytes were seen. Most
probable diagnosis is?
(a) Osteoarthritis
(b) Rheumatoid arthritis
(c) Psoriatic arthritis
(d) Gouty arthritis
Osteoarthritis symptoms often develop slowly and worsen over time. Signs and
symptoms of osteoarthritis include:
a) Prayer sign
b) Phalen sign
c) Tinel Sign
d) Schamroth sign
a) Aortic insufficiency
b) Mitral insufficiency
c) Pulmonary insufficiency
d) Tricuspid insufficiency
Lateral and/or inferior displacement of the apex beat usually indicates enlargement
of the heart, called cardiomegaly.
a) Apraxia
b) Agraphia
c) Hemianopia
d) Hemi-neglect
Ans. Apraxia
The clock-drawing test is used for screening for cognitive impairment and
dementia and as a measure of spatial dysfunction and neglect. It was originally used
to assess visuo-constructive abilities but we know that abnormal clock drawing
occurs in other cognitive impairments.
30. A 30-year-old patient presents with acute abdomen. X Ray Abdomen was done in
standing position. Which of the following is correct about management of this
condition?
Exploratory laparotomy has been the main stay of the treatment in patients
with acute abdomen. Laparoscopic management has previously been considered
as a relative contraindication for acute small bowel obstruction and peritonitis.
32. Comment on the diagnosis based on X Ray abdomen image shown here
33. A 60-year-old with restricted ambulation and prolonged bed rest due to acute
lumbago presented with severe pain in left leg below the knee. On examination
discoloration of left calf with significantly increased girth is noted. What is the
clinical diagnosis?
a) Acute lymphangitis
b) Superficial migratory thrombophlebitis
c) Deep vein thrombosis
d) Milroy disease
Deep vein thrombosis (DVT) occurs when a blood clot (thrombus) forms in one or
more of the deep veins in your body, usually in your legs. Deep vein thrombosis can
cause leg pain or swelling but also can occur with no symptoms.
You can get DVT if you have certain medical conditions that affect how your blood
clots. A blood clot in your legs can also happen if you don't move for a long time,
such as after you have surgery or an accident, when you're traveling a long distance,
or when you're on bed rest.
34. A cricket player was hit on the head with the cricket ball and was fine and then
collapsed suddenly. He woke up to play the match brilliantly but was later found
unconscious in the dressing room.Which of the following is correct about this
presentation?
a) Extradural hemorrhage
b) Subdural hemorrhage
c) Cortical venous thrombosis
d) Sub arachnoid hemorrhage
35. Boy baby is brought at 3 weeks of age with recurrent episodes of non-bilious
vomiting. Diagnosis is?
a) Duodenal atresia
b) Divarication of recti
c) CHPS
d) Peri-umblical hernia
Ans. CHPS
Clinical presentation
Diagnosis
The diagnosis is easily made if the presenting clinical features are typical, with
projectile vomiting, visible peristalsis, and a palpable pyloric tumor. An enlarged
pylorus, described as an "olive," can be palpated in the right upper quadrant or
epigastrium of the abdomen.
Ultrasound is the modality of choice, and Muscle wall thickness 3 mm or greater and
pyloric channel length 14 mm or greater are considered abnormal in infants younger
than 30 days.
Barium meal Upper GI study reveals delayed gastric emptying, peristaltic waves
(caterpillar sign), elongated pylorus with a narrow lumen (string sign). The entrance
to the pylorus looks like beak (beak sign)
36. An 8-year-old child presents with a localized painless swelling slowly growing in
size since childhood. Which is best treatment for this condition?
a) Excision
b) Chemotherapy
c) Intralesional steroids
d) Radiotherapy
Ans.Excision
Palpable mass
Ptosis
Proptosis
Strabismus
Pulsating proptosis with mastication
Globe displacement
Restriction in extraocular movements
Inflammation
Orbitocutaneous Fistula
Small, asymptomatic cysts may not require treatment. They may stabilize or even
decrease in size over years. However, some surgeons opt for early excision to avoid
the risk of traumatic rupture in the future
37. A young Guy comes to your OPD with the complaint that he develops pain in
penis during erection and cannot have sex. Which is the likely diagnosis?
a) Paraphimosis
b) Phimosis
c) Hypospadias
d) Peyronie disease
PHARMACOLOGY
SERM -DRUGS
Tamoxifen
Tamoxifen analogs
Toremifine, Droloxifene, Idoxifene
Fixed ring compounds :
Raloxifene, Lasofoxifene, Aroxif, Miproxifene, Levormeloxifene, EM652
Rituximab is the first MAb approved for cancer therapy. Clinical trials indicate that
rituximab is efficacious and safe for recurrent or chemotherapy-resistant, B-cell, low-
grade NHL.
(a) Valacyclovir
(b) Imipenem
(c) Carbapenem
(d) Penicillin
Valacyclovir is used to treat herpes virus infections, including herpes labialis (also
known as cold sores), herpes zoster (also known as shingles), and herpes simplex
(also known as genital herpes) in adults. It is also used to treat chickenpox and cold
sores in children.
(a) Rifampicin
(b) Erythromycin
(c) Valproate
(d) Phenobarbitone
(a) Vitamin K
(b) Protamine sulphate
Despite of the low therapeutic index, protamine is the only registered antidote of
heparins. The toxicology of protamine depends on a complex interaction of the high
molecular weight, a cationic peptide with the surfaces of the vasculature and blood
cells.
(a) Tamsulosin
(b) Phenoxybenzamine
(c) Terazosin
(d) Timolol
(a) Methicillin
(b) Cefotitan
(c) Imipenam
(d) Aztreonam
8. A new drug is introduced in the market after which phase of clinical trial?
(a) I
(b) II
(c) III
(d) IV
New Drug Application (NDA) is done following the Phase III Clinical Trials, the
drug manufacturer analyzes all the data from the studies and files an NDA with the
FDA (provided the data appear to demonstrate the safety and effectiveness of the
drug). The NDA contains all of the data gathered to date about the drug.
(a) Aspirin
(b) Dexamethasone
(c) MgSO4
(d) NSAID
10. All of the following conditions get worsened after timolol maleate 0.5%
administration except?
It works by blocking the action of certain natural chemicals in your body such as
epinephrine on the heart and blood vessels. This effect lowers the heart
rate, blood pressure, and strain on the heart. This medication is also used to
prevent migraine headaches. So timolol doesn’t worsen hypertension but used
to lower it .
11. Which anti-tussive should not be prescribed for the patients who are
driving?
(a) Codeine
(b) Ambroxol
(c) Diphenhydramine
(d) Hydrocodone
PHYSIOLOGY
(a) A
(b) B
(c) C
(d) D
Ans. (c) C
MICROBIOLOGY
Enterobius vermicularis, also called pinworm, is one of the most common nematode
infections in the world. Humans are the only natural host for this infection. Enterobius
vermicularis is an organism that primarily lives in the ileum and cecum.
Once E. vermicularis eggs are ingested, they take about 1 to 2 months to develop
into adult worms which happens in the small intestine. These do not usually cause
any symptoms when confined to the ileocecal area. The female adult worms and ova
migrate to the anal area mostly at night time and deposit thousands of eggs in the
perianal area. This migration causes pruritus. Eggs hatch near the anal area causing
perianal pruritus.
2. A patient with history of travel to Ohio shows lung symptoms. What could
be the most common cause?
(a) Blastomycosis
(b) Histoplasmosis
(c) Mucormycosis
(d) Coccidiodomycosis
(a) Shigella
(b) Cholera
(c) E.coli
(d) Salmonella
Ans. (b)Cholera
(a) Gluteraldehyde
(b) Alcohol
(c) Lysol
(d) Phenol
(a) Chlamydia
(b) Histoplasma
(c) Cryptococcus
(d) Aspergillus
India ink capsule stain is used to demonstrate cell capsules through microscopic
examination. This procedure is used to detect presence of encapsualted species,
e.g. Cryptococcus neoformans, which causes cryptococcosis in humans.
6. A pregnant woman with malaria presents in front of the doctor. What could
be the causative organism?
(a) P. vivax
(b)P. falciparum
(c) P. ovale
(d) P. malariae
(a) CJD
(b) Rabies
(c) Parkinson’s disease
(d) Alzheimer’s disease
(a) E. coli
(b) Entamoeba histolytica
(c) Shigella
(d) Giardia lamblia
10. A post covid-19 patient started to develop nasal congestion, black crusts in
nose, and blackish discoloration of skin on nasal and cheek area. Which of
the following micro-organisms is the causative agent ?
(a) Aspergillus
(b) Fusarium
(c) Mucormycosis
(d) Pencillium
ENT
1. An infant presented with inspiratory stridor and the clinical picture shows
omega shaped epiglottis. The probable diagnosis is?
(a) Rhinophyma
(b) Rhinosporidiosis
(c) Rhinoscleroma
(d) Rhinosinusitis
(a) FESS
(b) Young’s operation
(c) Nasal steroid spray
(d) Immunotherapy
Young's operation is a surgery designed for the treatment of atrophic rhinitis, first
described by Austen Young in 1967. Young's operation is a surgery designed for
the treatment of atrophic rhinitis, first described by Austen Young in 1967.
(a) HSV
(b) EBV
(c) HZV
(d) Parainfluenza virus
The nasopharynx is located at the very back of the nose near the Eustachian tubes.
Nasopharyngeal carcinoma is more common in Southeast Asia and is frequently, but
not always, caused by Epstein-Barr virus (EBV).
7. Identify the procedure shown below?
The Cottle's maneuver (CM) is a test in which the cheek on the side to be
evaluated is gently pulled laterally with one to two fingers to open the valve.
This test is used to determine if the most significant site of nasal obstruction is at the
valve or farther inside the nasal cavity.
a) Measles virus
b) Parainfluenza virus
c)Respiratory syncytial virus
d) Corynebacterium diphtheriae
A “halo” or “ring” sign, occurs when cerebrospinal fluid (CSF) mixes with blood on
an absorbent surface. The blood forms a spot in the center and a lightly stained
ring forms a halo around it.
(a) PET
(b) CT
(c) Angiography
(d) X-ray
Ans. (b) CT
11. A child presented with a high arched palate and mouth breathing and
failure to thrive. The probable diagnosis is?
(a) Tonsil hypertrophy
(b) Adenoid hypertrophy
(c) Lingual tonsil hypertrophy
(d) Turbinate hypertrophy
12. A patient presented with pulsation of the tonsil. The cause is?
RADIOLOGY
(a) 0.1
(b) 0.2
(c) 0.5
(d) 0.01
Lead aprons are the primary radiation protective garments used by personnel during
fluoroscopy. The radiation protection provided by a lead apron is approximately the
same as 0.25- to 1-mm thick lead. An apron with 0.5-mm thickness can attenuate
approximately 90% or more of the scatter radiation.
2. A female visits OPD with her 6-week child presenting with vomiting and
abdominal pain. X-ray of the infant shows?
3. A two-weeks child brought to the emergency room by her mother with the
symptoms of non-bilious vomiting and lump in the epigastric area. What is
the most appropriate diagnosis?
Pyloric stenosis should be suspected in any young infant with severe vomiting.
Infants with pyloric stenosis classically present with projectile, non bilious vomiting.
Vomiting may be intermittent or occur after each feedingOn physical exam, palpation
of the abdomen may reveal a mass in the epigastrium. This mass, which consists
of the enlarged pylorus, is referred to as the 'olive’.
4. Patient comes to emergency room with vomiting and colicky abdomen
pain. On investigation following Image is obtained. Likely diagnosis is?
Horseshoe kidney is a condition in which the kidneys fuse (bind) together at the
bottom, forming a “U” shape or horseshoe shape. Children who have horseshoe
kidney have one “fused” kidney instead of 2 separate kidneys.
5. A patient presents with hard palpable gall bladder with pain in right
hypochondrium. On performing USG following Image was seen. What could
be the diagnosis?
Gallbladder with multiple gallstones. Arrows indicate stones with associated posterior
acoustic shadowing
a) Popcorn calcification
b) Egg shell calcification
c) Ground Glass Appearance
d) Miliary opacities
Perthes disease is a rare childhood condition that affects the hip. It occurs when
the blood supply to the rounded head of the femur (thighbone) is temporarily
disrupted. Without an adequate blood supply, the bone cells die, a process called
avascular necrosis.
8. A 6-day-old neonate with respiratory distress showed the following findings
on the chest X-ray. What is the diagnosis?
Pericardial effusion is the buildup of extra fluid in the space around the heart. If
too much fluid builds up, it can put pressure on the heart. This can prevent it from
pumping normally. A fibrous sac called the pericardium surrounds the heart. This sac
consists of two thin layers.
The Holman-Miller sign (also called the antral sign) is seen in juvenile
nasopharyngeal angiofibroma; it refers to the anterior bowing of the posterior
wall of the maxillary antrum as seen on a lateral skull radiograph or cross-
sectional imaging
11. Identify the diagnosis on Imaging as shown below?
Radiographic features
Ultrasound
enlarged uterus
may be seen as an intrauterine mass with cystic spaces without any associated
fetal parts,the multiple cystic structures classically give a "snowstorm" or
"bunch of grapes" type appearance.
may be difficult to diagnose in the first trimester
o may appear similar to a normal pregnancy or as an empty gestational sac
o <50% are diagnosed in the first trimester
bilateral theca lutein cysts may also be seen on ultrasound
color Doppler interrogation may show high velocity with a low impedance flow
12. A cattle handler from a village with cough, hemoptysis and chest pain
underwent a chest radiograph and is shown below. What is the probable
diagnosis?
(a) Hydatid cyst
(b) Anthrax
(c) Byssinosis
(d) Abscess/Aspergillosis
OPHTHALMOLOGY
1. A tumor in the anterior pituitary causing pressure over optic chiasma will
present as?
When there is compression at optic chiasm, the visual impulse from both nasal
retina are affected, leading to inability to view the tempora, or peripheral, vision.
This phenomenon is known as bitemporal hemianopsia.
2. Ophthalmic surgical Instrument shown in the Image is used for surgery of ?
(a) Entropion
(b) Ectropion
(c) Chalazion
(d) Bitot spot
A chalazion forceps is a self-retaining type of thumb forceps that has discoid ends.
One arm has a flattened plate at its end, which is round and solid. The other arm has
a matching ring that fits into the opposite arm’s solid plate. There is an adjustable
screw at the middle of the plates, which can be tightened until it has a firm and fine
grip to the cyst or lesion.
Keratan sulfate (KS) also plays a role in maintaining the transparency of the cornea
by regulating the organization of collagen fibrils and maintaining corneal
hydration. Alteration of corneal KS may lead to reduced hydration and corneal
opacification. KS is also implicated in corneal wound repair.
4. 5-year-old suffering with chronic epiphora has appeared in the clinic. What
is the next step of management?
Epiphora is an overflow of tears onto the face, other than caused by normal
crying. It is a clinical sign or condition that constitutes insufficient tear film
drainage from the eyes, in that tears will drain down the face rather than
through the nasolacrimal system.
Dacryocystorhinostomy (DCR) is a surgery that creates a new path for
tears to drain between your eyes and your nose. You may need this
surgery if your tear duct has become blocked.
(a) Glaucoma
(b) Peripheral vascular disease
(c) COPD
(d) Diabetes
Ans. Glaucoma
Glaucoma is a group of eye conditions that damage the optic nerve, the
health of which is vital for good vision. This damage is often caused by an
abnormally high pressure in your eye.
Beta Blockers decrease the pressure inside your eyes by reducing how much
fluid (aqueous humor) is produced in the eyes. Reducing pressure in the
eyes, slows down optic nerve damage which greatly decreases the rate of
vision loss.
6. Patient wearing contact lens since two years develops redness and
decreased vision. What could be the cause?
8. A patient presented with anterior uveitis in OPD. Which of the following can
be associated with his condition?
(a) HLA B5
(b) HLA B27
(c) HLA B7
(d) HLA DR4
Acute anterior uveitis (AAU) is the most common form of uveitis, accounting for
approximately 90% of all cases. Half of all cases of AAU are HLA-B27 positive.
The disease is typically acute in onset, unilateral, no granulomatous inflammation
involving the iris and ciliary body, with a tendency to recurrent attacks.
Pterygium :
Triangular overgrowth of subconjunctival tissue.
Wedge shaped, pink -white translucent membrane with apex extending into
cornea.
Stocker's line - iron line at leading edge
Risk factors - Exposure to UVrays, hot, dry weather, wind, dust
Symptoms
Irritation, lacrimation, FB sensation, decreased vision , difficulty in contact lens
wear
Treatment
Excision with conjunctival autograft
Amniotic membrane transplant with fibrin glue / sutures.
11. A 15-year-old boy presents to clinic with difficulty in coordination of
movements and abdominal pain. On further examination copper deposition
in a ring form is seen in cornea. Most likely diagnosis?
(a) Keratitis
(b) Munson sign
(c) Wilson’s disease
(d) Neuroblastoma
OBG
1) A 55-year-old female presents with complaints of post coital bleeding. She is mother of 6 children
and has a history of early marriage. On colposcopic examination, inflammed cervix is seen which
bleeds on touch. What is the most probable diagnosis?
Cervical Cancer is a malignant tumour of the lower-most part of the uterus (womb) that can be
prevented by PAP smear screening and a HPV vaccine.
Symptoms include bleeding in between periods and after sexual intercourse. Foul smelling white
discharge and low back pain or lower abdominal pain may also occur. In some cases there may be no
symptoms.
2) Identify the procedure being done in the Image?
Maneuver I: The uterine contour is outlined. The fundus is palpated with the fingertips of both hands
facing toward the maternal xiphoid cartilage. This should allow the identification of the fetal parts in
the upper pole (fundus) of the uterus.
Maneuver II: Once an assessment is made of the fetal part present in the uterine fundus, the hands
are placed at either side of the maternal abdomen. With this maneuver, the examiner will be able to
determine the location of the fetal back.
Maneuver III: Using one hand, the examiner will grasp the presenting part between the thumb and
fingers. This is done on the lower abdomen, a few centimeters above the symphysis pubis. This will
allow the examiner to develop a further identification of the presenting part and assessment of its
engagement.
Maneuver IV: This last maneuver resembles the first one, but instead of facing the fundus, the
examiner faces the pelvis of the patient. The palms of both hands are placed on either side of the
lower maternal abdomen, with the fingertips facing toward the pelvic inlet. This maneuver should
allow the identification of the fetal parts in the lower pole of the uterus.
3) Look at the Image and diagnose the condition?
a. Kleinfelter
b. Down’s
c. CAH
d. Turner’s
Turner syndrome is a genetic disorder affecting girls and women. The cause of Turner syndrome
is a completely or partially missing X chromosome. Turner syndrome symptoms include short stature
and lack of breast development and periods. Treatment for Turner syndrome may include hormone
therapy.
Broad chest.
Cubitus valgus, where the arms point out slightly at the elbows.
Dental problems.
Eye problems, such as a lazy eye or drooping eyelids.
Scoliosis, when the spine curves sideways.
Low hairline at the back of the neck.
Many skin moles.
Missing knuckle in a particular finger or toe, making the digit shorter.
Narrow fingernails and toenails.
Small lower jaw.
Swelling of the hands and feet.
Unusually short, wide neck or webbed neck (extra skin folds).
4) An 8-weeks pregnant lady with spotting since the start of her pregnancy. On USG, gestational sac
corresponds to gestational age. What is the probable diagnosis?
SECOND TRIMESTER
Medical
Prostaglandins PGE1 (Misoprostol), 15 methylPGF2a (Carboprost), PGE2 (Dinprostone)
and their analogues (used-intravaginally, intramuscularly or intra-amniotically)
Surgical
Dilation and evacuation
Intrauterine instillation of hyperosmotic solutions
- Intra-amniotic hypertonic urea (40%), saline (20%)
- Extra-amniotic—Ethacrydine lactate, Prostaglandins
- (PGE2, PGF2a)
Hysterotomy (abdominal)— less commonly done
Mifepristone + Misoprostol
200 mg of mifepristone is given orally on day 1 followed 2 days / 48 hrs later by
Vaginal misoprostol 800 mcg (4 tablets of 200mcg each) is placed in the posterior fornix of vagina
The combipack of 1 tablet mifepristone and 4 tablets misoprostol (i.E. 5 tablets) is approved by
DGHS, govt, of india.
6) 35-year-old female, working in a factory presents within 8 hours after a sexual assault by a male
colleague. She is at day 13 of her menstrual cycle. What should be the emergency contraceptive
of choice in her case?
Levonorgestrel 1.5mg tablets can be used at any time during the menstrual cycle unless menstrual
bleeding is overdue.
a. Hydrosalpinx
b. Long para ovarian cyst
c. PID
d. Bicornuate uterus
Ans: (a) Hydrosalpinx
Hydrosalpinx is the blockage of a woman's fallopian tube caused by a fluid buildup and dilation of
the tube at its end. Most often it occurs at the fimbrial end of the tube next to the ovary, but it can also
occur at the other end of the tube that attaches to the uterus.
8) A G2P1 presents with history of child with down’s syndrome. Now her POG is 10 weeks. What is
the best possible method of Investigation?
9) Which of the following strain of HPV is more commonly associated with cervical cancer?
(a) HPV 6
(b) HPV 11
(c) HPV 16
(d) HPV 8
Ans. (c)HPV 16
Two HPV types (16 and 18) cause 70% of cervical cancers and pre-cancerous cervical lesions. There
is also evidence linking HPV with cancers of the anus, vulva, vagina, penis and oropharynx.
10) 25 year old married man came to infertility clinic, having azoospermia. normal sized testis, FSH
and testosterone levels also normal. What could be the reason?
Obstructive azoospermia: This type of azoospermia means that there is a blockage or missing
connection in the epididymis, vas deferens, or elsewhere along your reproductive tract. You are
producing sperm but it's getting blocked from exit so there's no measurable amount of sperm in your
semen.
In some men, the epididymis becomes blocked, preventing sperm from entering the vas deferens
and getting into the ejaculate. A blockage can occur on one side or on both sides. If a man has an
epididymal obstruction on one side, he may suffer a lowered sperm count.
11) A female in the labor room - head has not reached to the ischial spines with 3 cm cervical
dilatation in 3 hours will be considered at which stage of labor?
Latent phase
• It is the preparatory
phase of cervical
dilatation
• It is 8 hrs in primi and
4hrs in multi*
• The cervix dilates only
3 cms during the
latent phase*
12) Image showing a contraceptive device in uterus. The placement is best seen with:
(a) Laparoscopy
(b) Hysteroscopy
(c) Laparotomy
(d) Hysterectomy
Ans. (b) Hysteroscopy
A hysteroscopy is a procedure used to examine the inside of the womb (uterus). It's carried out
using a hysteroscope, which is a narrow telescope with a light and camera at the end. Images are
sent to a monitor so your doctor or specialist nurse can see inside your womb.
The hysteroscope is inserted into the vagina, through the cervix, and into the uterus, allowing the
surgeon to view, take a biopsy or to use special instruments to treat the underlying condition.
Hysteroscopy may also be used to remove adhesions, polyps or a malpositioned
IUD (intrauterine device).
13) A woman with secondary amenorrhea with history of curettage for abortion. FSH 7 IU/ml. What is
your likely diagnosis?
Uterine Synechiae are usually a consequence of uterine trauma caused by: Infections. Prolonged
use of intrauterine devices (IUDs) Post-miscarriage or postpartum curettage.
Uterine synechiae are associated with increased risk of preterm premature rupture of membranes,
placental abruption, and malpresentation. Uterine synechiae are intrauterine adhesions around
which chorioamniotic membranes wrap during pregnancy to produce amniotic sheets or shelves.
(a) Raloxifene
(b) Danazole
(c) Gosarelin
(d) tamoxifene
Endometriosis is an often painful disorder in which tissue similar to the tissue that normally lines the
inside of your uterus — the endometrium — grows outside your uterus. Endometriosis most
commonly involves your ovaries, fallopian tubes and the tissue lining your pelvis.
TREATMENT
• Treat all patients - regardless of clinical profile – even asymptomatic pts – progressive disease
– asymptomatic →symptomatic (30-60% patients in 1 yr.
• Chronic disease with ↑recurrence rate (even with hormonal and surgical treatment)
MEDICAL MANAGEMENT
• Aim - Relief from pain
o Pseudo pregnancy drugs - decidualization of endometrium – COC (Combined Oral Contraceptive),
Progesterone
Pseudomenopause drugs - atrophy of endometrium - Danazol, GnRH agonists
16) A patient presented with secondary amenorrhea after 5 years of her marriage. O/E she was found
to have retroverted uterus with some tenderness and swelling near the posterior wall of vagina.
What is the appropriate diagnosis?
A retroverted uterus (tilted uterus, tipped uterus) is a uterus that is oriented posteriorly, towards
the back of the body.
A retroverted uterus is due to a disease such as endometriosis
The other factors that can cause the uterus to become retroverted. Pelvic surgery, pelvic
adhesions, endometriosis, fibroids, pelvic inflammatory disease, or the labor of childbirth can change
the position of the uterus to retroverted.
17) Women with ovarian malignancy, what are all the procedures that can be done except?
Radiation therapy in ovarian cancers has been considered an outdated concept for many years,
mainly due to toxicity and failure to show benefit in terms of survival. Chemotherapy has been
extensively used after surgery for these cancers and it has almost replaced radiation therapy as an
adjuvant treatment.
18) A 14-year-old girl presented with primary amenorrhea, complains of urinary retention, pain in the
abdomen. On history talking, she explains that she has been suffering from cyclical abdominal
pain for last 6 months. O/E following image is seen. What is the diagnosis?
Imperforate hymen is when the hymen covers the whole opening of the vagina. Imperforate
hymen is the most common type of blockage of the vagina. Imperforate hymen is something a girl is
born with
19) A 30-year nulliparous lady being married for 5 years came to infertility center for treatment. She is
having secondary amenorrhea for last 45 days. Doctor had done test for her ovulation in which
she is ovulating regularly and normally. Under what parameters you can confirm that ovulation
has taken place?
In conception cycles, properly timed mid-luteal progesterone levels are over 10 ng/mL.
Progesterone is also often used to assess ovulation after induction of ovulation.
20) A 14-year-old girl presented with heavy bleeding per vaginum for last 10 days. What should be
done to know the cause other than pregnancy test?
Coagulation tests measure your blood's ability to clot, and how long it takes to clot. Testing can
help your doctor assess your risk of excessive bleeding or developing clots (thrombosis) somewhere
in your blood vessels.
21) A woman who is lactating came to OPD 6 months after delivery for checkup and to get advice for
contraception. She still wants to continue breast feeding for her child. Which is not to be adviced?
Contraceptives which contain estrogen have been linked to reduced milk supply and early
cessation of breastfeeding even when started after milk supply is well established and baby is
older.
22) A 32-year-old female came for regular check up found with ovarian cyst of 6*6 cm. What’s the
next line of management?
Ovarian cysts are growths that develop on or inside the ovaries. There are several types of cysts.
Functional cysts are common, not related to a disease and form as a result of ovulation. Disease-
related cysts can include those that result from polycystic ovary syndrome and ovarian cancer.
Functional ovarian cysts generally go away without treatment. Your healthcare provider may give
you medications containing hormones (such as birth control pills) to stop ovulation and prevent future
cysts from forming. If you do not ovulate, you will not form functional cysts.
23) A primigravida at 24-weeks of gestation comes to the hospital with an appointment with a report
of RBS of 126mg/dl. Fasting and OGTT after 2 hours was found to be 200mg/dl. On performing
the USG the fetus was found to be 2.5kg in weight. What should be the best management for
her?
(a) Reassure
(b) Call her to check again at 34 weeks
(c) Admit, start insulin and check for response
(d) Start insulin and send her home
Gestational diabetes mellitus (GDM), defined as any degree of glucose intolerance with onset or
first recognition during pregnancy, is associated with adverse outcomes in mothers and offspring
Target glucose values in women with GDM are ≤ 95 mg per dL (5.3 mmol per L) with fasting, ≤ 140
mg per dL (7.8 mmol per L) one-hour postprandial, or ≤ 120 mg per dL (6.7 mmol per L) two-hour
postprandial.
GDM Pregnant women should be managed by Medical Nutrition Therapy (MNT), and insulin
therapy/ metformin as required and check for response. In the postpartum period, OGTT should be
repeated at 6 weeks after delivery, if blood sugar <140 mg/dL, then women should be referred to NCD
clinic for Post Prandial Blood Sugar (PPBS) testing annually.
(a) HTN
(b) PID
(c) Obesity
(d) Anemia
Boundaries
Sacral promontory,
alae of the sacrum,
sacroiliac joints,
iliopectineal lines,
iliopectineal eminencies,
upper border of the superior pubic rami,
pubic tubercles,
pubic crests and
upper border of symphysis pubis.
Diameters
Antero -posterior diameters:
o Anatomical antero-posterior diameter (true conjugate) = 11cm
from the tip of the sacral promontory to the upper border of the symphysis pubis.
27) Which among the following is the most dependent part in a sitting female?
The broad ligaments reflect over the uterus, fallopian tubes, and parametrial uterine vessels and
serve as the anterior boundary of the rectouterine pouch of Douglas. The cul-de-sac or the pouch of
douglas is the most dependent portion of the peritoneal cavity and collects fluid, blood, abscesses,
and intraperitoneal drop metastases.
a) Hydatiform mole
b) PCOS
c) Milliary TB
d) Ectopic pregnancy
Hydatidiform mole (HM) is a rare mass or growth that forms inside the womb (uterus) at the
beginning of a pregnancy. It is a type of gestational trophoblastic disease (GTD).
Snowstorm sign in obstetric imaging is classically seen in complete Hydatiform mole. It is
characterized by the presence of many hydropic villi which gives the ultrasonographic appearance of
a central heterogeneous mass having a solid, hyperechoic area and interspersed with a multitude of
cystic areas and filling the entire uterine cavity.
DERMATOLOGY
A. Auspitz sign
B. Dermographism
C. Koebner’s Phenomenon
D. Nikolsky’s sign
Answer: C
Explanation: The Koebner phenomenon describes the appearance of new skin lesions of a pre-
existing dermatosis on areas of cutaneous injury in otherwise healthy skin. It is also known as
the Köbner phenomenon and isomorphic response.
2. From the given image is shown which nail finding that is the characteristic feature of
Psoriasis Vulgaris?
A. Subungual keratosis
B. Auspitz sign
C. Oil drop sign
D. Onycholysis
Answer: A
Explanation: Subungual hyperkeratosis refers to the accumulation of scales under
the nail plate, which is detached and uplifted. The nail bed often appears thickened.
The condition results from excessive proliferation of keratinocytes and failure to shed
off from the stratum corneum.
The nail findings in Psoriasis can be remembered by a simple mnemonic POISON
-P= Pitting
-OI=Oil drop sign
-S=Subungual hyperkeratosis
-O=Onycholysis
-N=Nail plate dystrophy
A. Leishmania Cutanea
B. Lepromatous Leprosy
C. Post Kala Azar Dermatitis
D. Tuberculoid Leprosy
Answer: B
Explanation: There is saddle nose
deformity and loss of eyebrows (lateral
madarosis) and we can see multiple
lobular lesions over the face leading us to
the answer Lepromatous Leprosy.
4. Which of the following is best treatment for the following condition?
A. Valacyclovir
B. Imipenem
C. Carbapenem
D. Penicillin
Answer: A
Explanation: we can see from the image that the lesion has clear midline demarcation
and is vesicular lesions on erythematous base leading us to the diagnosis Herpes
zoster
It is a reinfection of varicella virus
Painful condition
Appears as a vesicular lesion on erythematous base on dermatomal
distribution
Treatment is Acyclovir, Valacyclovir
5. A patient who has anti-convulsant medication presented with rashes which is shown
in the following image. What
could be the diagnosis?
A. Urticaria
B. Fixed drug eruption
C. Angioedema
D. Cellulitis
Answer: B
Explanation: in the question there is a history of drug intake, and the patient
developed rashes after taking a drug leading us to a diagnosis of Fixed drug eruption.
In angioedema and urticaria we can see a much bigger edematous swelling
In fixed drug eruption, after taking drug a rash develops that heals with
hyperpigmentation
And after when the drug is retaken, the rash appears on the same site and this
will heal with hypopigmentation.
6. Identify the image given below?
A. Carbuncle
B. Follicular abscess
C. Cellulitis
D. Erysipelas
Answer: A
Explanation: a furuncle is infection of hair follicle, single pus forming infection of a
single hair follicle but whereas a carbuncle is also a pus forming infection of
contiguous hair follicles
7. A patient presented with hypopigmented rash with satellite lesions as shown in the
image given below?
A. BT
B. BL
C. LL
D. TT
Answer: A
Explanation: satellite lesions and finger like projections (Pseudopodia) are seen in BT
type of leprosy
-In BB, we can see Swiss Cheese appearance or inverted saucer appearance
-other dermatological disorders where we can see satellite lesions are
BT Leprosy
Sporotrichosis
Candidiasis
8. A person is with chronic alcoholism, which deficiency
causes the following as shown in the image?
A. Riboflavin
B. Ascorbic acid
C. Biotin
D. Niacin
Answer: D
Explanation: patient is having dermatitis, diarrhea for 2 months, the patient has
appearance of CASAL’s Necklace. So, we conclude that it is Pellagra.
Pellagra is characterized by 4D’s
o Diarrhea
o Dermatitis
o Dementia
o Death
Pellagra is caused due to deficiency of vitamin B3 or Niacin. Frequently seen in Maize/Jowar
eaters due to deficiency of Tryptophan which leads to pellagra due to deficiency of niacin.
Excess of leucine inhibits the conversion of tryptophan to niacin (conversion ratio of
tryptophan to niacin is 60:1) in the body. Here the main culprit is Leucine.
A. Verrucous vulgaris
B. Elephantiasis
C. Leprosy
D. Leishmaniasis
Answer: B
Explanation: we can see unilateral swelling in lower limb,
In elephantiasis, there will be a chronic lymphatic obstruction
Transmitted by culex
Once the microfilaria enters the body, they convert into adult worms and they
finally destruct the lymphatics causing unilateral, non-pitting kind of an edema
of the lower limbs
Skin changes: Hypertrophy, verrucous changes and pigmentary changes
10. A truck driver with history of lesions on penis undergoing treatment presented with
rashes. What is the diagnosis?
A. Secondary syphilis
B. Drug eruption
C. TEN
D. Steven Johnson syndrome
Answer: A
Explanation: these rashes are typical for secondary syphilis along with the history of
lesions in the penis is leading us to the diagnosis of Secondary syphilis.
In (Primary) 10 Syphilis – Hard Chancre, Painless indurated genital ulcer
If not treated properly patient will have (secondary) 20 Syphilis
JUNE 2021
1. Identify the muscle marked 'X' in the given image?
a) Styloglossus
b) Genioglossus
c) Hyoglossus
d) Palatoglossus
Ans. B
Muscles of Tongue
Intrensic M/s 4 pairs Extrinsic M/s 4 pairs
Superior Longitudinal M/s Genioglossus
2. Arrange a, b, c, d in sequence?
a) A - Cordate nucleus B - Thalamus C - Lentiform Nucleus D – External capsule
b) A-Cordate nucleus B-lentiform nucleus C-thalamus D-external capsule
c) A-Thalamus B-lentiform nucleus C-cordate nucleus D-external capsule
d) A- Thalamus B-cordate nucleus C-lentiform nucleus D-external capsule
Ans. A
Ans. A
Ans. D
Ans. C
Functions:
Motor Functions
Skilled Voluntary monents (coriting)
6. Sutures was done to the finger during the surgery procedure. after the patient complains that he
is un able to
flex the middle finger. this can be due to injury of which of the following?
a) Median nerve
b) Flexor pollicis longus
c) Flexer digitorum profundus
d) Flexor retinaculum
Ans. A
Also called : LABOURES NERVE
Root value: C5 C6 C7 C8 T1
Sign:pointing index Benedictis hand sign
Test for nerve: PEN TEST : Muscle- abductor pollicis brevis
7. A new born present with the congenital anomaly of cleft lip. Which is the embrynological
development deftct?
a) Non fusion of palatine shelves
b) Non fussion of medial nasal process &maxillary process
c) Non fussion of lateral nasal process & maxillary process
d) Separation of nasal septem&vomer from palatine process
Ans. B
8. A patient came to the physician with complains of chest pain which is radiating to the left arm ,
which nerve will
causes pain?
a) Musculocutaneous nerve
b) Intercostobrachial nerve
c) Medial cutaneous nerve of arm
d) Medial cutaneous nerve of forearm
Ans. B
. A patient of covid-19 requires oxygen therapy, which of the following cylinders should be used to
provide
oxygen to this patient?
a. b. c. d.
a) O2 [Black body with white shoulders] pressure 2000 psi (140 kg/cm2)
b) N2O [Blue body] 760 psi
c) Entonox [Blue body with white shoulder] 2000 psi
d) CO2 [Brown]
Ans. A
10. A copd patient in ICU was on oxygen therapy with mask with 2 liter of oxygen. He was found to
be Hypo-
ventilating. His PCO2 on blood gas analysis was 64mmHg. Which of the following should be done?
a) Increase the FIO2
b) Decrease the FIO2
c) Start BIPAP
d) Intubate and start on ventilator
Ans. C
Ventilator
Non-invasive ventilator :
1. Continuous positive airway Pressure (CPAP) (Alveoli always) open so that gas exchange takes place
continuously
even during expiration]
2. Bilevel – positive airway Pressure [BI – PAP] positive pressure during inspiration and expiration
Ans. C
12. Which of the following anesthetic agent is best for cardiac patient?
a) Halothane
b) Isoflurane it minimally CO
c) Nitrous oxide
d) Sevoflurate
Ans. B
Desflurane
4.Renal Propofol Cisatracurium /
Atracurium
sevoflurane
5.Neuro surgeries Thiopentone Any one can use
13. Which of the following structure is not encountered during epidural anesthesia?
a) Supraspinous ligament
b) Interspinous ligament
c) Ligament flavum
d) Dura
Ans. D
Ans. B
Resistance to flow R = 8nL/ Π r4 resistance length , radiusresistance
( L- length , r – radius )
15. 7 A patient was given halothane for general anaesthesia. He developed rigidity and
hyperthermia. These
symptoms are most likely due to which ion?
a) Na+
b) k+
c) ca2+
d) cl-
Ans. C
Malignant hyperthermia :
Malignant hyperthermia is a hypermetabolic muscle disorder due to genetic mutation in
the ryanodine receptor
Mutation is seen most commonly in chr19
Mutation causes uncontrolled release of ca+ from sarcoplasmic reticulum causing
intense muscle contractions and subsequent complications of hypermetabolism
16. Macrocytic anemia case with normal methyl-malonyl with increaselevel of homocysteine.
Diagnosis?
a) B9 deficiency
b) B12, 9 deficiency
c) Iron deficiency
d) Anemia of chronic disease
Ans. A
Vit B12 Vit B12 Homocystein methyle Megaloblastic only B12 – source : mineal pratus
(Cobalamine) transferavo anemia B12 daf seen in Vegetarian
Ilium – (termaind) Methu melonyle Co-A Peripheral
Vit B9 / (Folic mutain nspropathy Megablastic anemia
Acid) B12 B9
Methameture N
Folate acid methameture
DHF DHF Reductars acid
THF DNA & Megaloblastic
H2 RNA Anemia
H2 Neural tube defects
Jejurum
17. A newborn develops vomiting, irritability, and lethargy several days afterbirth. His mother
states that his
diapers smell like "burned sugar". Laboratory studies confirm the diagnosis, and the patient is
started on the
appropriate dietary restrictions with subsequent improvement in his symptoms. The defective
enzyme
responsible for this patient's condition
a) Branched chain keto acid dehydrogenase
b) Pyruvate dehydrogenase complex
c) Glutathione peroxidase
d) Glucose-6-phosphatase
Ans. A
18. A 28 year old male has undergone surgery where would debriment wasperformed and (wound
was not
healing which one of the following vitamindeficiency is related with delayed would healing?
a) Vitamin C
b) Vitamin B2
c) Vitamin A
d) Vitamin D
Ans. A
Ans. C
20. Which of the following protein doesn't have a quaternary structure?
a) Hemoglobin
b) Collagen
c) Insulin
d) Myoglobin
Ans. D
21. A 8 month old child presents to with hypoglycemia in between feeding. On examination there
was hepatomegaly
and liver was palpable1 cm below subcostal margin. The child also presents with lacticacidosis,
ketosis and
xanthomas on the buttocks. What's the most likely enzyme deficiency leading to this?
a) Branching enzyme
b) Glucose 6 phosphatase
c) C. Glycogen synthase
d) Phosphofructokinase
Ans. B
SIGNS AND SYMPTOMS of Glycogen storage disease type 1-
Glycogen accumulation in liver and renal tubule cells(hepatomegaly &renomegaly)
Hypoglycemia
Lactic acidemia
Ketosis
Hyperlipemia
Disease Enzyme Deficient
Carbohydrates
Type I/Von Gierke’s Disease Glucose – 6-phosphatase
Type II/Pompe’s Disease Acid Maltase
Type III/Cori’s Disease Debranching Enzyme
Type IV / Anderson’s Disease Branching Enzyme
Type V/Mc Arlde’s Disease Muscle Glycogen Phosphorylase
Type VI/ Her’s Disease Hepatic Glycogen Phosphorylase
Essential Fructosuria Fructokinase
Fructose Intolerance Aldolase B
Galactossemia Galactose-1-phosphatase uridyl transferase
22. A newborn develops vomiting, irritability, and lethargy several days afterbirth. His mother
states that his diapers smell like "burned sugar". Laboratory studies confirm the diagnosis,
and the patient is started on the appropriate dietary restrictions with subsequent
improvement in his symptoms. The defective enzyme responsible for this patient's condition
a) Aromatic acid decarboxylase
b) Phenylalanine Hydroxylase
c) Branched Chain Alpha keto acid dehydrogenase complex
d) isovaleryl CoA dehydrogenase
Ans. C
Protein’s
Phenylketonuria (Mousy/Musty odour) Phenylalanine Hydroxylase
Ans. B
Ans. B
Types Locations
Type I Bone, skin, dentin, cornea, blood vessels,
fibrocartilage and tendon
Type II Cartilaginous tissues
Type III Skin, ligaments, blood vessels and internal organs
Type IV Basement membrane in various tissue
Type V Blood vessel wasll, synovium, corneal stoma,
tendon, lung, bone cartilage and skeletal muscle
25. A 73-year-old woman comes to the physician because of a 2-monthhistory of diffuse weakness
and tingling of
her arms and legs Neurologicexamination shows weakness of the extensor and flexor muscles
of thelower extremities. Sensation to vibration and position is decreased in allextremities.
This patient most likely has a deficiency of which of the following vitamins?
a) Vitamin B1
b) Vitamin B2
c) Vitamin B6
d) Vitamin B12
Ans. D
Vitamin Name Deficiency
Vitamin B1 Thiamine Deficiency causes beriberi. Symptoms of this disease of the
nervous system include weight loss, emotional disturbances
Vitamin B2 Riboflavin Deficiency causes ariboflavinosis. Symptoms may include
cheilosis (cracks in the lips), high sensitivity to sunlight,
Vitamin B3 Niacin Deficiency causes pellagra. Symptoms include aggression,
dermatitis, insomnia, weakness, mental confusion, and diarrhea
Vitamin B5 Pantothenic Acid Deficiency can result in acne and paresthesia, although it is
uncommon
Vitamin B6 Pyridoxine Deficiency may lead to microcytic anemia (because pyridoxyl
phosphate is the cofactor for heme synthesis, depression,
dermatitis
Vitamin B7 Biotin Deficiency does not typically cause symptoms in adults but may
lead to impaired growth and neurological disorders in infants
Vitamin B9 Folic Acid Deficiency results in a macrocytic anemia, and elevated levels of
homocysteine. Deficiency in pregnant women can lead to birth
defects
Ans. A
DNA REPAIR MECHANISMS:
1. Exonucleolytic Proofreading
2. Nucleotide Excision Repair -
a. Xeroderma Pigmentosa
3. Strand Directed Mismatch Repair -
a. Hereditary NON Polyposis Colon
b. Cancer (Lynch syndrome)
4. Base Excision Repair
5. Double Strand Break Repair
6. Transcription Coupled Repair
a. Cockayne's syndrome
27. A 25 old female presents to medicine OPD with acuteyearabdominal pain for which almost
exploratory
laparotomy was done. Heis also accompanied by neuropsychiatric symptoms and the urinesample
collected
turns to black with time in room temperature) Mostlikely diagnosis is
a) Porphyria cutanea tarda
b) Acute intermittent porphyria
c) Congenital erythropoietic porphyria
d) Variegate porphyria
Ans. B
28. Storage form of neutral lipids in our liver is
a) Cholesterol
b) Cholesterol ester
c) Triglycerides
d) Fatty acids
Ans. C
29. A 25 year male presents with hypercholesterolemia and increasedrisk of heart disease. His
elder brother also
had similar findings andsuccumbed to this. Workup reveals a congenital LDL receptordeficiency,
The most
likely defect will be?
a) Primary familial hypercholesterolemia
b) Abetalipoproteinemia
c) Familial combined Hyperlipidemia
d) Apolipoprotein E
Ans. A
30. Which of the following is a cofactor for Glutathione Peroxidase?
a) Zinc
b) Copper
c) Selenium
d) Calcium
Ans. C
31. A 35 years old male returns back to India from China. On immigrationhe was found to have
slightly elevated
temperature. One of the followingscreening test was used for this?
a) CBNAAT
b) Dimer
c) Real time RT-PCR
d) CRP
Ans. C
Ans. B
33. Which vitamin deficiency leads to cognitiveimpairment in old age?
a) Vitamin A
b) Vitamin B 6
c) Vitamin B 12
d) Vitamin C
Ans. A
Vitamin A – Retinol/Retinoic acid
Vitamin B 6 - Seizures in children
Vitamin B 12 - Methylcobalamine& cognitive impairment
Vitamin C – Iron absorbtion
34. An alcoholic patient with neurological symptoms. There is deficiency of vitamin ?
a) B1
b) A
c) B9
d) B12
Ans. A
B1- Thiamine
Deficiency seen in chronic alcoholics
Wernicks Encephalopathy
G-global confusion
O-opthalmoplegia
A-ataxia
Korsakoff psycosis
B9-folic acid
B12-methylcobalamine
35. A patient presented with the following lesion. Likely diagnosis?
a) Hypomelanosis of ito
b) Vitiligo (segmental)
c) Nevus of ito
d) Becker's nevus
Ans. B
Segmental vitiligo
Vitiligo vulgaris Most common type
Segmental vitiligo Seen in children
Most frequent in trigeminal nerve distribution
Generalized vitiligo Extensive lesions
Acrofacial vitiligo Periorificially and acral parts
Lip- tip vitiligo Lips, tip of penis, vulva, nipples
Vitiligo universalis Widespread ,multiple endocrinopathies
36. A female patient presented with a multiple warty lesions around vulva. The lesions are
gradually increasing.
On examination they are soft and sessile and didn’t bleed on touch. What is the lilely diagnosis?
Condyloma accuminata
a) Condyloma acuminata
b) Bowen disease
c) Condyloma lata
d) Hemorrhoids
Ans. A
Condyloma accuminata : Caused by HPV 6 and 11
Occurs on penis and vulva, in the urethra and vagina, at the uterine cervix, and in the perianal region.
May become large and invasive
Condyloma lata: Seen in secondary syphilis.
37. A female student presented with utricaria following intake of seafood.she wants a non-
sedating drug since this
is her exam time.which is the preferred antihistamine for this patient?
a) Diphenhydramine
b) Fexophenidine
c) Promethazine
d) Chlorphendimethate
Ans. B
Fexophenidine
38. What could be the diagnosis for this image where exclamation mark is seen in trichoscopy?
a) Alopecia areata
b) Telogen effluvium
c) Trichotillomania
d) Anagen effluvium
Ans. A
Alopecia areata
Patchy alopecia with exclamation mark hair (image)
Subtypes:
Ophiasis→ alopecia along margin of scalp
Alopecia totalis→ loss of hair on total scalp
Alopecia universalis→ loss of hair on whole body
Features:
Sharply defined alopecia
Exclamation mark hair
Sparing of grey /white hair
Going white over night phenomenon
First site affected: scalp
Nails: hammered brass( fine stippled pitting)
Treatment: few lesions → topical minoxidil, topical corticosteroids, topical psoralen + UVA
Extensive lesion → oral corticosteroids, oral psoralen+ UVA
39. A 70 years old patient develop the following lesion over face since 2 years. What is the
diagnosis?
Ans. A
Basal cell carcinoma
Underlying cause :
Albinisum
Arsenic
Psoralens + UVA
UV rays
Tar
Xeroderma pigmentosum
C/F:
Middle aged/ elderly males
Rodent ulcer ulcerated nodule with rolled, pearly, beaded edges.
Superficial BCC
Morphoeic BCC
Pigmented BCC
40. A young lady presents with lacy lesion in the oral cavity. What is the diagnosis?
a) Candidiasis
b) Psoriasis
c) Lichen planus
d) Oral leukoplakia
Ans. C
Oral lichen planus→Asymptomatic/ complain of intolerance of spicy food.
White, reticulate lacy pattern on the buccal mucosa, tongue and gingival.
Erosive gingivitis.
Ans. B
* Paired Unpaved
Arytenoid ● Thyroid
Cuneiform ●Cricoid:- Complete ring cartilage
Curricular * Epiglottis
43. Which of the following laryngeal cartilage doesn’t calcify with age:
a) Thyroid
b) Cricoid
c) Epiglottis
d) Croniculate
Ans. C
Epiglottis is a fibro elastic / soft cartilage will not ossify with age
44. A 40 years old trumpet blower has develoved a right sided swelling in upper part of neck. The
swelling keeps
changing in size . the image shown in x-ray find the appropriate diagnosis of the condition?
a) Throglossal cyst
b) Brachial cyst
c) Laryngocoele
d) Pharyngeal pouch
Ans. C
Laryngocele
It is the dilation of the saccule of laryngeal ventricle
On imaging, these lesions are generally characterized as well-defined,thin-walled, fluid or air-
filled cystic lesions in the paraglottic space
45. Which of the following is the m/c site of head and neck cancer in India?
a) Nasopharynx
b) Oropharynx
c) Oral cavity
d) Hypo pharynx
Ans. C
M/c site of oralcavity ca in India:-
Buccal mucosa (Gingio buccal sulcus)
Head &Neck:- Oral cavity cancer
Over all:- Brest cancer
Ans. C
Maxillary sines is the largest sines of all 4 sines
Maxillary sines drains into MIDDLE MEATUS
Ans. B
* Water view ⭢Occipto mental view (also Done with closedmouth)
*Cald well’s view Occipto frontal view (Frontal & ethmoid sinus more clearly)
* Schuller’s View
D x – ray mastoid view
* Towne’s View
48. Which of the following is the most common craw of neonatal and paediatric inspiratory
stridor?
a) Laryngesmalacia
b) Laryngeal web
c) Congenital subglottic – stenosis
d) Laryngeal cyst
Ans. A
Laryngomalacia is m/c congenitaialanomaly of laryngs
Weakness of supraglottis which leads to lnspiroy stridor shaped larynges omega
49. Which of the following laryngeal muscle is the abductor of vocal cord:-
a) Cricothyroid
b) Thyroartenoid
c) Posterior crico arytenoid
d) cateralcricoartenoid
Ans. C
NERVE SIPPLY
Ans. B
51. A 4 years old child has respiratory distress, fever, inspiratory stridor, x – ray neck lateral view
shows view
shows thumbsign on neck what could be causing organism?
a) Harmophilus influenza
b) Streptococcus pneumonia
c) Para influenza virus
d) Staphylococcus aureus
Ans. B
Epiglottitis
THUMB-PRINT SIGN is seen
Epiglottitis may be seen radiographically on a lateral neck x-ray
Epiglottic swelling(supraglottic) of three to four times normal is seen at the base of the tongue and
appears as if someone left a thumbprint
52. 7 years old child is complaining of hearing loss and sense of full ness in the ear. The
tympanometey show type
B curve what is the diagnoris in this child?
a) Chronic suppurativeotive media
b) Secretory otitis media
c) Acute suppurative otitis media
d) Otitis externa
Ans. B
M/c:- in school going children
2 – Prominent symptonr:- CHL
Fullners in the ear
TYPE A curve: NORMAL
TYPE B curve: serous /secretor otitis media
TYPR As curve:oto Sclerosis
TYPE C curve : Carcinoma (middle ear obstruction)
TYPE Ad curve: oscoles Disruption
53. A 17 year old female patient developed earache on right side post tonsillectomy. Which nerve
is responsible
for the ear pain?
a) Glossophrayngealnerve
b) Vague Nerve
c) Trigeminal nerve
d) Facial nerve
Ans. A
Ear ache right side post tonsillectomy – Glossophrayngeal nerve
Nerve lies in the bed tonsil Glossophrayngeal nerve
Sensory supply to middle ear – Tympanic branch Glossophrayngeal nerve
54. A neonate suffering from bilateral complete choanal atresia has respiratory distress since
birth. What is the
cause of respiratory difficulty in this neonate?
a) Less lung Volume
b) Associated vocal cord palsy
c) Newborns are obligatory nasal breathers
d) Insufficient lung surfactant
Ans. C
55. A 40year old diabeticwoman tested covid 19 positive and she started to have nasal congestion,
black crusts in
nose, and blackish discoloration of skin on nasal bridge and cheek area. What could be the cause?
a) Mucormycosis
b) Allergic fungal sinusitis
c) Nasal vestibulitis
d) Maxillary sinusitis
Ans. A
zygomycetes→seen in terminally ill patients
R → Rhizopus
A→Absidia
M→Mucor
Tissue biopsy :H&E stain (aseptate hyphae, obtuse angled)
Biopsy material on SDA→ cultural growth + LPCB→ Rhizopus
Mucor mycosis is commonly seen in:
DM
Frequent blood tansfusion
Chronic dialysis
56. A patient of Presbyacusis has been found to have high frequency sensorineural hearing loss on
audiometry.
Which of the following part of inner ear is affected in this patient?
a) Apexof cochlea
b) Basal turn of cochlea
c) Utricle
d) Saccule
Ans. B
57. Which of the following arteries is the main blood supply of Tonsil?
a) Descending palatine artery
b) Ascending palatine artery
c) Tonsillar branch facial artery
d) Ascending pharyngeal artery
Ans. C
58. A 60 year old diabetic patient has presented with severe ear ache and blood stained
eardischarge. The
examinationshows granulations in the external auditory canal What is the possible diagnosis in
this case?
a) Malignant otitis externa
b) Diffuse otitis externa
c) Otomycosis
d) Chronic suppurative otitis media
Ans. A
m/c/c –PSEUDOMONOS
59. A child aged 2 years, presented with severe SNHL was prescribed hearing aids, but showed no
improvement. Which is the next line of management?
a) Fenestration surgery
b) Conservative
c) Stapes mobilization
Cochlear implant
Ans. D
61. Cherry red colour post-mortem staining is associated with which poisoning ?
a) H2s
b) CO
c) Nitrate
d) Hydrogen cyanide
Ans. B
CO: Cherry red
62. A baby patient comes to casualty with history of unknown poisoning by some seed/drug, on
examination
patient presented with Dilated pupil, Dry mouth, Dilirium and decreased, secretions, skin
scratches,
Hallosination poisoning should the Dr suspect
a) Dhatura
b) Poppy seeds
c) Mushroom
d) Cannabis
Ans. A
Dathura contains anti-cholinergic substances that cause these symptoms
Mushroom contains cholinergic substances
Poppy seeds contains opium- morphine like substance
63. A person died due to starvation, which is correct in such case during autopsy?
a) Enlarged heart
b) Enlarged stomach
c) Distended gall bladder
d) Expanded lungs
Ans. C
When a person have his food. When if reaches Duodenium it stimulates to release of CCK which helps in
contraction of gall bladder and Bile is released.
As the person is in starvation:- bile secritons will not enter to deodinum so, gall bladder
distended.
All visceral organ size , Except BRAIN
Lungs will get collaps
Cause of death is starvation:- Circulatory failure
64. A person has committed a criminal offence, caught by police & later brought to a criminal court
but the court
directs police to transfer the case to Juvenile courts if it was proved that accused is of ___________
years of age?
a) 21
b) 17
c) 25
d) 19
Ans. B
Child < 18 according to Juvinile Justice Board.
Abdament in act:- any Juvinal of 16-18 years do’s Heanes offence like (Rape, murder) will come under
adult.
65. In Barberious test, yellow colourneede shaped crystals are seen under microscope. This is
done to detect
which stain?
a) Sputum
b) Semen
c) Blood
d) CSF
Ans. B
Boy friend:
1) Barberious
2) Florena = Semen
Blood:-T-Tetamans
T – Takayama
66. A farmer was brought to casualty with pinpoint pupil increased secretions and garlicky ordor,
which drug to
be used by physiation for treatment?
a) Oximes
b) Atropine
c) N-acetylcyteine
d) Physostigmile
Ans. B
organophosphate poisoning(OP): pinpoint pupil,garlicodour,increase secretions
DOC for OP: atropine
DOC for angiodiazepine: Flumazenil
Paracetmolpoisoning : N-acetylcysteine is used
Note: oximes are used in OP but it is not the drug of choice
67. A childaccidentaly consumes iron syrup used by the mother, Antidote to be used in such
poisoning is?
a) BAL
b) EDTA
c) Desferrioxamine
d) Trientene
Ans. C
Chelatorfor :copper poisoning &wilsons disease: TRIENTENE
Ans. D
Tanguntial gun shot:- # of only outer table of skull
69. In an unconscious patient with head injury requiring emergency Sx. If there are no relatives to
give consent the doctor should?
a) Operate without consent
b) Should not operate
c) Wait for the arrival of relatives
d) Take consent from the hospital authority
Ans. A
IPC: 92 state DOCTORCANoperate with out consent in case of emergency
70. A 40-years old patient was brought by police in unconscious state to casualty with history of
unknown poisoning, on examination patient had needle track ulcers over upper limbs, Bugs
crolling on skin, patient had Hallusions with scratch marks, increased blood pressure,
increased heart rate and respiratory rate of 30/ minute, which poisoning should Dr. suspect?
a) Cocaine
b) Heroin
c) Cannabis
d) LSD
Ans. A
Option we can easily eliminate by:
HEROIN : respiratory rate decreases
CANNABIS: BP decreases (due to :vomiting&diatthia)
LSD : is taken through oral rout so percutianeous ulcers are not found over upper limb
Ans. B
LIGATURE MARK DIFFERENCE BETWEEN
Ans. D
Lower central incisors :6-10 months
Upper central incisors :8-12 months
All teeth appear by 2.5 years
Ans. A
74. Identify the organism:
a) HIV
b) Herpes
c) Hepatitis B
d) Adenovirus
Ans. A
Family :Retroviridae
Subfamily: lentivirnae
It is a ssRNA virus
3 structural gene
→gag : matrix protein
→pol: polymerase
→Env : envelop, Gp41,Gp120
HIV affects CD4+ve T-cells by coreceptors CXCR4
HIV affects macrophages by coreceptors CCR5
Most sensitive test for HIV: ELISA
Most specific test for HIV: HIV RNA PCR ,WESTERN BLOT
Mother to child transmission –30%
Blood transfusion –90%
Needle stick injury—0-5%
75. A 5 year old child presents with fever, bull’s neck as shown in the image. Identify the disease.
a) Mumps
b) Lyme’s disease
c) Diphtheria
d) Measles
Ans. C
Diphtheria( klebs loeffler’s bacilli)
Gram positive bacilli
Chinese letter pattern/cuneiform arrangement is seen.
Staining—ponder’s, albert’s, neisser’s ( contents of albert’s stain: toludine blue,malachite green, glacial
acetic acid, kl + l2)
Pseudomembrane formation is seen → greyish site →M/C site faucial (tonsillar)
Bull’s neck( neck edema)
Chronic complication : myocarditis, polyneuropathies
Dx:
LSS( loeffler’s serum slope) ,culture media,enriched media(used for early diagnosis of diphtheria)
Potassium tellurite agar( best selective media)
Macleod’s media ( diphtheria toxin)
76. Most common viral agent causing pneumonia in < 1 year of age:
a) RSV
b) Parvovirus
c) HHV-6
d) Influenza virus
Ans. A
RSV( Respiratory Syncitial Virus) –RTI in <1 year of age.
Diagnosis :cytopathic effect (synctium form)—MN giant/mØ.
DOC—Ribavarin.
Yersinia pestis
77. The agent of the disease shown in the image is not found in which body fluid?
a) Saliva
b) Urine
c) Fluid from the leg
d) Ascitic fluid
Ans. A
78. Which among the following infection doesn’t form a membrane like covering over the tonsils?
a) Diphtheria
b) Infectious mononucleosis
c) Streptococcal tonsilitis
d) Ludwig angina
Ans. D
Ludwig's angina is a rare skin infection that occurs on the floor of the mouth, underneath the tongue. This
bacterial infection often occurs after a tooth abscess, which is a collection of pus in the center of a tooth.
79. A female with the following symptoms of sudden high fever, associated with headache and
vomiting along with
nuchal rigidity, presented to the hospital. CSF analysis of this patient showed the growth of this
organism aa
gram negative diplococci shown in the image,identify the organism:
a) Neisseria meningitidis
b) Streptococcus pneumoniae
c) Staphylococcus aureus
d) Mycobacterium tuberculosis
Ans. A
Neisseria meningitidis: Gram negative cocci(catalase/oxidase positive)
Ferrments glucose and maltose( capsulated)
Cultured media : thayer martin media, muller hinton agar
Transport media : stuwart amile media
Complications: water-house friderichsen syndrome( B/L adrenal haemorragic necrosis)
Deficiency of C5-C9(MAC): Prone for neisserial infections.
DOC: Ciprofloxacin( carrier), ceftrioxan(cases)
80. A covid patient presented with fever along with nasal congestion and unilateral headache.
Later he developed
facial pain and numbness along with blackish discharge, KOH amount reveals nonseptate hyphae
at right
angkes or obtuse angle, what is the most probable diagnosis?
a) Mucormycosis
b) Aspergillosis
c) Bacterial orbital cellulitis
d) Cavernous sinus thrombosis
Ans. A
a) Plasmodium Falciparum
b) Plasmodium vivax
c) Plasmodium ovale
d) Plasmodium malariae
Ans. A
Plasmodium vivax Plasmodium falciparum
Young aged RBC involved All age RBC are involved
↑ in size of RBC Normal size RBC
Schuffner’s dot Maurer’s dot
Gametocyte stage Banana shaped gametocytes
Single ring with in RBC Multiple ring within RBC
Reddish brown pigment Dark brown pigment
a) Rotavirus
b) Giardia lamblia
c) Cryptosporidium
d) Adenovirus
Ans. B
Giardia lamblia → fatty foul smelling diarrhea and fat malabsorption.
Feco-oral route transmission
Dx: string test
Falling leaf motility
4 pairs of flagella , tennis racket shape
DOC: Metronidazole, tinidazole
83. Orchitis is an important complication of which infection?
a) Mumps virus
b) Adeno virus
c) Polio virus
d) Pox virus
Ans. A
Myxo virus
Period of communicability: 4-6 days before and 1 week after.
Secondary attack rate: 86%
Mode of transmission : droplets
C/F: pain, swelling in parotid region, earache.
Complication: aseptic meningitis( children), orchitis( M/C extra salivary manifestation in adults), leading
cause of pancreatitis in children.
Vaccin: jerry-lynn stain, live attenuated
84. An immunocompromised patient came to the hospital with severe diarrhea. Flectal swab
revealed the
following image. Identify the causative organism.
a) Cryptosporidium parvum
b) Rotavirus
c) Giardia lamblia
d) Adenovirus
Ans. A
Cryptosporidium parvum → causes diarrhea in AIDS patients.
Oocysts and cyst are infective
Dx:Acid fast oocyts
DOC: Cotrimoxazole
85. A 10 year old child is diagnosed with amblyopia in one eye. What will be the best treatment ?
a) Observation
b) Penalisation
c) Occlusion
d) None of the above
Ans. C
Occlusion of the good eye to force the use of lazy eye.
86. A 3 months old baby presents with mucous exuding from eye on pressing the lacrimal sac.
What treatment
should be given?
a) Lacrimal sac massage
b) Syringing
c) Probing
d) DCR
Ans. A
Lacrimal sac /Pediatric massage/Crigglar sac massage: only till 1 year
87. Which lesion of the visual pathways produces a macular sparing field defect?
a) Optic chiasma
b) Occipital lobe
c) Optic nerve
d) Optic tract
Ans. B
Occipital lobe is supplied by 2 arteries: posterior cerebral artery and middle cerebral artery in which
posterior cerebral artery is more often got damaged. And middle cerebral artery still supplies so we see
macula is spared.
88. A child with history of malnutrition is examined and the following finding is noted. What is the
most likely
diagnosis?
a) Megalocornea
b) Buphthalmos
c) Anterior staphyloma
d) Corneal degeneration
Ans. C
89. A young male complains of watery eyes and itching and on eversion of upper eyelid the
following picture is
seen. What is the diagnosis?
a) Bacterial conjunctivitis
b) Trachoma
c) Vernal keratoconjunctivitis
d) Angular conjunctivitis
Ans. C
In question-- Itching is mostly seen in allergic conditions and we see cobble stone appearance in image,
above features are seen vernal keratoconjunctivitis.
Other options:
Bacterial conjunctivitis : no itching is seen
Trachoma: folicule formation is seen
Angular conjunctivitis : seen in chronic malnurised patients.
Ans. D
Ans. C
Excision with conjunctival auto graft
92. A lady present with a sudden painful loss of vision which worsens on eye movement. RAPD is
present on
perimetry a central scotoma is revealed. What is the diagnosis?
a) Optic neuritis
b) Papilledema
c) Retinoblastoma
d) CRVO
Ans. A
In optic neutritis (blurred disc margin) In retrobulbar neutritis( clear cut margin)
Other options:
Papilledema –no loss of vision.
Retinoblastoma – causes leukocoria, squint, glaucoma.
CRVO – painless loss of vision.
93. A patient presents with history of trauma with vegetable matter a few days back, with pain and
photophobia.
What is the most likely cause?
a) Herpes simplex keratitis
b) Acanthamoeba keratitis
c) Bacterial keratitis
d) Fungal keratitis
Ans. D
Other options:
Herpes simplex – painless ulcer
Acanthamoeba keratitis – painfull but due to contact lens
94. A child presents with proptosis, pain and loss of vision. O/E the following picture is seen. What
is the most
likely diagnosis?
a) Orbital cellulitis
b) Stye
c) Preseptal cellulitis
d) Blepharitis
Ans. A
Orbital cellulitis— proptosis, chemosis, congestion, pupil dilation and inflammation are seen in image.
Generally orbital cellulitis starts with a simple cold and cough→ ethmoid sinusitis →medial wall of orbit.
95. A boy gets punched in the eye. Which of the following is the most likely occurrence?
a) Subluxation of the lens
b) Inferior orbital wall fracture
c) Medial wall fracture
d) Lateral wall fracture
Ans. B
Blow out fracture-- Inferior wall gets fractured more often because it gets supported by maxillary sinus
which is cavernous.
(Tear- drop sign)
96. A patient with VA ˃1/60, but <3/60 in better eye. What type of blindness does he have?
a) Low vision
b) Economic blindness
c) Social blindness
d) Manifest blindness
Ans. C
Ans. A
Ectropion
lower eyelid
98. A patient presents with history of DM of more than 20 years duration and painless loss of
vision with the given
fundus findings. What is the most likely diagnosis?
a) NPDR
b) PDR
c) CRVO
d) CRAO
Ans. A
NPDR
Other options:
PDR— not the answer because no new blood vessels seen in the image.
CRVO—no tomato splash appearance in image.
CRAO—could see ocular spot.
99. A lady presents with ptosis, on eating and sipping of fluids her ptosis decreases. What is the
most likely diagnosis?
a) Mechanical ptosis
b) Horner’s syndrome
c) Complicated ptosis
d) Blepharophimosis syndrome
Ans. C
Complicated ptosis—Marcus Gunn Jaw- winking syndrome: a retraction of the ptotic lid in conjunction
with stimulation of the ipsilateral pterygoid muscle
100. A newborn child presents with watering and photophobia. O/E the following picture is seen.
What is the
diagnosis?
a) Buphthalmos
b) Congenital cataract
c) Megalocornea
d) Congenital dacryocystitis
Ans. A
Other options:
Congenital cataract—absence of water and megalocornea
Megalocornea—no watering
Congenital dacryocystitis—absence of megalocornea
101. A patient presents with complicated cataract and on slit lamp findings are shown in the given
image. What is
the type of cataract seen in this case?
a) Anterior subscapular cataract
b) Nuclear cataract
c) Posterior subscapular cataract
d) Cortical cataract
Ans. C
Ans. A
Myopia and astigmatisum due to irregular surface.
103. All of the following are true about below procedure, except:
Ans. C
The image shown here is direct ophthalmoscopy.
104. A patient presents with ocular motion defect. The position of the eyeball is shown in the
image below.
Identify the nerve involved in following defect.
a) Occulomotor nerve
b) Optic nerve
c) Trochlear nerve
d) Abducens nerve
Ans. A
105. Person with one month H/o hip pain, true limb shortening and diagnosed as a case of
tuberculosis. Which
stage is it ?
a) Late arthritis
b) early arthritis
c) early synovitis
d) advanced arthritis
Ans. A
Stage 1 : synovitis stage : hip in same poture ( i.e abduction and external rotation)
Stage 2 : early arthritis : hip is in adduction and internal rotation
Stage 3 : late arthritis : joint space distructed ( TRUE LIMB SHORTENING )
Stage 4 : advance arthritis : acetabulum will move freely
106. X-ray and clinical photo is given, which nerve is most commonly damaged ?
Ans : B
Posterior hip dislocation:
FADIR flexion,adduction,internal rotation
Limb shortening
Sciatic nerve damage
Lesser trochanter is not seen clearly
Anterior hip dislocation:
FABERflexion,abduction, external rotation
Femoral nerve damage
Lesser trochanter is seen clearly
107. Hormones and endogenous substances that are involved in regulation of calcium balance in
the body are ?
a) Vitamin D,PTH,Insulin
b) Vitamin D,PTH, Calcitonin
c) Vitamin D,Calcitonin,TSH
d) Vitamin D,PTH,TSH
Ans. B
ca + po4
I. Vit d
II. PTH
III. Calcitonin
108. A patient from bihar present with teeth and skeletal image as shown. most likely cause is ?
a) Osteropetrosis
b) Osteoporosis
c) Osteosclerosis
d) Osteomalacia
Ans. C
Dental flourosis
Normal value of flourosisis : 0.5 - 0.8 ppm
Upper limit : 1- 1.5 ppm
109. A alcoholic patient slept with compression. Around his arm region and had injury to radial
nerve. Most likely
this is which type of injury ?
a) Neuropraxia
b) Neurotmesis
c) Axonotmesis
Ans: A
110. A 40 year old female patient having history of bee allergy came to the emergency room with
complaints of
swelling of face and dyspnea after a bee sting . Which of the following inflammatory mediators
plays a major
role
a) Serotonin
b) Bradykinin
c) Histamine
d) Prostaglandin
Ans. C
111. When stained with Congo red and visualized under the polarising microscopewhich of the
followingsubstance would show apple green birefringence
a) Lipids
b) Amyloid
c) Collagen
d) Calcium
Ans. B
112. 24year oldfemale came with complains ofneurological deficits. Examination reveals
pallorHerblood
picture is given below. What is the most like diagnosis
a) Folate deficiency
b) Vitamin B12 deficiency
c) Thalassemia
d) Iron deficiency anemia
Ans. B
Vitamin B12 deficiencyanaemia occurs when a lack of vitamin B12 causes the body to produce
abnormally large red blood cells that cannot function properly.
113. A 35 year old lady presented with slow growing thyroid swelling for the past 10 months .
Histological
examination was done and the microscopic image of the biopsy had the presence of Orphan Annie
eye nuclei.
What is the patient suffering from
a) Follicular carcinoma of thyroid
b) Papillary carcinoma
c) Anaplastic carcinoma
d) Medullary carcinoma
Ans. B
114. Supravital staining is used for ?
a. Red blood cells
b. Platelets
c. Reticulocytes
d. White blood cells
Ans. C
115. Small or medium sizedsterile vegetations found in either or both sides heart valve commonly
associated with SLE patients is characteristic of which of the following disease?
a) Non bacterial thrombotic endocarditis
b) Rheumatic heart disease
c) Infective endocarditis
d) Libmann Sachs endocarditis
Ans. D
116. A person with hepatic jaundice, deranged liver function tests and presence of a clinical
finding as shown
alongside
a) Huntington chorea
b) Wilson's disease
c) Leigh syndrome
d) Hemochromatosis
Ans. B
117. A teenager presents with features of liver disease, tremor and poor incoordination, mask like
facies, and
psychiatric symptoms. Which of thefollowing is implicated in the pathogenesis of this condition?
a) ATP 7A gene on chromosome 13
b) ATP 7B genechromosome 13
c) ATP 7C gene on chromosome 13
d) ATP 7B gene on chromosome 6
Ans. B
118. A child presented with history of pallor with fatigue, petechial bleeding and fever since last 7
days. On
examination he was havingenlargement of liver and spleen with sternal tenderness. Which of the
following
is the likely explanation for the symptoms in the child?
a) Aplastic anemia
b) Chronic myeloid leukemia
c) Acute lymphoblastic leukemia
d) Acute myelogenous leukemia
Ans. C
119. A person is working with 30 year history of working in cardboard manufacturing and
develops
breathlessness and likely to be associated withmottling in lungs is
a) Bysinossis
b) Bagasossis
c) Asbestosis
d) Nasopharyngeal carcinoma
Ans. B
Ans. B
Klinefelter-xxy-and-turner-xo-syndrome
121. A 16 year old boy had abdominal discomfort on the left side andhe went to his physician. His
father had a
similar condition during his teenage and later developed some GI malignancy. He then underwent
a
colonoscopy which is shown alongside. What doyou conclude about his condition?
Ans. A
FAP
˃100 polyps are seen.
Autosomal dominant
Mutation in APC gene (5q21)
122. A 30 year male patient has a complaint of a painless testicular mass. His blood sample
revealed non elevated
alpha fetoprotein levels. Which of the following is the most likely cause of his testicular mass?
a) Yolk sac tumor
b) Classical seminoma
c) Teratoma
d) Choriocarcinoma
Ans. B
123. An elderly male has been working in the dye industry for 40years.He develops
somefrequency and urgency,
Bladder symptoms like hematuria. What is the likely subtype of the conditionpresent in him?
a) Transitional cell cancer
b) Adenocarcinoma
c) Squamous cell cancer
d) Small cell cancer
Ans. A
124. Which of the following is the likely diagnosis in a lady with extremely low TSH and high T3
and T4?
a) Subclinical hypothyroidism
b) Subclinical hyperthyroidism
c) Graves disease
d) Reidel thyroiditis
Ans. C
125. A middle aged lady presents with weight gain, constipation, hoarseness of voice and lethargy.
On
examination, she is found to be having anti TPO antibodies. What is the likely diagnosis in her?
a) Reidel's thyroiditis
b) Graves' disease
c) Hashimoto thyroiditis
d) De quervain thyroiditis
Ans. C
126. A lady has been diagnosed with breast cancer. Her biopsy report revealing the presence of
tumor cells with
signet ring appearance, mucin deposition and "Indian file pattern" is most likely suggestive of
a) Invasive lobular carcinoma
b) Invasive ductal carcinoma
c) Medullary cancer
d) Mucinous
Ans. A
127. Histological examination of a bone tumor is as follows . What is thelikely diagnosis?
a) Osteosarcoma
b) Giant cell tumor
c) Ewings sarcoma
d) Chrondrosarcoma
Ans. B
128. Which of the following anti-diabetic drug does not require dose reduction in renal failure?
a) Linagliptin
b) Sitagliptin
c) Vildagliptin
d) Saxagliptin
Ans. A
Normally liptins are avoided in renal failure except: linagliptin is the only drug safe in renal failure
Liptins are dipeptidyl peptidase-4 inhibitorthey inhibit breakdown of GLPGLPrelease
insulinblood sugar
129. Which of the following drugs used in emergency management of asthma, which doesnot act by
causing
bronchodilation?
a) Salbutamol
b) Ipratropium
c) Deriphylline
d) Hydrocortisone
Ans. D
There are 3 major types of bronchodilators:
I. Beta2 agonists [eg:salbutamol,salmetrol]
II. ACE inhibitors [eg:ipratropium,thiotropium]
III. PDE inhibitors [eg:theophylline,deriphylline,aminophylline]
Ans. A
Low dose: <1-2μg/kg/min
At low dose dopamine stimulates only D1 receptors
D1 receptors are present on renal blood vessels
So when dopamine stimulate D1 receptors renal vasodilation increase in renal blood
flow
Intermideatedose : 2-10μg/kg/min
High dose :>10μg/kg/min
131. A female patient presented with pain and redness in great toe.serum uric acid level is
9.6mg/dl. A part from
prescribing analgesics for relieving pain , the physician prescribed a drug which can decrease the
formation
of uric acid. Which of the following enzyme is likely to be inhibited by this drug ?
a) Xanthine oxidase
b) Thymidylate synthase
c) Phosphpribosyl transferase
d) Dhfr
Ans. A
Uric acid formed from purines which are metabolized to generate xanthine and convert
xanthine into uricacid by xanthineoxidase
Ans. C
2 major substances help in increase in absorption of iron :
I. Hydrochloric acid
II. Vitc/ascorbic acid
2 substances which decrease absorption of iron:
I. Phytates
II. Oxalates
III. Tannates
Ans. C
PDE inhibitor
USES:
Erectile dysfunction
Pulmonary HTN
Ans. B
Potency: It defines left and right side of the curve
more the drug is on left side more potent the drug
more the drug is on right side less the potent the drug
Efficacy: It defines by height of the curve
Higher the curve more efficacy
Lower the curve less efficacy
135. Which of the following antimicrobials is contraindicated in a patient with seizure disorder?
a) Ampicillin
b) Ofloxacin
c) Moxifloxacin
d) Cefixime
Ans. B
2 major groups of anti-microbials contraindicated in seizure disorders:
I. Flouroquinolol [floxacindrugs] - ofloxacin > moxifloxacin
II. Carbapenems [imipenem drugs]
136. Which of the following drug is preffered for closure of ductus arteriosus in a preterm baby
with PDA ?
a) Indomethacin
b) Ibuprofen
c) Mefenamic acid
d) Paracetamol
Ans. B
NSAIDS are used to close ductus arteriosus
NSAIDS are: ibuprofen[best],indomethacin,aspirin
137. A patient on long term digitalis therapy develops arrythmia. What will be the next step in the
management
a) Adenosine
b) Amiodarone
c) Atropine
d) Lignocaine
Ans. D
Lidocaine: drug of choice for ventricular arrhythmias
Atropine: drug of choice for atrial arrhythmias
Amidarone: it causes toxicity
Digoxin toxicity can emerge during long-term therapy as well as after an overdose. It can occur even
when the serum digoxin concentration is within the therapeutic range.Toxicity causes anorexia,
nausea, vomiting and neurological symptoms. It can also trigger fatal arrhythmias.
"Digitalis" arrhythmias were treated with beta-adrenergic blockers: Inderal, Viskene, Eraldin, Trasicor
and Aptin. These drugs proved effective in most cases with atrial arrhythmias and in some--with
ventricular arrhythmias. Lidocain was more effective in cases of ventricular arrhythmias. Effective
drugs of a broad spectrum are also Aimalin, Pulsenorma and Ritmodan.
Ans. A
Explanation:
Edrophonium is ACHesterase inhibitor
It act only for 10min
By giving edrophonium if the condition improves then it diagnose as Myasthenia gravis
By edrophonim if the condition worsens then it diagnose as cholinergic crisis
139. Which of the following drugs can be used for the treatment of undescended testes?
a) Testosterone
b) GnRH
c) Cold Water Compress
d) Anti -MIH Therapy
Ans. B
Pulsatile GnRH analogues [leuprolide] testosterone help in movement of testis to scrotum
140. An elderly male developed the symptoms of dementia and was diagnosed as Alzhiemer’s
disease. Which
neurotransmitter level is reduced in this disease ?
a) Dopamine
b) Acetylcholine
c) Adrenaline
d) Serotonin
Ans. B
Alzhiemer’s dementia occurs in basal nucleus of MEYNERT in brain
MEYNERT function: is to maintain memory, this area contains plenty of Ach
So if there is any problem in MEYNERT nucleus then Ach level will decrease
141. Which of the following combination of seizure and its first line drug is not correct ?
a) Generalised seizure – Valproate
b) Myoclonic- Topiramate
c) Focal- Levetiracetam
d) Absence- Ethosuximide
Ans. B
Myoclonic -valproate
Ans. D
Measure:- Total chlorine – free chlorine A/M combine chlorine
O T test: Measures free chlorine (within 10 seconds) and free + combined chlorine (15-20 mint) in water.
Ans. D
Stage 1: (High stationary):- High BR
High DR
144. A health worker goes house to house for every 15 days taking fever history and preparing
malarial slide.
Which form of surveillance is this?
a) Active
b) Passive
c) Sentinel
d) Any of the above
Ans. A
Malarial surveillance was done by MPW (Male)Subcenter level
Passive: Hospital reporting Most convenient way of sample
Sentinel: Done for air & water quality
145. Two vaccines MB and Pentavalent used for immunization round given for the worker which
was open. Which
of the following is vaccine is discarded?
a) MR discard, USE pentavalent
b) Pentavalent
c) Both MR & Pentavalent
d) MR can be used, Pentavalent was discarded
Ans. A
VACCINES WHICH WILL NOT FOLLOW OPEN VIAL POLICY
BCG
MEASLES
MR
JE
YELLOW FEVER
Ans. B
IPR: Most sensitive index for recent transmission of Malaria
147. According to Bio medical wastage compare. Which Bag of Bio medical wastage can be
incinerated?
a) Red
b) Blue
c) Yellow
d) White
Ans. C
Yellow: Incinevates
Red
Blue Disinfected and later discarded
White
a) JE
b) Dengue
c) Zika virus
d) Malaria
Ans. A
GIVEN IMAGE :.CULEX
Causes:Failarisis
JE
.ANOPHELES
Causes:
Maliaria
.ADES
Causes :
Dengue ,Riftvally fever, Yellow fever, Chicken gunia
149. The average number children that would be born to a woman over her reproductive life span
refers to?
a) Total fertility rate
b) Gross reproduction rate
c) Net reproduction rate
d) General fertility rate
Ans. A
TFR:- Give magnitude of complete family size
GRR: Measures the no. of daughters a woman would have in her life time
NRR: No. of daughters a newborn girl will bear during her life time assuming fixed age specific fertility
and mortality rate
NRR: 1
To active NRR=1, CPR> 60%
GFR:- No. of CB for 1000 people who are in reproductive age group
150. A pregnant lady was bit by a rabid dog, what should be the next step?
a) Treat the wound locally
b) Give vaccine immediately
c) Local treatment of wound + immunoglobulin + vaccine
d) Local treatment of wound + vaccination
Ans. D
Ans. B
Image:- PM-JAY
Full form :Pradhan mantra janarogyayojana
Ans. B
Population covered by PHC
Plain :30000
Hilly: 20000
Population covered by Sub center
Plain :5000
Hilly: 3000
Population covered by CHC
Plain :120000
Hilly: 80000
CHC is the first referral level
Ans. D
4th principle :Equity of distribution
154. Survey of 4 cities. A, B, C, D reported IFR as 2.1, 2.4, 2.6, 2.8. which city had achieved desire
goal of TFR?
a) 2.1 (A)
b) 2.4 (B)
c) 2.6 (C)
d) 2.7 (D)
Ans. A
TOTAL FERTILITY RATE :- Single most important factor in population growth
Ans. B
Open vial vaccines can be used for 4 weeks
Vaccines follows open vial policy are:
.PENTAVALENT .OPV
. HEP B . IPV
. DPT . Td
156. Which vaccine has seroconversion time less than incubation period for that disease?
a) Flu
b) Measle
c) Rubella
d) Mumps
Ans. B
Flu:- I.P:- 7 days after vaccine AB develops > 7 days
Measles: I.P:-10-14 days:- after exposure If we give vaccines Ab will develops within 7 days
Rubella:- I.P:- 14-21 days: AB take time: 2-4 weeks
Measles vaccine:
a) Pre exposure:
Live attenustaved
Strain: Edmostron ZAEREB
Powered: We use: distilled water
0.5 ml s/c to Rt shoulder
2 dose 1) = 9-12 months
2) = 1 1/2 year (16-24 months)
b) Pos Exposures
1) Measles vaccine
Within 3 days of exposure
2) Ig (Immuno compromised child)
0.25 mg/kg
IM
157. A person reported to ophthalmic OPD with vision of <3/6 in RT eye and finger movement at 1
meter distance
in LT eye. His state of vision is?
a) Severe visual impairment
b) Economical Blindness
c) Social Blindness
d) Manifest Blindness
Ans. C
Normal: 6/6 – 6/18
158. A man travelled to Assam and after returning back he developed fever, giddiness, and on
investigation
confirmed to be a case of PI falciparum, Tx of choice?
a) Doxycline
b) Chloroquine
c) I/V artemether
d) Artesunale, Salfadoxine, Pyridethamine
Ans. C
P. vivax:-Chloroquin in all trimesless
No primaquive
159. A worker in the wheat section of a food facility eats peanutsregularly and develops hepatic
carcinoma after
many years.Which of the following is implicated in the pathogenesis of his condition?
a) BOAA
b) Aflotoxin
c) Pyrolizidine
d) Sanguanarine
Ans. B
BOAA:-Latherysm
Aflatoxin:- Aspergillus flavours fungus : causes :hepatocellular carcinoma
Pyrocizidine:- Endemic asitis
SANGUANARINE:- Epidemic drop (mustard oil+ argenomonc oil)
160. In a hospital the management wants to know that how much time that each doctor is
spending with patient
on an average. They want to know increase or decrease in the number of doctors as per
requirement, to
Improve the quality of care. Which management technique should be used?
a) FGD
b) Systemic Analysis
c) Network Analysis
d) Work Sampling
Ans. D
161. A case control study was done in a group of people who are ordering food and who are
preparing food to see
in which group chances of Obesity is more what sample should the investigator should take?
a) Person with or without ordering food
b) Person with or without exerces
c) Person with and without ordering food
d) Person without ordering food
Ans. C
Ans. B
Ans. C
Ans. D
Ans. A
166. Study the graph given below of different occupational group’s. Analyse and mark who has the
highest chance
of developing fever?
a) Industrial Worker
b) White collar Worker
c) Farmer
d) Housewives
Ans.
167. A person with depression prescribed a serotonin and norepinephrine reuptake inhibitor.
Identify the drug?
a) Fluoxeitine
b) Venlafaxine
c) Sertraline
d) All of the above
Ans. B
Serotonin nor-epinephrine reuptake inhibitors:
Venlafaxine
Desvenlafaxine
Duloxetine
Selective serotonin reuptake inhibitor:
Escitalopram
Paroxetine
Fluoxetine
Ans. A
Alcohol withdrawal
Symptoms
After 6-8 hours: coarse tremors (m/c), nausea, vomiting, anxiety.
After 12- 24 hours: alcoholic hallucinations (auditory).
After 24-48 hours: alcohol withdrawal seizures (GTCS)—comes in clusters.
After 48-72 hours: delirium tremens—(lillputian hallucination-patient sees everything small)
169. A chronic alcoholic patient develops delirium tremens after stopping alcohol. Patient should
be treated with?
a) I/V methycobalamine
b) I/V thiamine
c) I/V thiamine + lorazepam
d) I/V methylcobalamine + lorazepam
Ans. C
Ans. A
Panic attack →sudden, discrete period of intense fear or discomfort that are accompanied by significant
physical and cognitive symptoms.
C/F: palpitations, chest discomfort, dizziness,discomfort, sweating, trembling, shaking.
Cognitive symptoms : disorganized thinking, irrational fears, depersonalization, decreased ability to
communicate.
Fear of death: mimics heart attack
Peak at 10 minutes, lasts upto 30 minutes.
171. A 32 years old women presented with symptoms of mild depression first time. What should
be the initial
treatment?
a) Cognitive behaviourtherapy(CBT)
b) Antidepressant
c) CBT+ antidepressant
d) Electro convulsive therapy
Ans. A
Treatment of depression:
Mild depression
For 1st time:
CBT- Multiple sessions maybe needed.
Exercise-early morning
Active monitoring
If not getting better start anti-depressants
Antidepressents:
SSRI’S, SNRI’S, TCA’S, MAOI’S, Atypical antidepressant (choosen on the basis of side effects).
172. A patient suffering from RTA 6 hrs back he was brought to the emergency with altered
sensorium. his
Glasgow coma score was 12. CT scan of head done which showed the following. he is most likely
suffering
from –
a) Sub dural hematoma
b) Epidural hematoma
c) Subarachnoid hemorrhage
d) Intracerebral hematoma
Ans. B
Explanation: EPIDURAL HEMATOMA
Ans. A
ACHALASIA CARDIA
a) Pleural effusion
b) Perforation peritonitis
c) Sigmoid volvulus
d) Intestinal obstruction
Ans. B
PERFORATION PERITONITIS
Air under diaphragmpneumoperitoneum
Ans. A
Congenital diaphragmatic hernia
Depending on the location and size of the defect retroperitoneal or intra-abdominal organs and
tissues can prolapse intothoraciccavity due to negative intra-thoracic pressure
176. Barium enema image demonstrates
a) Meckel's diverticulum
b) Diverticulosis of colon
c) Sigmoidvulvulus
d) Colon cancer
Ans. B
Explanation: DIVERTICULOSIS
SAW-TOOTH APPEARANCE
Description: serrated/concertina-like appearance
Mc site: sigmoid colon
Seen in: Diverticulitis
177. A chronic smoker presented to the emergency . His X-ray chest was done which showed the
following.
Possible diagnosis in this patient
a) Pneumothorax
b) Tension pneumothorax
c) Hemothorax
d) Hydropneumothorax
Ans. B
TENSION PNEUMOTHORAX
Ans. B
PANCREATIC PSEUDOCYST
Duration:>4weeks
Fibrous wall is seen
Homogenous collection is seen
a) Caudate
b) Thalamus
c) Lentiform nucleus
d) Insular cortex
Ans.
T2 is the time taken for hydrogen atoms to diphase
T1 is the time taken for return to relaxed state ( and release energy)
T1 Image T2 Image
Time to echo (TE) 40 ms 120 ms
Time to repetition (TR) 400 ms 1000 ms
Useful for Demonstrating anatomy Demonstrating pathology
Bright for Fat, methaemoglobin, contrast, Fluid (oedema), fat
calcium, blood
Ans. A
181. A 65 year old female presented with imbalance while standing. On examination there is loss of
vibration and
proprioception. What will be next line of treatment
a) Iron
b) Vitamin B12
c) Niacin
d) Folic acid
Ans. B
Vitamin B12deficiency symptoms include:
strange sensations, numbness, or tingling in the hands, legs, or feet.
difficulty walking (staggering, balance problems)
anemia.
a swollen, inflamed tongue.
difficulty thinking and reasoning (cognitive difficulties), or memory loss.
weakness.
fatigue
182. A patient presented with complaints of fatigue, dyspnea on exertion.Lab tests shows
macrocytic RBCS. Serum
methylcobalamin level is within normal limits.What will be next line of treatment
a) Iron
b) Folic acid
c) Vitamin B12
d) Pyridoxine
Ans. B
The symptoms of anemia that occur due to folate deficiency include:
Persistent fatigue.
Weakness.
Lethargy.
Pale skin.
Shortness of breath.
Irritability.
183. Which of the following presentations has worst prognosis
a) Paralysis
b) Parasthesia
c) Pallor
d) Pulselessness
Ans. D
Pulseless electrical activity (PEA), a cardiac arrest rhythm scenario with an associated poor prognosis, is
defined as cardiac electrical activity without a palpable pulse.
Ans. D
Central venous pressure (CVP) has a normal range of 5–7 mm Hg in an adult spontaneously breathing
patient while supine. The CVP is elevated in obstructive or cardiogenic shock, while it is decreased in
septic, neurogenicand hypovolemic shock.
185. A patient diagnosed to have empyema. Which of the following is the best treatment
a) Antibiotics only
b) Thoracotomy + antibotics
c) Pleurodesis + antibiotics
d) Intercostal chest tube drainage + antibiotics
Ans. D
Empyema is a collection of pus in the space between the lung and the inner surface of the chest wall
(pleural space).
Treatment is aimed at removing the pus and fluid from the pleura and treating the infection. Antibiotics
are used to treat the underlying infection. The specific type of antibiotic depends on what type of bacteria
is causing the infection. The method used to drain the pus depends on the stage of the empyema.
186. A female presented with symptoms of lethargy, weight gain, constipation, hair loss, dry skin
and intolerance
tocold. What is the most likely diagnosis
a) Hypothyroidisim
b) Hyperthyroidism
c) Hypoparathyroidism
d) Hyperparathyroidism
Ans. A
187. An elderly male presented with dyskinesia and rigidity. There was some pathology in
substantia nigra.Which
neurotransmitter is most likely involved
a) Acetylcholine
b) Serotonin
c) Glycine
d) Dopamine
Ans. D
The substantia nigra (SN) is an area of deeply pigmented cells in the midbrain that regulates movement
and coordination. ... Neurons of the SNc produce Dopamine, which stimulates movement. In contrast,
GABAergic neurons of the SNr can stimulate or inhibit movement depending on the input signal.
188. Elderly male with low backache and sclerotic lesions in vertebra is most likely having which
condition
a) Ca prostate
b) Multiple myeloma
c) TB of vertebra
d) Waldenstrom macroglobulinemia
Ans. A
Osteoblastic lesions are asso- ciated with increased bone formation and appear sclerotic.
Bone lesions resulting from prostate cancer are primarily osteoblastic, but are also associated with
increased bone resorption
189. A patient has serum K of 3.2 meq / L. What will be next line of treatment
a) IV KCl infusion
b) Oral KCI
c) IV Calcium gluconate
d) IV Insulin
Ans. B
Potassium supplements are generally prescribed for low potassium levels. If the situation is
severe, potassium might be given as an intravenous (IV) solution.
190. A young female with malar rash and fever.On Echocardiography there are vegetations on both
sides of mitral
valve. What is the likely diagnosis
a) A.RHD
b) B. Infective endocarditis
c) C. Non bacterial thrombotic endocarditis
d) Libman Sacks endocarditis
Ans. D
191. A patient had diarrhea associated with blood in stools. Physician diagnosed shigella as the
etiological agent of diarrhea. Which clinical feature supported shigella over cholera
a) Abdominal pain
b) Fever
c) Vomiting
d) Stool frequency
Ans. B
Symptoms Cholera = Shigella =
acute watery diarrhoea acute bloody diarrhoea
Stool > 3 loose stools per day. Watery > 3 loose stools per day, with
like reice water blood or pus
Fever No Yes
Abdominal cramps Yes Yes
Vomiting Yes a lot No
Rectal pain No Yes
192. A young boy was found to have wing beating tremors.Which is the most likely diagnosis
a) Sydenham Chorea
b) Wilson disease
c) Huntington chorea
d) Neurodegenative disease
Ans. B
Wing-beating tremor is a coarse, irregular tremor that is most prominent when the limbs are held
outstretched, reminiscent of a bird's flapping its wings; due to up and down excursion of arm at abducted
shoulder. Seen mainly with Wilson disease.
194. Which of the following agent doesn't cause bronchodilatation still used in emergency
management of bronchial asthma
a) Salbutamol
b) lpratropium
c) MgSO4
d) Hydrocortisone
Ans. D
Corticosteroids reduce the mucus secretion by inhibiting the release of secretagogue from macrophages.
Corticosteroids inhibit the late phase reaction by inhibiting the inflammatory response and interfering with
chemotaxis. This action may be due to the inhibition of LTB4 release.
Ans. D
The corticospinal tract, along with the corticobulbar tract, form two pyramids on either side of the
medulla of the brainstem—and give their name as pyramidal tracts. ... The lateral corticospinal
tract neurons cross the midline at the level of the lower medulla of medulla oblongata, and controls the
limbs and digits.
196. A young patient brought to the ED after road traffic accident. CT head shows evidence of raised
intracranial
pressure. What will be the next line of management
a) Mannitol
b) Furosemide
c) Acetazolamide
d) Glycerol
Ans. A
197. A 30 year old presented with vomiting and pain epigastrium which is radiating to back.On
examination the
findings asshown in the figure are noted.What is the diagnosis
a) Acute pancreatitis
b) Acute cholecystitis
c) Acute hepatitis
d) Ectopic ruptured pregnancy
Ans. A
Cullen's sign is superficial edema and bruising in the subcutaneous fatty tissue around the
umbilicus . It is named for gynecologist Thomas Stephen Cullen (1869–1953), who first described the
sign in ruptured ectopic pregnancy in 1916. This sign takes 24–48 hours to appear and can predict acute
pancreatitis, with mortality rising from 8–10% to 40%
198. A patient with deep X descent and rapid Y descent in JVP is most likely having
a) Constrictive pericarditis
b) Tamponade
c) Acute pericarditis
d) D.TR
Ans. A
199. A middle aged male presented with fatigue and weakness more in evening.His symptoms
improved after
administeringedrophonium.What is the most likely diagnosis
a) ALS
b) LES
c) Myasthenia Gravis
d) Botulism
Ans. C
The Tensilon test uses the drug Tensilon (edrophonium) to help you to diagnose myasthenia gravis.
Tensilon prevents the breakdown of the chemical acetylcholine, a neurotransmitter that nerve cells release
to stimulate your muscles.
200. A patient while being operated for a thoracic surgery have a sudden cardiovascular
collapse.There are
decreasedbreath sounds on right side.What is the likely cau
a) Tension pneumothorax
b) ARDS
c) Cardiac tamponade
d) Atelechesis
Ans. A
Tension pneumothorax is a major life‐threatening condition that must be recognized by clinical findings
and immediately treated. Air from a leak in the thoracic wall or the lung becomes trapped in the pleural
space, decreasing venous return to the heart and collapsing the opposite lung.
201. A male patient presented to ED with plasma glucose of 350 mg / dl and positive urine ketones.
Serum K 3.8
mmol / L,serum na- 130 mmol / L. Patient is dehydrated. You start normal saline. What will be
added in normal
saline
a) Glucose
b) NAHCO3
c) Insulin
d) KCI
Ans. C
Diabetic ketoacidosis is a serious complication of diabetes that occurs when your body produces high
levels of blood acids called ketones. The condition develops when your body can't produce enough insulin
202. A patient presented with these eye examination findings. He has behavior abnormalities and
poor hand
writing. Whichgene mutation is responsible for this presentation
Ans. B
203. The definition of steroid resistant nephrotic syndrome is absence of remission even after
taking steroids for
a) 4 weeks
b) 6 weeks
c) 8 weeks
d) 12 weeks
Ans. A
Steroid-resistant nephrotic syndrome (SRNS) is defined as NS resistant to steroid therapy, defined by
the absence of complete remission after four weeks of daily prednisone therapy at a dose of 60 mg/m2 per
day
204. Which of the following is true about severe vitamin D deficiency
a) Low calcium, high PTH
b) Low calcium, Low PTH
c) Normal calcium, High PTH
d) High calcium, high PTH
Ans. A
When vitamin D level is low, the absorption of calcium in the intestines becomes less, which then causes
the level of calcium in the blood to go down. As a consequence the parathyroid glands become more
active and produce more PTH that causes calcium to come out of the bones, therefore weakening the
bones.
205. A 5 year old boy has hyperflexible fingers and can touch back of hand with wrist extended.
What is the most
likelydiagnosis
a) Marfans syndrome
b) Ehler Danlos syndrome
c) Alport syndrome
d) Homocystinuria
Ans. B
Ehlers-Danlos syndrome is a group of inherited disorders that affect your connective tissues — primarily
your skin, joints and blood vessel walls. Connective tissue is a complex mixture of proteins and other
substances that provide strength and elasticity to the underlying structures in your body.
206. A 22 year old male presents with history of recurrent fall and difficulty in ambulation.On
examination
atrophied legmuscles are observed, Elder brother has same complaints.What is the diagnosis
Ans. C
A group of hereditary disorders that damage the nerves in the arms and legs.
Charcot-Marie-Tooth is a degenerative nerve disease that usually appears in adolescence or early
adulthood.
Muscle weakness, decreased muscle size, decreased sensation, hammer toes and high arches are symptoms.
The main treatments are physiotherapy and occupational therapy. Medication may reduce pain.
Muscular: abnormality walking, flaccid muscles, instability, lack of coordination, muscle weakness,
rhythmic muscle contractions, or loss of muscle
Foot: hammer toe, high foot arches, constantly walking on tip toe, or difficulty raising the foot
Sensory: pins and needles or reduced sensation of touch
Also common: hand contractures, slow reflexes, or tremor
207. A young female presents in her first trimester of pregnancy. She is taking warfarin for
prosthetic heart
valve. What should be next appropriate step
a) Continue warfarin
b) Add vitamin K
c) Stop warfarin and switch to heparin
d) MTP
Ans. C
Warfarin therapy should be avoided during pregnancy. If warfarintherapy is essential, it should be
avoided at least during the first trimester (because of teratogenicity) and from about 2 to 4 weeks before
delivery to reduce risk of hemorrhagic complications.
208. What is the arthropathy in diabetes with given x ray of ankle known as
a) Osteomyelitis
b) Charcoat arthropathy
c) Gout
d) Osteoarthritis
Ans. B
Charcot foot, also called Charcot arthropathy, is a disease that attacks the bones, joints, and soft tissue in
your feet. When it starts, you may not realize something's wrong. But eventually, it can cause painful sores
or change the shape of your foot
209. What is the acid base abnormality in patient presented with multiple episodes of vomiting
a) Metabolic acidosis
b) Metabolic alkalosis
c) Respiratory alkalosis
d) Respiratory acidosis
Ans. B
Vomiting or nasogastric (NG) suction generates metabolic alkalosis by the loss of gastric secretions,
which are rich in hydrochloric acid (HCl). Whenever a hydrogen ion is excreted, a bicarbonate ion is gained
in the extracellular space.
210. An arterial blood gas is performed and reveals-pH-7.2, paCO2-68 mmHg, and
serum HCO3- 26 mmol / L,
Diagnosis?
a) Respiratory acidosis partially compensated
b) Respiratory acidosis fully compensated
c) Metabolic acidosis with respiratory acidosis
d) Respiratory acidosis uncompensated
Ans. D
211. What is the GCS in a patient with eye opening to pain, inappropriate words and limb withdrawl
is
a) E2, M4,V3
b) E2, M3.V4
c) E2, V2,M4
d) E1, V3, M3
Ans. A
212. A female presents with chronic diarrhea, fatigue and dyspnea on exertion. On examination the
findings are
shown inthe figure. Lab test shows low Hb. What will be the appropriate treatment
a) Doxycycline
b) Folic acid
c) High protein diet
d) Iron
Ans. D
Koilonychia affects the shape of nails. The nails start to curve like a spoon. Possible causes include
nutritional deficiencies and autoimmune conditions.
Iron deficiency is the most frequent cause of koilonychia. Iron deficiency anemia is the world’s most
common nutritional deficiency disease. It most commonly affects children and women of childbearing age.
Koilonycia is a nail finding seen in iron deficiency anemia, syphilis, ischemic heart disease.
213. All the following conditions are associated with the sign shown in figure except
a) Pregnancy
b) Hypo estrogen state
c) Rheumatoid arthritis
d) Cirrhosis
Ans. B
Palmar erythema is a skin condition that makes the palms of your hands turn red, palms have turned red
because of dilated capillaries, which are the smallest blood vessels in the body.Mostly associated with
Pregnancy,Liver disease,Autoimmune diseases,Thyroid disease and it is a hyper estrogenic state.
214. Where is this pacemaker lead located ?
a) LA
b) RA
c) LV
d) RV
Ans. D
There are three basic kinds of pacemakers:
1-Single chamber: One lead attaches to the upper or lower heart chamber.
2-Dual-chamber: Uses two leads, one for the upper and one for the lower chamber.
3-Biventricular pacemakers(used in cardiac resynchronization therapy).
215. A patient presented with recurrent syncope episodes. ECG strip shows the following findings.
What is the most
appropriate treatment option for him
a) Pacemaker
b) ICD
c) Anticoagulant
d) Antiarrhythmic drugs
Ans. A
Sinus bradycardia is a type of slow heartbeat. A special group of cells begin the signal to start your
heartbeat. These cells are in the sinoatrial (SA) node. Normally, the SA node fires the signal at about 60 to
100 times per minute at rest. In sinus bradycardia, the node fires less than 60 times per minute.
Treatment of post infectious bradycardia usually requires permanent pacing. In patients with
hypothermia who have confirmed sinus bradycardia with a pulse, atropine and pacing are usually not
recommended because of myocardial irritability.
216. A 30-year-old patient was having breathingdifficulty on doing day to day activity. His
physiciannoticed
dullness onpercussion in right infra-axillaryarea with reduced air entry. CXR showed
pleuraleffusion and
pleural tapping was done. What findings will suggest an exudative pleural effusion?
a) Pleural fluid protein = 3.5 gm% and LDH= 100 IU
b) Pleural fluid protein=4.5 gm% and glucose= 30mg%
c) Pleural fluid LDH =90 IU and glucose=60mg %
d) Pleural fluid protein=3.5gm% and glucose=90 mg%
Ans. A
217. In ED a patient becomes unresponsive and ECG showed arrhythmia. DC shock is immediately
given. But he is
still unresponsive and arrhythmia is persisting. What is the next step
a) Repeat DC shock AV
b) CPR
c) Start bag and mask ventilation
d) Check carotid pulse
Ans. B
218. In ICU a patient is on mechanical ventilator. His ABG shows features of hypoventilation leading
to respiratory
acidosis. What will you do next to improve the ventilation of the patient
a) Increase FiO2
b) Increase respiratory rate
c) Increase PEEP
d) Increase expiratory time
Ans. B
Respiratory failure is divided into type I and type II. Type I respiratory failure involves low oxygen,
and normal or low carbon dioxide levels. Type II respiratory failure involves low oxygen, with high
carbon dioxide.
Type 1 respiratory failure may require only supplementary oxygen, but type 2 failure may require
additional support such as continuous positive airway pressure (CPAP) or biphasic positive airway
pressure (BiPAP) to increase exchange of both gases and, where possible, reverse any causes for low tidal
volumes or low respiratory rates.
219. Which blood vessel is involved in Type II Takayasu arteritis
a) Arch of aorta
b) Arch of aorta and ascending aorta
c) Arch of aorta and descending aorta
d) Abdomen aorta
Ans. B
Takayasu arteritis can be divided into the following six types based on angiographic
involvement :
Type I - Branches of the aortic arch
Type IIa - Ascending aorta, aortic arch, and its branches
Type IIb - Type IIa region plus thoracic descending aorta
Type III - Thoracic descending aorta, abdominal aorta, renal arteries, or a combination
Type IV - Abdominal aorta, renal arteries, or both
Type V - Entire aorta and its branches
Ans. D
Pulsus paradoxus (a decrease in the systolic blood pressure during inspiration) results from a decrease
in cardiac stroke volume with inspiration due to greatly increased left-ventricular afterload. Pulsus
paradoxus can be observed in cardiac tamponade and in conditions where intrathoracic pressure swings
are exaggerated or the right ventricle is distended, such as severe acute asthma or exacerbations of chronic
obstructive pulmonary disease.
221. 20-year-old Boy presented with pain in scrotum with red discoloration of scrotal skin. (On
lifting the testis pain was decreased) Diagnosis?
a) Torsion testis
b) Inguinal hernia
c) Hydrocele
d) Epididmyo-orchitis
Ans. D
Prehn's sign is an evaluation used to determine the cause of testicular pain. It is performed by lifting the
scrotum and assessing the consequent changes in pain. A positive Prehn's sign indicates relief of pain upon
elevation of the scrotum and is associated with epididymitis but exacerbates the pain of torsion.
222. A Patient has been admitted to the ICU since he is having fever for past (4 days with new onset
of breathlessness which was unresponsive to incremental flow rate of oxygen and saturation is
progressively decreasing. What can be the diagnosis?
a) Atelectasis
b) Idiopathic Pulmonary fibrosis
c) ARDS
d) Mediastinitis chest pain
Ans. C
Refractory hypoxemia, and this term usually considered when there is inadequate arterial oxygenation
despite optimal levels of inspired oxygen.
A hallmark of COVID-19 is the rapid development of refractory hypoxemia with a poor response to oxygen
supplementation, suggesting intrapulmonary shunting. Refractory hypoxemia in Covid 19 patients
occurs due to moderate to severe ARDS.
223. A 5-year-old boy starts choking at dinner table and is not able to cough or speak. What is the
immediate
management to be done in this child?
a) Abdominal Thrust
b) Drink water / Carbonated drinks fast
c) Give back blows
d) Identify the food consumed
Ans. C
224. 46 xx, absent uterus, vagina, normal secondary sexual characteristics. Ovary present
diagnosis, fsh and Lh
within normal limits?
a) MRKH
b) AIS
c) TURNER syndrome
d) Kliniefilter Syndrome
Ans. A
Bilateral mullerian duct agenesis:
Mullerian duct is absent so no fallopian tube ( distal FT is present ) , uterus, cervix, vagina 2/3
Gonad ovary
Lower 1/3 rd of vagina is present ( bcz it is from urogenital sinus )
Phenotype : female
Karyotype : 46xx
Clinical features :primary amennorhea
no mensus( no uterus )
no cyclic abdominal pain
normal secondary sexual characters ( breast development )
Examination :per vaginal : blind pouch vagina
USG : gonads present , no uterus
Gold standard investigation :laproscopy
Treatment : vaginoplasty ( done just before or after marriage ) Williams , Mc indoe’s
225. A 25 year old female attends gynae OPD with c/o secondary amenorrhea. She has history of
previous d and c
and her FSH levels are 6 IU/L . The probable cause of amenorrhea is
a) Sheehan syndrome
b) Ashermann syndrome
c) Premature ovarian failure
d) Pregnancy
Ans. B
Ashermansyndrome :
It is a rare acquired condition of the uterus
Scar tissue or adhesions form due to some form of trauma
Causes :c- section
fibroids/ polyps
Symptoms :no periods , pain but no bleeding bcz the blood is unable to leave the uterus bcz the
exit is blocked by scar tissue
Asherman syndrome increases risk during pregnancy : placenta previa
placenta increta
excesssive bleeding
Diagnosis : Hysteroscopy
Treatment : operative hysteroscopy : small instruments are attached to the end of the hysteroscope
and used to remove the adhesions
226. 16 yr old girl with primary amenorrhea. Ssc well developed. On examination there is bluish
bulging
hymen. What is the best management?
Ans. A
Imperforate hymen :
Means when the hymen covers whole opening of vagina
Symptoms :cryptomenorrhea
lack of menstrual cycle
secondary sexual characters are present
cyclic abdominal pain
Diagnosis : on P/V : bluish hymen is seen
Treatment : cruciate incision : x-shaped cut made in the hymen
227. A 55 year old women was found to have a carcinoma cervix figo stage 3b. What is the
management?
a) Concurrent chemoradiotherapy
b) Radiotherapy + hpv vaccine
c) Schauta procedure
d) Intracavitary brachytherapy followed by ebrt
Ans. A
228. A couple presented with infertility semen analysis is normal, female has menorrhagia. On
hsg there is distal
dilatation with fimbria. What is the likely cause?
a) Tubal polyp
b) Tubal endometriosis
c) Tubal spasm
d) Tubal salphingitis
Ans. D
Gonorrhea : distal dilatation of fimbria is seen
Etiology :neisseriagonorrheae ( gram –ve diplococcic)
m/c site : endo cervix >bartholinglan> urethra
clinical features : asymptomatic
mucopurulent endocervical discharge
urethral syndrome dysuria
pelvic pain
post coital bleeding
Investigation: culture : MC COY / HELA CELLS
NAAT ( nucleic acid amplification test ) - best test
urine test ( 1st in the mrng )
complications : 1.FITZ-HUGH CURTIS SYNDROME
2.REITER’S SYNDROME
3.ACUTE SALPINGITIS
4.ECTOPIC PREGNANCY
5. INFERTILITY
Treatment : doxycycline or azithromycin
In pregnancy cyclines are contraindicated so RX of choice in pregnancy azithromycin
229. Women with history of previous cs presented with prolonged labor. Patient is
hypotensive. Fhr is not
audible, fetal parts are easily palpable on pa examination, what is the diagnosis?
a) Abruption
b) Uterine rupture
c) Fetal intrauterine death
d) Polyhydramnios
Ans. B
Most common cause is previous caesarean section
Mcc innon scarred uterus is obstructed labor
Sign of impending rupture is fetal tachycardia
Sigh of rupture is fetal bradycardia
Management:
Uterine rupture : immediate laprotomy if not possible do hysterectomy
Impending rupture : immediate caesarean section
230. P212 postpartum presents with a few weeks of spotting and history of bleeding
per vaginum since 3 months
now comes with heavy bleeding. She gives h / o d and c in some other hospital but still continues
to
bleed. What could be the reason?
a) Normal Lochia
b) DIC
c) Retained bits of placenta
d) Gestational trophoblastic neoplasia
Ans. D
Normal lochia- is present for only first 14 days (no sudden bleeding will be seen in this case)
Retained bits of placenta is removed by Dilation and curettage (D&C)
Continuous bleeding per vaginum beyond 12weeks or beyond
Puerperium suspects Choriocarcinoma or Gestational Trophoblastic Neoplasia.
a) A
b) B
c) C
d) D
Ans. B
Usually around the midpoint of your cycle — LH secretion surges to really high levels(LH peak). This
hormone surge is what triggers ovulationabout 24 to 36 hours later.
233. A 23 year old primigravida presents at 19 weeks of pregnancy gestation for the first time. She
ahs no
significant medical history. Her family history is significant for diabetes running in the family. Her
ecg is
normal. Urinalysis is normal
Her FBS- 126 mg/dl
Hba1c- 6.5
Likely diagnosis is
a) Pregestational diabetes
b) Gestational diabetes
c) Normal pregnancy
d) Glucose intolerance
Ans. A
Women with pregestational diabetes mellitus defined by fasting ≥7 mmol/L (126 mg/dL) or 2 h ≥11.1
mmol/L (200 mg/dL),hba1c >6.5
Ans. C
Manual vacuum aspiration (MVA) is a way of surgically treating miscarriage when there is pregnancy
tissue remaining within the womb. MVA uses a narrow tube to enter and empty the womb
using aspiration (gentle suction).There are two methods of vacuum aspiration (also called suction
aspiration): Manual vacuum. This procedure can be used around 5 to 12 weeks after the last menstrual
period (early first trimester)
235. A 24 year old primigravida attends anc clinic with h / o bp 160/120 mmhg. Her lab findings
shows increased
liver enzymes and increased LDH. Low platelet count. Diagnosis?
a) AFLP
b) HELLP
c) Hepatitis B
d) Obstetric cholestasis
Ans. B
HELLP syndrome is a potentially life-threatening disorder that’s usually associated with preeclampsia
Hemolysis
EL: elevated liver enzymes
LP: low platelet count
Ans. B
Android pelvis- MC OP position and associated with deep transverse arrest, it’s a triangular or a male type
pelvis.
Anthropoid pelvis- its anterio posteriorly oval which is associated with persistent op position and face to
pubis delivery.
Gynecoid pelvis- MC and the best pelvis.
Platypelloid pelvis- Rarest pelvis, it is Flat and transversely oval, its associated with face presentation and
Asynclitism
Ans. B
Uterine inversion means the placenta fails to detach from the uterine wall, and pulls the uterus inside-
out as it exits.Uterine inversion is an unusual and potentially life‐threatening event occurring in the third
stage of labour. It is associated with significant blood loss, and shock, which may be out of proportion to
the haemorrhage, When managed promptly and aggressively, uterine inversion can result in minimal
maternal morbidity and mortality.
238. A 25 year old primigravida female is on lithium for treatment of her psychiatric illness for
past 2 years . The
m/c anomaly seen in fetus whose mother is on lithium during pregnancy is
a) VSD
b) PDA
c) Ebstein anomaly
d) NTD
Ans. C
Lithium causes Ebstein’s anomaly if used during pregnancy
Ebstein’s anomaly is a congenital cardiac malformation
239. A 25 year old women visits Iabour room at 37 weeks. Pt has mild labor pain for 10 hrs and
cervix is
persistently 1 cm dilated and uneffaced. Management?
a) LSCS
b) Arm
c) Sedation and wait
d) Augmentation with oxytocin
Ans. C
After sedation false labor pain subsides
Braxton Hicks contractions are the "false" labor pains that a pregnant woman might have before
“true” labor. They're your body's way of getting ready for the real thing. But they don't mean labor has
started or is about to begin.Before "true" labor begins, you may have "false" labor pains.
240. A pregnant women with prosthetic valve on warfarin tests positive for pregnancy. What is
the advice to be
given?
a) Discontinue warfarin
b) To replace warfarin with heparin
c) Add vit k with warfarin
d) To continue warfarin throughout pregnancy
Ans. B
241. A G4 P0+3 , 24 year old female presents at 22 weeks with previous history of three
midtrimester abortions,
On usg her cervical length is 20 mm What could be the most probable cause of her recurrent
pregnancy loss
a) Cervical Incompetence
b) Uterine Anomaly
c) Hormonal Disturbance
d) Chromosomal Anomalies
Ans. A
Common causes of miscarriage in the second trimister include cervical insufficiency (the premature
dilation of the cervix) or preterm labor (also known as premature birth). With cervical insufficiency (also
known as an incompetent cervix), the baby may be born too early to survive.
242. A pregnant women comes for checkup at 18 weeks. On examination it was found that the
uterine size
corresponds to only 16 weeks. On usg, oligohydrominos was found. Which of the following is likely
cause?
a) Fetal Anemia
b) Anencephaly
c) Renal agenesis
d) DM
Ans. C
243. Which is most cost effective cervical cancer screening test?
a) VIA
b) HPV DNATesting
c) PAP smear
d) Colposcopy
Ans. A
Cost effective
1. Lugol iodine
2. Visual inspection of Acetic acid
BEST :Pap smear
244. A 55 year old post menopausal female attends obgy clinic with history of chronic pelvic pain.
An Xray pelvis
showed the following image The most likely diagnosis is
a) Ovarian mass
b) PID
c) Calcified fibroid
d) Bladder stone
Ans. C
leiomyoma is a benign lesion in postmenopausal age group. It causes diagnostic confusion with
solid calcified adnexal mass and large bladder calculi at the pelvic region. A calcified fibroid is when
a fibroid has reached the final stage of degeneration, or cell death and calcium deposits develop on the
remaining fibroid tissue Clinical and radiological diagnoses were confirmed by histopathology of the
hysterectomy specimen.
245. Which of the following have a false positive Barr body
a) Turners syndrome
b) Trisomy 21
c) Klinefelter syndrome
d) Androgen insensitivity syndrome
Ans. C
A Barr body (named after discoverer Murray Barr) is an inactive X chromosome in a cell with more than
one X chromosome, rendered inactive in a process called lyonization, in species with XY sex-
determination.
Turners syndrome-45XO
Klinefelter syndrome-47XXY
Androgen insensitivity syndrome-46XY
Ans. C
A total hysterectomy is the removal of the uterus and cervix. A total hysterectomy with bilateral salpingo-
oophorectomy is the removalof the uterus, cervix, fallopian tubes (salpingo) and ovaries (oophor)
247. A female after delivery complains of cramps during breast feeding Which of the following
hormone is
responsible for it
a) Prolactin
b) Oxytocin
c) Estrogen
d) Progesterone
Ans. B
Nipple stimulation during breastfeeding causes a hormone known as oxytocin to be released into your
bloodstream. This hormone causes the contraction of all smooth muscles and helps your uterus contract
back into its pre-pregnancy shape and size. These contractions also help reduce postpartum blood loss,
so although you may be uncomfortable, this cramping is helping your body heal.
248. After binge drinking of alcohol a young male presented with vomiting and upper
Glbleed.Most likely cause
is-
a) Boerrhave syndrome
b) Mallory Weiss tear
c) Duodenal ulcer perforation
d) Tension pneumothorax
Ans. B
In Mallory Weiss syndrome, vigorous vomiting produces a vertical split in the gastric mucosa,
immediately below the squamocolumnar junction at the cardia in 90% of the cases. In only 10% is the
tear in the oesophagus.
Mallory Weiss tear occurs just below GE junction, partial tear involving mucosa and submucosa.
Patient present with painless hematemesis.
Ans. C
Ans. A
In image we can see dilated bowel loops with
valvulae conniventes. Valvulae conniventes is seen in jejunem means obstruction is in ileum. Bowel loops
are placed centrally (in large bowel obstruction bowel lops are placed peripherally and we can see
haustration)
251. A 35 year old patient presented with a one year history of low grade fever, anorexia and weight
loss. During
investigation with a contrast evaluation of the bowel, following images were seen. Possible
diagnosis?
a) Ileocaecal tuberculosis
b) Volvulus
c) Cancer colon
d) Diverticulosis of colon
Ans. A
In image the position of caecum is subhepatic (almost located in the right hypochondrium).
Generally the ileo-caecal angle is 900. In tuberculosis we see pulled up caecum i.e., obtuse ileo- caecal
angle which is known as GOOSE NECK DEFORMITY.
252. A female patient presented to the hospital with complaint of vomiting, diarrhea, wheezing,
flushing and palpitations. On laboratory evaluation, high serotonin was found. What is the
diagnosis?
a) VIPoma
b) Somatostatinoma
c) Pheochromocytoma
d) Carcinoid syndrome
Ans. D
carcinoid syndrome can produce number of vasoactive peptides, most
commonly 5- hydroxytryptamine (serotonin), but also histamine,
prostaglandins and kallikrein. When they metastasise to the liver, the
‘carcinoid syndrome’ can become evident, because the vasoactive
substances escape the filtering actions of the liver. The clinical
syndrome itself consists of reddish- blue cyanosis, flushing attacks,
diarrhea, borborygmi, asthmatic attacks and, eventually, pulmonary and
tricuspid stenosis. Classically, the flushing attacks are induced by
alcohol.
other options-
VIPoma →is aka Vernal Morrisons syndrome/WDHA syndrome.
Somatostatinoma→ these patients have DM, steatorrhea and gall stones.
Pheochromocytoma→ headache + diaphoresis + palpitations.
253. In an asymptomatic patient, on routine screening a small mass was detected in the rectum.
Carcinoma rectum was suspected. What is the next step in the management?
a) Colonoscopy with lesion biopsy
b) Sigmoidoscopy with lesion biopsy
c) CT pelvis
d) Barium enema
Ans. B
Rigid sigmoidoscopy can be performed in the outpatient clinic and is useful to identify the neoplasm and
possibly obtain biopsies. However, it requires the rectum to be empty of faces and may require a prior
rectal enema, which may not be practical in the outpatient setting. As colonoscopy is almost always
required to visualize the whole colorectum, it is often easier and safer to obtain biopsies at this time.
Proctoscope 10-12 cm
Rigid sigmoidoscope 25 cm
Flexible sigmoidoscope 60 cm
Colonoscope 160
Ans. A
255. Patient presented with history of fever and pain in right lower abdomen for last 5 days. On
palpitation, painful lump was palpable in right iliac fossa. On laboratory investigations,
leukocytosis was found. What is the most probable diagnosis?
a) Hydronephrosis
b) Appendicular lump
c) C.Tuberculosis
d) D.Carcinoma colon
Ans. B
256. A 34 year old female presented with a mass in the inguinal region which produced gurgling
sound on reduction. The mass reduced completely on pressing it through the deep inguinal ring.
Possible diagnosis in this lady?
a) Indirect inguinal hernia
b) Direct inguinal hernia
c) Femoral hernia
d) Pantaloon hernia
Ans. A
Indirect inguinal hernia
Other option—
Direct inguinal hernia (not seen in females)
Femoral hernia (hernia occurs in femoral ring)
Pantaloon hernia (have dual sac)
257. Very old male presented with a ballotable mass in the left flank region. On examination
painless hematuria,
RBC's were found in urine, while there were no WBC's in urine. Most probable diagnosis in this
patient?
a) Hydronephrosis
b) Pyelonephritis
c) Renal cell carcinoma
d) D.Polycystic kidney disease
Ans. C
mass+ pain( in lumbar/loin region) + hematuria
258. An old man presented to hospital with low back pain and sclerotic lesion in vertebrae and
pelvic bone. What
is the most likely diagnosis in this patient?
a) Testicular tumor
b) Lung cancer
c) Colon cancer
d) Prostate cancer
Ans. D
Prostate cancer metastasis to lumbar vertebra.( pelvic bones are also involved)
RCC, colon cancer—lungs
259. Type of bladder cancer seen in the patient with history of working with dye- related
industry?
a) Adenocarcinoma
b) Squamous cell carcinoma
c) Transitional cell carcinoma
d) Papillary carcinoma
Ans. C
260. A patient with history of trauma has ruptured bulbar urethra. What is the location for
accumulation of urine?
a) Superficial pouch
b) Deep pouch
c) Retrouterine pouch
d) Uterovesical pouch
Ans. A
In bulbar urethral injury there is superficial extravasation.
Urethral injury is of 2 types
Anterior and posterior ( C/F- urinary retention, blood in meatus and hematoma)
In anterior urethral injury – perineal hematoma
In posterior urethral injury – pelvic hematoma
261. 37 years old women came to the hospital with the folloing presentation (image) what is the t-
staging ofof
the cancer?
a) T4a
b) T4b
c) T4c
d) T4d
Ans. B
262. 60 years old lady presented with a hard breast mass of 5x6cm in size, single lymph node mass
present. There
was no evidence of metastasis? What should be the ideal treatment?
a) Radical mastectomy with chemotherapy
b) Modified radical mastectomy followed by adjuvant chemotherapy
c) Neoadjuvant chemotherapy with Modified radical mastectomy with radio therapy
d) Palliative mastectomy.
Ans. A
263. A women with breast malignancy as shown in image,it is not associated with lymph node
metastasis, not movable and attached to the rib ,what is the diagnosis?
a) Phyllodes tumour
b) Brest abscess
c) Inflammatory breast cancer
d) Leiomyosarcoma
Ans. A
264. A lactating female with history of breast feeding for 15 days presented to emergency with
fever. On examination, breast was red as shown in the image .what is the probable diagnosis?
a) Breast abscess
b) Milk fever
c) Tuberculosis
d) None of the above
Ans. A
Ans. D
I-123 =13 Hrs used for diagnosis RAI(radio active iodine up take scan)
I-131 =8 days used for diagnostic and therapitic
Also used for Radio active ablation
266. A patient came with the pain in right hypochondrium .bloodinvestigantion shows AST-22 IU/l
and ALT-27 IU/ L,serum bilirubine levels -6mg% and ALP-200 mg % what should be the
investingation in thus patient?
a) USG abdomen
b) CECT abdomen
c) Hepatitis A and Hepatitis E markers
d) HIDA scan
Ans. A
267. A female patient presented to the hospital with history of recurrent pain in the right
hypochondrium for 1 year. what is the diagnosis?
a) Hydatid cyst
b) Acute pancreatitis
c) Acute cholecystitis
d) Amobic liver abscess
Ans. C
268. A 60 years old male who is heavy alcoholic present with 12 hours history of sharp central
abdominal pain and vomiting, preferred to nsit, rather lying down as pain increases when lying
down. pulse rate 132/min, bp 118/82 mmhg, temperature is 38.3 c. on examibnation he has mild
jaundice. There is periumbilical and flank discoloration as shown in the image and minimum
tenderness in epigastrium. What is the diagnosis?
a) acute pancreatitis
b) pyelonephritis
c) acute viral hepatitis
d) perforated duodenal ulcer
Ans. A
key points
BLUISH DISCOLERATION OF THE FLANKS =GREY TURNER SIGN
DIS COLERATION AROUND UMBILICUS =CULLEN SIGN
269. Alcoholic male presented with pain in epigastrium radiating to the back. What is the likely
diagnosis in the given CT image ?
Ans. B
Ans. A
also called : PUESTOW’S PROCEDURE LATERAL PANCREATIOJEJUNOSTOMY
Involves a retrocolic side-to-side roux-en-ypancreaticojejunostomy
a) Pancreaticojejunostomy
b) Whipples procedure
c) Radical pancreatectomy
d) Duodenojejunostomy
Ans. B
272. In a new born male baby, mother found empty scrotum.on examination, left testis is in
inguinal region and
right testis is in perineum. Which of the following is correct?
a) Right testis is undescended and left testis is ectopic
b) Left testis is undescended and right testis is ectopic
c) Both are undescended
d) Both are ectopic
Ans. B
left – is at inguinal region –undescended
right is at perienguinal- ectopic
273. In a new born bady with empty scrotum, testis was was found to be laying close to the
peritoneum.which of
the following helps the testis to reach the scrotum?
a) GnRH Analogues
b) Testosteron
c) Cold compression
d) Manually bring testis down to the bottom of scrotum
Ans. A
Maternal chorionic gonadotrophin growth of the testis and may stimulate its migration so we give baby
GnRH analog that helps the testis to reach the scrotum
274. 18 years old boy presented in agony with sudden onset of severe pain in the groin region,
associated with
redness and swelling of the scrotum. On examination the right testis as gone up. Possible
diagnosis in this
boy is?
a) Testicular torsion
b) Hydrocele
c) Strangulated inguinal hernia
d) Acute epididymo-orchitis
Ans. A
Testicular torsion:
Hydrocele:
275. A 35 years old male presented with scrotal enlargement on investigation, LDH was significantly
high but the HCG and alpha fetoprotin were within normal limits .histopathology image is shown
below what is the most
likely diagnosis?
a) Teratoma of testis
b) Seminoma of testis
c) Choriocarcinoma
d) Yolk sac tumor
Ans. B
276. A young male underwent lumbar sympathectomy and bilateral L1 ganglions were removed.
What is the most commion side effect?
a) Impotence
b) Retrograde ejaculation
c) Bladder dysfunction
d) Incontinence
Ans. B
patients with burgers diseas who have rest pain we perform lumbar sympathectomy,
in this we remove L1 and L2 ganglion on one side
if we remove both sides the patient will have retrograde ejaculation
it can be prevented by sparing of L1 at one side
277. A patient under went surgery, from which she was immobilized.patient started complaining
of leg pain on the 5th post operative day, on examination Homans sign was found to be positive.
What is the most probable
diagnosis?
a) Deep venous thrombosis
b) Urinary sepsis
c) Peripheral vasculsar disease
d) Lymphangitis
Ans. A
278. A patient with old burn scar develop symptoms over burnt area . it was indicative of
malignancy.what is the type of malignancy?
Ans. B
given condition-MARJOLINS ULCER
279. A patient with blunt trauma was brought to the emergency with the pulse rate 120/min
bp80/50. What is the next best step?
a) FAST
b) DPL
c) Exploratory laprotomy
d) CECT
Ans. A
Patient whether stable or unstable first FAST should be done
(FAST-focused assessment with sonography for trauma)
280. A patient of blunt abdominal trauma was brought in the emergency with cold clammy skin
and increased puls rate. What is the next step of management?
a) CECT
b) Exploratory laprotomy
c) Resuscitation with blood products
d) Resuscitation with intervenous crystalloids
Ans. D
given only one liter of warm isotonic crystalloids for adults
child <40kg given 20ml/kg of warm isotonic crystalloids
281. A patient under went a surgery for ingunial hernia 5 days before. Now comes to the hospital
with complaints of fever. What can be the cause?
a) Atelectasis of lung
b) Surgical site infection
c) Thromboembolism
d) Chest infection
Ans. B
cause of post-op fever
POD 1-2: Atelectasis
POD 2-3: UTI
POD 3-7: Wound infection (ssi)
POD 5-7: DVT/Thrombophlebitis
POD >7: drug fever
282. Identify the movement taking place in the intestine?
a) Segmented movement
b) Peristaltic movement
c) Passive diffusion
d) Intersegmental contraction
Ans. B
GI – Motility
Types
2- types
Food
Major inititoing response
for persislassis is – local
stretching of gest
Yes No
Fed state pattern food Fasting state pattern
Para & of each
Food F
Contraction felaxation 2
Ans. A
a) A
b) B
c) C
d) D
Ans. A
About 2/3rd of the filtered water is reabsorped in proximal convoluted tubule along with salt
absorption.other molecules absorped here are bicarbonates, glucose, amino acids and phosphates.
Ans. C
Ans. D
Calcium Regulation :
3 HORMONES 3 organs modal
1. Parathormone
2. Vit D Ca2+ 1. Bone (Bank for Ca2+)
3. Calcitonin Ca2+ 2. GIT
3. Kidneys
Parathormone (Actions)
1. Bone 2 cells PTH PTH
Osteo blast Osteo.Clast Bone resorption
PTH receptor + Ca2+ - PO42-
RANK ligand Crystal
Rx : Osteoporosis
Rank Ligand Blocker
DENOSUMAB Ca2+ PO42-
Bone (PTH)
2) Kidneys G PTH
Excreation of Ca2+
Excreation of PO42-
PTM
Phosphaturic action
Over all action
PTH PO42- Terminating Hormone
Ans. A
Protiens not filted by GC Glossullary capillary
3 reason
1. Charge Repals
-Ve
Protein Anions
Gc
?
Heparan, sulfate
Heparan SO42-
(Proteoglycans)
2. Size
GC – Small gap
Fenestrations Fenestrated Endothelium
10 – 90 nm
3. Slit diaphram :
Structure
Proteins
i. Neprin NPHS 1 geno
mutation
Congenital. N syndrome = (protein Excretion)
Finish type
Ans. B
Type I pneumocytes Type II pneumocytes
Large Small
Less in number More in number
More area SURFACTANT
Stem cells
Surfactant :
Lipids (major—lecithin ), (minor –spingomyelin)
Protein :A,B,C,D
In amniotic fluid—bubble test, shake test, clement test.
Function of surfactant:
↓ in surface tension
Prevent collapse of alveoli
Stability of alveoli
SP : A and D → lung maturity
Surfactant stored in Lamellar body.
Regulation of surfactant :
↑surfactant : steroids, glucocorticoids, cortisol, T3 and T4.
Ans. A
Palmar grasp (immature) should logically also appear first followed by transferring objects
Bi dexterous appear by 4th month
Unidextrous appear by 6th month
290. A 4 years old child,presented with thisty,drinks eagerly,irritable with dehydration, what the
treatment ?
a) ORS
b) Ringer lactate
c) Plain normal saline
d) Refer to district hospital
Ans. A
DEHYDRATION:
Skin pinch test
>1yr 30 min 2 ½ hr
Patient comes with an enlarged leg with sulphur crystals coming out (madura foot as shown in the
image). Organisum responsible is:
Explanation :
Actinomycotic mycetoma: Sinus ,swelling,granules(discharging)
Actinomycetoma Eumycetoma
Numerous sinus and swelling Single sinus and swelling
Colour of granules→ yellow/white except Black →madurella mycetomatis
in Actinomadural pellitri where they have White
red granules Acremonium
Respond to antibiotics Pseudallescheria boydii
Amputation
CTEV
Image 1: Area of consolidation middle lobe which silhouette right heart border
Image 2: cardiac shadow is seen on the middle lateral lobe
Plain radiograph
For implantable cardio-defibrillator ( ICD) , a chest x-ray should be obtained to confirm lead
position and examine lead integrity
Most pulse generators, both pacemakers and ICD’s, can be identified from their appearance
under radiography: most helpful method of identifying unknown devices
Q. What is the radioisotope used in PET scan ?
Ans: 18FDG
ANNEXURE 5. HALF – LIVES OF RADIOACTIVE ISOTOPES
Substance Half – life
Iodine – 132 2.3 hours
Technetium – 99 6 hours
Iodine – 123 13 hours
Gold – 198 2.7 days
Gollium -67 3.3 days
Iodine – 131 8 days
Phosphorus -32 14.3 days
Iodine -125 59 days
Iridium – 192 74 days
Cobalt -60 5.3 days
Strontium – 90 29.1 years
Caesium -137 30 years
Radium – 226 1600 years
Uranium – 238 4.5 billion years
All
the
Best
Follow us on
f cy d
ARISE - Hyderabad
+91 7680929292
040 2351 5252
# 2ⁿ Floor, Above Indian Bank,
Opp. Olive Hospital,
Kakatiya Nagar Colony,
Mehdipatnam, Hyderabad - 500 028
arisemedicalacademy@gmail.com
ARISE - Kerala (Kannnur) ARISE - Delhi
+91 8136932666 +91 9100728811
+86 15778411369
# 2ⁿ Floor, Kingdom Tower, Goutham Nagar,
Manna, Taliparamba,
Kannur, Kerala.
arisemedicalacademy.kerala@yahoo.com arisemedicalacademy@gmail.com